Apuntes MAT023 Completo UTFSM

1 MAT023 A PUNTES DE CLASES Departamento Matem´atica UTFSM Santiago Parte I Ecuaciones diferenciales ordinarias 5

Views 160 Downloads 1 File size 6MB

Report DMCA / Copyright

DOWNLOAD FILE

Recommend stories

Citation preview

1

MAT023 A PUNTES DE CLASES

Departamento Matem´atica UTFSM Santiago

Parte I Ecuaciones diferenciales ordinarias

5

´Indice general

´Indice general

1

I

5

Ecuaciones diferenciales ordinarias

1. Ecuaciones diferenciales ordinarias de primer orden

6

1.1. Definiciones . . . . . . . . . . . . . . . . . . . . . . . . . . . . . . . . . . .

6

1.2. Modelos simples . . . . . . . . . . . . . . . . . . . . . . . . . . . . . . . . .

11

1.3. M´etodos Elementales de Resoluci´on . . . . . . . . . . . . . . . . . . . . . .

19

1.3.1. Integraci´on directa . . . . . . . . . . . . . . . . . . . . . . . . . . .

19

1.3.2. Ecuaciones de variable separable . . . . . . . . . . . . . . . . . . . .

20

1.3.3. Ecuaci´on lineal de primer orden . . . . . . . . . . . . . . . . . . . .

24

1.3.4. Ecuaci´on de Bernoulli . . . . . . . . . . . . . . . . . . . . . . . . .

28

1.3.5. Ecuaci´on de Ricatti . . . . . . . . . . . . . . . . . . . . . . . . . . .

30

1.3.6. Ecuaciones homog´eneas

. . . . . . . . . . . . . . . . . . . . . . . .

34

1.3.7. Otros cambios de variables . . . . . . . . . . . . . . . . . . . . . . .

37

1.4. Modelos simples: Segunda parte . . . . . . . . . . . . . . . . . . . . . . . .

42

1.5. An´alisis cualitativo . . . . . . . . . . . . . . . . . . . . . . . . . . . . . . .

51

1.5.1. M´etodos cualitativos . . . . . . . . . . . . . . . . . . . . . . . . . .

52

1.5.2. Ecuaciones diferenciales aut´onomas . . . . . . . . . . . . . . . . . .

55

1.5.3. Equilibrio y estabilidad . . . . . . . . . . . . . . . . . . . . . . . . .

59

1.6. Ejercicios del cap´ıtulo . . . . . . . . . . . . . . . . . . . . . . . . . . . . . .

67

2. Ecuaciones diferenciales lineales de orden superior

76

2.1. Elementos de transformaciones lineales . . . . . . . . . . . . . . . . . . . .

76

2.1.1. Definiciones . . . . . . . . . . . . . . . . . . . . . . . . . . . . . . .

76

2.1.2. N´ ucleo e imagen

. . . . . . . . . . . . . . . . . . . . . . . . . . . .

85

2.1.3. Isomorfismo . . . . . . . . . . . . . . . . . . . . . . . . . . . . . . .

92

2.1.4. Matriz asociada a una transformaci´on lineal . . . . . . . . . . . . .

93

2.1.5. C´alculo con coordenadas . . . . . . . . . . . . . . . . . . . . . . . . 101 2.1.6. Ejercicios resueltos de Transformaciones lineales . . . . . . . . . . . 105 1

Apuntes Mat023 (versi´on preliminar actualizada 23-05-2014)

2.2. Ecuaciones diferenciales lineales . . . . . . . . . . . . . . . . . . . . . . . . 142 2.2.1. Definiciones . . . . . . . . . . . . . . . . . . . . . . . . . . . . . . . 142 2.3. Teorema de Existencia y Unicidad . . . . . . . . . . . . . . . . . . . . . . . 148 2.4. El wronskiano . . . . . . . . . . . . . . . . . . . . . . . . . . . . . . . . . . 152 2.5. Ecuaciones diferenciales a coeficientes constantes . . . . . . . . . . . . . . . 158 2.5.1. Definiciones . . . . . . . . . . . . . . . . . . . . . . . . . . . . . . . 158 2.5.2. La ecuaci´on de orden 2 . . . . . . . . . . . . . . . . . . . . . . . . . 158 2.5.3. La ecuaci´on de orden superior . . . . . . . . . . . . . . . . . . . . . 163 2.6. M´etodo de variaci´on de par´ametros . . . . . . . . . . . . . . . . . . . . . . 167 2.7. M´etodo del anulador . . . . . . . . . . . . . . . . . . . . . . . . . . . . . . 174 2.8. Movimiento vibratorio . . . . . . . . . . . . . . . . . . . . . . . . . . . . . 184 2.9. Ejercicios del cap´ıtulo . . . . . . . . . . . . . . . . . . . . . . . . . . . . . . 190 3. Sistemas de ecuaciones diferenciales

196

3.1. Definiciones . . . . . . . . . . . . . . . . . . . . . . . . . . . . . . . . . . . 196 3.2. Ecuaci´on con coeficientes constantes . . . . . . . . . . . . . . . . . . . . . . 200 3.2.1. Matriz A diagonalizable . . . . . . . . . . . . . . . . . . . . . . . . 201 3.3. Variaci´on de par´ametros en sistemas . . . . . . . . . . . . . . . . . . . . . 206 3.4. An´alisis cualitativo de sistemas . . . . . . . . . . . . . . . . . . . . . . . . 213 3.4.1. Valores propios reales y distintos (no nulos) . . . . . . . . . . . . . 214 3.4.2. Valores propios complejos . . . . . . . . . . . . . . . . . . . . . . . 218 3.4.3. Valores propios repetidos (no nulos) . . . . . . . . . . . . . . . . . . 222 3.5. Ejercicios del cap´ıtulo . . . . . . . . . . . . . . . . . . . . . . . . . . . . . . 226 4. Transformaci´ on integral de Laplace

231

4.1. Definiciones y teoremas fundamentales . . . . . . . . . . . . . . . . . . . . 231 4.2. C´alculo de transformadas . . . . . . . . . . . . . . . . . . . . . . . . . . . . 235 4.3. Primer Teorema de la Traslaci´on

. . . . . . . . . . . . . . . . . . . . . . . 238

4.4. Transformada de la derivada . . . . . . . . . . . . . . . . . . . . . . . . . . 239 4.5. Funciones escalonadas y Segundo Teorema de la Traslaci´on . . . . . . . . . 246 4.6. La Transformada de integrales de convoluci´on . . . . . . . . . . . . . . . . 250 4.7. Ejercicios del cap´ıtulo . . . . . . . . . . . . . . . . . . . . . . . . . . . . . . 256

2

Apuntes Mat023 (versi´on preliminar actualizada 23-05-2014)

5. Series de Fourier

263

5.1. Definiciones . . . . . . . . . . . . . . . . . . . . . . . . . . . . . . . . . . . 263 5.1.1. El espacio SC [a, b] . . . . . . . . . . . . . . . . . . . . . . . . . . . 263 5.1.2. Teorema de la mejor aproximaci´on . . . . . . . . . . . . . . . . . . 269 5.2. Convergencia Puntual de series de Fourier . . . . . . . . . . . . . . . . . . 279 5.3. Series de Fourier de senos y cosenos . . . . . . . . . . . . . . . . . . . . . . 284 5.4. Derivaci´on e integraci´on de Series de Fourier . . . . . . . . . . . . . . . . . 294 5.5. Ejercicios del cap´ıtulo . . . . . . . . . . . . . . . . . . . . . . . . . . . . . . 295

II

C´ alculo diferencial en varias variables

6. Elementos de topolog´ıa de Rn

301 302

6.1. El espacio euclidiano Rn . . . . . . . . . . . . . . . . . . . . . . . . . . . . 302 6.2. Producto interno y norma . . . . . . . . . . . . . . . . . . . . . . . . . . . 302 6.3. Elementos de topolog´ıa de Rn . . . . . . . . . . . . . . . . . . . . . . . . . 306 6.4. Ejercicios del cap´ıtulo . . . . . . . . . . . . . . . . . . . . . . . . . . . . . . 310 7. Funciones de varias variables

314

7.1. Definiciones b´asicas . . . . . . . . . . . . . . . . . . . . . . . . . . . . . . . 314 7.2. Gr´aficos, conjuntos de nivel y trazas . . . . . . . . . . . . . . . . . . . . . . 316 7.3. Ejercicios del cap´ıtulo . . . . . . . . . . . . . . . . . . . . . . . . . . . . . . 321 8. L´ımites y continuidad

323

8.1. Definiciones . . . . . . . . . . . . . . . . . . . . . . . . . . . . . . . . . . . 323 8.2. C´alculo de l´ımites . . . . . . . . . . . . . . . . . . . . . . . . . . . . . . . . 329 ´ 8.2.1. Algebra de l´ımites . . . . . . . . . . . . . . . . . . . . . . . . . . . . 330 8.2.2. Desigualdades y Teorema del Sandwich . . . . . . . . . . . . . . . . 331 8.3. Continuidad . . . . . . . . . . . . . . . . . . . . . . . . . . . . . . . . . . . 332 ´ 8.4. Algebra de funciones continuas . . . . . . . . . . . . . . . . . . . . . . . . 334 8.5. Continuidad de funciones vectoriales . . . . . . . . . . . . . . . . . . . . . 336 8.6. Ejercicios del cap´ıtulo . . . . . . . . . . . . . . . . . . . . . . . . . . . . . . 338

3

Apuntes Mat023 (versi´on preliminar actualizada 23-05-2014)

9. Diferenciaci´ on en varias variables

341

9.1. Derivadas parciales . . . . . . . . . . . . . . . . . . . . . . . . . . . . . . . 341 9.2. Interpretaci´on de la derivada parcial

. . . . . . . . . . . . . . . . . . . . . 348

9.3. Diferenciabilidad . . . . . . . . . . . . . . . . . . . . . . . . . . . . . . . . 355 9.4. Derivadas de orden superior y funciones de clase C n . . . . . . . . . . . . . 369 9.5. Gradiente y matriz jacobiana . . . . . . . . . . . . . . . . . . . . . . . . . 373 9.6. La regla de la cadena . . . . . . . . . . . . . . . . . . . . . . . . . . . . . . 377 9.7. Gradiente y planos tangentes

. . . . . . . . . . . . . . . . . . . . . . . . . 388

9.8. Derivada direccional y direcciones de crecimiento m´aximo . . . . . . . . . . 391 9.9. Ejercicios del cap´ıtulo . . . . . . . . . . . . . . . . . . . . . . . . . . . . . . 396 10.M´ aximos y m´ınimos

409

10.1. Extremos locales . . . . . . . . . . . . . . . . . . . . . . . . . . . . . . . . 409 10.2. M´aximos y m´ınimos en compactos y/o con restricciones . . . . . . . . . . . 425 10.3. Extremos restringidos Multiplicadores de Lagrange . . . . . . . . . . . . . 428 10.3.1. Criterio de la segunda derivada para extremos condicionados . . . . 434 10.4. Ejercicios del cap´ıtulo . . . . . . . . . . . . . . . . . . . . . . . . . . . . . . 439 11.Funciones impl´ıcitas e inversas

444

11.1. El teorema de la funci´on impl´ıcita . . . . . . . . . . . . . . . . . . . . . . . 444 11.2. El teorema de la funci´on inversa . . . . . . . . . . . . . . . . . . . . . . . . 460 11.3. Ejercicios del cap´ıtulo . . . . . . . . . . . . . . . . . . . . . . . . . . . . . . 470

III

Evaluaciones de a˜ nos anteriores

473

12.Controles

474

13.Cert´ amenes

489

Bibliograf´ıa

524

4

Parte I Ecuaciones diferenciales ordinarias

5

Cap´ıtulo 1 : Ecuaciones diferenciales ordinarias de primer orden

Definiciones

En general, entenderemos el modelamiento matem´atico como el proceso de establecer un modelo matem´atico (es decir, un sistema expresado en t´erminos de variables, funciones, ecuaciones, etc.) que represente una situaci´on —principalmente de naturaleza f´ısica—, su resoluci´on matem´atica, y finalmente la interpretaci´on de los resultados en los t´erminos f´ısicos originales. Como muchos conceptos de la naturaleza, tales como velocidad, aceleraci´on, las reacciones qu´ımicas, los cambios de temperatura observados en un cuerpo, etc. se expresan como razones de cambio tiene pleno sentido el uso de derivadas de funciones adecuadas. En este tipo de situaciones, un modelo matem´atico es frecuentemente una ecuaci´on que contiene una o m´as derivadas de una funci´on desconocida. Tal modelo matem´atico es llamado una ecuaci´on diferencial [1]. Ejemplo 1.1.1. Son ejemplos de ecuaciones diferenciales las siguientes expresiones matem´aticas: 1.

d dx

 dy (1 − x2 ) dx + n (n + 1) y = 0 (Ecuaci´on de Legendre) 2

d y dy 2 2 2. x2 dx on de Bessel) 2 + x dx + (x − α ) y = 0 (Ecuaci´ 3

2

dy d y 2d y 3. x3 dx on de Euler) 3 + 4x dx2 + x dx + 5y = 0 (Ecuaci´

4. y 00 + ky = A sin(ωo x),

A, ωo ∈ R (Problemas de resortes)

5. y 00 − xy = 0 (Ecuaci´on de Airy)

Definici´ on 1.1.1. Una ecuaci´on diferencial se dice de orden n si n corresponde al mayor orden de derivada de la variable dependiente y presente en la ecuaci´on. 2

d y 3 Ejemplo 1.1.2. La ecuaci´on dx = y 6 es una ecuaci´on diferencial de segundo 2 + (5x) y  3 4 d y dy grado. La ecuaci´on dx = dx + 5y es una ecuaci´on diferencial de grado tres. 3

6

Apuntes Mat023 (versi´on preliminar actualizada 23-05-2014)

Comenzaremos el estudio de las ecuaciones diferenciales con las ecuaciones diferenciales de primer orden. Definici´ on 1.1.2. Sea f : U ⊆ R2 → R una funci´on de dos variables. Una ecuaci´ on diferencial de primer orden es una ecuaci´on de la forma: y 0 = f (x, y)

(1.1)

La variable x en este caso se conoce como variable independiente. Si la variable independiente es el tiempo t, frecuentemente una ecuaci´on diferencial se anota como: y 0 = f (t, y)

o bien y˙ = f (t, y)

Observaci´ on 1.1.1. Una ecuaci´on diferencial (ordinaria) de orden n es una ecuaci´on de la forma:  f x, y, y 0 , y 00 , . . . , y (n) = 0 para una cierta funci´on f . Ejemplo 1.1.3. Son ecuaciones diferenciales de primer orden: 1. y 0 = f (x) ,

f funci´on integrable.

2. y 0 + y = cos x 3. x−3 y 0 − y 2 = 0,

x>0

Definici´ on 1.1.3. Sea I ⊆ R un intervalo abierto del tipo ]a, b[, o posiblemente intervalos abiertos infinitos del tipo ]−∞, b[ , ]a, +∞[, o bien ]−∞, +∞[. Una funci´on ϕ : I ⊆ R → R se dice soluci´ on de la ecuaci´on diferencial (1.1) en el intervalo I si:  1. t, ϕ (t) ∈ U,

∀t ∈ I

 2. ϕ0 (t) = f t, ϕ (t) ,

∀t ∈ I

Ejemplo 1.1.4. La funci´on ϕ : ]1, +∞[ → R, t → ϕ (t) = dy = −y 2 dt 7

1 t−1

es soluci´on de la EDO

Apuntes Mat023 (versi´on preliminar actualizada 23-05-2014)

pues, para t ∈ ]1, +∞[ d dt



1 t−1



1 (t − 1)2  2 1 = − t−1

= −

Ejemplo 1.1.5. Verifique que la funci´on ϕ (x) = sin x definida en R es soluci´on de la ecuaci´on diferencial de segundo orden: y 00 + y = 0 Soluci´ on. Derivando la funci´on ϕ obtenemos:

dϕ dx

= cos x y

d2 ϕ dx2

= − sin x se sigue

d2 ϕ (x) + ϕ (x) = 0 dx2 para todo x ∈ R. Observaci´ on 1.1.2. Note que al escribir: ϕC (x) = sin x + C con C una constante cualquiera, ϕC tambi´en es soluci´on de la ecuaci´on diferencial. Ejemplo 1.1.6. Consideremos la ecuaci´on: xy 0 − x2 − y = 0,

x>0

Se sabe que toda funci´on de la forma: y = x2 + Cx,

C∈R

(1.2)

es soluci´on de la ecuaci´on diferencial. En efecto, derivando la ecuaci´on (1.2), tenemos: y 0 = 2x + C Reemplazando en la ecuaci´on diferencial, se tiene que:  x (2x + C) − x2 − x2 + Cx = 0 para todo x > 0. 8

Apuntes Mat023 (versi´on preliminar actualizada 23-05-2014)

Ejemplo 1.1.7. Hallar una ecuaci´on diferencial para la familia de par´abolas: y = C1 (x − C2 )2 donde C1 y C2 constantes arbitrarias. Soluci´ on. Despejamos C1 y = C1 (x − C2 )2 luego d dx esto es



y (x − C2 )2

 =0

y 0 (x − C2 )2 − y2 (x − C2 ) =0 (x − C2 )4

se sigue y 0 (x − C2 )2 − y2 (x − C2 ) = 0 luego y 0 (x − C2 ) = 2y de donde obtenemos x − C2 = derivando  1=

2y y0

2y y0 0

se sigue 1 (y 0 )2 − yy 00 = 2 (y 0 )2 luego (y 0 )2 = yy 00 2

9

Apuntes Mat023 (versi´on preliminar actualizada 23-05-2014)

Ejercicios de la secci´ on 1. Establezca el orden de la ecuaci´on diferencial dada: 00

0

(b)

+ ex y = 0

(d)

(a)

(1 − x) y − 4xy + 3y = tan x

(c)

d3 y x 3− dx



dy dx

4



dy y − 2xy y = dx s  2 2 dy dy 1 + = dx2 dx (6) 00

(4)

7

2. Comprobar que las siguientes funciones satisfacen las ecuaciones dadas y dar un intervalo en el cual esto se cumpla: (a) y = A sin (x + B) ; y 00 + y = 0

(b) y = ex − e−x ; y 00 − y = 0

y = tan (x) ; y 0 = 1 + y 2

(d) y 0 = 25 + y 2 ; y = 5 tan 5x

(c)

3. Demostrar que la ecuaci´on dada define impl´ıcitamente una soluci´on de la ecuaci´on dada y3 =1 3 (b) x3 + 3xy 2 = 1

(a)

x2 y −

dy dx

=

−2xy x2 −y 2

para x 6= y

2xyy 0 + x2 + y 2 = 0 para x ∈ ]0, 1[

4. Muestre que la familia de funciones y=e

−x2

Z

x

2

et dt + Ce−x

2

0

son soluciones de la ecuaci´on dy + 2xy = 1 dx 5. Muestre que la funci´on definida por tramos   −x2 si x < 0 ϕ (x) =  x2 si x ≥ 0 es una soluci´on de la ecuaci´on diferencial xy 0 − 2y = 0 en R. 6. Determine α ∈ R para que la funci´on y = xα sea soluci´on de 2x2 y 00 − y = 0. Si encuentra m´as de un valor, muestre que cualquier combinaci´on lineal de esas dos funciones resulta ser una soluci´on del problema. 7. Encontrar valores de m para los cuales la funci´on es soluci´on de la ecuaci´on dada: a) y (x) = emx donde y 000 − 3y 00 − 4y 0 + 12y = 0 10

Apuntes Mat023 (versi´on preliminar actualizada 23-05-2014)

b) y (x) = xm donde x2 y 00 + 2xy 0 − 6y = 0 8. Encontrar una E.D.O. de la forma y 00 + A (x) y 0 + B (x) y = 0 que tenga entre sus soluciones las funciones y1 (x) = ex y y2 (x) = xex . 9. Juan, Leo y Roberto est´an tomando caf´e y un joven del paralelo 19 les pregunta por la soluci´on de la ecuaci´on diferencial dy y+1 = dt t+1 despu´es de un rato, Juan dice y (t) = t, Leo y (t) = 2t + 1 y Roberto y (t) = t2 − 2 ¿Qui´en esta en lo correcto? 10. Construir una ecuaci´on diferencial de la forma dy = 2y − t + g (y) dt que tenga la funci´on y (t) = e2t como soluci´on. dy = f (t, y) que tenga por soluci´on dt 2 y (t) = et donde f (t, y) dependa expl´ıcitamente de t y y.

11. Construir una ecuaci´on diferencial de la forma

12. Hallar una ecuaci´on diferencial para la familia de curvas: y = C1 + (x − C2 )2 donde C1 y C2 constantes arbitrarias. 13. Construya una ecuaci´on diferencial que no tenga ninguna soluci´on real.

Modelos simples Estudiaremos algunos ejemplos elementales de modelamiento matem´atico: Problema 1.2.1 ([2]). Desde una cierta altura se ha arrojado un cuerpo de masa m. Determinar la ley seg´ un la cual var´ıa la velocidad de ca´ıda v, si sobre el cuerpo, adem´as

11

Apuntes Mat023 (versi´on preliminar actualizada 23-05-2014)

de la fuerza de gravedad, act´ ua la fuerza de resistencia del aire que es proporcional a la velocidad v.

Soluci´ on. Sea m la masa del cuerpo en ca´ıda libre. En virtud de la Segunda Ley de Newton: X

Fi = ma

i

donde

P

i

Fi representa la suma (vectorial) de fuerzas aplicadas al cuerpo y a representa la

aceleraci´on del cuerpo, se obtiene: m

dv = mg − kv dt

(1.3)

donde g es la aceleraci´on de gravedad y k es la constante (positiva) de proporcionalidad. Resolver esta ecuaci´on diferencial significa encontrar una funci´on v = f (t) que satisfaga id´enticamente la ecuaci´on diferencial dada. Existen una infinidad de funciones de este tipo (esto ser´a probado m´as adelante). Es f´acil comprobar que toda funci´on del tipo: k

v (t) = Ce− m t +

mg k

(1.4)

satisface la ecuaci´on (1.3), cualquiera que sea la constante C. Pero, ¿cu´al de estas funciones dar´a la dependencia buscada entre v y t? Para encontrar dicha relaci´on, se debe utilizar un condici´on adicional. Esta condici´on adicional se llama condici´on inicial. Supongamos que 12

Apuntes Mat023 (versi´on preliminar actualizada 23-05-2014)

en el momento inicial del experimento arrojamos el cuerpo con una velocidad inicial v0 conocida. As´ı, la funci´on v = f (t) que deseamos encontrar debe satisfacer la condici´on: f (0) = v0 Es decir, v = v0 en t = 0. As´ı, reemplazando en (1.4), se obtiene: C = v0 −

mg k

Por lo tanto, la ley seg´ un la cual var´ıa la velocidad de ca´ıda est´a dada por la ecuaci´on:  mg  − k t mg e m + v (t) = v0 − k k Observaci´ on 1.2.1. Una cuesti´on de inter´es es el comportamiento asint´otico de la soluci´on. Es decir, el comportamiento de la soluci´on para t suficientemente grande. M´as precisamente, nos interesa: l´ım v (t)

t→+∞

En particular, para el ejemplo: l´ım v (t) = l´ım

t→+∞

t→+∞

=

n

v0 −

mg  − k t mg o e m + k k

mg k

La interpretaci´on del resultado anterior, es que cuando el tiempo t es suficientemente grande, la velocidad final del cuerpo no depende de la velocidad inicial. Observaci´ on 1.2.2. Observar que desde la ecuaci´on (1.3), si suponemos k = 0, es decir, suponemos que la resistencia del aire es tan peque˜ na que puede ser despreciada, se obtiene: v (t) = v0 + gt Definici´ on 1.2.1. Se llamar´a soluci´ on general de la ecuaci´on diferencial de primer orden: y 0 = f (x, y) a la funci´on: y = ϕ (x, C) que depende de una constante arbitraria C y satisface las condiciones siguientes: 13

Apuntes Mat023 (versi´on preliminar actualizada 23-05-2014)

1. satisface la ecuaci´on diferencial para cualquier valor de la constante C. 2. cualquiera que sea la condici´on inicial y = y0 para x = x0 , es decir, y (x0 ) = y0 se puede encontrar un valor C = C0 tal que la funci´on y = ϕ (x, C0 ) satisfaga la condici´on inicial dada. Problema 1.2.2 (Espejo parab´olico). Hallar la forma que debe tener un espejo convexo, sim´etrico respecto de un eje, llamado eje focal, de tal modo que si un haz de luz es apuntado hacia el espejo, paralelo al eje focal, se refleje directamente en un punto F fijo (llamado foco) del eje focal.

Soluci´ on. Obtendremos tal espejo mediante la rotaci´on de una curva en el plano. Sea C : y = f (x) tal curva. Consideremos un sistema de coordenadas de tal modo que el origen del sistema coincida con el foco F de la curva. En particular, el eje focal de la curva coincide con el eje de las abscisas de tal sistema de referencia. Considere un punto P (x, y) en la curva C y sea T la recta tangente a C en el punto P (x, y). Denotemos por Q el punto de intersecci´on de T con el eje focal (o de las abscisas del sistema de referencia). Si L es una recta que representa el haz de luz paralelo al eje focal y que incide en P , por la ley de Snell, el ´angulo de incidencia ]T P L y ´angulo de reflexi´on ]QP F coinciden. Es decir, se tiene que: ]T P L = ]QP F = α Entonces, del tri´angulo 4QP F se obtiene: tan 2α = 14

y x

(1.5)

Apuntes Mat023 (versi´on preliminar actualizada 23-05-2014)

por ser F el origen del sistema de coordenadas. Como T es tangente a la curva C, se tiene que ]P QF es tambi´en α. Luego: tan α = y 0

(1.6)

Reemplazando las ecuaciones (1.5) y (1.6), en la identidad trigonom´etrica siguiente: tan 2α =

2 tan α 1 − tan2 α

se obtiene la ecuaci´on diferencial de primer orden: y 2y 0 = x 1 − (y 0 )2

(1.7)

Problema 1.2.3 (Braquist´ocrona). [3] Hallar la forma que debe tener un alambre de modo que una argolla que se desliza por ´el, sin roce, bajo la acci´on de la gravedad de un punto A a un punto B de menor altura en el mismo plano y no exactamente bajo el punto A, lo haga en el menor tiempo posible.

Soluci´ on. Este problema fue planteado en 1696 por Jean Bernoulli a la comunidad cient´ıfica de su ´epoca. La soluci´on que ´el mismo encontr´o (independientemente tambi´en lo hicieron Leibnitz, L’Hˆopital, Newton) usa una versi´on generalizada de la Ley de Snell de la ´optica. Comenzaremos modelando primeramente esta situaci´on: la versi´on generalizada de la Ley de Snell. A modo de ejercicio lo haremos utilizando las herramientas del c´alculo diferencial, en particular minimizaci´on. Considere, entonces, el siguiente problema:

15

Apuntes Mat023 (versi´on preliminar actualizada 23-05-2014)

Supongamos que un atleta, situado en la orilla oriental de un r´ıo de ancho a debe atravesarlo nadando a velocidad constante v1 hasta un cierto punto C en la orilla occidental. Luego de esto, debe correr a velocidad constante v2 por sobre la arena de la ribera de ancho b del r´ıo hasta la meta en el punto B. Se supone que v1 < v2 . Se desea encontrar el punto C en la orilla occidental de tal modo que el tiempo empleado por el atleta desde el punto A hasta la meta en B sea el menor posible.

Consideremos las rectas paralelas (y verticales) l1 , l2 y l3 . Supongamos que la recta l1 representa la orilla oriental del r´ıo, l2 representa la ribera del r´ıo y l3 la l´ınea de meta. Por la condiciones del problema tenemos: 1. A ∈ l1 2. C ∈ l2 3. B ∈ l3 4. a es la distancia entre l1 y l2 5. b es la distancia entre l2 y l3 Supongamos, adem´as, que c y x son las distancias verticales de A a B, y de A a C, respectivamente. Entonces, el tiempo total de la carrera est´a dado por: q √ (c − x)2 + b2 x 2 + a2 T (x) = + v1 v2 Derivando e igualando a 0, se obtiene la coordenada x0 de tiempo m´ınimo. Es decir, x0 debe cumplir con: x0 v1

p

x20

+

a2

=

c − x0 q v2 (c − x0 )2 + b2

Si introducimos los a´ngulos de incidencia α y β respecto de la recta normal a l2 para este valor de x0 se obtiene: sin α sin β = v1 v2 que es la conocida Ley de Snell. 16

Apuntes Mat023 (versi´on preliminar actualizada 23-05-2014)

Ahora bien, si consideramos otro segmento (es decir, otra recta vertical l4 ) por el cual el atleta se deba desplazar a velocidad constante v3 , se obtendr´a: sin α1 sin α2 sin α3 = = = constante v1 v2 v3 De manera an´aloga, la argolla de masa m que cae bajo la acci´on de la gravedad g tiene una velocidad v que va en aumento de acuerdo a la distancia vertical recorrida y. As´ı, igualando energ´ıas potencial y cin´eticas se obtiene la ecuaci´on: 1 2 mv = mgy 2 de donde: v=

p 2gy

(1.8)

Por esta raz´on, Bernoulli conjetur´o la Ley de Snell generalizada siguiente: sin α = λ, v

λ constante

(1.9)

donde α es el a´ngulo que instant´aneamente forma la tangente a la curva en la posici´on de la argolla y la vertical. Finalmente, notando que: sin α = cos

π 2

−α

1 = q 1 + tan2



π 2

−α



1 = q 1 + (y 0 )2 Utilizando, entonces, las f´ormulas (1.8) y (1.9) se obtiene la ecuaci´on: p 1 2gy = q λ 1 + (y 0 )2 O bien:   2 y 1 + (y 0 ) = k 2 donde k =

1 . 2gλ2

17

Apuntes Mat023 (versi´on preliminar actualizada 23-05-2014)

Ejercicios de la secci´ on 1. Determine la ecuaci´on que debe cumplir la familia de curvas que forman un ´angulo de 45 grados en la intersecci´on con la familia de curvas y (x + c) = 1. 2. Determine la ecuaci´on de la familia de curvas ortogonales a la familia y 2 = cx3 . 3. En este problema se analiza la ca´ıda de una gota de agua. Supongamos que al caer esta se evapora y mantiene su forma esf´erica, la rapidez con que se evapora es proporcional al a´rea con una constante de proporcionalidad ζ < 0 y no se considera la resistencia del aire. Designemos por ρ la densidad del agua, r0 el radio de la gota cuando t = 0 y la direcci´on positiva es hacia abajo. a) Muestre que el radio de la gota r (t) disminuye de acuerdo a la ley   ζ r (t) = t + r0 ρ b) Obtener la ecuaci´on diferencial que satisface la velocidad v (t) de la gota en su ca´ıda libre. Determine la velocidad si la gota cae del reposo. 4. En la figura suponga que el eje y y la recta x = 1 corresponden a las orillas oeste y este de un r´ıo de 1 [km] de ancho. El r´ıo fluye hacia el norte con una velocidad vr donde kvr k = vr [km/h]. Un hombre entra al r´ıo en el punto (1, 0) en la costa este y nada en direcci´on a la ribera contraria a una velocidad constante de kvs k = vs [km/h]. El hombre quiere llegar al punto (0, 0) de modo que nada de forma tal que su vector velocidad vs siempre apunta a (0, 0). Muestre que la trayectoria que sigue el nadador satisface la ecuaci´on p vs y − vr x2 + y 2 dy = dx vs x

18

Apuntes Mat023 (versi´on preliminar actualizada 23-05-2014)

M´ etodos Elementales de Resoluci´ on Recordaremos primeramente un teorema esencial para resolver ecuaciones diferenciales elementales: el Teorema Fundamental del C´alculo. Teorema 1.3.1. (Teorema Fundamental del C´alculo) Sea f : [a, b] → R funci´on integrable en [a, b]. Dado x0 ∈ [a, b] e y0 ∈ R se tiene que la funci´on F : [a, b] → R dada por Zx F (x) =

f (t)dt + y0

con x ∈ [a, b],

x0

es continua en [a, b] y F (x0 ) = y0 . Adem´as, si f es continua en [a, b], entonces la funci´ on F es derivable en [a, b] y satisface la ecuaci´on: F 0 (x) = f (x) Observaci´ on 1.3.1. Recordemos que en este contexto la funci´on F es conocida como una primitiva de f .

Integraci´ on directa Definici´ on 1.3.1. Una ecuaci´on diferencial ordinaria de primer orden, digamos y 0 = f (x, y), se dice que es resoluble por integraci´ on directa si existe una funci´on g integrable sobre un intervalo abierto I ⊆ R tal que f (x, y) = g(x)

19

Apuntes Mat023 (versi´on preliminar actualizada 23-05-2014)

Observaci´ on 1.3.2. En particular, una ecuaci´on diferencial de integraci´on directa es de la forma: y 0 = g(x)

(1.10)

para todo x ∈ I. As´ı, si g : D ⊆ R → R es una funci´on integrable, entonces integrando en ambos lados de la igualdad 1.3.2 y usando el Teorema Fundamental del C´alculo, obtenemos que la soluci´on general de la ecuaci´on diferencial es de la forma: φ(x) = F (x) + C, donde F es una primitiva de g y C ∈ R es una constante. Ejemplo 1.3.1. Una part´ıcula se mueve a lo largo de una l´ınea recta de manera que su velocidad en el instante t es 2 sin t. Si f (t) indica su posici´on en el tiempo t medio a partir del punto de partida, se tiene que f 0 (t) = 2 sin t. Por la observaci´on anterior, se concluye que: f (t) = −2 cos t + C Note que para fijar la funci´on posici´on se necesita alg´ un otro dato. En particular, si se conoce el valor de f en alg´ un instante en particular, entonces se puede determinar C. Por ejemplo, si f (0) = 0, entonces C = 2 y la funci´on posici´on es f (t) = −2 cos t + 2. Ejemplo 1.3.2. Determine la soluci´on del problema dy 2 = e−x dx y (0) = 5 R 2 Soluci´ on. Se puede demostrar que e−x dx no es una funci´on elemental, sin embargo, podemos expresar la soluci´on por x

Z

2

e−t dt + 5

y (x) = 0

Ecuaciones de variable separable Definici´ on 1.3.2. Una ecuaci´on diferencial de primer orden y 0 = f (x, y) se dice una ecuaci´ on de variable separable si: f (x, y) = Q (x) R (y) 20

(1.11)

Apuntes Mat023 (versi´on preliminar actualizada 23-05-2014)

para Q y R funciones continuas sobre alg´ un intervalo abierto I ⊆ R. Observaci´ on 1.3.3. [5] Si R (y) 6= 0 se puede dividir por R (y) y escribir la ecuaci´on (1.11) en la forma: A (y) y 0 = Q (x)

(1.12)

donde A (y) = 1/R (y). Recordemos, adem´as, que y representa una cierta funci´on desconocida y = Y (x), luego la ecuaci´on (1.12), se expresa como: A (Y (x)) Y 0 (x) = Q (x) Integrando la ecuaci´on anterior, se sigue que: Z Z 0 A (Y (x)) Y (x) dx = Q (x) dx + C Haciendo la sustituci´on y = Y (x) en la integral de la izquierda, se tiene que dy = Y 0 (x) dx. Por consiguiente, se obtiene: Z

Z A (y) dy =

Q (x) dx + C

Ahora bien, si G es una primitiva de A y H es una primitiva de Q, la ecuaci´on anterior se puede escribir como: G (y) = H (x) + C

(1.13)

Rec´ıprocamente, si y es una funci´on que satisface la ecuaci´on (1.13), entonces: G0 (y) y 0 = H 0 (x) es decir, tenemos A (y) y 0 = Q (x). Por lo tanto, la ecuaci´on (1.13) da una representaci´on impl´ıcita de una familia de soluciones a un par´ametro. Observaci´ on 1.3.4. En particular, el formalismo anterior se reduce considerablemente si se considera la notaci´on de Leibnitz para las derivadas. Es decir, escribimos y 0 = dy/dx y separamos las variables, con lo cual la ecuaci´on (1.12) se escribe directamente como: A (y) dy = Q (x) dx y la ecuaci´on anterior tiene sentido al ser dy la diferencial de la funci´on desconocida y. Formalmente, podemos integrar la ecuaci´on anterior, obteniendo: Z Z A (y) dy = Q (x) dx + C pero esto s´olo es formal, pues las variables de integraci´on son distintas. 21

Apuntes Mat023 (versi´on preliminar actualizada 23-05-2014)

Ejemplo 1.3.3. Resolver la ecuaci´on dy = ey−x dx Soluci´ on. Se trata de una E.D.O. de variables separadas pues dy = ey−x = e−x ey dx se sigue e−y(x) y 0 (x) = e−x integrando respecto a x Z

Z

−y(x) 0

e

e−x dx

y (x) dx =

se sigue −e−y(x) = −e−x + C despejando e−y(x) = e−x + K luego y (x) = − ln e−x + K



es una familia de soluciones. Ejemplo 1.3.4. Resolver la ecuaci´on dy = x2 y + x2 dx Soluci´ on. Note que x2 y+x2 = x2 (y + 1) luego se trata de una EDO de variables separables dy = x2 (y + 1) dx se sigue Z

dy = y+1

Z

x2 dx

integrando ln |y + 1| =

x3 +C 3

⇒ x3

y + 1 = Ke 3 22

Apuntes Mat023 (versi´on preliminar actualizada 23-05-2014)

as´ı x3

y (x) = Ke 3 − 1 on diferencial lineal Ejemplo 1.3.5. Diremos que una ecuaci´on diferencial es una ecuaci´ de primer orden homog´ enea si es de la forma: y 0 + P (x) y = 0

(1.14)

si P es una funci´on continua sobre su dominio abierto. Soluci´ on. Separando las variables e integrando: Z Z dy = − P (x) dx + K y con K una constante de integraci´on. Luego, la ecuaci´on anterior implica que: Z ln |y| = − P (x) dx + K As´ı: y = ±eK e−

R

P (x)dx

Por consiguiente: y = C e−

R

P (x)dx

Observaci´ on 1.3.5. Note que en el procedimiento anterior, se ha dividido por y, por tanto, debe explicarse que toda soluci´on de (1.14) se puede expresar mediante la f´ormula y = C e−

R

P (x)dx

. Sea y una soluci´on de (1.14) y considere la funci´on g definida por: R

g (x) = y e

P (x)dx

Luego: g 0 (x) = y 0 e R

= e

R

P (x)dx

P (x)dx

R

+ P (x) ye

P (x)dx

(y 0 + P (x) y)

= 0 como x pertenece a un intervalo abierto, se obtiene que g (x) = C, por el Teorema del Valor Medio. Por tanto: y = C e− 23

R

P (x)dx

Apuntes Mat023 (versi´on preliminar actualizada 23-05-2014)

Observaci´ on 1.3.6. El razonamiento anterior da origen a un m´etodo de resoluci´on de ecuaciones diferenciales del tipo: y 0 + P (x) y = Q (x) donde P y Q son funciones continuas en un intervalo abierto I ⊆ R. La ecuaci´on diferencial anterior se llama ecuaci´on diferencial lineal de primer orden.

Ecuaci´ on lineal de primer orden Definici´ on 1.3.3. Una ecuaci´on diferencial de primer orden se dice lineal si es de la forma: y 0 + P (x) y = Q (x)

(1.15)

para dos funciones P y Q continuas en un intervalo abierto I ⊆ R. Definici´ on 1.3.4. Un factor integrante o un factor de integraci´ on para la ecuaci´on (1.15) es una expresi´on de la forma: µ (x) = e

R

P (x)dx

(1.16)

Observaci´ on 1.3.7. Multipliquemos la ecuaci´on diferencial (1.15) por el factor integrante en (1.16), luego: R

e

P (x)dx

R

{y 0 + P (x) y} = e

P (x)dx

Q (x)

Note que la ecuaci´on anterior, puede escribirse como: R d n R P (x)dx o ye = e P (x)dx Q (x) dx

Integrando respecto de x, obtenemos: R

ye

P (x)dx

Z =

R

P (x)dx

e

R

e

Q (x) dx + C

Finalmente: y=e



R

P (x)dx

Z

P (x)dx

24

 Q (x) dx + C

Apuntes Mat023 (versi´on preliminar actualizada 23-05-2014)

Teorema 1.3.2 (F´ormula de Leibnitz). Sean P y Q dos funciones continuas sobre un intervalo abierto I ⊆ R. La soluci´on general de la ecuaci´on diferencial lineal: y 0 + P (x) y = Q (x) est´a dada por la f´ormula: −

y (x) = e

R

P (x)dx

R

Observaci´ on 1.3.8. Si µ (x) = e y (x) =

Z

R

e

P (x)dx

 Q (x) dx + C

P (x)dx

1 µ (x)

es el factor integrante, la f´ormula anterior queda: Z  µ (x) Q (x) dx + C

Ejemplo 1.3.6. Resolver la ecuaci´on lineal de primer orden: xy 0 + (1 − x) y = e2x Soluci´ on. Supongamos que x 6= 0. La ecuaci´on anterior queda como:   e2x 1 0 −1 y = y + x x Utilizando las notaciones del teorema anterior, tenemos que P (x) = 1/x−1 y Q (x) = e2x /x. Calculamos, primeramente, el factor integrante: 1 µ (x) = e ( x −1)dx

R

= |x| e−x Luego: Z  1 y (x) = µ (x) Q (x) dx + C µ (x) Z  2x ex −x e = |x| e dx + C |x| x Si x > 0, se tiene que |x| = x y la soluci´on es: Z  ex x e dx + C y = x e2x ex = +C x x 25

Apuntes Mat023 (versi´on preliminar actualizada 23-05-2014)

Por otro lado, si x < 0, se tiene que |x| = −x y la soluci´on es:  Z  ex x − e dx + C y = −x ex e2x −C = x x Ahora bien, como C es una constante arbitraria, si x 6= 0 se puede escribir: y=

e2x ex +C x x

Ejemplo 1.3.7. Deternine una funci´on f : D ⊆ R → R tal que Z t t f (t) = t + e e−u f (u) du + tf (t) 0

Soluci´ on. Notemos que f (0) = 0 y −t

(f (t) − t − tf (t)) e

Z =

t

e−u f (u) du

0

usando el teorema fundamental del c´alculo (f 0 (t) − 1 − f (t) − tf 0 (t)) e−t − (f (t) − t − tf (t)) e−t = e−t f (t) se sigue f 0 (t) − 1 − f (t) − tf 0 (t) − f (t) + t + tf (t) = f (t) as´ı (1 − t) f 0 (t) − (3 − t) f (t) = 1 − t es una EDO de primer orden lineal 3−t df − f =1 dt 1−t el factor de integraci´on es R

µ (t) = e

− 3−t dt 1−t

= e2 ln(t−1)−t

= (t − 1)2 e−t se sigue  d (t − 1)2 e−t f = (t − 1)2 e−t dt 26

Apuntes Mat023 (versi´on preliminar actualizada 23-05-2014)

integrando Z

2 −t

(t − 1) e f =

(t − 1)2 e−t dt

es decir  (t − 1)2 e−t f = −e−t t2 + 1 + C de donde f (t) =

−e−t (t2 + 1) + C (t − 1)2 e−t

como f (0) = 0 se sigue 0 = −1 + C ⇒ C = 1 finalmente f (t) =

−e−t (t2 + 1) + 1 (t − 1)2 e−t

Ejemplo 1.3.8. Resolver la ecuaci´on dy − 2xy = x dx R

Soluci´ on. Es una ecuaci´on lineal, el factor integrante es µ (x) = e cando la ecuaci´on por µ (x) se tiene e−x

2

dy 2 2 − 2xe−x y = xe−x dx

luego d  −x2  2 e y = xe−x dx integrando −x2

e

Z y =

2

xe−x dx

1 2 = − e−x + C 2 as´ı 1 2 y (x) = − + Cex 2

27

−2xdx

2

= e−x , multipli-

Apuntes Mat023 (versi´on preliminar actualizada 23-05-2014)

Ecuaci´ on de Bernoulli Observaci´ on 1.3.9. Numerosas aplicaciones pueden ser modeladas con ecuaciones diferenciales ordinarias que no son lineales, sin embargo, mediante cambios de variables adecuados y algo de manipulaci´on algebraica, estas ecuaciones pueden ser transformadas en ecuaciones diferenciales lineales. Un caso importante es la llamada ecuaci´on de Bernoulli. Definici´ on 1.3.5. Sean P (x) y Q (x) funciones continuas sobre un intervalo abierto I ⊆ R. Una ecuaci´on diferencial de la forma: y 0 + P (x) y = Q (x) y α ,

α 6= 1

(1.17)

se llama ecuaci´ on de Bernoulli. Observaci´ on 1.3.10. Multipliquemos la ecuaci´on de Bernoulli en (1.17) por y −α , de donde obtenemos: y −α y 0 + P (x) y 1−α = Q (x)

(1.18)

Sea z = y 1−α . Luego, por la regla de la cadena, obtenemos: z 0 = (1 − α) y −α y 0 Reemplazando en la ecuaci´on (1.18), se obtiene: 1 z 0 + P (x) z = Q (x) 1−α O bien: z 0 + (1 − a) P (x) z = (1 − α) Q (x)

(1.19)

que es una ecuaci´on diferencial lineal de primer orden. Es importante notar que una vez resuelta la ecuaci´on (1.19) se debe volver a la variable original y = y (x). Ejemplo 1.3.9. Hallar la soluci´on de la ecuaci´on diferencial: dy + xy = x3 y 3 dx Soluci´ on. Dividiendo todos los t´erminos por y 3 , tenemos: y −3 y 0 + xy −2 = x3 28

(1.20)

Apuntes Mat023 (versi´on preliminar actualizada 23-05-2014)

Consideremos, tal como en la observaci´on anterior, el cambio de variables z = y −2 . Luego: dy dz = −2y −3 dx dx Reemplazando la ecuaci´on anterior en (1.20), se obtiene la ecuaci´on diferencial lineal: dz − 2xz = −2x3 dx cuya soluci´on es: z = 1 + x2 + Cex

2

Por consiguiente, la soluci´on general de la ecuaci´on diferencial dada es: y=√

1 1+

x2

+ Cex2

con C una constante cualquiera. Ejemplo 1.3.10 (Ecuaci´on log´ıstica). Resuelva la siguiente ecuaci´on de Bernoulli, conocida como ecuaci´ on log´ıstica: dN = αN (β − N ) dt

(1.21)

con α, β > 0. Soluci´ on. Note que podemos escribir la ecuaci´on (1.21) como: dN = αβN − αN 2 dt Sean A = αβ y B = α, entonces: dN − AN = −BN 2 dt Dividiendo la ecuaci´on anterior por N 2 , obtenemos: N −2

dN − AN −1 = −B dt

Sea z = N −1 , luego: dN dz = −N −2 dt dt Reemplazando en la ecuaci´on (1.22), obtenemos: −

dz − Az = −B dt 29

(1.22)

Apuntes Mat023 (versi´on preliminar actualizada 23-05-2014)

Es decir: dz + Az = B dt cuya soluci´on exacta es: z = Ce−At + Pero z =

1 , N

B A

luego: N=

1 1 = −At z Ce + B/A

Finalmente, como A = αβ y B = α, obtenemos: N (t) =

β 1 + Ce−αβt

Ecuaci´ on de Ricatti Definici´ on 1.3.6. Una ecuaci´ on de Ricatti es una ecuaci´on diferencial de la forma: y 0 + P (x) y + Q (x) y 2 = R (x)

(1.23)

donde P, Q y R son funciones continuas sobre un intervalo I ⊆ R. Observaci´ on 1.3.11. Existe un m´etodo para obtener una familia de soluciones de una ecuaci´on de Ricatti si se conoce una soluci´on particular u = u (x). Supongamos esto y consideremos el cambio de variables: y =u+

1 v

(1.24)

con v = v (x) la nueva variable. Derivando la ecuaci´on anterior, obtenemos: y 0 = u0 −

1 0 v v2

(1.25)

Reemplazando las ecuaciones (1.24) y (1.25) en la ecuaci´on de Ricatti (1.23), se sigue que: 

  2   1 1 0 1 u − 2 v + P (x) u + + Q (x) u + = R (x) v v v 0

Reordenando los t´erminos de la ecuaci´on anterior, tenemos que:    0 v0 1 u 1 2 u + P (x) u + Q (x) u − 2 + P (x) + Q (x) 2 + 2 = R (x) v v v v 30

Apuntes Mat023 (versi´on preliminar actualizada 23-05-2014)

Ahora bien, como u es una soluci´on particular de la ecuaci´on (1.23), obtenemos:   u 1 v0 1 − 2 + P (x) + Q (x) 2 + 2 = 0 v v v v Amplificando la ecuaci´on anterior por −v 2 , finalmente se obtiene la ecuaci´on: v 0 − P (x) v − Q (x) u (x) v − Q (x) = 0 Es decir: v 0 − {P (x) + u (x) Q (x)} v = Q (x) que es una ecuaci´on lineal de primer orden. Ejemplo 1.3.11. Resuelva la ecuaci´on de Ricatti: y0 = y2 − 1 x

Soluci´ on. Notamos primeramente que u =

2 x2

(1.26)

es una soluci´on particular de la ecuaci´on.

En efecto: u2 −

Ahora bien, sea y =

1 x

1 2 2 = − x2 x2 x2 1 = − 2 x 0 = u

+ v1 , luego: dy 1 1 dv =− 2 − 2 dx x v dx

Reemplazando la ecuaci´on anterior y el cambio de variables y = obtenemos: 1 1 dv − 2− 2 = x v dx



1 1 + x v

2 −

Simplificando y agrupando t´erminos semejantes, se tiene: −

1 dv 2 1 = + 2 2 v dx vx v

Amplificando por −v 2 , finalmente se obtiene: 2 v 0 + v = −1 x 31

2 x2

1 x

+ v1 en la ecuaci´on (1.26),

Apuntes Mat023 (versi´on preliminar actualizada 23-05-2014)

Como se puede observar, la ecuaci´on anterior es una ecuaci´on lineal de primer orden cuya soluci´on exacta es: v= Sin embargo, recordemos que y =

1 x

C x − x2 3

+ v1 . Luego, la soluci´on obtenida es: 1 −

y=

1 + x

y=

1 3x2 + x C − x3

O bien:

C x2

x 3

Ejemplo 1.3.12. Obtenga una soluci´on del problema 1 1 dy = 2 y2 − y + 1 dx x x y (1) = 3 Indicaci´ on:Primero buscar una soluci´on dela ecuaci´on de la forma y = ax + b. Soluci´ on. Usamos la indicaci´on para buscar una soluci´on (la ecuaci´on es de Ricatti) entonces x2 a = (ax + b)2 − x (ax + b) + x2 se sigue  ax2 = a2 − a + 1 x2 + (2ab − b) x + b2 as´ı b=0 y a = a2 + 1 − a que tiene soluci´on a = 1, se sigue que una soluci´on es y=x (la cual no cumple y (1) = 3) hacemos el cambio de variables y=

1 +x u 32

Apuntes Mat023 (versi´on preliminar actualizada 23-05-2014)

entonces 

1 1 du +1= 2 − 2 u dx x

1 +x u

2

1 − x



 1 +x +1 u

as´ı −

1 1 1 du +1= + 2 2 +1 2 u dx ux u x

eliminando −

1 1 1 du = + 2 2 2 u dx ux u x

luego du u 1 =− − 2 dx x x es lineal y tiene soluci´on u= as´ı, como y =

1 u

1 C − ln x x x

+ x se sigue y=

1 C x

+x

− x1 ln x

pero y (1) = 3 as´ı 3= luego C =

1 2

1 +1 C

reemplazando 1

y=

1 2x

− x1 ln x

+x

es la soluci´on del problema. Ejemplo 1.3.13. Resolver la ecuaci´on y 0 − xy 2 + (2x − 1) y = x − 1 si se sabe que tiene una soluci´on constante. Soluci´ on. La soluci´on constante es y = 1 (verificar) entonces podemos hacer el cambio 1 +1 z dy dz = −z −2 dx dx y=

reemplazamos −z

−2 dz

dx

 −x

1 +1 z

2



 1 + (2x − 1) +1 −x+1 = 0 z dz 1 − 2 (x + z) = 0 −z −2 dx z 33

Apuntes Mat023 (versi´on preliminar actualizada 23-05-2014)

as´ı dz + (x + z) = 0 dx la cual es lineal, la soluci´on es z = Ae−x − x + 1 as´ı y=

1 +1 Ae−x − x + 1

Ecuaciones homog´ eneas Definici´ on 1.3.7. Una funci´on f : U ⊆ R2 → R se dice homog´ enea de grado n si: f (tx, ty) = tn f (x, y) para todo t ∈ R tal que (tx, ty) ∈ U . Observaci´ on 1.3.12. Si M y N son funciones homog´eneas de grado n, entonces la ecuaci´on: M (x, y) dx + N (x, y) dy = 0 se escribe como:

(1.27)

 y M 1, xy dy M (x, y)  = F =− =− dx N (x, y) x N 1, xy

donde F : R → R. Luego, la ecuaci´on (1.27) puede escribirse como: y 0 y =F x Definici´ on 1.3.8. La ecuaci´on de primer orden: y 0 = f (x, y) se llama homog´ enea, si la funci´on f (x, y) es homog´enea de grado 0. Observaci´ on 1.3.13. En vista de la observaci´on anterior, toda ecuaci´on homog´enea puede escribirse en la forma: 0

y =F

y

(1.28) x haciendo t = x1 . Para resolver esta ecuaci´on se considera el cambio de variables z = xy ; o bien, y = zx. Luego, al derivar respecto de x tenemos que: dy dz =z+x dx dx 34

Apuntes Mat023 (versi´on preliminar actualizada 23-05-2014)

Reemplazando la ecuaci´on anterior en la ecuaci´on (1.28), se obtiene: z+x

dz = F (z) dx

la cual es una ecuaci´on de variable separable. Finalmente, podemos escribir: Z Z dx dz = +C F (z) − z x con C una constante arbitraria. Ejemplo 1.3.14. Resuelva la ecuaci´on diferencial: y0 =

x+y x−y

Soluci´ on. Se trata de una ecuaci´on con funci´on homog´enea de grado cero pues y0 = ponemos u =

y x

1+ x+y = x−y 1−

y x y x

=F

y x

luego u + xu0 = y 0 reemplazando u + xu0 =

1+u 1−u

luego   1 1+u du = −u dx x 1−u   1 u2 + 1 = x 1−u esta ecuaci´on es de variables separadas, se sigue Z Z 1−u dx du = 2 1+u x luego arctan u −

 1 ln 1 + u2 = ln |x| + C 2

volvemos a la variable   y 2  1 arctan − ln 1 + = ln |x| + C x 2 x y

35

Apuntes Mat023 (versi´on preliminar actualizada 23-05-2014)

Ejemplo 1.3.15. Resuelva el P.V.I. 1/2 dy = y + x2 − y 2 dx y (1) = 0 x

Soluci´ on. Note que   y 2 1/2 dy y = + 1− dx x x es homog´enea, hacemos el cambio u=

y du dy ⇒u+x = x dx dx

luego u+x

√ du = u + 1 − u2 dx

se sigue du = dx

√ 1 − u2 x

resolvemos esta ecuaci´on de variables separables Z Z du dx √ = 2 x 1−u arcsin u = ln |x| + C as´ı u = sin (ln |x| + C) luego y = x sin (ln |x| + C) evaluando 0 = sin (ln |1| + C) = sin C se sigue C = kπ con k ∈ Z, as´ı y = x sin (ln |x| + kπ) con k ∈ Z

36

Apuntes Mat023 (versi´on preliminar actualizada 23-05-2014)

Otros cambios de variables Observaci´ on 1.3.14. La regla de la cadena nos permite cambiar variables en las ecuaciones diferenciales para llevarlas a ecuaciones diferenciales que se resuelven mediante los m´etodos elementales. Consideremos los siguientes ejemplos: Ejemplo 1.3.16. Recordemos la ecuaci´on (1.7) obtenida en el problema del espejo parab´olico. Es decir, la ecuaci´on:

y 2y 0 = x 1 − (y 0 )2 Utilizando un cambio de variables adecuado, resolveremos esta ecuaci´on diferencial. En primer lugar, despejando y 0 de la ecuaci´on, se obtiene: 2

y (y 0 ) + 2xy 0 − y = 0 Por la f´ormula de la ecuaci´on de segundo grado, encontramos que: p −x ± x2 + y 2 y0 = y p Por la simetr´ıa de la curva y = f (x) y el hecho que |x| < x2 + y 2 , se obtiene que y 0 > 0. Luego: y0 =

−x +

p

x2 + y 2

y Consideremos, ahora, el cambio de variables z = x2 + y 2 . Entonces: dy dz = 2x + 2y dx dx Reemplazando la ecuaci´on anterior en la ecuaci´on (1.29), se obtiene: ! p −x + x2 + y 2 dz = 2x + 2y dx y p = 2x − 2x + 2 x2 + y 2 Es decir, obtenemos la ecuaci´on de variable separable: √ dz =2 z dx As´ı: Z

dz √ = 2 z

Z

37

dx + C

(1.29)

Apuntes Mat023 (versi´on preliminar actualizada 23-05-2014)

implica que:

√ z =x+C

Por tanto, elevando al cuadrado y como z = x2 + y 2 , se obtiene finalmente: x2 + y 2 = x2 + 2Cx + C 2 O bien: y 2 = 2Cx + C 2 con C ∈ R. Ejemplo 1.3.17. Resuelva la ecuaci´on diferencial: dy = (x + y + 1)2 − 2 dx utilizando para ello un cambio de variables adecuado. Soluci´ on. Haciendo z = x + y + 1, tenemos z 0 = 1 + y 0 , de modo que: z0 − 1 = z2 − 2 o bien: z0 = z2 − 1 Separando variables, obtenemos: Z

dz = 2 z −1

Z dx + C

Integrando se tiene que: 1 {ln (z − 1) − ln (z + 1)} = x + C 2 o: ln

z−1 = 2x + C z+1

Luego: z−1 = e2x+C z+1 Es decir, obtenemos: z − 1 = Ce2x (z + 1) Despejando z y recordando que z = x + y + 1, se obtiene finalmente: y=

1 + Ce2x −x−1 1 − Ce2x 38

Apuntes Mat023 (versi´on preliminar actualizada 23-05-2014)

Ejemplo 1.3.18. Para un valor de n ∈ N adecuado, el cambio de variables u = xn y transforma la ecuaci´on diferencial x2

p dy + 2xy = 1 − y 2 x4 x2 dx

en una ecuaci´on de variables separables. Determine tal valor de n y resolver la ecuaci´on. Soluci´ on. Note que y = x−n u luego dy du = −nx−n−1 u + x−n dx dx reemplazando    q −n−1 −n du + 2x x−n u = 1 − (x−n u)2 x4 x2 x −nx u+x dx 2

ordenando −nx−n+1 u + x2−n

 p du + 2 x−n+1 u = 1 − (x4−2n u2 )x2 dx

se sigue  p du + (2 − n) x−n+1 u = 1 − (x4−2n u2 )x2 dx tomando n = 2 obtenemos du √ = 1 − u 2 x2 dx que es de variables separables, se sigue Z Z du √ = x2 dx 1 − u2 x2−n

as´ı

x3 +C 3

arcsin u = luego u = sin



x3 3

 + C y finalmente sin y=



x3 3

+C



x2

Ejemplo 1.3.19. Resolver la ecuaci´on yy 00 = y 0 y 2 + (y 0 ) haciendo el cambio u = y 0 . 39

2

Apuntes Mat023 (versi´on preliminar actualizada 23-05-2014)

Soluci´ on. Si u = y 0 entonces du dy dy dx du = u dy

y 00 =

reemplazando du = uy 2 + (u)2 dy du y = y2 + u dy

yu

esta ecuaci´on es lineal du 1 − u=y dy y que tiene soluci´on u = y 2 + cy luego dy = y 2 + cy dx es de variables separadas, tiene soluci´on y (x) =

c Ae−cx

−1

Obs.: Esta t´ecnica reduce el orden de la E.D.O. si es de la forma F (y, y 0 , y 00 ) es decir, no depende de la variable x. Observaci´ on 1.3.15. Como muestran los ejemplos anteriores, en general, se puede efectuar cualquier cambio de variables o sustituci´on que se desee al intentar resolver una ecuaci´on diferencial. Sin embargo, cualquier no asegura el ´exito en facilitar la resoluci´on de la ecuaci´on diferencial. Por tanto, debe buscarse el cambio de variables adecuado, [4].

Ejercicios de la secci´ on 1. Resuelva las siguientes ecuaciones mediante integraci´on directa  dy dy (a) x2 + 4 =4 (b) = x ln x dx dx 1 dy 1 (c) y0 = (d) e−x = sin x 2 arctan x (1 + x ) dx 40

Apuntes Mat023 (versi´on preliminar actualizada 23-05-2014)

2. Resuelva las siguientes ecuaciones de variables separables: (a) yy 0 − x = xy 2

(b)

y 0 = ex+y

(c)

y ln y + xy 0 = 0

(d)

(1 + ex ) y 0 = ey

(e)

y 0 = 1 + x + y + xy

(f)

y 0 = x 2 y 2 + x2

3. Verifique que las siguientes ecuaciones son homog´eneas y resu´elvalas: p dy dy (a) 3x − y + (2y − x) = 0 (b) x = y + y 2 − x2 dx dx  dy  dy 2 2 2 2 (d) 3x2 − y 2 −5x + 2xy + y = 0 = 2xy (c) 4x + xy − 3y + dx dx dy x+y dy (e) = (f) x2 y = x3 + y 3 dx x−y dx 4. Resuelva las siguientes ecuaciones lineales de primer orden: (a) xy 0 + y = xex

y 0 + 2y = sin x 1 1 (e) (f) y 0 + y = 2 x x 5. Resolver las siguientes ecuaciones de Bernoulli: (c)

y 0 + 2y = x2 + 2x  1 + x2 y 0 + xy = 1

(b) y 0 + y cos (x) = sin (x) cos (x) (d)

1.- y 0 − 4y = 2ex y 1/2

3.- xy 0 − 2y = 4x3 y 1/2

2.- y 0 − y + y 2 (x2 + x + 1) = 0

4.- xy 0 + y = y 2 x2 ln x

6. Las ecuaciones en este ejercicios pueden ser transformadas en lineales mediante un cambio de variables adecuado, descubra tal cambio y resuelva la ecuaci´on: a) y 0 + x tan y = x2 sec y b) 2xyy 0 + (1 + x) y 2 = ex 7. Sean A, B, C, D, E, F constantes. Muestre que utilizando un cambio de coordenadas conveniente podemos transformar una ecuaci´on del tipo   dx Ax + Bt + C =f dt Dx + Et + F en una ecuaci´on de variables separables o en una homog´enea. Ind.: x = X+h, y = Y +k donde h, k son constantes por determinar. 8. Resolver el P.V.I. dx 2x + t − 1 = dt x + 2t + 1 x (0) = 1 41

Apuntes Mat023 (versi´on preliminar actualizada 23-05-2014)

9. Resuelva la ecuaci´on de Ricatti:  y 0 − 2x3 + 1 y = −x2 y 2 − x4 − x + 1 sabiendo que u (x) = x es una soluci´on particular de la ecuaci´on. 10. Si u (x) = x2 es una soluci´on particular de la ecuaci´on:  y 0 + 3 − 2x2 sin x y = −y 2 sin x + 2x + 3x2 − x4 sin x Hallar una familia infinita de soluciones.

Modelos simples: Segunda parte Observaci´ on 1.4.1. Ahora consideraremos algunos ejemplos m´as: el problema de mezclas, o en t´erminos m´as generales, el problema de an´alisis de compartimientos, [4], el problema de las curvas de persecuci´on y un problema geom´etrico.

Observaci´ on 1.4.2 (An´alisis de compartimientos, [4]). Un proceso f´ısico o biol´ogico complejo puede ser dividido algunas veces en varios estados distintos. El proceso total puede describirse por la interacci´on entre los estados individuales. Cada estado se llama compartimiento (lo podemos considerar como un tanque) y se supone adem´as que el contenido de cada compartimiento est´a mezclado homogeneamente. En cada compartimiento se transfiere material que es inmediatamente incorporado al siguiente en el sistema. Considere un sistema formado por un solo compartimiento, suponga que un material es introducido en tal compartimiento a una raz´on e (t), el cual se incorpora a una cantidad x (t) de material existente al interior del compartimiento, y luego se extrae material (que puede pasar a otro compartimiento) a una raz´on de s (t). Por consiguiente, la variaci´on de material al interior del compartimiento est´a dada entonces por la ecuaci´on diferencial: dx = e (t) − s (t) dt Consideremos algunos ejemplos:

42

Apuntes Mat023 (versi´on preliminar actualizada 23-05-2014)

Ejemplo 1.4.1. Considere un tanque que contiene 100 litros de agua, en el cual se han disuelto 50 kilogramos de sal. Suponga que 2 litros de salmuera cada uno con 1 kilogramo de sal disuelta, entran por minuto al tanque, y la mezcla que se mantiene homog´enea revolvi´endola a gran velocidad, sale del tanque a raz´on de 2 litros por minuto. Hallar la cantidad de sal al interior del tanque en el tiempo t.

Soluci´ on. Sea x (t) el n´ umero de kilogramos de sal disueltos en el tanque en t minutos. Notamos que las unidades ayudan bastante en la extracci´on de informaci´on. En efecto, dx/dt est´a en [kg/ m´ın] y entonces, e (t) y s (t) deben esta en las mismas unidades. As´ı:      kg lt kg e (t) = 2 =2 lt m´ın m´ın y 

     lt x (t) kg x (t) kg s (t) = 2 · = m´ın 100 lt 50 m´ın Por tanto, la ecuaci´on diferencial queda: dx x (t) =2− dt 50 la cual es una ecuaci´on lineal de primer orden. As´ı, por la f´ormula de Leibnitz, tenemos que: −t/50

x (t) = e

 Z  t/50 2 e dt + C

= 100 + Ce−t/50 43

Apuntes Mat023 (versi´on preliminar actualizada 23-05-2014)

pero sabemos que en t = 0, x (0) = 50. As´ı, 50 = 100 + C. Por tanto: x (t) = 100 − 50e−t/50 Observaci´ on 1.4.3 (Curvas de persecuci´on, [4]). Utilizando la propiedad geom´etrica de que la pendiente de la recta tangente a una curva y en un punto dado de la curva es y 0 , se pueden construir ecuaciones diferenciales que permiten estudiar la trayectoria que describe un depredador tras su presa.

Consideremos algunos ejemplos:

Ejemplo 1.4.2. Un esquiador acu´atico P localizado en el punto (a, 0), con a > 0, es halado por un bote de motor Q localizado en el origen y que viaja hacia arriba a lo largo del eje Y . Hallar la trayectoria del esquiador si ´este se dirige en todo momento hac´ıa el bote. La trayectoria se denomina tractriz. ←→ Soluci´ on. Observemos que la recta que une P y Q, digamos P Q es tangente al camino recorrido por P . Por tanto, su pendiente est´a dada por: √ dy a2 − x 2 =− dx x

(1.30)

puesto que la longitud del segmento P Q es a. La ecuaci´on diferencial (1.30) es de integraci´on directa, luego:

Z √ 2 a − x2 y=− dx + C x

As´ı1 :  y = a ln 1

Z √

a+



a2 − x 2 x

 −

a + √a2 ± x2 p a2 ± x2 dx = a2 ± x2 − a ln +C x x

44

√ a2 − x 2 + C

Apuntes Mat023 (versi´on preliminar actualizada 23-05-2014)

Como y = 0 cuando x = a, vemos que C = 0, de modo que la ecuaci´on de la trayectoria es: √   √ a + a2 − x 2 y = a ln − a2 − x 2 x Ejemplo 1.4.3. Suponga que un halc´on P situado en el punto (a, 0) descubre una paloma Q en el origen, la cual vuela a lo largo del eje Y a una velocidad v. El halc´on emprende el vuelo inmediatamente hacia la paloma a una velocidad w. ¿Cu´al ser´a el camino seguido por el halc´on en su vuelo? Soluci´ on. Sea t = 0 el instante en que el halc´on comienza a volar hacia la paloma. Despu´es de t segundos la paloma estar´a en el punto Q = (0, vt) y el halc´on en P (x, y). Como la ←→ recta T = P Q es otra vez tangente a la trayectoria, encontramos que su pendiente mT = y 0 . Luego, se obtiene la ecuaci´on diferencial: y0 =

y − vt x

(1.31)

Debemos ahora eliminar t de la ecuaci´on anterior, pues y 0 = dy/dx. Para ello debemos calcular la longitud del camino recorrido por el halc´on. Si ds representa un elemento diferencial de longitud del arco formado por la trayectoria, tenemos que: Z a wt = ds x

q pero ds = 1 + (y 0 )2 dx. As´ı, la f´ormula anterior queda como: 1 t= w

Z

a

q

1 + (y 0 )2 dx

x

Despejando t de la ecuaci´on (1.31) e igualando con la ecuaci´on anterior, obtenemos: Z q y − xy 0 1 a = 1 + (y 0 )2 dx v w x Derivando:

y 0 − (y 0 + xy 00 ) 1 = v w

q 1 + (y 0 )2

Ordenando la ecuaci´on anterior, se tiene que: q v 00 xy = 1 + (y 0 )2 w 45

(1.32)

Apuntes Mat023 (versi´on preliminar actualizada 23-05-2014)

Note que la ecuaci´on anterior es una ecuaci´on diferencial de segundo orden. Sin embargo, mediante el cambio de variables: u = y0 la ecuaci´on (1.32) queda como: xu0 =

v√ 1 + u2 w

que es una ecuaci´on de primer orden de variable separable. Entonces, separando variables e integrando, obtenemos: Z Luego:

du v √ = 2 w 1+u

Z

dx +C x

  √ v 2 ln u + 1 + u = ln x + K w

pero u = y 0 = 0 cuando x = a, se sigue que K = (v/w) ln a. Tomando exponenciales a ambos lados de la ecuaci´on anterior se tiene: u+



1 + u2 =

 x v/w a

que, despu´es de algunas operaciones algebraicas, nos da:   1  x v/w  x −v/w dy = − dx 2 a a Suponiendo que w > v, se obtiene finalmente que: ( ) a (x/a)1+v/w (x/a)1−v/w − +C y= 2 1 + v/w 1 − v/w

Ejemplo 1.4.4. Determine la curva que pasa por

1 3 , 2 2



y corta a cada miembro de la

familia x2 + y 2 = c2 con c ∈ R+ formando un ´angulo de 45o . Soluci´ on. El ´angulo entre dos curvas es dado por el ´angulo entre sus rectas tangentes luego tan θ =

m1 − m2 1 + m1 m2

note que la pendiente de las rectas tangentes a la familia de curvas es 2x + 2yy 0 = 0 ⇒ m1 = 46

−x y

Apuntes Mat023 (versi´on preliminar actualizada 23-05-2014)

as´ı tan

π  4

esto es 1=

=

−x y

−x y

− y0

1 − xy y 0 − y0

1 − xy y 0

se sigue −x x − y0 1 − y0 = y y ⇔ x x 0 1+ = y − y0 y y luego

y 1 + xy +1 dy = x = x y dx −1 1− x y

que es una ecuaci´on homogenea , hacemos el cambio u = u+x

y x

de donde

u+1 du = dx 1−u

luego x

du dx

=

u+1 −u 1−u

⇔ 

du dx

=

u2 + 1 1−u



1 x

resolvemos Z

1−u du = 1 + u2

Z

1 dx x

se sigue arctan u −

 1 ln 1 + u2 = ln |x| + C 2

volvemos a la variable y,  1 ln x2 + y 2 = C x 2  y determinamos la constante con el punto 21 , 23   1 5 arctan (3) − ln =C 2 2 as´ı la curva es   y 1  1 5 2 2 arctan − ln x + y = arctan 3 − ln x 2 2 2 . arctan

y



47

Apuntes Mat023 (versi´on preliminar actualizada 23-05-2014)

Ejercicios de la secci´ on 1. Determinar la curva que pasa por

1 3 , 2 2



y corta a cada miembro de la familia de

curvas x2 + y 2 = c2 formando un a´ngulo de 30o . 2. Encuentre la curva que pertenece a la familia de trayectorias ortogonales de la familia de curvas x + y = cey que pasa por (0, 5). 3. Suponga que un halc´on situado en (a, 0) descubre una paloma en el origen, la cual vuela a lo largo del eje Y a una velocidad v; El halc´on emprende el vuelo inmediatamente hacia la paloma con una velocidad de w. ¿Cual es el camino seguido por el halc´on en su vuelo persecutorio?. 4. Un destructor est´a en medio de una niebla muy densa que se levanta por un momento y deja ver un submarino enemigo en la superficie a cuatro kil´ometros de distancia. Suponga: a) que el submarino se sumerge inmediatamente y avanza a toda m´aquina en una direcci´on desconocida. b) que el destructor viaja tres kil´ometros en l´ınea recta hacia el submarino. ¿Qu´e trayectoria deber´ıa seguir el destructor para estar seguro que pasar´a directamente sobre el submarino, si su velocidad v es tres veces la del submarino? 5. Suponga que el eje Y y la recta x = b forman las orillas de un r´ıo cuya corriente tiene una velocidad v en la direcci´on negativa del eje Y . Un hombre esta en el origen y su perro esta en el punto (b, 0). Cuando el hombre llama al perro, este se lanza al r´ıo y nada hacia el hombre a una velocidad constante w (con w > v). ¿Cu´al es la trayectoria seguida por el perro?. 6. Cuatro caracoles situados en las esquinas de un cuadrado [0, a] × [0, a] comienzan a moverse con la misma velocidad, dirigi´endose cada uno hacia el caracol situado a su derecha. ¿Qu´e distancia recorrer´an los caracoles al encontrarse?

48

Apuntes Mat023 (versi´on preliminar actualizada 23-05-2014)

7. Hallar la ecuaci´on de todas las curvas que tienen la propiedad de que el punto de tangencia es punto medio del segmento tangente entre los ejes coordenados.

8. Un cuerpo se calienta a 110o C y se expone al aire libre a una temperatura de 100 C. Si al cabo de una hora su temperatura es de 60o C. ¿Cu´anto tiempo adicional debe transcurrir para que se enfr´ıe a 30o C? 9. Una persona de un pueblo de 1000 habitantes regres´o con gripe. Si se supone que la gripe se propaga con una rapidez directamente proporcional al n´ umero de agripados como tambi´en al n´ umero de no agripados. Determinar el n´ umero de agripados cinco d´ıas despu´es, si se observa que el n´ umero de agripados el primer d´ıa es 100. 10. Un colorante s´olido disuelto en un l´ıquido no vol´atil, entra a un tanque a una velocidad v1 galones de soluci´on/minuto y con una concentraci´on de c1 libras de colorante/gal´on de soluci´on. La soluci´on bien homogeneizada sale del tanque a una velocidad de v2 galones de soluci´on/min. y entra a un segundo tanque del cual sale posteriormente a una velocidad de v3 galones de soluci´on/min. Inicialmente el primer tanque ten´ıa P1 libras de colorante disueltas en Q1 galones de soluci´on y el segundo tanque P2 libras de colorante disueltas en Q2 galones de soluci´on. Encontrar dos ecuaciones que determinen las libras de colorante presentes en cada tanque en cualquier tiempo t. 11. Un teatro de dimensiones 10 × 30 × 50m3 , contiene al salir el p´ ublico 0,1 % por volumen de CO2 . Se sopla aire fresco a raz´on de 500 m3 por minuto y el sistema de aire acondicionado lo extrae a la misma velocidad. Si el aire atmosf´erico tiene 49

Apuntes Mat023 (versi´on preliminar actualizada 23-05-2014)

un contenido de CO2 del 0,04 % por volumen y el l´ımite saludable es de 0,05 % por volumen. ¿ En que tiempo podr´a entrar el p´ ublico?. 12. Un tanque contiene inicialmente agua pura. Salmuera que contiene 2 libras de sal/gal. entra al tanque a una velocidad de 4 gal./min. Asumiendo la mezcla uniforme, la salmuera sale a una velocidad de 3 gal./min. Si la concentraci´on alcanza el 90 % de su valor m´aximo en 30 minutos, calcular los galones de agua que hab´ıan inicialmente en el tanque. 13. Un tanque de una cierta forma geom´etrica est´a inicialmente lleno de agua hasta una altura H. El tanque tiene un orificio en el fondo cuya a´rea es A pie2 . Se abre el orificio dQ dt

es proporcional a la

aplicando la ecuaci´on de energ´ıa 12 mv 2 = mgh se obtiene v =

√ 2gh donde g = 32

y el l´ıquido cae libremente. La raz´on volum´etrica de salida velocidad de salida y al a´rea del orificio, es decir, dQ = −kAv dt

pie/seg2 . La constante k depende de la forma del orificio: a) Si el orificio es de forma rectangular, la constante k = 0, 8. b) Si el orificio es de forma triangular, la constante 0, 65 ≤ k ≤ 0, 75. c) Si el orificio es de forma circular, la constante k = 0, 6. Con estos datos: a) Un tanque semiesf´erico tiene un radio de 1 pie; el tanque est´a inicialmente lleno de agua y en el fondo tiene un orificio de 1 pulg. de di´ametro. Calcular el tiempo de vaciado. b) Modelar el caso: Cilindro circular de altura H0 pies y radio r pies, dispuesto en forma vertical y con un orificio circular de di´ametro ρ (pulgadas), suponga que esta lleno de agua y calcule el tiempo de vaciado.

50

Apuntes Mat023 (versi´on preliminar actualizada 23-05-2014)

14. Un torpedo se desplaza a una velocidad de 60 millas/hora en el momento de agotarse el combustible; si el agua se opone al movimiento con una fuerza proporcional a su velocidad y si en una milla de recorrido reduce su velocidad a 30 millas/hora. ¿A que distancia se detendr´a? 15. Una bala se introduce en una tabla de h = 10 cm. de espesor con una velocidad v0 = 200 m/seg, traspas´andola con v1 = 80 m/seg. Suponiendo que la resistencia de la tabla al movimiento de la bala es proporcional al cuadrado de la velocidad. Hallar el tiempo que demora la bala en atravesar la tabla. 16. Una cadena de 4 pies de longitud tiene 1 pie de longitud colgando del borde de una mesa. Despreciando el rozamiento, hallar el tiempo que tarda la cadena en deslizarse fuera de la mesa.

An´ alisis cualitativo Es equivocado pensar que el objetivo principal del estudio de las ecuaciones diferenciales consiste en encontrar artificios de c´alculo que permitan resolverlas. Anteriormente presentamos una selecci´on de t´ecnicas que permiten resolver algunas ecuaciones diferenciales. Como en toda selecci´on la lista no es completa. Existen tratados en donde se elaboran tablas de soluciones de manera an´aloga a las tablas de antiderivadas. La pericia para resolver ecuaciones diferenciales va perdiendo poco a poco importancia con la llegada de los computadores y el dise˜ no de software especializado para computaci´on simb´olica. La tendencia actual es dejar al computador este tipo de tareas de c´alculo. Un programa como Mathematica puede resolver mediante instrucciones sencillas casi todas las ecuaciones diferenciales tratadas en este curso. Sin quitarle importancia a este tipo de programas debe quedar claro que ni el m´as refinado de los software ni el m´as ingenioso de los matem´aticos puede resolver en t´erminos de funciones elementales todas las ecuaciones diferenciales, ni siquiera las m´as importantes de ellas. El problema m´as que de habilidad es de principio. En casos tan simples como dx 1 =− +t dt x 51

Apuntes Mat023 (versi´on preliminar actualizada 23-05-2014)

se desconocen soluciones cl´asicas. La b´ usqueda de recetas para resolver todas las ecuaciones diferenciales en t´erminos de funciones elementales es una b´ usqueda sin esperanzas. Ante este hecho se presentan algunas alternativas: Los m´etodos cualitativos, los m´etodos num´ericos, y los m´etodos de aproximaci´on. No es parte de los objetivos de estas notas un estudio detallado al respecto. Se quiere sin embargo ilustrar los m´etodos cualitativos.

M´ etodos cualitativos En muchos problemas, m´as que c´alculos cuantitativos puntuales, lo que interesa es el comportamiento cualitativo de las soluciones en t´erminos de las condiciones iniciales o de valores de los par´ametros. Saber que una soluci´on es creciente, que es c´oncava o que tiene un l´ımite en el infinito puede ser de ayuda en el entendimiento de un modelo. Ocurre, que bajo ciertas circunstancias, podemos obtener tal informaci´on sin resolver expl´ıcitamente la ecuaci´on diferencial. Analizaremos primero el siguiente modelo:

El modelo de Verhulst Resumiremos los principales resultados concernientes al modelo de Verhulst para la din´amica de poblaciones dx = x (a − bx) dt

(1.33)

donde a, b > 0, esta ecuaci´on tiene dos soluciones constantes x1 (t) = 0 y x2 (t) = ab . Estas soluciones dividen al plano xt en 3 regiones de poblaciones n o n a ao R1 = (t, x) : < x , R2 = (t, x) : 0 < x < , R3 = {(t, x) : x < 0} b b tales que el gr´afico de cualquier soluci´on no constante x = x(t) de (1.33) permanece confinado en una y s´olo una de estas regiones. M´as a´ un, podemos determinar cu´ando es creciente, y cu´ando es decreciente la soluci´on x = x(t) de (1.33) a partir de la condici´on inicial x(t0 ) = x0 .

52

Apuntes Mat023 (versi´on preliminar actualizada 23-05-2014)

Z

dx x (a − bx)  Z  1 b 1 + ax a a − bx   1 1 ln |x| − ln |a − bx| a a x ln a − bx se sigue

x a−bx

Z =

dt

= t+C = t+C = at + C

= Keat , x (t) =

1. Si x0 < 0 entonces x0 =

a Kaeat = at Kbe + 1 b + Ce−at

a , b+Ce−at0

x (t) =

 1 entonces C = − e−at b− 0

a x0

 , se sigue

ax0 bx0 + (a − bx0 ) e−a(t−t0 )

note que ax0 < 0 y bx0 + (a − bx0 ) e−a(t−t0 ) = 0 ⇔ (a − bx0 ) e−a(t−t0 ) = −bx0 ⇔ e−a(t−t0 ) =

−bx0 >0 (a − bx0 )

as´ı 1 −bx0 T = − ln + t0 > t0 a (a − bx0 ) i h −bx0 el intervalo de definici´on de esta soluci´on es −∞, − a1 ln (a−bx + t 0 ≡ ]−∞, T [ en 0) este intervalo d ax0 a2 x0 e−a(t−t0 ) (a − bx0 ) = 0 = dt bx0 + (a − bx0 ) e−a(t−t0 ) (bx0 + aeat0 −at − bx0 eat0 −at )2 la funci´on es estrictamente creciente y ax0 a = t→+∞ bx0 + (a − bx0 ) e−a(t−t0 ) b l´ım

adem´as ax0 =0 t→−∞ bx0 + (a − bx0 ) e−a(t−t0 ) l´ım

3. Si x0 >

a b

entonces x (t) =

ax0 bx0 +(a−bx0 )e−a(t−t0 )

i h −bx0 esta bien definida en − a1 ln (a−bx + t , +∞ ≡ 0 0)

]T, +∞[ donde T < t0 adem´as ax0 a = −a(t−t ) 0 t→+∞ bx0 + (a − bx0 ) e b l´ım

y d ax0 a2 x0 e−a(t−t0 ) (a − bx0 ) = ab .

54

Apuntes Mat023 (versi´on preliminar actualizada 23-05-2014)

En el gr´afico se muestra el comportamiento de las soluciones

Veremos ahora que es posible analizar los comportamientos de las soluciones sin la necesidad de resolver la ecuaci´on.

Ecuaciones diferenciales aut´ onomas Definici´ on 1.5.1. Diremos que una ecuaci´on diferencial de primer orden es aut´onoma si se puede expresar en la forma dx = f (x) dt

(1.34)

donde f : Ω → R es una funci´on definida en el intervalo abierto Ω. Las ecuaciones

dx dt

= x (1 − x2 );

dx dt

= x (1 − x);

ecuaciones aut´onomas, mientras la ecuaci´on

dx dt

dx dt

2

= e−x sin x son ejemplos de

= et x + t no lo es.

Teorema 1.5.1. Sean f : Ω → R una funci´ on de clase C 1 (Ω), x0 ∈ Ω y t0 ∈ R entonces existe una u ´nica soluci´on del P.V.I. dx = f (x) dt x (t0 ) = x0 x : I → R de clase C 1 (I) definida en un intervalo abierto que contiene a t0 .

55

Apuntes Mat023 (versi´on preliminar actualizada 23-05-2014)

Definici´ on 1.5.2. Llamaremos intervalo maximal de definici´on al mayor intervalo abierto donde esta definida la soluci´on del P.V.I. anterior.

Ejemplo 1.5.1. Determine el intervalo maximal de la soluci´on del P.V.I. dx = x2 dt x (0) = 2 Soluci´ on. Aplicando la t´ecnica de separaci´on de variables Z 0 Z x (t) dt = 1dt x (t)2 ⇔ −1 = t+C x (t) ⇒ −1 x (t) = t+C usando la condici´on inicial 2 =

−1 C

entonces c = −1/2 se sigue x (t) =

−1 t − 12

  el mayor intervalo que contiene a t = 0 en el cual esta funci´on esta definida es I = −∞, 12 el cual corresponde al intervalo maximal. Observaci´ on 1.5.1. Note que las ecuaciones aut´onomas son de variables separadas.

Definici´ on 1.5.3. Las soluciones constantes x (t) = c, t ∈ R de la ecuaci´on (1.34) son llamadas soluciones de equilibrio. Note que si x (t) = c es una soluci´on de equilibrio 0=

dx (t) = f (x (t)) = f (c) dt

en otras palabras las soluciones de equilibrio corresponden a las ra´ıces de la ecuaci´on f (x) = 0.

56

Apuntes Mat023 (versi´on preliminar actualizada 23-05-2014)

Ejemplo 1.5.2. Determine las soluciones de equilibrio de la ecuaci´on dx = sin x + sin2 x dt Soluci´ on. Las soluciones de equilibrio corresponden a las ra´ıces de la ecuaci´on sin x + sin2 x = 0 esto es (sin x) (1 + sin x) = 0 luego x = kπ con k ∈ Z o bien x =

3π 2

+ 2lπ con l ∈ Z.

Teorema 1.5.2. Las soluciones de la ecuaci´ on (1.34) son soluciones de equilibrio o funciones mon´otonas estrictas. Si x (t) es una soluci´on definida en su intervalo maximal I, mostraremos que la derivada no se puede anular a menos que la soluci´on sea de equilibrio. Supongamos que x0 (tc ) = 0 para alg´ un tc ∈ I y definamos φ (t) = x (tc ) = α una funci´on constante, como 0 = φ0 (t) y x0 (tc ) = f (x (tc )) se sigue 0 = φ0 (t) = f (φ (t)) luego φ (t) es soluci´on de equilibrio pero dx = f (x) dt x (tc ) = α tiene 2 soluciones, por el teorema de existencia y unicidad la soluci´on debe ser la misma, es decir x (t) = α para todo t. Esta demostraci´on tiene otra consecuencia 57

Apuntes Mat023 (versi´on preliminar actualizada 23-05-2014)

Teorema 1.5.3. Las gr´aficas de dos soluciones distintas de (1.34) no se intersectan.

Teorema 1.5.4. Sean Ω = ]α, β[, x0 ∈ Ω, t0 ∈ R. Si x = x (t) es soluci´on de (1.34) con x (t0 ) = x0 y su intervalo maximal de definici´on es ]a, b[ entonces los l´ımites l´ım x (t) = A y l´ım− x (t) = B

t→a+

t→b

existen o son ±∞, m´as a´ un, A debe tomar el valor α o β si a = 6 −∞, y B debe tomar uno de los valores α o β si b 6= ∞.

Ejemplo 1.5.3. Consideremos la ecuaci´on dx = x (1 − x) dt en este caso Ω = R as´ı α = −∞ y β = +∞. Para 0 < x0 < 1 sean x (t) definida en ]a, b[ la soluci´on que cumple x (t0 ) = x0 , note que x1 (t) = 0 y x2 (t) = 1 son soluciones de equilibrio luego 0 < x (t) < 1 se sigue que 0 ≤ A, B ≤ 1 entonces a = −∞ y b = +∞. La soluci´on estar´a definida en todo R.

Teorema 1.5.5. Si c ∈ Ω y x (t) es una soluci´on de (1.34) tal que l´ımt→+∞ x (t) = c o l´ımt→−∞ x (t) = c entonces φ (t) = c, t ∈ R es una soluci´on de equilibrio.

Demostraci´on. Tenemos que probar que f (c) = 0. Supongamos que f (c) > 0 entonces por la continuidad de f existir´ıa un intervalo ]c − δ, c + δ[ tal que x ∈ ]c − δ, c + δ[ implica f (x) >

f (c) 2

> 0, como l´ım x (t) = c

t→+∞

58

Apuntes Mat023 (versi´on preliminar actualizada 23-05-2014)

se sigue que existe un t0 tal que t ≥ t0 implica |x (t) − c| < δ se sigue Z t x0 (u) du x (t) = x (t0 ) + t0 Z t = x (t0 ) + f (x (u)) du t0 Z t f (c) > x (t0 ) + du 2 t0 f (c) (t − t0 ) = x (t0 ) + 2 para t ≥ t0 pero esto contradice l´ımt→+∞ x (t) = c (se puede llegar a una contradicci´on similar si f (c) < 0).

Equilibrio y estabilidad Notemos que en la ecuaci´on aut´onoma dx = f (x) dt se nos indica la pendiente de la recta tangente a la gr´afica de la funci´on soluci´on x = x (t) esta viene dada por f (x), esto es, dado un valor de x las pendientes siempre son las mismas (independiente de t), las curvas isoclinas corresponde x = c, adem´as en los intervalos en los cuales f (x) > 0 la soluci´on es estrictamente creciente y en los intervalos en los cuales f (x) < 0 la funci´on soluci´on es estrictamente decreciente. Si x (t) es una soluci´on de dx = f (x) dt entonces la funci´on ϕ (t) = x (t + c) tambi´en es soluci´on, en efecto ϕ0 (t) = x0 (t + c) = f (x (t + c)) = f (ϕ (t)) 59

Apuntes Mat023 (versi´on preliminar actualizada 23-05-2014)

esto significa que las traslaciones de una soluci´on de la ecuaci´on aut´onoma tambi´en es soluci´on, note que si se conoce la soluci´on x (t) de dx = f (x) dt x (t0 ) = x0 entonces la soluci´on de dx = f (x) dt x (µ0 ) = x0 es ϕ (t) = x (t + (t0 − µ0 )), esto nos dice que en cada banda limitada por las las soluciones de equilibrio las soluciones son traslaciones de una soluci´on dada. Ejemplo 1.5.4. Considere la ecuaci´on dx =x dt en este caso la soluci´on de equilibrio es x (t) = 0. Por teorema las dem´as soluciones son mon´otonas estrictas. Para x > 0 las soluciones son estrictamente crecientes y para x < 0 estrictamente decrecientes. En este caso es f´acil resolver expl´ıcitamente x1 (t) = et es una soluci´on en la regi´on x > 0 las dem´as soluciones son xc (t) = et+k = ek et = Cet donde C > 0 son traslaciones de la soluci´on anterior. Note tambi´en que x (t) = −et es una soluci´on en la regi´on x < 0 y las dem´as soluciones en esa regi´on son xK (t) = −et+k = −Ket En el ejemplo anterior el comportamiento de las soluciones en el entorno de la soluci´on 60

Apuntes Mat023 (versi´on preliminar actualizada 23-05-2014)

de equilibrio es como indica el diagrama

diremos en este caso que el punto de equilibrio es un repulsor, las soluciones se alejan de esta soluci´on de equilibrio. Llamaremos diagrama de fases o l´ıneas de fases al gr´afico del comportamiento de las soluciones en el plano xt. Definici´ on 1.5.4. Dependiendo del comportamiento local de las soluciones de la ecuaci´on alrededor de un punto de equilibrio aislado x0 en las l´ıneas de fases, se distinguen los siguientes tipos: 1. x0 es llamado Repulsor si existe δ > 0 tal que

signo f (x)

x0 − δ < x < x0

x = x0

x0 < x < x 0 + δ

−−−

0

+++

x0 − δ < x < x0

x = x0

x0 < x < x 0 + δ

+++

0

−−−

2. x0 es llamado Atractor si existe δ > 0 tal que

signo f (x)

3. x0 es llamado Atractor-repulsor si existe δ > 0 tal que

signo f (x)

x0 − δ < x < x0

x = x0

x0 < x < x 0 + δ

+++

0

+++

61

Apuntes Mat023 (versi´on preliminar actualizada 23-05-2014)

4. x0 es llamado Repulsor-atractor si existe δ > 0 tal que x0 − δ < x < x0

x = x0

x0 < x < x 0 + δ

−−−

0

−−−

signo f (x)

Para analizar entonces el tipo de soluci´on de equilibrio tenemos que analizar el signo de la funci´on en el entorno de la soluci´on de equilibrio, adicionalmente se cuenta con el siguiente teorema: Teorema 1.5.6. Si x (t) = c es una soluci´on de equilibrio de dx = f (x) dt entonces: 1. Si f 0 (c) < 0, c es un atractor 2. Si f 0 (c) > 0, c es un repulsor. Ejemplo 1.5.5. Bosquejar el diagrama de fases de la ecuaci´on dx = x2 (2 − x) (x − 3) dt Desarrollo: La funci´on x2 (2 − x) (x − 3) esta bien definida y es de clase C ∞ (R) existe soluci´on u ´nica en cada punto (t0 , x0 ) del plano xt. Las soluciones de equilibrio corresponden a x1 (t) = 0, x2 (t) = 2 y x2 (t) = 3, en el siguiente diagrama se analiza el signo de f 0

2

3

x2

+++

0

+++

+

+++

+

+++

x−2

−−−



−−−

0

+++

+

+++

x−3

−−−



−−−



−−−

0

+++

f (x) = −x2 (x − 2) (x − 3)

−−−

0

−−−

0

+++

0

−−−

62

Apuntes Mat023 (versi´on preliminar actualizada 23-05-2014)

se sigue que x1 (t) = 0 es un repulsor-atractor, x2 (t) = 2 un repulsor y x2 (t) = 3 un atractor.

Ejercicios de la secci´ on 1. En una red de computadores sea x (t) la proporci´on de m´aquinas infectadas con el virus MTA320 en un instante t dado (note que 0 ≤ x (t) ≤ 1). x0 (t) mide la rapidez de propagaci´on sobre la red. Los estudios de expertos de internet determinan que el virus satisface el problema con valores iniciales    1 2 0 x = αx −x − x (1 − x) 3 3 x (0) = x0 donde x0 es la proporci´on de computadores en que el virus se implanta inicialmente y α > 0 es una constante. a) Encuentre las soluciones de equilibrio, clasificarlas y dibujar el diagrama de fase del sistema. b) Demostrar que si 0 < x0 < 2/3 entonces el virus alcanzar´a finalmente a un tercio del total. 63

Apuntes Mat023 (versi´on preliminar actualizada 23-05-2014)

c) ¿Cu´al debe ser la cantidad de m´aquinas infectada inicialmente para que todas se infecten? 2. Sea x : I → R la soluci´on maximal del P.V.I. dx 2 = x2 (1 − x) (2 − x) ex dt 3 x (0) = 2 muestre que I = R y determine l´ımt→+∞ x (t), l´ımt→−∞ x (t). 3. Considere la ecuaci´on diferencial dy = y 2/3 dt a) Demuestre que y1 (t) = 0 para todo t ∈ R es soluci´on. b) Comprobar que y2 (t) = t3 /27 es tambi´en soluci´on. c) Notar que y1 (0) = y2 (0) pero ambas funciones no son iguales para toda t ¿Por qu´e este ejemplo no contradice el teorema de unicidad? 4. Esboce las l´ıneas de fase para la ecuaci´on diferencial dada. Identifique los puntos de equilibrio: (a)

dy dt

= 3y (1 − y)

(b)

dy dt

= y 2 − 6y − 16

(c)

dy dt

= cos y

(d)

dw dt

= w cos w

(e)

dw dt

= (w − 2) sin w

(f)

dy dt

=

(g)

dw dt

= w2 + 2w + 10

(h)

dy dt

= tan y

5. Considere la ecuaci´on diferencial dy = y 2 − 4y + 2 dt con las siguientes condiciones iniciales: a) y (0) = 0 b) y (0) = 1 64

1 y−2

Apuntes Mat023 (versi´on preliminar actualizada 23-05-2014)

c) y (0) = −1 d ) y (0) = −10 e) y (0) = 10 f ) y (3) = 1 describir el comportamiento a largo plazo de la soluci´on. 6. Describir el diagrama de fases de la ecuaci´on  dx = (x + λ) x2 + λ (cos x + 2) dt para los distintos valores del par´ametro λ. 7. Determine los valores de α ∈ R para los cuales la soluci´on de equilibrio x (t) =    1 dx = (x − α) − x x2 − α dt 3

1 3

de

es: a) Un atractor. b) Un repulsor. o justifique la no existencia de tales valores. 8. Sea x (t) la soluci´on del P.V.I. dx 2 = ex−x sin (x) arctan x dt x (3) = 4 a) Determine su intervalo maximal de definici´on. b) ¿Es x (t) estrictamente creciente? c) Si existen, calcular el valor de l´ımt→+∞ x (t) y l´ımt→−∞ x (t) 9. Sean α, β > 0. La ecuaci´on dP = αP 2/3 − βP dt modela el peso de un pez en el tiempo t. Sin resolver la ecuaci´on, determine el peso m´aximo del pez si P (0) = p0 > 0 (justificar sus c´alculos). 65

Apuntes Mat023 (versi´on preliminar actualizada 23-05-2014)

10. Muestre que la soluci´on de equilibrio y (t) ≡ 0 de la ecuaci´on   dy = y cos y 5 + 2y − 27πy 4 dt corresponde a un repulsor.

66

Apuntes Mat023 (versi´on preliminar actualizada 23-05-2014)

Ejercicios del cap´ıtulo 1. Resuelva las siguientes ecuaciones diferenciales: a)

dy dx

=

1−x2 y2

b)

dy dx

= y (2 + sin x)

c)

dy dx

d)

dy dx

=

1 xy 3

e)

dy dx

= 3xy 2

f)

dv x dx =

g)

dx dt

+ x2 = x

h)

dy dx

= 3x2 (1 + y 2 )

i)

dy (x + xy 2 ) + ex y dx =0

=

sec2 x 1+x2

1−4v 2 3v

2

2. Resuelva los siguientes problemas de valor inicial: a) y 0 = x3 (1 − y) , 3x2 +4x+2 , 2y+1

c)

dy dx

=

e)

dy dx

= y sin x,

√ 3

y (0) = 3

b)

dy dx

= (1 + y 2 ) tan x,

y (0) =

y (0) = −1

d)

dy dx

√ = 2 y + 1 cos x,

y (0) = 2

f)

dy x2 + 2y dx = 0,

y (π) = −3

y (1) = 2

3. Resuelva las siguientes ecuaciones lineales: a)

dy dx

− y = e3x

dy d) x dx + 3y + 2x2 = x3 + 4x

b)

dy dx

e)

dy (x2 + 1) dx + xy = x

=

y x

+ 2x + 1

c)

dr dθ

+ r tan θ = sec θ

f)

dy dx

= x2 e−4x − 4y

4. Considere el problema con valor inicial: p dy + y 1 + sin2 x = x, dx

y (0) = 2

Utilice la integral indefinida para mostrar que el factor integrante para la ecuaci´on diferencial se puede escribir como: Z µ (x) = exp

x

 p 2 1 + sin t dt

0

5. Resuelva las siguientes ecuaciones diferenciales:

67

Apuntes Mat023 (versi´on preliminar actualizada 23-05-2014)

a)

dy =0 (x − y) + x dx

d) y 0 + 1 =



x+y

b)

dy dx

= (3x + 2y − 1)2 + 2

e)

dx dt

=

√ x2 +t t2 +x2 tx

c)

dy dx

= sin (x − y)

f)

dy dx

=

y(ln y−ln x+1) x

6. Hallar la soluci´on general de la ecuaci´on de Bernoulli: 2y 0 + y tan x = (−2x sec x) y 3 7. Resuelva la ecuaci´on: 3y 0 = (1 − 2t) y 4 − y sabiendo que y (0) = 1. 8. Resolver: dy 1 =y−x−1+ dx x−y−2 9. Considere la ecuaci´on diferencial:  y − xy 0 = a 1 + x2 y 0 ,

a>1

a) Hallar la soluci´on general de la ecuaci´on. b) Encuentre una soluci´on particular de la ecuaci´on, tal que y (0) = 1. c) Hallar el intervalo m´as grande donde la soluci´on particular anterior est´e definida. 10. Demuestre que la ecuaci´on diferencial: 2x4 y y 0 + y 4 = 4x6 se reduce a una ecuaci´on homog´enea mediante el cambio de variables: y = zn para cierto n ∈ Z. Determine el valor de n y resuelva la ecuaci´on. 11. Muestre que la ecuaci´on:   2 x3 y − y 3 dy = 3 x5 + x4 y 4 dx 68

Apuntes Mat023 (versi´on preliminar actualizada 23-05-2014)

se reduce a una ecuaci´on homog´enea, realizando el cambio de coordenadas: x = up



y = vq

para ciertas constantes adecuadas p, q ∈ R. Hallar tales constantes y resolver la ecuaci´on diferencial. 12. Hallar la soluci´on general de la ecuaci´on: (x − 2y − 4) + (2x − y + 2)

dy =0 dx

13. Considere la ecuaci´on de Ricatti: y 0 + 2 (1 − x) y − y 2 = x (x − 2) a) Hallar constantes A, B ∈ R de modo que: y = Ax + B sea una soluci´on particular de la ecuaci´on. b) Determine la soluci´on general de la ecuaci´on diferencial. 14. Resuelva la ecuaci´on: dy y = x3 (y − x)2 + dx x 15. Sea x > 0. Considere la ecuaci´on: y 0 + e−2x y 2 −

  1 e2x 1 + 4x + 2x2 y = − 1 + x + 2x2 + x3 x x

a) Hallar una soluci´on particular de la forma: y = e2x (Ax + B) b) Resuelva la ecuaci´on diferencial. 16. Un esquiador acu´atico ubicado en el punto (a, 0) es tirado por un bote que se encuentra en el origen O y viaja hacia el norte en direcci´on OY . Hallar la trayectoria que sigue el bote si este se dirige en todo momento hacia el bote. 69

Apuntes Mat023 (versi´on preliminar actualizada 23-05-2014)

17. Considere un tanque que contiene inicialmente 1000 litros de agua pura, dentro del cual empieza a fluir una soluci´on salina a raz´on de 6 litros por minuto. La soluci´on dentro del estanque se mantiene bien agitada y fluye hacia el exterior del tanque a una velocidad de 5 litros por minuto. Si la concentraci´on de sal en la salmuera que entra en el estanque es de 1 kilogramo por litro, determine el instante en que la concentraci´on de sal dentro del tanque sea de

63 64

kilogramo por litro.

18. El modelo de Malthus para el crecimiento de poblaciones est´a dado por la ecuaci´on: dN = rN, dt

N (0) = N0

donde r es una constante positiva intr´ınseca a la poblaci´on y N0 es la poblaci´on inicial. a) Resuelva la ecuaci´on diferencial. b) En 1790, la poblaci´on de EE.UU. era de 3, 93 millones de habitantes, y en 1890 era de 62, 98 millones de habitantes. Estime la poblaci´on para EE.UU para el a˜ no 2000. c) El censo realizado en el a˜ no 2000 en EE.UU. estim´o en 281, 42 millones de habitantes d ) El modelo de Malthus s´olo considera muertes por causas naturales, sin embargo, un an´alisis m´as detallado indica que hay muertes debida a enfermedades, a desnutrici´on, a cr´ımenes. En resumen, por la competencia entre los miembros de la poblaci´on. El modelo log´ıstico implementa dichas consideraciones: dN = rN (N − K) , dt

N (0) = N0

para constantes positivas adecuadas r y K. Resuelva la ecuaci´on diferencial. e) Para la poblaci´on de EE.UU. se sabe, adem´as, que ´esta alcanz´o los 17, 07 millones de habitantes en 1840. Calcule, usando el modelo log´ıstico, la poblaci´on de EE.UU. en el a˜ no 2000. Compare respecto al censo del a˜ no 2000. f ) Considerando el modelo log´ıstico, calcule la poblaci´on l´ımite de EE.UU.

70

Apuntes Mat023 (versi´on preliminar actualizada 23-05-2014)

19. La sangre conduce un medicamento a un o´rgano a raz´on de 3

h

cm3 seg

i

y sale de ´el a la

misma raz´on. El ´organo tiene un volumen l´ıquido de 125 [cm3 ]. Si la concentraci´on  gr  del medicamento en la sangre que entra al o´rgano es de 0, 2 cm 3 : a) ¿Cu´al es la concentraci´on del medicamento en el ´organo en el instante t, si inicialmente no hab´ıa rastros de dicho medicamento? b) ¿En qu´e momento llegar´a la concentraci´on del medicamento en el ´organo a  gr  0, 1 cm 3 20. Una taza de caf´e caliente, inicialmente a 95◦ C, se enfr´ıa hasta 80◦ C en 5 minutos, al estar en un cuarto con temperatura de 21◦ C. Use s´olo la ley de enfriamiento de Newton (ver MAT022) y determine el momento en que la temperatura del caf´e estar´a a unos agradables 50◦ C. 21. Era el mediod´ıa en un fr´ıo d´ıa de julio en Vi˜ na del Mar: 16◦ C. Un inspector de la Polic´ıa de Investigaciones (PDI) llega a la escena de un crimen, encontrando un detective sobre un cad´aver. El detective dijo que hab´ıa varios sospechosos detenidos. Dadas las coartadas se podr´ıa hallar al culpable sabiendo el momento exacto de la muerte. El inspector sac´o un term´ometro y midi´o la temperatura del cad´aver: 34◦ C. Luego de realizar peritos en la escena que duraron exactamente una hora, midi´o nuevamente la temperatura del cuerpo: 33, 7◦ C. Bajo que la hip´otesis de que la temperatura normal de un cuerpo humano es de 37◦ C. Determine el momento en que ocurri´o el crimen. 22. El economista ingl´es Thomas Malthus fue unos de los primeros en intentar modelar el crecimiento poblacional humano por medio de matem´aticas en 1978. B´asicamente, el fundamento del modelo maltusiano es la suposici´on de que la rapidez a la que crece la poblaci´on de un pais en cierto tiempo es proporcional a la poblaci´on del pa´ıs en ese momento. a) Proponer una ecuaci´on para este modelo. b) Determine la ecuaci´on diferencial si se permite inmigraci´on a tasa constante r > 0. 71

Apuntes Mat023 (versi´on preliminar actualizada 23-05-2014)

23. La ley de enfriamiento (o calentamiento) de Newton establece que la rapidez a la que cambia la temperatura de un cuerpo es proporcional a la diferencia entre la temperatura del cuerpo y la temperatura del medio circundante. Obtener una E.D.O. para esta ley. Si la temperatura del medio cambia en el tiempo, escribir la ecuaci´on e identificarla. 24. Una enfermedad contagiosa se disemina en una comunidad por medio de la gente que entra en contacto con otras personas. Suponer que la rapidez con que se disemina la enfermedad es proporcional al n´ umero de interacciones entre las personas contagiadas y no contagiadas (puede ser considerado como el producto de los contagiados y no contagiados). Suponga que en una poblaci´on peque˜ na de n personas se introduce un enfermo obtener una EDO cuya soluci´on permita obtener el n´ umero de contagiados en un tiempo t. 25. Resolver las siguientes ecuaciones diferenciales ordinarias:

72

Apuntes Mat023 (versi´on preliminar actualizada 23-05-2014)

1)

1+y dy = dx 1+x

2)

dy − 2xy = x dx

3)

dy − y tan x = cos x dx

4)

dy yx = 2 dx x − y2

5)

2y + x − 2 dy = dx y−x+1

6)

dy 1 cos x + y= 2 dx x y

7)

1 dy + x2 y = 2 y 4 dx x

8)

dy cos y + 2x sin y = −2x dx

9)

dy = y 2 + 2x − x4 dx

10)

dy + dx

11)

dy − 4y = 2ex y 1/2 dx

12)

dy y+1 = + exp dx x+2

13)

dy = 3 |y|2/3 dx

14)

dy e−y = dx y (2x + x2 )

15)

dy = (x − y + 3)2 dx

16) x

17)

dy + y sin x = sin3 x dx

18)

dy 19) y = x + dx





 1 e2x −1 y = x x 

y+1 x+2



2

dy dx

p dy = y + x2 + y 2 dx

dy y + = (1 + x) y 4 dx 1 + x

2

26. Suponga que un gran tanque de mezclado tiene inicialmente 600 galones de salmuera. Otra soluci´on de salmuera se bombea hacia en tanque a una rapidez de 4 galones por minuto, la concentraci´on de sal en el flujo de entrada es de 2 libras por gal´on. Cuando la soluci´on en el tanque esta bien agitada se bombea a 2 galones por minuto. Determinar una EDO cuya soluci´on corresponda a la cantidad de sal en el tanque en el momento t.

73

Apuntes Mat023 (versi´on preliminar actualizada 23-05-2014)

27. Identificar el campo de direcciones con la ecuaci´on correspondiente; (A) y 0 = x + y (B) y 0 = − xy (C) y 0 = y + 1 (D) y 0 = y 2 − x2

Obs: el campo de direcciones nos indica la direcci´on de las rectas tangentes a las soluciones en un punto (x, y) dado. 28. Considere la ecuaci´on diferencial aut´onoma dS = S 3 − 2S 2 + S dt a) Hacer el diagrama de las l´ıneas de fase.. b) Usando el dibujo anterior delinear las gr´aficas de las soluciones S (t) con las condiciones iniciales siguientes: S (0) = 0; S (0) = 12 ; S (0) = c) ¿A que tienden sus soluciones cuando t → +∞? 74

3 2

y S (0) = − 12 .

Apuntes Mat023 (versi´on preliminar actualizada 23-05-2014)

29. Considere la ecuaci´on dy 1 = dt 1−y a) Determine los valores de y ∈ R en los cuales es aplicable el teorema de existencia y unicidad. b) Obtener las soluciones expl´ıcitamente y mostrar que llegan la y = 1 en un tiempo finito, determine los intervalos maximales de soluci´on 30. La ardilla negra es un peque˜ no mam´ıfero nativo de las monta˜ nas rocallosas. Esas ardillas son muy territoriales, por lo que si su poblaci´on es grande, su raz´on de crecimiento decrece y puede llegar a ser negativa. Por otra parte, si la poblaci´on es demasiado peque˜ na, los adultos f´ertiles corren el riesgo de no ser capaces de encontrar parejas adecuadas y su tasa de crecimiento nuevamente es negativa. Muestre que el modelo dP = kP dt



P 1− N



 P −1 M

donde k, M, N > 0 y M < N puede ser utilizado para describir la poblaci´on de Ardillas ¿Qu´e interpretaci´on tienen N y M ?

75

Cap´ıtulo 2 : Ecuaciones diferenciales lineales de orden superior

Elementos de transformaciones lineales Definiciones En lo que sigue K representa R o bien C. on lineal Definici´ on 2.1.1. Sean U y V espacios vectoriales sobre K. Una transformaci´ es una funci´on T : U → V tal que para todo u, v ∈ U y α ∈ K cumple: 1. T (u + v) = T (u) + T (v) 2. T (α u) = α T (u) Observaci´ on 2.1.1. Usualmente, los puntos (1) y (2) de la definici´on anterior, pueden escribirse como 1. ∀u, v ∈ U, ∀α ∈ K, T (α u + v) = α T (u) + T (v) o bien 2. ∀u, v ∈ U, ∀α, β ∈ K, T (α u + βv) = α T (u) + βT (v) las tres definiciones obtenidas son equivalentes.

Observaci´ on 2.1.2. Note que si U, V son K espacios vectoriales, T : U → V es una transformaci´on lineal, para i = 1, · · · , n, αi son escalares en K y ui ∈ U entonces ! n n X X T αi ui = αi T (ui ) i=1

i=1

luego las transformaciones lineales env´ıan combinaciones lineales del espacio de partida a combinaciones lineales del espacio de llegada.

76

Apuntes Mat023 (versi´on preliminar actualizada 23-05-2014)

Ejemplo 2.1.1. La funci´on nula 0 y la funci´on identidad 1 de un espacio vectorial en s´ı mismo son transformaciones lineales. En efecto, Si V es un K espacio vectorial y 0 : V → V , v → 0 (v) = θV (el neutro de la adici´on) entonces 0 (αu + v) = θV = αθv + θv = α0 (u) + 0 (v) de manera similar, si 1 : V → V , v → 1 (v) = v entonces 1 (αu + v) = αu + v = α1 (u) + 1 (v) Ejemplo 2.1.2. T : R2 → R4 , (x, y) → T (x, y) = (x − y, 2x, x + y, 2y) es una transformaci´on lineal. En efecto: 1. Para cada (a, b), (u, v) ∈ R2 se tiene T ((a, b) + (u, v)) = T (a + u, b + v) = (a + u − b − v, 2a + 2u, a + u + v + b, 2b + 2v) = ((a − b) + (u − v) , 2a + 2u, (a + b) + (u + v) , 2b + 2v) = (a − b, 2a, a + b, 2b) + (u − v, 2u, u + v, 2v) = T (a, b) + T (u, v) 2. Para cada (u, v) ∈ R2 y α ∈ R se cumple T (α (u, v)) = T (αu, αv) = (αu − αv, 2αu, αu + αv, 2αv) = α (u − v, 2u, u + v, 2v) = αT (u, v) de esta forma T verifica las condiciones de transformaci´on lineal.

77

Apuntes Mat023 (versi´on preliminar actualizada 23-05-2014)

Ejemplo 2.1.3. T : R → R2 , x → T (x) = (x, 2x + 1) no es una transformaci´on lineal. En efecto, T (x + y) = (x + y, 2x + 2y + 1) = (x, 2x + 1) + (y, 2y) 6= T (x) + T (y) basta usar un contraejemplo: T (1 + 2) = T (3) = (3, 7) pero T (1) + T (2) = (1, 3) + (2, 5) = (3, 8) de esta forma T (1 + 2) 6= T (1) + T (2) luego no es una transformaci´on lineal. Ejemplo 2.1.4. Si R3 [x] es el espacio vectorial de los polinomios de grado menor igual a 3 entonces T : R3 [x] → p

M (R)  2×2  p (0) p0 (0)  → T (p) =  p00 (0) p000 (0)

Es una transformaci´on lineal. En efecto: 1. Si p, q ∈ R3 [x] entonces  T (p + q) = 

(p + q)0 (0)



(p + q)00 (0) (p + q)000 (0)



(p + q) (0)

pero recordar que (p + q)0 (x) = p0 (x) + q 0 (x), (p + q)00 (x) = p00 (x) + q 00 (x) y

78

Apuntes Mat023 (versi´on preliminar actualizada 23-05-2014)

(p + q)000 (x) = p000 (x) + q 000 (x) se sigue   0 (p + q) (0) (p + q) (0)  T (p + q) =  00 000 (p + q) (0) (p + q) (0)   p (0) + q (0) p0 (0) + q 0 (0)  =  p00 (0) + q 00 (0) p000 (0) + q 000 (0)     0 0 p (0) p (0) q (0) q (0) +  =  p00 (0) p000 (0) q 00 (0) q 000 (0) = T (p) + T (q) 2. Si α ∈ R y p ∈ R3 [x] entonces     (αp) (0) (αp)0 (0) αp (0) αp0 (0) =  T (αp) =  (αp)00 (0) (αp)000 (0) αp00 (0) αp000 (0)   0 p (0) p (0)  = αT (p) = α p00 (0) p000 (0) Note que no es necesario considerar la expresi´on del polinomio p (x) = a3 x3 + a2 x2 + a1 x + a0 en esta demostraci´on de que es transformaci´on lineal, lo que utilizamos son las propiedades de la derivada y matrices. Ejemplo 2.1.5. Sea T : R3 → R2 la funci´on definida por: T (x, y, z) = (x + y + z, x − y) muestre que T es una transformaci´on lineal. Soluci´ on. Sean α ∈ R, (u, v, w) , (x, y, z) ∈ R3 entonces T (α (u, v, w) + (x, y, z)) = T ((αu, αv, αw) + (x, y, z)) = T (αu + x, αv + y, αw + z) = ((αu + x) + (αv + y) + (αw + z) , (αu + x) − (αv + y)) = (αu + αv + αw, αu − αv) + (x + y + z, x − y) = α (u + v + w, u − v) + (x + y + z, x − y) = αT (u, v, w) + T (x, y, z) 79

Apuntes Mat023 (versi´on preliminar actualizada 23-05-2014)

Ejemplo 2.1.6. Sean I ⊆ R un intervalo abierto y C 1 (I) el espacio vectorial real de las funciones de clase C 1 sobre I. Defina la transformaci´on D : C 1 (I) → C (I) mediante: D (f ) (x) =

df (x) , dx

∀x ∈ I

Entonces, D es una trasformaci´on lineal. Soluci´ on. Por propiedades de la derivada, si α ∈ R, f, g ∈ C 1 (I) entonces d {(αf + g) (x)} dx df (x) dg (x) = α + dx dx = αD (f ) (x) + D (g) (x)

D (αf + g) (x) =

= (αD (f ) + D (g)) (x) as´ı D (αf + g) = αD (f ) + D (g) Ejemplo 2.1.7. Sea C el espacio vectorial de todas las funciones continuas de R en R y sea T : C → C, f → T (f ) donde T (f ) es la funci´on definida por: Z x T (f ) (x) = f (t) dt 1

Entonces, T es una transformaci´on lineal. Soluci´ on. Sean α ∈ R, f, g ∈ C entonces x

Z T (αf + g) (x) =

(αf (t) + g (t)) dt Z x Z x = αf (t) dt + g (t) dt 1 1 Z x Z x = α f (t) dt + g (t) dt 1

1

1

= αT (f ) (x) + T (g) (x) = (αT (f ) + T (g)) (x) as´ı T (αf + g) = αT (f ) + T (g) 80

Apuntes Mat023 (versi´on preliminar actualizada 23-05-2014)

Ejemplo 2.1.8. Sean x, y ∈ Rn . Considere la funci´on P : Rn → R definida por: x 7→ P (x) =

n X

xi y i

i=1

Entonces, P es una transformaci´on lineal. Esto es directo por las propiedades del producto punto en Rn . Observaci´ on 2.1.3. El conjunto de todas las transformaciones lineales de U en V se denotar´a por LK (U, V ), o m´as sencillamente por L (U, V ) si el cuerpo de escalares no presenta confusi´on. Es decir: n L (U, V ) = T : U → V

o T es transformaci´ o n lineal

Teorema 2.1.1. Sea T : U → V una transformaci´on lineal, entonces T (θU ) = θV . Demostraci´on. Note que T (θU ) = T (θU + θU ) = T (θU ) + T (θU ) sumando el inverso aditivo −T (θU ) ∈ V se tiene T (θU ) + −T (θU ) = (T (θU ) + T (θU )) + −T (θU ) luego θV

= T (θU ) + (T (θU ) + −T (θU )) = T (θU ) + θV

y as´ı T (θU ) = θV

Ejemplo 2.1.9. Considere la funci´on T : R2 → R3 definida por la ecuaci´on: T (x, y) = (x + y, 1 − y, x) En vista del teorema anterior, la funci´on T no puede ser transformaci´on lineal, pues: T (0, 0) = (0, 1, 0) no es el vector nulo de R3 . 81

Apuntes Mat023 (versi´on preliminar actualizada 23-05-2014)

Teorema 2.1.2. Sean U, V, W espacios vectoriales sobre K: 1. L (U, V ) es un espacio vectorial sobre K. 2. Si T ∈ L (U, V ) y S ∈ L (V, W ) entonces S ◦ T ∈ L (U, W ). 3. Si T ∈ L (U, V ) es biyectiva entonces T −1 ∈ L (V, U ) Demostraci´on. En efecto: 1. Mostraremos que es un subespacio del espacio de las funciones de U en V . L (U, V ) no es vac´ıo pues existe la transformaci´on lineal θ : U → V definida por u → θ (u) = θV . Sean α, β ∈ K y T, L ∈ L (U, V ) entonces αT + βL es una transformaci´on lineal pues si γ ∈ K, u1 , u2 ∈ U entonces (αT + βL) (γu1 + u2 ) = αT (γu1 + u2 ) + βL (γu1 + u2 ) = α (γT (u1 ) + T (u2 )) + β (γL (u1 ) + L (u2 )) = γ (αT (u1 ) + βL (u1 )) + (αT (u2 ) + βL (u2 )) = γ (αT + βL) (u1 ) + (αT + βL) (u2 ) luego L (U, V ) es no vac´ıo, cerrado para la suma y producto escalar. 2. Sean u, v ∈ U y α ∈ K, entonces: (S ◦ T ) (α u + v) = S (T (α u + v)) = S (α T (u) + T (v)) = α S (T (u)) + S (T (v)) = α (S ◦ T ) (u) + (S ◦ T ) (v) Luego, S ◦ T es una transformaci´on lineal. 3. Si T −1 : V → U existe entonces es una transformaci´on lineal, en efecto, sean v1 , v2 ∈ V y α ∈ K, por la biyectividad existen u ´nicos u1 , u2 ∈ U tales que T (u1 ) = v1 T (u2 ) = v2 82

Apuntes Mat023 (versi´on preliminar actualizada 23-05-2014)

luego T −1 (αv1 + v2 ) = T −1 (αT (u1 ) + T (u2 )) = T −1 (T (αu1 + u2 )) = αu1 + u2 = αT −1 (v1 ) + T −1 (v2 )

Observaci´ on 2.1.4. Con respecto al teorema anterior, denotaremos la composici´on de transformaciones lineales S ◦ T mediante la notaci´on producto ST . Teorema 2.1.3. Sean U un espacio vectorial sobre K tal que u1 , u2 , . . . , un es una base para U y v1 , v2 , . . . , vn un subconjunto cualquiera de un espacio vectorial V arbitrario (se pueden repetir elementos), entonces existe una u ´nica transformaci´on lineal T : U → V tal que: ∀i = 1, 2, . . . , n

T (ui ) = vi ,

(2.1)

Observaci´ on 2.1.5. Note que si u1 , u2 , . . . , un es una base para un espacio vectorial U y u ∈ U , entonces existen u ´nicos escalares x1 , x2 , . . . , xn ∈ K tales que: u=

n X

xi u i

i=1

Si, adem´as, T est´a definida por las ecuaciones (2.1), entonces se tiene que: ! n X Tu = T xi ui i=1

=

n X

xi T (ui )

i=1

=

n X

xi vi

i=1

Ejemplo 2.1.10. Determinaremos la f´ormula T (x, y, z) de la transformaci´on lineal T : R3 → R3 [x] tal que: T (1, 0, 2) = x3 + 2x2 + 1 T (−1, 1, 1) = −2x2 − 2 T (1, −2, 1) = x3 − 1 83

(2.2)

Apuntes Mat023 (versi´on preliminar actualizada 23-05-2014)

Notamos primeramente que u1 = (1, 0, 2) , u2 = (−1, 1, 1) y u3 = (1, −2, 1) forman una base para R3 , pues: 

1

 det   −1 1

0

2



 1  =5 −2 1 1

Por otro lado, si: (a, b, c) = x1 · (1, 0, 2) + x2 · (−1, 1, 1) + x3 · (1, −2, 1) entonces x1 = 35 a + 25 b + 15 c, x2 = 25 c − 15 b − 45 a y x3 = 15 c − 35 b − 25 a, as´ı:       2 1 2 1 4 1 3 2 3 a + b + c (1, 0, 2)+ c − b − a (−1, 1, 1)+ c − b − a (1, −2, 1) (x, y, z) = 5 5 5 5 5 5 5 5 5 por la linealidad de T y su definici´on en la base, tenemos que:



 3 2 1 T (a, b, c) = a + b + c T (1, 0, 2) + 5 5 5     1 4 1 3 2 2 c − b − a T (−1, 1, 1) + c − b − a T (1, −2, 1) + 5 5 5 5 5 5       3 2 1 2 1 4 3 2 = a + b + c x + 2x + 1 + c − b − a −2x2 − 2 + 5 5 5 5 5 5    1 3 2 + c − b − a x3 − 1 5 5 5       1 1 2 14 6 2 13 7 4 3 2 = a− b+ c x + a+ b− c x + a+ b− c 5 5 5 5 5 5 5 5 5 Es decir, T : R3 → R3 [x] est´a definida por:       1 1 2 14 6 2 13 7 4 3 2 T (a, b, c) = a− b+ c x + a+ b− c x + a+ b− c 5 5 5 5 5 5 5 5 5 Ejemplo 2.1.11. Hallar la f´ormula de una transformaci´on lineal T de R2 en:  U = (x, y, z) ∈ R3 : x + y + z = 0 tal que T (−1, 1) = (1, 2, −3) y T (2, −1) = (5, −3, −2).

84

Apuntes Mat023 (versi´on preliminar actualizada 23-05-2014)

Soluci´ on. Note que (1, 2, −3), (5, −3, −2) pertenecen a U , como (−1, 1) y (2, −1) son linealmente independiente, por el teorema existe una u´nica transformaci´on lineal T : R2 → U tal que T (−1, 1) = (1, 2, −3) T (2, −1) = (5, −3, −2) para determinar una f´ormula, notemos que (a, b) = α (−1, 1) + β (2, −1) implica  

−1 1

2

a

−1 b





∼

1 0 a + 2b 0 1

a+b

 

as´ı (a, b) = (a + 2b) (−1, 1) + (a + b) (2, −1) as´ı T (a, b) = (a + 2b) T (−1, 1) + (a + b) T (2, −1) = (a + 2b) (1, 2, −3) + (a + b) (5, −3, −2) = (6a + 7b, b − a, −5a − 8b)

N´ ucleo e imagen Definici´ on 2.1.2. Sean U y V espacios vectoriales sobre K y T : U → V una transformaci´on lineal. Llamaremos: 1. N´ ucleo de T o Kernel de T al subconjunto de U definido como: ker T = {u ∈ U : T (u) = 0} 2. Imagen de T al subconjunto de V definido como: ImT = {v ∈ V : ∃ u ∈ U, T (u) = v}

85

Apuntes Mat023 (versi´on preliminar actualizada 23-05-2014)

Observaci´ on 2.1.6. El siguiente teorema indica que el n´ ucleo y la imagen de T poseen estructura de espacio vectorial.

Teorema 2.1.4. Sea T ∈ L (U, V ), entonces: 1. ker T ≤ U . 2. ImT ≤ V

Ejemplo 2.1.12. Hallar el n´ ucleo y la imagen de T : R3 → R3 definida por: T (x, y, z) = (2x − y + z, x − y + z, x) Soluci´ on. Primero determinaremos el n´ ucleo ker (T ) =



(x, y, z) ∈ R3 : T (x, y, z) = (0, 0, 0)

(x, y, z) ∈ R3 : (2x − y + z,   2x − y + z   3 = (x, y, z) ∈ R : x − y + z    x

=



x − y + z, x) = (0, 0, 0)  = 0    = 0 = 0

  

0

0

note que 

2 −1 1 0





1 0



     1 −1 1 0  ∼  0 1 −1 0      1 0 0 0 0 0 0 0 luego  

 x = 0  ker (T ) = (x, y, z) ∈ R3 :  y−z = 0   = (0, y, y) ∈ R3 : y ∈ R = h{(0, 1, 1)}i

86

Apuntes Mat023 (versi´on preliminar actualizada 23-05-2014)

un espacio de dimensi´on 1. Determinemos la imagen ImT =



(a, b, c) ∈ R3 : ∃ (x, y, z) ∈ R3 , T (x, y, z) = (a, b, c)

(a, b, c) ∈ R3 : ∃ (x, y, z) ∈ R3 , (2x − y + z,   2x − y + z   3 3 = (a, b, c) ∈ R : ∃ (x, y, z) ∈ R , x − y + z    x =



x − y + z, x) = (a, b, c)  = a   



= b

= c

  

en otras palabras, para que valores de a, b, c ∈ R el sistema 2x − y + z = a x−y+z

= b

x

= c

tiene soluci´on, notemos que    2 −1 1 a 2 −1 1 a     1 −1 1 b  ∼  0 −1 1 2b − a    1 0 0 c 0 0 0 −c − b + a

   

luego el sistema tiene soluci´on si y solo si −c − b + a = 0 se sigue (a, b, c) ∈ R3 : −c − b + a = 0  = (b + c, b, c) ∈ R3 : b, c ∈ R

ImT =



= h{(1, 1, 0) , (1, 0, 1)}i es un espacio de dimensi´on 2. Definici´ on 2.1.3. Sea T ∈ L (U, V ) con U y V espacios de dimensi´on finita, entonces: 1. Llamaremos nulidad de T , denotado η (T ), al n´ umero definido como: η (T ) = dim ker T

87

Apuntes Mat023 (versi´on preliminar actualizada 23-05-2014)

2. Llamaremos rango de T , denotado ρ (T ) al n´ umero definido como: ρ (T ) = dim ImT

Teorema 2.1.5. Sean U, V un espacios vectoriales sobre K, u1 , u2 , . . . , un una base para U y T ∈ L (U, V ), entonces: ImT = h{T (u1 ) , T (u2 ) , . . . , T (un )}i Demostraci´on. Suponga que v ∈ ImT entonces, por definici´on, existe un u ∈ U tal que T (u) = v, como {u1 , u2 , . . . , un } es una base de U existir´an escalares α1 , α2 , . . . , αn tales que u=

n X

αi ui

i=1

se sigue que v = T (u) = T

n X

! αi ui

=

i=1

n X

αi T (ui )

i=1

es decir, v es una combinaci´on lineal de los elementos T (u1 ) , T (u2 ) , . . . , T (un ), luego {T (u1 ) , T (u2 ) , . . . , T (un )} genera ImT .

Ejemplo 2.1.13. Si T : M2×2 (R) → M2×2 (R)        x y x y 1 1 x y  →T =   z w z w 1 1 z w sabemos que  

1 0 0 0

  ,

0 1 0 0

  ,

88

0 0 1 0





 y 

0 0 0 1

 

Apuntes Mat023 (versi´on preliminar actualizada 23-05-2014)

forman una base de M2×2 (R) luego    1 0  =  T 0 0    0 1  =  T 0 0    0 0  =  T 1 0    0 0  =  T 0 1 generan la imagen, se sigue *   ImT =  *   = 

1 0 1 0 1 0 1 0

1 1 1 1 1 1 1 1 1 1 1 1 1 1 1 1

  ,   ,

       

0 1 0 1 0 1 0 1

1 0 0 0 0 1 0 0 0 0 1 0 0 0 0 1





= 



= 



= 



=

1 0 1 0 0 1 0 1 1 0 1 0 0 1 0 1

       

+ 1 0 0 1  , ,  1 0 0 1  +     

 

de donde obtenemos ρ (T ) = 2. Ejemplo 2.1.14. Demuestre que existe una transformaci´on lineal T : M2 (R) → R3 tal que: ImT = {(x, y, z) : x − 2y + z = 0} y la nulidad de T sea 2. Soluci´ on. Notemos que ImT = {(x, y, z) : x − 2y + z = 0} = {(x, y, z) : x = 2y − z} = {(2y − z, y, z) : y, z ∈ R} = h{(2, 1, 0) , (−1, 0, 1)}i este espacio tiene dimensi´on 2. Si necesitamos que la nulidad sea 2 entonces el N´ ucleo debe

89

Apuntes Mat023 (versi´on preliminar actualizada 23-05-2014)

estar generado por dos elementos L. I.  1 T 0  0 T 0  0 T 1  0 T 0

Definamos  0  = (2, 1, 0) 0  1  = (−1, 0, 1) 0  0  = (0, 0, 0) 0  0  = (0, 0, 0) 1

existe una u ´ nica T : M2 (R) → R3 que cumple lo anterior, para esta transformaci´on se tiene:        + *   1 0 0 1 0 0 0 0          ImT = T ,T ,T ,T  0 0 0 0 1 0 0 1  = h{(2, 1, 0) , (−1, 0, 1)}i Note que  T

a b c d

  = (2a − b, a, b)

demuestre que el N´ ucleo tiene dimensi´on 2.. Observaci´ on 2.1.7. El siguiente resultado relaciona la nulidad y el rango de una transformaci´on lineal: Teorema 2.1.6. Sean U y V espacios vectoriales sobre K con dim U < ∞. Considere T ∈ L (U, V ), entonces: dim U = η (T ) + ρ (T ) Demostraci´on. Supongamos que dim U = n. Sea u1 , u2 , . . . , ur (r ≤ n) una base del n´ ucleo de T . Considere ur+1 , ur+2 , . . . , un una completaci´on de la base u1 , u2 , . . . , ur del n´ ucleo de T al espacio vectorial U . Luego, si v ∈ U , entonces: v=

n X i=1

90

αi ui

Apuntes Mat023 (versi´on preliminar actualizada 23-05-2014)

As´ı: n X

T (v) = T

! αi ui

i=1 n X

=

αi T (ui )

i=1 n X

=

αT (ui )

i=r+1

y por tanto, T (ur+1 ) , T (ur+2 ) , . . . , T (un ) generan la imagen de T . Basta verificar, ahora, que son linealmente independientes. En efecto, supongamos que: n−r X

αi T (ur+i ) = 0

i=1

Luego: n−r X

T

! αi ur+i

=0

i=1

es decir, si w =

Pn−r i=1

αi ur+i , se tiene entonces que w ∈ ker T . Por tanto, existen escalares

β1 , β2 , . . . , βr ∈ K tales que: w=

r X

βi ui

i=1

As´ı:

r X

βi ui =

i=1

O bien:

r X i=1

βi ui −

n−r X

αi ur+i

i=1 n−r X

αi ur+i = 0

i=1

Ahora bien, como u1 , u2 , . . . , ur , ur+1 , . . . , un es una base de U se tiene que: β1 = β2 = · · · = βr = α1 = α2 = · · · = αn−r = 0 As´ı, T (ur+1 ) , T (ur+2 ) , . . . , T (un ) es una base para la imagen de T . Por tanto, η (T ) = r y ρ (T ) = n − r.

Definici´ on 2.1.4. Sea T ∈ L (U, V ). Diremos que:

91

Apuntes Mat023 (versi´on preliminar actualizada 23-05-2014)

1. T es inyectiva si: T (u) = T (v) =⇒ u = v para todos u, v ∈ U . 2. T es epiyectiva si: ImT = V 3. T es biyectiva si es inyectiva y epiyectiva, simult´aneamente.

Ejemplo 2.1.15. No existe transformaci´on lineal T : R2 → R3 que sea sobreyectiva. En efecto, si T es sobreyectiva ImT = R3 luego ρ (T ) = 3 pero del teorema de las dimensiones  η (T ) + ρ (T ) = Dim R2 = 2 como ρ (T ) = 3 se obtiene η (T ) = −1 esto es una contradicci´on pues η (T ) ≥ 0. Teorema 2.1.7. Sea T ∈ L (U, V ). Entonces, T es inyectiva, si y solo si, ker T = {θU }. Demostraci´on. Supongamos que ker T = {θU }. Sean u, v ∈ U tales que T (u) = T (v) . Como T es una transformaci´on lineal, se tiene T (u − v) = θV , es decir, u − v ∈ ker T . As´ı, u − v = θU , y por tanto, T es inyectiva. Por otro lado, supongamos que T es inyectiva, como {θU } ⊆ ker T , bastar´a verificar que ker T ⊆ {0}. En efecto, sea u ∈ ker T , esto es T (u) = θV y como T es transformaci´on lineal, se tiene que T (θU ) = θV . Es decir, T (u) = T (θU ), luego u = θU , pues T es inyectiva. As´ı, ker T = {θU }.

Isomorfismo Definici´ on 2.1.5. Dos espacios vectoriales U y V sobre K se dicen isomorfos si existe una transformaci´on lineal T : U → V biyectiva. Tal transformaci´on T biyectiva se llamar´a isomorfismo entre U y V . El concepto de isomorfismo entre dos espacios se representar´a por: U 'V. 92

Apuntes Mat023 (versi´on preliminar actualizada 23-05-2014)

Ejemplo 2.1.16. Sea U un espacio vectorial sobre R tal que u1 , u2 , . . . , un es una base de U , entonces L : Rn → U definida por: (x1 , x2 , . . . , xn ) 7→ L (x1 , x2 , . . . , xn ) =

n X

xi ui

i=1

es un isomorfismo entre U y Rn , esto se debe a que (x1 , x2 , . . . , xn ) ∈ ker T



n X

xi ui = θU

i=1

⇔ ∀i, xi = 0 (pues u1 , u2 , . . . , un es una base de U ) as´ı (x1 , x2 , . . . , xn ) ∈ ker T ⇔ (x1 , x2 , . . . , xn ) = (0, 0, . . . , 0) luego L es inyectiva y por el teorema de las dimensiones n = dim Rn = 0 + ρ (L) as´ı ρ (L) = n esto implica ImT = U . Teorema 2.1.8. Sean U y V espacios vectoriales de dimensi´on finita sobre un cuerpo K. Entonces, U ' V si y solo si dim U = dim V . Ejemplo 2.1.17. M2 (R) ' R3 [x] ' R4 Ejemplo 2.1.18. Kmn ' Mm×n (K)

Matriz asociada a una transformaci´ on lineal Definici´ on 2.1.6. Sean U un espacio vectorial sobre K y B = {u1 , u2 , . . . , un } una base de U . Diremos que B es una base ordenada de U , si se considera B como la sucesi´on finita de vectores en U : B = (u1 , u2 , . . . , un )

93

Apuntes Mat023 (versi´on preliminar actualizada 23-05-2014)

Definici´ on 2.1.7. Sea B = {u1 , u2 , . . . , un } una base ordenada de un espacio vectorial U y sea u ∈ U . Los coeficientes αi en la combinaci´on u = α1 u1 + α2 u2 + · · · + αn un son llamadas coordenadas de u con respecto a B y desde ahora usaremos la notaci´on   α1    α   2  [u]B =  .   ..    αn Observaci´ on 2.1.8. Estos escalares existen y son u´nicos por la definici´on de base, el orden de los elementos de la base es importante.

Ejemplo 2.1.19. Calcular las coordenadas del vector −1 + 2x + 3x2 en la base de R2 [x] dada por B = {1 + x, x2 , 1} . Soluci´ on. Tenemos que encontrar escalares α, β, γ tales que −1 + 2x + 2x2 = α (1 + x) + βx2 + γ1 desarrollando −1 + 2x + 3x2 = (α + γ) + αx + βx2 as´ı −1 = α + γ 2 = α 3 = β se sigue γ = −3 entonces  −1 + 2x + 3x2 = 2 (1 + x) + 3 x2 − 3 (1) de donde obtenemos 

2



     −1 + 2x + 3x2 B =  3   −3 94

Apuntes Mat023 (versi´on preliminar actualizada 23-05-2014)

Observaci´ on 2.1.9. Dado en vector de coordenadas [v]B y conocida la base B se puede recuperar el vector v. Ejemplo 2.1.20. Si el determinado vector v en M2×2 (R) tiene vector de coordenadas   1    −1    [v]B =    2    −2          1 1 1 1 1 1 1 0  , , ,  entonces en la base B =   1 1 1 0 0 0 0 0   v = 1

1 1





 + (−1) 

1 1   0 2  =  0 1

1 1 1 0





 + 2

1 1 0 0





 + (−2) 

1 0 0 0

 

Observaci´ on 2.1.10. Sean U y V espacios vectoriales sobre K tales que B = {u1 , u2 , . . . , un } y D = {v1 , v2 , . . . , vm } son bases ordenadas de U y V , respectivamente. Note que para cada i = 1, 2, . . . , n se tiene que T (ui ) ∈ V , por tanto, existen escalares Aji ∈ K tales que: T (ui ) = A1 i v1 + A2 i v2 + · · · + Am i vm =

m X

Aji vj ,

∀i = 1, 2, . . . , n

j=1

Es decir, los escalares A1 i , A2 i , . . . , Am i son las coordenadas de T (ui ) en la base D. As´ı:   A1 i    A   2i  [T (ui )]D =  .  , ∀i = 1, 2, . . . , n  ..    Am i Definici´ on 2.1.8. Con respecto a las notaciones de la observaci´on anterior, se define la matriz de T respecto de las bases B y D como la matriz A tal que A = (Ai j ) ∈ Mm×n (R). Esto es:

95

Apuntes Mat023 (versi´on preliminar actualizada 23-05-2014)

[T ]D B

=



[T (u1 )D ] [T (u2 )D ] · · · [T (un )D ]   A11 A12 · · · A1n    A  A · · · A 21 22 2n   =  . .. ..  ..  .. . . .    Am1 Am2 · · · Amn



Ejemplo 2.1.21. Dada la transformaci´on lineal T : R3 → R2 definida por: T (x, y, z) = (3x − 2y + z, x − y) Determine la matriz asociada a T respecto de las bases: 1. can´onicas de R3 y R2 , respectivamente. Soluci´ on. T (1, 0, 0) = (3, 1) = 3 (1, 0) + 1 (0, 1) T (0, 1, 0) = (−2, −1) = −2 (1, 0) − 1 (0, 1) T (0, 0, 1) = (1, 0) = 1 (1, 0) + 0 (0, 1) as´ı

 C

[T ]CR23 =  R

3 −2 1 1 −1 0

 

2. las bases B = {(1, 1, 0) ; (1, 0, 1) ; (1, 2, 3)} y D = {(2, 1) ; (−1, 1)} Soluci´ on. T (1, 1, 0) = (3 − 2, 1 − 1) = (1, 0) = α1 (2, 1) + β 1 (−1, 1) T (1, 0, 1) = (3 + 1, 1) = (4, 1) = α2 (2, 1) + β 2 (−1, 1) T (1, 2, 3) = (3 − 4 + 3, 1 − 2) = (2, −1) = α3 (2, 1) + β 3 (−1, 1) determinemos los escalares, los determinaremos en un solo sistema     5 1 1 2 −1 1 4 2 1 0 3 3 3   ∼ 1 2 0 1 − 3 − 3 − 43 1 1 0 1 −1 96

Apuntes Mat023 (versi´on preliminar actualizada 23-05-2014)

se sigue   [T ]D B =

1 3

5 3



1 3

− 31 − 23 − 43



Ejemplo 2.1.22. Sea D : R3 [x] → R3 [x] el operador lineal derivada, es decir, p → D (p) = p0 y considere las bases de R3 [x] dadas por 2x3 , 3x − x2 , 1 − x, 1  = 1, x, x2 , x3 

B1 = B2 luego D 2x3



= 0 · 1 + 0x + 6x2 + 0x3

D 3x − x2



= 3 (1) − 2 (x) + 0x2 + 0x3

D (1 − x) = −1 (1) + 0x + 0x2 + 0x3 D (1) = 0 (1) + 0x + 0x2 + 0x3 entonces

 [D]BB21

0

3

  0 −2  =  6 0  0 0

−1 0 0 0 0



 0    0   0

usted puede calcular [D]B1 a que es una matriz diferente. B2 y ver´ Ejemplo 2.1.23. Si B = {v1 , v2 , . . . , vn } es una base del espacio vectorial V y I : V → V la transformaci´on identidad entonces [I]BB = In×n

(la matriz identidad)

En efecto, Iv1 = v1 = 1v1 + 0v2 + 0v3 + · · · + 0vn se sigue   1    0    [Iv1 ]B =  .   ..    0 97

Apuntes Mat023 (versi´on preliminar actualizada 23-05-2014)

similarmente Iv2 = v1 = 0v1 + 1v2 + 0v3 + · · · + 0vn se sigue   0    1       [Iv2 ]B =   0   .   ..    0 as´ı [I]BB es [I]BB = In×n Definici´ on 2.1.9. Sean U un espacio vectorial sobre K tales que B = {u1 , u2 , . . . , un } y D = {v1 , v2 , . . . , vn } son bases ordenadas distintas de U . Sea 1 : U → U la transformaci´on lineal identidad. Se define la matriz cambio de base como la matriz asociada a 1 respecto de las bases B y D.

Teorema 2.1.9. Sean U y V espacios vectoriales finito dimensionales sobre K con bases B y D, respectivamente. Sean, adem´as, S, T : U → V dos transformaciones lineales, entonces: D D [αT + S]D B = α [T ]B + [S]B

Teorema 2.1.10. Sean U, V y W espacios vectoriales de dimensi´ on finita sobre K tales que B, D y E son bases de U, V y W , respectivamente. Suponga que T : U → V y S : V → W son transformaciones lineales, entonces: [ST ]EB = [S]ED · [T ]D B Observaci´ on 2.1.11. Sean U y V espacios vectoriales finito dimensionales sobre K con bases B y E para U , y D y F para V . Considere, adem´as, T : U → V una transformaci´on lineal. Note que en vista del teorema anterior el cambio de representaci´on matricial puede expresarse mediante el siguiente diagrama conmutativo: [T ]F

E UE −→ VF

[1]EB



↓ [T ]D

B UB −→ VD

98

[1]D F

Apuntes Mat023 (versi´on preliminar actualizada 23-05-2014)

donde la notaci´on del tipo UE , por ejemplo, representa el espacio U considerado con la base E. En efecto, utilizando la convenci´on usual para la composici´on de funciones, tenemos que: D F E [T ]D B = [1]F · [T ]E · [1]B

Teorema 2.1.11. Sean U y V espacios vectoriales finito dimensionales sobre K con bases B y D, respectivamente. Sea, adem´as, T : U → V una transformaci´on lineal entonces T es invertible si y solo si [T ]D as B es invertible, adem´  −1 B  D −1 T D = [T ]B Teorema 2.1.12. Sean U y V espacios vectoriales sobre el mismo cuerpo K tales que B = {u1 , u2 , . . . , un } es una base ordenada para U y D = {v1 , v2 , . . . , vm } es una base ordenada para V , entonces: [T u]D = [T ]D B [u]B para todo u ∈ U . Ejemplo 2.1.24. Considere las bases de R3 B1 = {(1, 1, 1) , (1, 1, 0) , (1, 0, 0)} y B2 = {(0, 0, 1) , (1, 0, 0) , (0, 1, 0)} 1. Calcular [I]BB21 y [I]BB12 . 2. Calcular [v]Bi para i = 1, 2 donde v = (3, 1, 3) 1. Notemos que I (1, 1, 1) = (1, 1, 1) = 1 (0, 0, 1) + 1 (1, 0, 0) + 1 (0, 1, 0) I (1, 1, 0) = (1, 1, 0) = 0 (0, 0, 1) + 1 (1, 0, 0) + 1 (0, 1, 0) I (1, 0, 0) = (1, 0, 0) = 0 (0, 0, 1) + 1 (1, 0, 0) + 0 (0, 1, 0)

99

Apuntes Mat023 (versi´on preliminar actualizada 23-05-2014)

entonces 

−1  = [I]BB21 como [I]BB12 

1 0 0



   [I]BB21 =  1 1 1   1 1 0 −1  se sigue que [I]BB12 = [I]BB21

1 0 0 1 0 0





1 0 0

1

0

0



     1 1 1 0 1 0  ∼  0 1 0 −1 0 1      1 1 0 0 0 1 0 0 1 0 1 −1 as´ı 

1

0

0



   [I]BB12 =   −1 0 1  0 1 −1 2. Notemos que (3, 1, 3) = 3 (0, 0, 1) + 3 (1, 0, 0) + 1 (0, 1, 0)   3    [(3, 1, 3)]B2 =   3  1 luego [(3, 1, 3)]B1 = [I]BB12 [(3, 1, 3)]B2    3 1 0 0      =   −1 0 1   3  0 1 −1 1   3    =  −2   2 Ejemplo 2.1.25. Sean B = {(1, 1, 1) ; (0, 0, 1) ; (1, 0, 1)} y D = {(1, 1) ; (0, 1)} bases de R3 y R2 , respectivamente. Sea T una transformaci´on lineal tal que:   4 2 −2   [T ]D B = −3 −1 1 Hallar expl´ıcitamente T (x, y, z). 100

Apuntes Mat023 (versi´on preliminar actualizada 23-05-2014)

Soluci´ on. Primero buscamos [(x, y, z)]B esto es (x, y, z) = α (1, 1, 1) + β (0, 0, 1) + γ (1, 0, 1) luego 

1 0 1 x





1 0 0

y



     1 0 0 y ∼ 0 1 0 z−x      1 1 1 z 0 0 1 x−y as´ı 

y



   [(x, y, z)]B =  z − x   x−y se sigue que [T (x, y, z)]D = [T ]D B [(x, y, z)]B    y 4 2 −2   z − x =   −3 −1 1 x−y   6y − 4x + 2z  =  2x − 4y − z

   

de donde obtenemos T (x, y, z) = (6y − 4x + 2z) (1, 1) + (2x − 4y − z) (0, 1) = (6y − 4x + 2z, 2y − 2x + z)

C´ alculo con coordenadas Observaci´ on 2.1.12. Como se puede observar de la secci´on anterior, las matrices asociadas a las transformaciones lineales dependen de las coordenadas. Es decir, dependen de las bases consideradas en los espacios vectoriales involucrados. Por consiguiente, el c´alculo de los subespacios notables (n´ ucleo e imagen) asociados a una transformaci´on lineal no es directo por definici´on. Sin embargo, con los procedimientos algebraicos adecuados se pueden obtener los espacios notables asociados a las transformaciones a trav´es de c´alculos con coordenadas. Veamos algunos ejemplos: 101

Apuntes Mat023 (versi´on preliminar actualizada 23-05-2014)

Ejemplo 2.1.26. Sean: B = {(1, 1, 1) ; (0, 1, 1) ; (1, 0, 1)} y: D = {(1, 1, 1) ; (1, 0, 1) ; (1, 2, 3)} bases de R3 y T ∈ L (R3 , R3 ) tales que: 

4

2

−2



   −3 −1 1  [T ]D = B   1 2 −2 Calcule el n´ ucleo y la imagen de T , sin explicitar T (x, y, z). ucleo de T , es decir T (u) = 0. Por tanto, Soluci´ on. Sea u un vector cualquiera en el n´ [T · u]D = [0]D = 0. Sin embargo,  4   −3  1 en donde [u]B =



a b c

T



sabemos que [T · u]D = [T ]D B [u]B , luego:     2 −2 a 0         −1 1   b  =  0  2 −2 c 0

. Escalonando la matriz de coeficientes: 4

2

−2





1 0

0



     −3 −1 1  →  0 1 −1      1 2 −2 0 0 0 se obtiene que a = 0 y b = c. Por lo tanto: 

0



   [u]B =   c , c

c∈R

Luego: u = 0 · (1, 1, 1) + c · (0, 1, 1) + c · (1, 0, 1) = (c, c, 2c) , Por lo tanto, obtenemos de lo anterior que: ker T = h (1, 1, 2) i 102

c∈R

Apuntes Mat023 (versi´on preliminar actualizada 23-05-2014)

Para calcular la imagen de T , debemos notar primeramente que de:   4 2 −2    −3 −1 1  [T ]D = B   1 2 −2 se obtiene que: 

4



  , [T (1, 1, 1)]D =  −3   1



2



  , [T (0, 1, 1)]D =  −1   2



−2

   [T (1, 0, 1)]D =  1   −2

Por consiguiente: T (1, 1, 1) = 4 (1, 1, 1) + (−3) (1, 0, 1) + 1 (1, 2, 3) = (2, 6, 4) T (0, 1, 1) = 2 (1, 1, 1) + (−1) (1, 0, 1) + 2 (1, 2, 3) = (3, 6, 7) T (1, 0, 1) = (−2) (1, 1, 1) + 1 (1, 0, 1) + (−2) (1, 2, 3) = (−3, −6, −7) Ahora bien: ImT = hT (1, 1, 1) , T (0, 1, 1) , T (1, 0, 1)i = h{(2, 6, 4) , (3, 6, 7) , (−3, −6, −7)}i = h{(2, 6, 4) , (3, 6, 7)} i note que el rango de T es 2. Ejemplo 2.1.27. Sean: B = {(1, 1, −1) ; (0, 2, −1) ; (1, 0, 1)} y:  D = 1 + 2x + x2 ; 1 + 3x + 2x2 ; 2 + x + 3x2 bases de R3 y R2 [x], respectivamente. Sea T ∈ L (R3 , R2 [x]) tales que:   5 2 −2    [T ]D 1  B =  −3 −1  1 4 −3 Verifique que T es un isomorfismo entre R3 y R2 [x]. Calcule, adem´as, T −1 . 103



Apuntes Mat023 (versi´on preliminar actualizada 23-05-2014)

Soluci´ on. Para verificar que T es un isomorfismo, basta notar que det [T ]D 6 0. En efecto: B = 5 2 −2 D det [T ]B = −3 −1 1 1 4 −3

=1

Por otro lado, para calcular T −1 , procedemos por coordenadas. En efecto:  −1   B T (a, b, c) B = T −1 D [(a, b, c)]D  −1 = [T ]D [(a, b, c)]D B Luego:  

T −1

B D

5

2

 =   −3 1  −1  =   −8 −11

−2

−1

 1   4 −3  −2 0  −13 1   −18 1

−1

Necesitamos, ahora, las coordenadas de a + bx + cx2 respecto de la base D. Esto es:    α 1 + 2x + x2 + β 1 + 3x + 2x2 + γ 2 + x + 3x2 = a + bx + cx2 Resolviendo el sistema anterior, obtenemos: 

α



     a + bx + cx2 D =   β  γ   1 5 7 a + b − c 4 4   4 1 5  =  4 b − 4 a + 43 c   1 1 1 c + 4a − 4b 4

104

Apuntes Mat023 (versi´on preliminar actualizada 23-05-2014)

Luego:  

T −1 a + bx + cx2

 B

−1

−2

0



   =  −8 −13 1   −11 −18 1   3 3 1 a − b − c 4 4   4 5 11  =  2 a − 2 b + 21 c   15 1 7 a − 2 b + 2c 2

7 a+ 4 1 b− 4 1 c+ 4

1 b 4 5 a 4 1 a 4

− 54 c



 + 43 c   1 − 4b

As´ı: T

−1

a + bx + cx

2





   3 5 3 1 11 1 = a − b − c (1, 1, −1) + a − b + c (0, 2, −1) 4 4 4 2 2 2   15 1 7 a − b + c (1, 0, 1) + 2 2 2   17 33 1 23 47 3 1 5 1 = a − b + c, a − b + c, a − b + c 4 4 4 4 4 4 4 4 4

Ejercicios resueltos de Transformaciones lineales 1. Considere la transformaci´on T : M2×2 (R) → R4     a b a b  → T  = (a + b, a + c − b, c, d)  c d c d a) Determine una base de Ker(T ) e Im(T ).

105

Apuntes Mat023 (versi´on preliminar actualizada 23-05-2014)

Soluci´ on. Vamos a buscar el N´ ucleo de la transformaci´on,       a b  a b  = (0, 0, 0, 0)   ∈ M2×2 (R) : T  ker (T ) =  c d  c d      a b   ∈ M2×2 (R) : (a + b, a + c − b, c, d) = (0, 0, 0, 0) =   c d     a + b = 0            a b a−b+c = 0    ∈ M2×2 (R) : =  c d c = 0           d = 0       a b   ∈ M2×2 (R) : a = b = c = d = 0 =   c d    0 0    =  0 0  del teorema de las dimensiones se sigue que Dim (ker (T )) + Dim (Im (T )) = Dim (M2×2 (R)) 0 + Dim (Im (T )) = 4 as´ı Dim (Im (T )) = 4 y luego Im (T ) = R4 (T es biyectiva) b) Si          1 −1 0 −1 0 1 1 1          B1 = , , ,  2 0 2 1 1 1 0 3  y B2 = {(1, 0, 2, 0) , (1, 0, 2, 1) , (0, 0, 1, 1) , (1, 1, 0, −3)} son bases de M2×2 (R) y R4 respectivamente determinar [T ]BB21 .

106

Apuntes Mat023 (versi´on preliminar actualizada 23-05-2014)

Soluci´ on. Como  T  T

1 −1 2

0

0 −1

2 

T  T

1 0 1 1 1 1 1 0 3

  = (0, 4, 2, 0)   = (−1, 3, 2, 1)   = (1, 0, 1, 1)   = (2, 0, 0, 3)

se sigue que necesitamos resolver los sistemas (0, 4, 2, 0) = α1 (1, 0, 2, 0) + β1 (1, 0, 2, 1) + γ1 (0, 0, 1, 1) + δ1 (1, 1, 0, −3) (−1, 3, 2, 1) = α2 (1, 0, 2, 0) + β2 (1, 0, 2, 1) + γ2 (0, 0, 1, 1) + δ2 (1, 1, 0, −3) (1, 0, 1, 1) = α3 (1, 0, 2, 0) + β3 (1, 0, 2, 1) + γ3 (0, 0, 1, 1) + δ3 (1, 1, 0, −3) (2, 0, 0, 3) = α4 (1, 0, 2, 0) + β4 (1, 0, 2, 1) + γ4 (0, 0, 1, 1) + δ4 (1, 1, 0, −3) y as´ı la matriz asociada ser´a  [T ]BB21

vamos a  1   0    2  0

α1 α2 α3 α4

   β β β β   1 2 3 4  =   γ1 γ2 γ3 γ4    δ1 δ2 δ3 δ4

resolver los 4 sistema de una sola vez   1 0 1 0 −1 1 2 1 0    0 0 1 4 3 0 0    0 1 ∼    2 1 0 2 2 1 0    0 0 0 0 1 1 −3 0 1 1 3

as´ı

 [T ]BB21



0 0 −6 −4 −1 −5 0 0

1 0 10 0 1

−6 −4 −1 −5

  2  =  10  4 107

2 4 

 7    10 −1 −4   3 0 0 0

2



 7    10 −1 −4   3 0 0 0

2

Apuntes Mat023 (versi´on preliminar actualizada 23-05-2014)

c) Si la transformaci´on es un isomorfismo (biyectiva) determine 

T −1

B1 B2

Soluci´ on. ya sabemos que la transformaci´on es biyectiva, como  −1 B1  B2 −1 T B2 = [T ]B1 se sigue  −1



−6 −4 −1 −5    2 0 2 7       10 10 −1 −4    4 3 0 0

3 22

     −2  11  =    23   11    7 − 11  B = T −1 B12

3 − 22

9 − 22

17 11

2 11

6 11

− 19 11

21 11

8 11

4 11

4 − 11

1 − 11

6 − 11

               

2. Construir una transformaci´on lineal T : R3 → R3 que cumpla las siguientes condiciones (simult´aneamente) a) Ker(T ) = {(x, y, z) ∈ R3 : (x, y, z) = t (1, 0, −1) para t ∈ R} b) Im(T ) = {(x, y, z) ∈ R3 : x − 2y + z = 0} Soluci´ on. La transformaci´on tiene que cumplir las dos condiciones, notamos que ker (T ) = h(1, 0, −1)i y (x, y, z) ∈ R3 : x − 2y + z = 0  = (x, y, z) ∈ R3 : x + z = 2y    x+z 3 = x, , z ∈ R : x, z ∈ R 2     1 1 = 1, , 0 , 0, , 1 2 2

Im (T ) =



108

Apuntes Mat023 (versi´on preliminar actualizada 23-05-2014)

desde el punto de vista del teorema de las dimensiones estas dos condiciones no son incompatibles puesto que  Dim (ker (T )) + Dim (Im (T )) = Dim R3 = 3 para la posible transformaci´on. Vamos a utilizar el teorema que nos permite construir transformaciones si la conocemos en una base   1 T (1, 0, 0) = 1, , 0 2   1 T (0, 1, 0) = 0, , 1 2 T (1, 0, −1) = (0, 0, 0) notamos que los vectores (1, 0, 0) , (0, 1, 0) y (1, 0, −1) son linealmente independientes pues 1 0 1 0 1 0 0 0 −1

= −1 6= 0

y luego forman una base de R3 estas condiciones determinan por completo una transformaci´on lineal, note tambi´en que Im (T ) = h{T (1, 0, 0) , T (0, 1, 0) , T (1, 0, −1)}i      1 1 = 1, , 0 , 0, , 1 , (0, 0, 0) 2 2     1 1 = 1, , 0 , 0, , 1 2 2  3 = (x, y, z) ∈ R : x − 2y + z = 0 y (1, 0, −1) ∈ ker (T ) que tiene dimensi´on 1 (por el teorema de las dimensiones) as´ı ker (T ) = h{(1, 0, −1)}i  = (x, y, z) ∈ R3 : (x, y, z) = t (1, 0, −1) para t ∈ R podemos tambi´en determinar en forma expl´ıcita esta transformaci´on (x, y, z) = α (1, 0, 0) + β (0, 1, 0) + γ (1, 0, −1) 109

Apuntes Mat023 (versi´on preliminar actualizada 23-05-2014)

α+γ = x β = y −γ = z de donde α = x + z, β = y, γ = −z as´ı (x, y, z) = (x + z) (1, 0, 0) + y (0, 1, 0) + (−z) (1, 0, −1) luego T (x, y, z) = T ((x + z) (1, 0, 0) + y (0, 1, 0) + (−z) (1, 0, −1)) = (x + z) T (1, 0, 0) + yT (0, 1, 0) + (−z) T (1, 0, −1)     1 1 = (x + z) 1, , 0 + y 0, , 1 + (−z) (0, 0, 0) 2 2   1 1 1 = x + z, x + y + z, y 2 2 2 3. Sea T ∈ L (R3 , R2 ) con B = {(0, 0, 1) ; (0, 2, 1) ; (3, 2, 1)} y C = {(1, −3) ; (2, −5)} bases ordenadas de R3 y R2 , respectivamente. Supongamos que:   −1 3 2  ∈ M2×3 (R) [T ]CB =  4 −12 −8 Con respecto a las hip´otesis anteriores: a) Hallar ker T sin calcular T (x, y, z). Soluci´ on. v ∈ ker (T ) ⇔ [v]B es soluci´on de [T ]CB x = 0 luego     −1 3 2 0 1 −3 −2 0  ∼  4 −12 −8 0 0 0 0 0 luego 

α



   [v]B =   β  γ es soluci´on si y solo si α − 3β − 2γ = 0 110

Apuntes Mat023 (versi´on preliminar actualizada 23-05-2014)

de esta forma 

α





     [v]B =   β = γ

3β + 2γ β

   para β, γ ∈ R 

γ

luego v ∈ ker (T ) ⇔ (x, y, z) = (3β + 2γ) (0, 0, 1) + β (0, 2, 1) + γ (3, 2, 1) para β, γ ∈ R esto es (x, y, z) = (3γ, 2β + 2γ, 4β + 3γ) para β, γ ∈ R as´ı ker (T ) = h(3, 2, 3) , (0, 2, 4)i b) Determinar T (x, y, z). Soluci´ on. Notemos que [T (x, y, z)]C = [T ]CB [(x, y, z)]B luego (x, y, z) = α (0, 0, 1) + β (0, 2, 1) + γ (3, 2, 1) as´ı 3γ = x 2β + 2γ = y α+β+γ = z luego α = z − 21 y, β = 21 y − 13 x, γ = 13 x de donde tenemos 

z − 12 y



  1 1  [(x, y, z)]B =  y − x  2  3 1 x 3

111

Apuntes Mat023 (versi´on preliminar actualizada 23-05-2014)

as´ı 

−1

3

2





z − 12 y



    1y − 1x    2 3 4 −12 −8 1 x 3   2y − 13 x − z   = 4 x − 8y + 4z 3

[T (x, y, z)]C = 

as´ı     4 1 x − 8y + 4z (2, −5) T (x, y, z) = 2y − x − z (1, −3) + 3 3   7 17 = x − 14y + 7z, 34y − x − 17z 3 3 

x



   c) ¿Por qu´e es falso que T (x, y, z) = [T ]CB ·   y ? z Soluci´ on. Esta igualdad no es posible ni por los ordenes de las matrices. d ) ¿Cu´ando es v´alida la igualdad anterior? Soluci´ on. Al considerar las bases can´onicas en los espacios de partida y llegada se cumple una desigualdad del tipo 

x



  onica   T (x, y, z)t = [T ]can´ · can´ onica  y  z (adem´as la transformaci´on este definida de Rn en Rm ) e) Hallar la nulidad de T y el rango de T sin hallar la Im(T )? Soluci´ on. El rango de la transformaci´on lineal es igual al rango de la matriz asociada en este caso es 1. f ) Hallar Im(T ) y una base para Im(T ).

112

Apuntes Mat023 (versi´on preliminar actualizada 23-05-2014)

Soluci´ on. Como ya tenemos la transformaci´on podemos determinar la imagen como Im (T ) = hT (1, 0, 0) , T (0, 1, 0) , T (0, 0, 1)i    7 17 = ,− , (−14, 34) , (7, −17) 3 3 = h(7, −17)i esto es una recta que pasa por el origen con direcci´on (7, −17). 4. Sean U = {(x, y, z) ∈ R3 : 2x − 3y + 5z = 0} un espacio vectorial y A una base ordenada de U y C la base can´onica de R2 . Suponga que T : U → R2 es una transformaci´on lineal definida por T (x, y, z) = (x − y, x − z) tal que:   1 2  ∈ M2×2 (R) [T ]CA =  1 1 Determine la base A. Soluci´ on. Sea A = {(α, β, γ) , (ϕ, κ, λ)} la base de U del ejercicio (el espacio tiene dimensi´on 2) entonces  [T (α, β, γ)]C =   [T (ϕ, κ, λ)]C = 

1 1 2 1

   

adem´as 2α − 3β + 5γ = 0 2ϕ − 3κ + 5λ = 0 pero  [T (α, β, γ)]C =   [T (ϕ, κ, λ)]C =  113

α−β α−γ ϕ−κ ϕ−λ

   

Apuntes Mat023 (versi´on preliminar actualizada 23-05-2014)

de donde tenemos el sistema de ecuaciones α−β = 1 α−γ = 1 ϕ−κ = 2 ϕ−λ = 1 2α − 3β + 5γ = 0 2ϕ − 3κ + 5λ = 0 que tiene soluci´on α = 12 , κ = − 49 , β = − 12 , λ = − 54 , γ = − 12 , ϕ = − 14 as´ı     1 1 1 1 9 5 A= ,− ,− , − ,− ,− 2 2 2 4 4 4 5. Considere la transformaci´on lineal T :

M (R) → R4  2×2    x y x y   → T  = (x − y, x − z, x − w, x) z w z w

y suponga que          1 0 −2 0 0 0 0 −1          B1 = , , ,  0 1  0 0 1 0 0 0 B2 = {(1, 1, 1, 1) , (1, 1, 0, 0) , (1, 0, 0, 0) , (1, 1, 1, 0)} a) Calcular [T ]BB21 , muestre que T es invertible (isomorfismo) y calcule T −1 en forma expl´ıcita. Soluci´ on. Como  T

x y z w

  = (x − y, x − z, x − w, x)

114

Apuntes Mat023 (versi´on preliminar actualizada 23-05-2014)

se sigue 

1 0

T  T

0 1

−2 0 0

 T  T

0

0 0 1 0

0 −1 0

0

  = (1, 1, 0, 1)   = (−2, −2, −2, −2)   = (0, −1, 0, 0)   = (1, 0, 0, 0)

luego tenemos que resolver los sistemas (1, 1, 0, 1) = α1 (1, 1, 1, 1) + β1 (1, 1, 0, 0) + γ1 (1, 0, 0, 0) + δ1 (1, 1, 1, 0) (−2, −2, −2, −2) = α2 (1, 1, 1, 1) + β2 (1, 1, 0, 0) + γ2 (1, 0, 0, 0) + δ2 (1, 1, 1, 0) (0, −1, 0, 0) = α3 (1, 1, 1, 1) + β3 (1, 1, 0, 0) + γ3 (1, 0, 0, 0) + δ3 (1, 1, 1, 0) (1, 0, 0, 0) = α4 (1, 1, 1, 1) + β4 (1, 1, 0, 0) + γ4 (1, 0, 0, 0) + δ4 (1, 1, 1, 0) resolveremos  1 1   1 1    1 0  1 0

los 4 sistemas 1 1 1 −2

0

1





1 0 0 0

1

   0 1 1 −2 −1 0    0 1 0 0 1 ∼  0 1 0 −2 0 0    0 0 1 0 0 0 0 0 1 −1 0 0 1 −2 0 0

se sigue  [T ]BB21

1

  1  =  0  −1

 −1 b) Encontrar [T ]BB21

115

−2 0 0 0

0

0



 −1 0    1 1   0

0

−2 0 0 0

0

0



 −1 0    1 1   0 0

Apuntes Mat023 (versi´on preliminar actualizada 23-05-2014)

Soluci´ on. 

[T ]BB21

−1

B T −1 B12  1 −2   1 0  =   0 0  −1 0  0 0   −1 0  =  2  0 −1  0 1

=



0

0

−1

 −1 0    1 1   0 0  0 −1  0 − 12    0 −1   1 1

c) Considere las bases can´onicas deM2×2 (R) y R4          1 0 0 1 0 0 0 0          C1 = , , ,  0 0 0 0 1 0 0 1  C2 = {((1, 0, 0, 0)) , (0, 1, 0, 0) , (0, 0, 1, 0) , (0, 0, 0, 1)} calcular [T ]CC21 usando la matrices de cambio de base adecuadas. Soluci´ on. Para calcular [T ]CC21 necesitamos hacer [I]CB22 [T ]BB21 [I]BC11 as´ı  α

1 0 0 1





 +β 

de donde tenemos  1 −2 0   0 0 0    0 0 1  1 0 0

−2 0 0

0





 +γ 

0 0 1 0



 +δ 

que resolver 4 sistemas   0 1 0 0 0    −1 0 1 0 0    ∼  0 0 0 1 0    0 0 0 0 1

116



0 −1 0

0

1 0 0 0





=

0

0 1 0 0 − 12 0 0 1 0

0

0 0 0 1

0

0

α − 2β −δ γ

α

0 1



    0 1 0   −1 0 0 0

0

1 2

 

Apuntes Mat023 (versi´on preliminar actualizada 23-05-2014)

de aqu´ı 



0 0 0 1    −1 0 0 1   2  [I]BC11 =  2    0 0 1 0   0 −1 0 0  −1 (note que es mas f´acil pensarlo como [I]CB11 = [I]BC11 si la can´onica esta en la llegada es escribir los vectores como columna)  1 1 1   1 1 0  C2 [I]B2 =   1 0 0  1 0 0

y 1



 1    1   0

as´ı [T ]CC21 = [I]CB22 [T ]BB21 [I]BC11   1 1 1 1    1 1 0 1    =    1 0 0 1    1 0 0 0  1 −1 0 0   1 0 −1 0  =   1 0 0 −1  1 0 0 0

1

−2

1

0

0

0

−1 

0

0

0



0

0

0 1



   −1 0 0 1  −1 0   2 2      0 1 1  0 1 0   0 0 0 −1 0 0

     

6. Considere la transformaci´on lineal T : R2 [x] → R2 [x] dada por:  T ax2 + bx + c = (a + b) x2 + (a + b + c) x + c a) Encuentre una base para ker (T ) b) Encuentre una base para Im (T ) c) Considere en el dominio la base B = {1 + x2 , 1 + x + x2 , −1} y en el codominio la base D = {1 − x, 1 + x, x2 } encontrar la matriz asociada a T respecto a estas bases. 117

Apuntes Mat023 (versi´on preliminar actualizada 23-05-2014)

a) Por definici´on ker (T ) =



 ax2 + bx + c ∈ R2 [x] : T ax2 + bx + c = 0

=



ax2 + bx + c ∈ R2 [x] : (a + b) x2 + (a + b + c) x + c = 0

ax2 + bx + c ∈ R2 [x] : (a + b = 0) ∧ (a + b + c = 0) ∧ (c = 0)         1 1 0 a 0           2     = ax + bx + c ∈ R2 [x] :   1 1 1   b  =  0       0 0 1 c 0

=



luego 



1 1 0



1 1 0



     1 1 1 ∼ 0 0 1      0 0 1 0 0 0 as´ı ax2 + bx + c ∈ R2 [x] : (a + b = 0) ∧ (c = 0)  = ax2 + bx + c ∈ R2 [x] : (b = −a) ∧ (c = 0)  = ax2 − ax ∈ R2 [x] : a ∈ R

 2 = x −x

ker (T ) =



luego el kernel tiene dimensi´on 1 (la nulidad es 1). b) Notemos que T (ax2 + bx + c) = (a + b) x2 + (a + b + c) x + c y luego   (a + b) x2 + (a + b + c) x + c = a x2 + x + b x2 + x + c (x + 1) as´ı   T ax2 + bx + c ∈ x2 + x, x2 + x, x + 1 como hay un elemento repetido, el conjunto es L.D. as´ı



x2 + x, x2 + x, x + 1



=



x2 + x, x + 1

se sigue Im (T ) ⊆



118

x2 + x, x + 1





Apuntes Mat023 (versi´on preliminar actualizada 23-05-2014)

por el teorema de las dimensiones se sigue Dim (ker (T )) + Dim (Im (T )) = Dim (R2 [x]) as´ı, de la parte a) se tiene 1 + Dim (Im (T )) = 3 de donde se obtiene Dim (Im (T )) = 2 y as´ı Im (T ) =



x2 + x, x + 1



de esta forma, una base para la imagen es {x2 + x, x + 1} c) Como T (ax2 + bx + c) = (a + b) x2 + (a + b + c) x + c se sigue que T 1 + x2

= x2 + 2x + 1 = α (1 − x) + β (1 + x) + γx2



= γx2 + (β − α) x + (a + β) por determinar a, β, γ. T 1 + x + x2



= 2x2 + 3x + 1 = α (1 − x) + β (1 + x) + γx2 = γx2 + (β − α) x + (a + β)

por determinar a, β, γ. T (−1) = −x − 1 = α (1 − x) + β (1 + x) + γx2 = γx2 + (β − α) x + (a + β) por determinar a, β, γ. Vamos a resolver esos 3 problemas de una sola vez, porque hay que resolver los sistemas γ = 1 (β − α) = 2 (a + β) = 1 119

Apuntes Mat023 (versi´on preliminar actualizada 23-05-2014)

y γ = 2 (β − α) = 3 (a + β) = 1 y γ = 0 (β − α) = −1 (a + β) = −1 en todas la matriz de coeficientes es la misma, formamos una tripe matriz ampliada  0   −1  1

y escalonamos 0 1 1 2

0





1 0 0 − 21 −1

 Operaciones Elementales   0 1 0 ∼ 1 0 2 3 −1    1 0 1 1 −1 0 0 1    [T ]D B = 

− 12 −1

0

2

1

2



 −1   0



 −1   1 2 0  (notar que T (1 + x2 ) = x2 + 2x + 1 = − 12 (1 − x) + 3 2

3 2

0

2

3 2



(1 + x) + 1x2 ,

T (1 + x + x2 ) = 2x2 + 3x + 1 = (−1) (1 − x) + 2 (1 + x) + 2x2 y T (−1) = −x − 1 = 0 (1 − x) + (−1) (1 + x) + 0x2 ) 7. Suponga que S : R2 [x] → R2 [x] es una transformaci´on lineal con   − 21 −1 0    3  [T ]D = 2 −1 B  2  1 2 0 B = {1 − x, 1 + x, x2 } y D = {1 + x2 , 1 + x + x2 , −1} (No es el ejercicio anterior ver las bases) a) Encontrar ker (T ) 120

Apuntes Mat023 (versi´on preliminar actualizada 23-05-2014)

b) Encontrar Im (T ) c) Calcule T (2 + x − x2 ) d ) Encontrar una expresi´on para T (ax2 + bx + c) a) Note que p ∈ ker (T ) si y solo si T (p) = 0 si  0  [T p]D =   0 0

y solo si    

pero [T p]D = [T ]D B [p]B luego si resolvemos  0   0  0





   = [T ]D [p] =  B B  

− 21 −1 3 2

2

1

2

0



se sigue que p ∈ ker (T ) si y solo si 

α





1 0 −2

  ∼ 0 1   0 0



   β  = [p] B   γ cumple α = 2γ y β = −γ 121



    −1   β  γ 0

estaremos encontrando las coordenadas   α    β  = [p] B   γ de los vectores de ker (T ). Pues bien  − 12 −1 0   3 2 −1  2 1 2 0

α



 1   0

Apuntes Mat023 (versi´on preliminar actualizada 23-05-2014)

as´ı p ∈ ker (T ) si y solo si p (x) = (2γ) (1 − x) + (−γ) (1 + x) + γx2 para alg´ un γ ∈ R es decir ker (T ) =



−3x + x2 + 1



b) De la matriz asociada tenemos 

− 21

 [T (1 − x)]D =  

   as´ı 

3 2

1       1 3 2 T (1 − x) = − 1+x + 1 + x + x2 + (1) (−1) 2 2 3 = x2 + x 2 tambi´en 

−1



   as´ı [T (1 + x)]D =  2   2   T (1 − x) = (−1) 1 + x2 + (2) 1 + x + x2 + (2) (−1) = x2 + 2x − 1 y  

0



   −1  as´ı =   D 0    2 T x = (0) 1 + x2 + (−1) 1 + x + x2 + (0) (−1)

T x2



= −x2 − x − 1 luego  Im (T ) =

 3 2 2 x + x, x + 2x − 1, −x − x − 1 2 2

122

Apuntes Mat023 (versi´on preliminar actualizada 23-05-2014)

 del teorema de las dimensiones x2 + 32 x, x2 + 2x − 1, −x2 − x − 1 deber´ıa ser un conjunto L.D. en efecto     1 1 −1 1 0 −2      3 2 −1  ∼  0 1 1   2    0 −1 −1 0 0 0    3 2 (−2) x + x + (1) x2 + 2x − 1 = −x2 − x − 1 2 se sigue  Im (T ) =

 3 2 x + x, x + 2x − 1 2 2

c) Para calcular T (2 + x − x2 ) podemos hacer lo siguiente  

2 + x − x2

 B

α



   =  β  γ

donde 2 + x − x2 = α (1 − x) + β (1 + x) + γx2 γ = −1 β−α = 1 α+β = 2 α = 21 , β = 23 , γ = −1 luego  

T 2 + x − x2

 D

 = 

− 21 −1 3 2

2

1

2

0



1 2 3 2





    = −1     0 −1

− 74



19 4 7 2

  

se obtiene T 2+x−x

2



        7 19 7 2 2 = − 1+x + 1+x+x + (−1) 4 4 2 19 1 = 3x2 + x − 4 2

123

Apuntes Mat023 (versi´on preliminar actualizada 23-05-2014)

d ) Encontremos una expresi´on para T ax2 + bx + c



notemos que ax2 + bx + c = α (1 − x) + β (1 + x) + γx2 si y solo si γ = a β−α = b α+β = c esto es 

0

0 1 a





1 0 0

    −1 1 0 b  ∼  0 1 0    0 0 1 1 1 0 c

1 c 2 1 b 2

− 21 b



 + 12 c   a

luego 2



ax + bx + c =

   1 1 1 1 c − b (1 − x) + b + c (1 + x) + ax2 2 2 2 2

podemos encontrar T (ax2 + bx + c) en dos formas       1 1 1 1 2 c − b T (1 − x) + b + c T (1 + x) + aT x2 T ax + bx + c = 2 2 2 2       1 1 3 1 1 2 c− b x + x + b + c x2 + 2x − 1 = 2 2 2 2 2  2 +a −x − x − 1     1 7 1 1 2 = (c − a) x + −a + b + c x + − b − c − a 4 4 2 2 la otra forma es  

T ax2 + bx + c

 D

 =     =   124

− 12 −1

0



  −1   1 2 0  − 14 b − 34 c  1 7  b − a + c 4 4  1 3 b + 2c 2 3 2

2

1 c 2 1 b 2

 − 12 b  + 21 c a

   

Apuntes Mat023 (versi´on preliminar actualizada 23-05-2014)

y as´ı 2

T ax + bx + c



     1 3 7 1 2 b − a + c 1 + x + x2 = − b− c 1+x + 4 4 4 4   1 3 + b + c (−1) 2 2     1 7 1 1 2 = (c − a) x + −a + b + c x + − b − c − a 4 4 2 2 

8. Muestre que T : M2×2 (R) → R3 [x] dada por   a b  = (a + b) x3 + cx2 + (a + d) x + (c − d) T  c d es una transformaci´on lineal adem´as determine el ker (T ) e Im(T ). Soluci´ on. En general, si V y W son espacios vectoriales, una funci´on F : V → W es llamada transformaci´on lineal si cumple: i) ∀α ∈ K,∀v ∈ V , F (αv) = αF (v) y ii) ∀v1 , v2 ∈ V , F (v1 + v2 ) = F (v1 ) + F (v2 ) (esto quiere decir, que env´ıa combinaciones lineales en combinaciones lineales). En nuestro ejercicio los elementos del espacio de partida son matrices de orden 2 × 2 luego para probar la primera propiedad necesitamos considerar un escalar arbitrario y una matriz arbitraria y en la segunda propiedad, dos matrices arbitrarias como sigue:   a b  ∈ M2×2 (R) se tiene i) Sea α ∈ R y  c d   T α 

a b c d





 = T 

αa αb



 αc αd   A B  donde A = αa, B = αb, C = αc, D = αd = T  C D = (A + B) x3 + Cx2 + (A + D) x + (C − D)

= ((αa) + (αb)) x3 + (αc) x2 + (αa + αd) x + (αc − αd)  = α (a + b) x3 + cx2 + (a + d) x + (c − d)   a b  = αT  c d 125

Apuntes Mat023 (versi´on preliminar actualizada 23-05-2014)

(Explicaci´on: tenemos que mostrar que el escalar se puede “sacar”  de la funci´  on a b  = y para esto utilizamos solamente la definici´on, en la linea T α  c d   αa αb  no sabemos como act´ T  ua la funci´on T sobre el producto escalar αc αd   a b  es por eso que aplicamos la definici´on de producto escalar de matrices y α c d as´ı     a b αa αb =  α αc αd c d   αa αb  (en este caso si y ahora podemos aplicar la definici´on de T a la matriz  αc αd sabemos como act´ ua T ), para que se vea mas claro agregu´e la linea     αa αb A B  =  donde A = αa, B = αb, C = αc, D = αd αc αd C D pero no es necesario, entonces por definici´on   A B  T  C D = (A + B) x3 + Cx2 + (A + D) x + (C − D) y ahora reemplazamos para obtener la igualdad deseada)     a11 a12 b b  ,  11 12  ∈ M2×2 (R) matrices arbitrarias, se tiene ii) Sean  a21 a22 b21 b22  T 

a11 a12 a21 a22





+

b11 b12 b21 b22





 = T 

a11 + b11 a12 + b12

a21 + b21 a22 + b22  A B  = T  C D

 



donde A = a11 +b11 , B = a12 +b12 , C = a21 +b21 , D = a22 +b22 , aplicando la definici´on

126

Apuntes Mat023 (versi´on preliminar actualizada 23-05-2014)

de T se sigue   A B  = (A + B) x3 + Cx2 + (A + D) x + (C − D) T  C D = ((a11 + b11 ) + (a12 + b12 )) x3 + (a21 + b21 ) x2 + ((a11 + b11 ) + (a22 + b22 )) x + ((a21 + b21 ) − (a22 + b22 )) = ((a11 + a12 ) + (b11 + b12 )) x3 + (a21 + b21 ) x2 + ((a11 + a22 ) + (b11 + b22 )) x + ((a21 − a22 ) + (b21 − b22 ))  = (a11 + a12 ) x3 + a21 x2 + (a11 + a22 ) x + (a21 − a22 ) +  (b11 + b12 ) x3 + b21 x2 + (b11 + b22 ) x + (b21 − b22 )     b11 b12 a11 a12   + T   = T  b21 b22 a21 a22 de esto concluimos  T   = T 

a11 a12 a21 a22 a11 a12 a21 a22





+

b11 b12

 

b21 b22   b11 b12  + T   b21 b22 

de i) y ii) T es una transformaci´on lineal. Vamos a buscar el ker (T ) ker (T ) = {v ∈ V   a  =  c   a  =  c   a  =  c

: T (v) = θW }      b a b  ∈ M2×2 (R) : T   = 0 (el polinomio 0)  d c d    b  ∈ M2×2 (R) : (a + b) x3 + cx2 + (a + d) x + (c − d) = 0  d    b  ∈ M2×2 (R) : (a + b) = 0 ∧ c = 0 ∧ (a + d) = 0 ∧ (c − d) = 0  d

127

Apuntes Mat023 (versi´on preliminar actualizada 23-05-2014)

 de esta forma buscamos todas la matrices 

a b c d

  tales que

a+b = 0 c = 0 a+d = 0 c−d = 0 o matricialmente



1 1   0 0    1 0  0 0

0





a       1 0   b     =     0 1   c   1 −1 d

resolvemos el sistema escalonando  1 1 0 0   0 0 1 0    1 0 0 1  0 0 1 −1

0



0





1

1

0

0



 0    0   0

0

0



    0 −1 0 1 0  0      ∼   0   0 0 1 0 0   0 0 0 −1 0 0

de esto se obtiene a = b = c = d = 0 es decir    0 0   ker (T ) =   0 0  (se concluye que la transformaci´on es inyectiva) Por el teorema de las dimensiones se tiene lo siguiente Dim (ker (T )) + Dim (Im (T )) = Dim (M2×2 (R)) = 4 pero Dim (ker (T )) = 0 as´ı Dim (Im (T )) = 4

128

Apuntes Mat023 (versi´on preliminar actualizada 23-05-2014)

como Dim (T ) < R3 [x] y R3 [x] tiene dimensi´on 4, se sigue Im (T ) = R3 [x]. Otra forma para determinar la imagen es utilizar el resultado que la im´agenes de una base generan la imagen de la transformaci´on, luego       + *   1 0 0 1 0 0 0 0  ,T  ,T  ,T   Im (T ) = T  0 0 0 0 1 0 0 1  pero notemos que  T

1 0 0 0

  = (1 + 0) x3 + 0x2 + (1 + 0) x + (0 − 0) = x3 + x

 T

0 1 0 0

  = (0 + 1) x3 + 0x2 + (0 + 0) x + (0 − 0) = x3

 T

0 0 1 0

  = (0 + 0) x3 + 1x2 + (0 + 0) x + (1 − 0) = x2 + 1

 T

0 0 0 1

  = (0 + 0) x3 + 0x2 + (0 + 1) x + (0 − 1) = x−1

as´ı Im (T ) =



x3 + x, x3 , x2 + 1, x − 1

y mostrar que {x3 + x, x3 , x2 + 1, x − 1} es un conjunto L.I. luego Im (T ) tiene dimensi´on 4 y por tanto tiene que ser R3 [x]. 9. Encontrar por lo menos una transformaci´on lineal T : M2×2 (R) → R3 tal que     a b    ker (T ) = ∈ M2×2 (R) : a + d + c = 0  c d  129

Apuntes Mat023 (versi´on preliminar actualizada 23-05-2014)

Soluci´ on. Notemos que      a b   ∈ M2×2 (R) : a + d + c = 0 ker (T ) =   c d      a b   ∈ M2×2 (R) : a = −c − d =   c d      −c − d b   ∈ M2×2 (R) : b, c, d ∈ R =   c d           −1 0 0 1 −1 0  + b  + d  ∈ M2×2 (R) : b, c, d ∈ R c =   0 0 0 1 1 0     + *  −1 0 0 1 −1 0  , ,    0 0 0 1  1 0 luego ker (T ) tiene dimensi´on 3 (no puede ser base de M2×2 (R) que tiene dimensi´on 4), sabemos que para conocer completamente una T.L. necesitamos conocerla sobre una base, buscamos alg´ un elemento L.I. con las tres matrices que forman el kernel, para ello,  α

−1 0 1

0





+β

0 1 0 0





+γ

−1 0 0

1





+δ

a b c d





=

debe tener por u ´nica soluci´on α = β = γ = δ = 0. Esto se traduce a −α − γ + aδ = 0 β + bδ = 0 α + δc = 0 γ + δd = 0 matricialmente la ampliada  −1 0 −1   0 1 0    1 0 0  0 0 1

a 0





−1 0 −1

   b 0    0 ∼    c 0    0 d 0

0 130

a

0



 0    0 −1 a+c 0   0 0 a+c+d 0 1

0

b

0 0 0 0

 

Apuntes Mat023 (versi´on preliminar actualizada 23-05-2014)

la soluci´on es u ´ nica si y solo si a + c + d = 6 0, de esta forma, podemos considerar la matriz

 



1 1



1 1

pues 1 + 1 + 1 = 3 6= 0. Ahora para definir   −1 0  T 1 0   0 1  T 0 0   −1 0  T 0 1   1 1  T 1 1

la transformaci´on (por ejemplo) = (0, 0, 0)

= (0, 0, 0)

= (0, 0, 0)

= (1, 1, 1) 

esto determina completamente quien es T (notar que   son elementos del kernel pero 

1 1

−1 0 1

0

  ,

0 1 0 0

  ,

−1 0 0

  no lo es). La T en forma expl´ıcita se deter-

1 1 mina de la siguiente manera, Como          −1 0 −1 0 1 1  0 1 ,  , , B=   1 0 0 0 0 1 1 1   es base, dado una matriz 

x y z w

  arbitraria, deben existir escalares α, β, γ, δ tales

que  

x y z w





 = α

−1 0 1

0





+β

0 1 0 0

131





+γ

−1 0 0

1





+δ

1 1 1 1

 

1

 

Apuntes Mat023 (versi´on preliminar actualizada 23-05-2014)

para buscar tales  −1   0    1  0

escalares resolvemos el sistema   0 −1 1 x 1 0 0    1 0 1 y    0 1 0 ∼    0 0 1 z    0 0 1 0 0 0 0 1 1 w

0 0 y 0 1

2 z − 13 x − 13 w 3 − 31 x − 13 w − 13 z 2 w − 13 x − 13 z 3 1 w + 31 x + 31 z 3

      

de donde se tiene 2 1 1 z− x− w 3 3 3 1 1 1 β = y− x− w− z 3 3 3 2 1 1 γ = w− x− z 3 3 3 1 1 1 δ = w+ x+ z 3 3 3

α =

as´ı  

x y z w

  =





−1 0 1 1 2  z− x− w  3 3 3 1 0     0 1 1 1 1  + y− x− w− z  3 3 3 0 0     −1 0 2 1 1  + w− x− z  3 3 3 0 1     1 1 1 1 1  + w+ x+ z  3 3 3 1 1 



luego 

x y





 2 1 1  = T z − x − w (0, 0, 0) 3 3 3 z w   1 1 1 + y − x − w − z (0, 0, 0) 3 3 3   2 1 1 + w − x − z (0, 0, 0) 3 3 3   1 1 1 + w + x + z (1, 1, 1) 3 3 3   1 1 1 1 1 1 1 1 1 = w + x + z, w + x + z , w + x + z 3 3 3 3 3 3 3 3 3 132

Apuntes Mat023 (versi´on preliminar actualizada 23-05-2014)

as´ı M2×2 (R) → R3     x y x y 1 1 1 1 1 1 1 1 1   → T = w + x + z, w + x + z , w + x + z 3 3 3 3 3 3 3 3 3 z w z w 

T 

:

10. Sea B la base can´onica de R2 y considere las transformaciones T : R2 → R2 y S : R2 → R2 definidas por T (a, b) = (2a + 3b, 4a − 7b) y S (a, b) = (3a − 2b, a − b) calcular [T ]B , [S]B y [T ◦ S]B comprobar que [T ]B · [S]B = [T ◦ S]B Soluci´ on. Al ser la matriz con respecto a la base can´onica se tiene     2 3 3 −2  y [S] =   [T ]B =  B 4 −7 1 −1 as´ı

 [T ]B · [S]B = 

2



3

4 −7



3 −2 1 −1





=

9 −7 5 −1

 

por otro lado T ◦ S (a, b) = T (3a − 2b, a − b) = (2 (3a − 2b) + 3 (a − b) , 4 (3a − 2b) − 7 (a − b))   = 9a − 7b 5a − b as´ı

 [T ◦ S]B = 

133

9 −7 5 −1

 

Apuntes Mat023 (versi´on preliminar actualizada 23-05-2014)

11. Sea T : R4 → R4 una transformaci´on lineal tal que T (1, 1, 0, 0) = (0, 1, 0, −1) T (1, 0, 1, 0) = (1, 1, 1, 0) si adem´as se tiene que T ◦ T = IR4 determinar la matriz asociada a T respecto a la base can´onica C de R4 . ¿Es T un isomorfismo? Justifique. Soluci´ on. Por la propiedad T ◦ T = IR4 se sigue T (T (1, 1, 0, 0)) = T (0, 1, 0, −1) = (1, 1, 0, 0) T (T (1, 0, 1, 0)) = T (1, 1, 1, 0) = (1, 0, 1, 0) as´ı T (1, 1, 0, 0) = (0, 1, 0, −1) T (1, 0, 1, 0) = (1, 1, 1, 0) T (0, 1, 0, −1) = (1, 1, 0, 0) T (1, 1, 1, 0) = (1, 0, 1, 0) queremos determinar la matriz asociada a la base can´onica T (1, 1, 1, 0) − T (1, 1, 0, 0) = T (0, 0, 1, 0) = (1, 0, 1, 0) − (0, 1, 0, −1) = (1, −1, 1, 1) T (0, 0, 1, 0) = (1, −1, 1, 1) T (1, 0, 0, 0) = T (1, 0, 1, 0) − T (0, 0, 1, 0) = (1, 1, 1, 0) − (1, −1, 1, 1) = (0, 2, 0, −1) T (0, 1, 0, 0) = T (1, 1, 0, 0) − T (1, 0, 0, 0) = (0, 1, 0, −1) − (0, 2, 0, −1) = (0, −1, 0, 0) T (0, 0, 0, 1) = T (0, 1, 0, 0) − T (0, 1, 0, −1) = (0, −1, 0, 0) − (1, 1, 0, 0) = (−1, −2, 0, 0) 134

Apuntes Mat023 (versi´on preliminar actualizada 23-05-2014)

as´ı



0

0

−1

1



   2 −1 −1 −2    [T ]CC =    0 0 1 0    −1 0 1 0 es f´acil ver que es isomorfismo, pues T ◦T

= I ⇒ [T ◦ T ]CC = I4 ⇒ [T ]CC [T ]CC = I4

as´ı [T ]CC es invertible y luego T es invertible. Otra forma 0 1 −1 0 2 −1 −1 −2 =1 0 0 1 0 −1 0 1 0 y luego T es invertible. Otra forma es utilizar la matriz de cambio de base  1 1 0 1   1 0 1 1  [I]CB =   0 1 0 1  0 0 −1 0 as´ı  [I]BC

1 1

0

1

−1

   1 0 1 1    =   0 1 0 1    0 0 −1 0

135



1

      

0

  1 −1  =  0 0  −1 1

−1 0 0 1

0



 −1    −1   1

Apuntes Mat023 (versi´on preliminar actualizada 23-05-2014)

luego [T ]CC = [T ]CB [I]BC   0 1 1 1    1 1 1 0    =    0 1 0 1    −1 0 0 0  0 0 1 −1   2 −1 −1 −2  =   0 0 1 0  −1 0 1 0

1

0

−1

1

−1

0

0

0

0

−1 

1

1

0



 −1    −1   1

     

Ejercicios de la secci´ on 1. Determine si las siguientes funciones son transformaciones lineales: a) T : R2 → R, T (x, y) = |x + y| b) T : R3 → R2 , c) T : R → R3 , d ) T : R3 → R3 ,

T (x, y, z) = (2x + y, z − y) T (x) = (x, 2x, x2 ) T (x, y, z) = (cos x, sin y, z)

2. ¿Existe un transformaci´on lineal de R2 en R2 que transforma los vectores (1, 4) y (−3, 2) en los vectores (2, 1) y (4, 2), respectivamente? En caso afirmativo determine la imagen del vector (−2, a), con a ∈ R. 3. Sea U un espacio vectorial tal que dim U = n < ∞. Suponga que u1 , u2 , . . . , un es una base ordenada de U . Suponga que T : U → U es tal que: T (ui ) = ui ,

∀i = 1, 2, . . . , n

(2.3)

Entonces: a) Demuestre que existen funciones T : U → U definidas por la condici´on (2.3) que son transformaciones lineales. 136

Apuntes Mat023 (versi´on preliminar actualizada 23-05-2014)

b) Verifique que si T es una transformaci´on lineal que satisface la condici´on (2.3), entonces T = IU , donde IU es la funci´on identidad de U en U . 4. Sea T : R3 → R2 la transformaci´on lineal definida por: T (x, y, z) = (x + y − 2z, x + y) Calcule ker T e ImT . 5. Sea S ∈ L (R3 , M2 (R)) definida por:  S (x, y, z) = 

x − 3y y − 2z x

z

 

a) Calcule condiciones sobre a ∈ R de modo que: 3e1 + ae2 − e3 ∈ kerT donde ei es el i–´esimo vector de la base can´onica de R3 . b) Calcule ImT 6. Sea T : R2 [x] → M2 (R) una funci´on definida por:   R1 00  p (0) 0 p (x) dx  T p (x) =  p (−1) 0 a) Demuestre que T es una transformaci´on lineal. b) Hallar una base para el kernel de T . c) Hallar ImT y su dimensi´on. 7. Sea T ∈ L (U, V ) tal que ker T = {0}. Suponga que B es un conjunto linealmente independiente en U . Demuestre que: T (B) = {T (u) : u ∈ B} es linealmente independiente en V . 8. Sea T ∈ L (U, U ). Entonces, las siguientes proposiciones son equivalentes: 137

Apuntes Mat023 (versi´on preliminar actualizada 23-05-2014)

(i) ker T ∩ ImT = {0}. (ii) Si T 2 (u) = 0, entonces T (u) = 0. 9. Sea T : R3 → R3 la transformaci´on lineal definida por: T (x, y, z) = (3x, x − y, 2x + y + z) ¿Es T invertible? En caso afirmativo, hallar una f´ormula para T −1 . 10. Sean U y V espacios vectoriales sobre el mismo cuerpo K, tales que dim U = dim V = n < ∞. Sea, adem´as, T ∈ L (U, V ) tal que: T u = 0 =⇒ u = 0 Entonces, existe una base u1 , u2 , . . . , un de U tal que T u1 , T u2 , . . . , T un es una base de V . 11. Sea B =



(1, 2, 1) ; (2, 1, 1) , (1, 1, 2)



una base de R3 y T : R3 → R3 una transfor-

maci´on lineal tal que: 

−3 −3 −5

 [T ]BB =   1 2



 1   4

1 2

a) Determinar una base del ker T . b) Determinar una base del ker T 2 . c) Determinar una base de ImT . 12. Sea T : R3 → R3 tal que: 

1

2

0



   [T ]D 1 1  B =  1  3 −1 1 donde B = {(1, 1, 2) ; (0, 1, 1) ; (−1, 0, 0)} y D = {(1, 1, 1) ; (0, 1, 1) ; (0, 0, 1)} son bases de R3 . Entonces: a) Encuentre T (1, 3, 1), sin calcular T (x, y, z). b) Determine T −1 (x, y, z), si acaso existe. 138

Apuntes Mat023 (versi´on preliminar actualizada 23-05-2014)

c) Hallar [T ]F E , sin calcular T (x, y, z), si E = {(0, 1, 2) ; (1, 1, 1) ; (0, 1, 0)} y F = {(1, 1, 0) ; (1, −1, 1) ; (1, 0, 0)} son, tambi´en, bases de R3 . 13. Sean T : R3 [x] → R3 [x] una funci´on definida por:  T p (x) = p0 (x) − x · p00 (x) y B1 = {1 − x2 , 1 + x2 , 1 − x3 , 2 + x}, B2 = {1, x2 + x3 , x3 − x2 , 1 − 2x} dos bases ordenadas de R3 [x]. Entonces: a) Pruebe que existe T es una transformaci´on lineal. b) Hallar ker T y una base para ImT . ¿Es T invertible? c) Calcule [T ]BB21 . 14. Diremos que un espacio vectorial V es suma directa de los subespacios W1 y W2 , lo cual anotaremos, V = W1 ⊕ W2 , si W1 ∩ W2 = {0}, y para cada v ∈ V , existen w1 ∈ W1 y w2 ∈ W2 tales que v = w1 + w2 . Sean V un espacio vectorial de dimensi´on finita y T : V → V una transformaci´on lineal. Pruebe que:  V = ker T ⊕ ImT ⇐⇒ ker T 2 = ker T 15. Sean U y V espacios vectoriales sobre el mismo cuerpo K y sea S un isomorfismo de U en V . Demuestre que: T 7→ S T S −1 es un isomorfismo de L (U, U ) en L (V, V ). 16. Hallar expl´ıcitamente, esto es, calculando T (ax3 + bx2 + cx + d), una transformaci´on lineal T : R3 [x] → R3 tal que:  ImT = (x, y, z) ∈ R3 : x + 2y − z = 0 y cuya nulidad sea 2. 17. Sean: B = {(1, −2, 1) , (0, 1, −2) , (1, 1, 1)} 139

Apuntes Mat023 (versi´on preliminar actualizada 23-05-2014)

y:          1 2 0 0 1 −2 2 1  ,  ,  ,   D=   −2 1 1 −1 3 −3 2 1  bases ordenadas de R3 y M2 (R), respectivamente. Considere T : R3 → M2 (R) definida por:  T (x, y, z) = 

x + 2y x − y + 3z 0

x+y+z

 

a) Hallar [T ]D B. b) Determine condiciones sobre a ∈ R para que (x, y, z) ∈ ker T , sabiendo que:   −1    [(x, y, z)]B =  a   2 c) Hallar ImT , usando [T ]D B. 18. Sean T : M2 (R) → M2 (R) una transformaci´on lineal y:          1 0 1 1 1 0 1 1  , , ,  B=   0 1 0 1 1 1 2 2  una base ordenada de M2 (R) tal que: 

1 0 1 0



   1 2 3 0    [T ]BB =    1 1 2 0    1 1 2 1 Determinar una base D de M2 (R) de modo que [T ]D D sea una matriz diagonal. (Ayuda: El polinomio caracter´ıstico de T es fT (λ) = λ (λ − 4) (λ − 1)2 ) 19. Sean:  B = 1 + x − x2 , 2x − x2 , 1 + x2 y: D = {(1, 2, 1) , (1, 3, 2) , (2, 1, 3)} 140

Apuntes Mat023 (versi´on preliminar actualizada 23-05-2014)

bases ordenadas de R2 [x] y R3 , respectivamente. Consideremos T : R2 [x] → R3 tal que: 

5

−2

2



   [T ]D 1  B =  −3 −1  1 4 −3 a) Demuestre que T es un isomorfismo. b) Calcule ker T −1 e ImT −1 . c) Hallar expl´ıcitamente T −1 . 20. Sean B = {1 + x, 1 − x2 , 1} y D = {1, 1 + x, 1 + x + x2 } bases ordenadas de R2 [x] . Considere T : R2 [x] → R2 [x] una transformaci´on lineal tal que: 

1 0 1



    [T ]D B =  0 1 1  1 1 2 a) Hallar condiciones sobre a ∈ R para que: 1 − (a + 1) x − 2x2 ∈ ker T b) Hallar condiciones sobre b ∈ R para que: 1 − 2x + (a − 1) x2 ∈ ker T −1 21. Sea:  W = (x, y, z) ∈ R3 : 2x − 3y + 5z = 0 un subespacio de R3 tal que B es una base W . Considere D = {(1, 1) , (2, 1)} una base de R2 . Sea T ∈ L (W, R2 ) definida por T (x, y, z) = (x + y − z, 3x − y + 2z) tal que:   [T ]D B = Determine la base B.

141

1

2

−3 −1

 

Apuntes Mat023 (versi´on preliminar actualizada 23-05-2014)

22. Considere el plano en R3 dado por:  W = (x, y, z) ∈ R3 : ax + by + cz = 0 con a, b, c ∈ R. Para cualquier u ∈ R3 , denotaremos por PW (u) al punto del plano W que se encuentra a menor distancia de u. Tal funci´on PW (u) se llama la proyecci´on ortogonal de u sobre W . Entonces: a) Dado u ∈ R3 , hallar una expresi´on para PW (u). b) Demuestre que la funci´on PW (u) : R3 → R3 definida por: u 7→ PW (w) es una transformaci´on lineal. c) Calcule [PW ]CC , donde C es la base can´onica de R3 , y verifique que: T 2   = [PW ]CC [PW ]CC = [PW ]CC ∧ [PW ]CC 23. Considere el subespacio vectorial W < R2 [x] definido por:

W = 1 − x2 , x + 2x2 Demuestre que existe una transformaci´on lineal T : R2 [x] → R2 [x] diagonalizable tal que −1 y 1 sean sus valores propios, y que el espacio propio asociado a −1 sea W .

Ecuaciones diferenciales lineales Definiciones Definici´ on 2.2.1. Sean I ⊆ R un intervalo, A0 , A1 , . . . , An y R funciones continuas sobre I tales que A (x) 6= 0, para cada x ∈ I. Una ecuaci´on diferencial de la forma: An (x) y (n) + An−1 (x) y (n−1) + An−2 (x) y (n−2) + · · · + A1 (x) y 0 + A0 (x) y = R (x) (2.4) se denomina ecuaci´ on diferencial lineal de orden n. Las funciones A0 , A1 , . . . , An son llamados coeficientes de la ecuaci´on diferencial. Si R ≡ 0, la ecuaci´on lineal se denomina ecuaci´ on homog´ enea. 142

Apuntes Mat023 (versi´on preliminar actualizada 23-05-2014)

Observaci´ on 2.2.1. En general, los puntos en los cuales An (x) = 0 se conocen como puntos singulares y su estudio no ser´a considerado en este curso. Observaci´ on 2.2.2. La ecuaci´on (2.4) puede escribirse como: y (n) + pn−1 (x) y (n−1) + pn−2 (x) y (n−2) + · · · + p1 (x) y 0 + p0 (x) y = Q (x)

(2.5)

al dividir la ecuaci´on (2.4) por An (x), de esta forma pi (x) =

Ai (x) para i = 0, . . . , n − 1 An (x)

y Q (x) =

R (x) An (x)

esta forma de la ecuaci´on es llamada forma normal. Ejemplo 2.2.1. Son ecuaciones diferenciales lineales las siguientes ecuaciones: 1. y (4) + 3xy 00 − 2 (cos x) y 0 + y = e2x cos 3x 2. y 00 + y = 0 3. Las ecuaciones de Euler, definidas como: (ax + b)n y (n) + A1 (ax + b)n−1 y (n−1) + · · · + An−1 (ax + b) y 0 + An y = Q (x) con ax + b > 0. 4. las ecuaciones de Legendre, definidas como:  1 − x2 y 00 − 2xy 0 + α (α + 1) y = 0 con α ∈ R. Ejemplo 2.2.2. Considere la ecuaci´on diferencial: y 00 − 2y 0 + 2y = 0 1. Verifique que las funciones u1 (x) = ex cos x y u2 (x) = ex sin x son soluciones de la ecuaci´on diferencial. 143

Apuntes Mat023 (versi´on preliminar actualizada 23-05-2014)

2. Sean C1 , C2 ∈ R. Verifique que y (x) = C1 u1 (x) + C2 u2 (x) es soluci´on de la ecuaci´on diferencial. 3. Hallar una soluci´on y que satisfaga las condiciones iniciales y (0) = 1 e y 0 (0) = 4. Soluci´ on. Por partes: 1. Notemos que d x (e cos x) = ex cos x − ex sin x dx d x u001 (x) = (e cos x − ex sin x) = −2ex sin x dx u01 (x) =

se sigue u001 − 2u01 + 2u1 = (−2ex sin x) − 2 (ex cos x − ex sin x) + 2 (ex cos x) = 0 de manera similar d x (e sin x) = ex (cos x + sin x) dx d x u002 (x) = (e (cos x + sin x)) = 2 (cos x) ex dx u02 (x) =

luego u002 − 2u02 + 2u2 = (2 (cos x) ex ) − 2 (ex (cos x + sin x)) + 2 (ex sin x) = 0 2. Notar que y 0 (x) = C1 u01 (x) + C2 u02 (x) y 00 (x) = C1 u001 (x) + C2 u002 (x) luego y 00 − 2y 0 + 2y = (C1 u001 (x) + C2 u002 (x)) − 2 (C1 u01 (x) + C2 u02 (x)) + 2 (C1 u1 (x) + C2 u2 (x)) = (C1 u001 (x) − 2C1 u01 (x) + 2C1 u1 (x)) + (C2 u002 (x) − 2C2 u02 (x) + 2C2 u2 (x)) = C1 (u001 (x) − 2u01 (x) + 2u1 (x)) + C2 (u002 (x) − 2u02 (x) + 2u2 (x)) = C1 0 + C2 0 = 0 144

Apuntes Mat023 (versi´on preliminar actualizada 23-05-2014)

3. Sabemos que y (x) = C1 ex cos x + C2 ex sin x es soluci´on de la ecuaci´on, para cumplir las condiciones y (0) = 1 e y 0 (0) = 4 las constantes las debemos escoger en forma adecuada 1 = y (0) = C1 4 = y 0 (0) = C1 + C2 as´ı 1 = C1 4 = C1 + C2 que tiene soluci´on C1 = 1, C2 = 3. As´ı y (x) = ex cos x + 3ex sin x Observaci´ on 2.2.3. Con el objeto de simplificar el estudio de las ecuaciones diferenciales lineales y el de aprovechar la terminolog´ıa de las transformaciones lineales es que consideraremos la noci´on de operador diferencial asociado a una ecuaci´on diferencial lineal de orden n. Para ello recordemos que C n (I) denota el espacio vectorial de todas las funciones de clase C n definidas sobre I, esto es el conjunto de todas las funciones f : I → R que son n veces derivables con f (n) continua sobre I. Definici´ on 2.2.2. Sean p0 , p1 , . . . , pn−1 funciones continuas sobre un intervalo I ⊆ R. Llamaremos operador diferencial asociado a la ecuaci´on (2.5) a la funci´on L : C n (I) → C (I) definida por: L (f ) = f (n) + pn−1 f (n−1) + · · · + p0 f

(2.6)

Observaci´ on 2.2.4. El operador diferencial dado por la ecuaci´on (2.6) puede escribirse como: L = Dn + pn−1 Dn−1 + · · · + p0 Id en donde Dk representa la derivada de orden k y Id representa la transformaci´on lineal identidad. Teorema 2.2.1. Sea I ⊆ R un intervalo cualquiera. El operador diferencial L : C n (I) → C (I) es una transformaci´on lineal. 145

Apuntes Mat023 (versi´on preliminar actualizada 23-05-2014)

Observaci´ on 2.2.5. Con la introducci´on del operador diferencial L toda ecuaci´on lineal: y (n) + pn−1 (x) y (n−1) + · · · + p0 (x) y = Q (x) puede escribirse como: L (y) = Q

(2.7)

As´ı, la ecuaci´on: L (y) = 0 se llamar´a ecuaci´on homog´enea asociada a la ecuaci´on (2.7). Observaci´ on 2.2.6. La ecuaci´on (2.7) tiene un an´alogo finito dimensional con sistemas de ecuaciones lineales y consecuentemente con la respectiva ecuaci´on matricial asociada. En efecto, sea S un sistema de m ecuaciones lineales con n inc´ognitas. Luego, existe una matriz A ∈ Mm×n (K) que contiene los coeficientes del sistema, una matriz X ∈ Mm×1 (K) con las inc´ognitas y adem´as, una matriz B ∈ Mm×1 (K) con las constantes. El producto de matrices permite representar el sistema S como la ecuaci´on matricial siguiente: AX = B Observe que las propiedades del ´algebra de matrices nos permiten escribir X como: X = X h + Xp donde Xh representa la soluci´on general del sistema homog´eneo AX = 0 y Xp representa una soluci´on particular del sistema que se asume conocida. En efecto: AX = A (Xh + Xp ) = AXh + AXp = 0+B = B Ahora bien, mediante el isomorfismo LK (U, V ) ' Mm×n (K), debe existir una transformaci´on lineal T : U → V tal que: A = [T ]CB 146

Apuntes Mat023 (versi´on preliminar actualizada 23-05-2014)

para ciertas bases ordenadas B y D, de U y V , respectivamente. Por tanto, el espacio soluci´on de la ecuaci´on matricial es, salvo cambio de coordenadas, el n´ucleo o espacio nulo de la transformaci´on lineal T . Por esta raz´on, nos interesar´a calcular el n´ ucleo o kernel del operador diferencial L. Teorema 2.2.2. Suponga que yh representa la soluci´on de la ecuaci´on homog´enea asociada a L (y) = Q y que yp es una soluci´ on particular (esto es, L (yp ) = Q) de la misma ecuaci´on. Entonces, la soluci´on general de: L (y) = Q es de la forma: y = yh + yp En efecto, siguiendo los razonamientos de la observaci´on anterior, tenemos que: L (y) = L (yh + yp ) = L (yh ) + L (yp ) = 0+Q = Q Observaci´ on 2.2.7. Consideremos la ecuaci´on diferencial: y (n) + pn−1 (x) y (n−1) + pn−2 (x) y (n−2) + · · · + p1 (x) y 0 + p0 (x) y = 0 Por lo expuesto anteriormente, el operador lineal asociado L : C n (I) → C (I) definido por: L = Dn + pn−1 Dn−1 + · · · + p1 D + p0 Id permite expresar la ecuaci´on como L (y) = 0. Por tanto, una funci´on y es soluci´on de la ecuaci´on diferencial homog´enea si y solo si y ∈ ker L, notamos adem´as que: ker L ≤ C n (I) y el espacio vectorial C n (I) no es de dimensi´on finita (basta ver que para cada n ≥ 1, el conjunto 1, x, x2 , . . . , xn es linealmente independiente). Una aplicaci´on muy importante del Teorema de Existencia y Unicidad para ecuaciones diferenciales ser´a la de mostrar que el 147

Apuntes Mat023 (versi´on preliminar actualizada 23-05-2014)

n´ ucleo de un operador diferencial no s´olo es de dimensi´on finita, sino que, adem´as, es de dimensi´on n, en donde n es el orden de la ecuaci´on diferencial lineal. Es decir, si: L = Dn + pn−1 Dn−1 + · · · + p1 D + p0 Id es un operador diferencial asociado a una ecuaci´on diferencial lineal, entonces: dim ker L = n

Teorema de Existencia y Unicidad Observaci´ on 2.3.1. El siguiente teorema juega un rol crucial en establecer que la dimensi´on del n´ ucleo del operador diferencial L asociado a una ecuaci´on diferencial lineal de orden n del tipo: y (n) + pn−1 (x) y (n−1) + · · · + p0 (x) y = Q (x) es de dimensi´on finita. En particular, en esta secci´on se utilizar´a el Teorema de Existencia y Unicidad para demostrar que no s´olo es finita esa dimensi´on, sino que adem´as es de dimensi´on coincide con el orden de la ecuaci´on lineal. Introduzcamos primeramente la siguiente definici´on: Definici´ on 2.3.1. Diremos que la funci´on ϕ : I ⊆ R → R es soluci´on del problema de valores iniciales de orden n: y (n) = f x, y, y 0 , y 00 , . . . , y (n−1)



y (x0 ) = y0 , y 0 (x0 ) = y1 , . . . , y (n−1) (x0 ) = yn−1 si ϕ : I ⊆ R → R es soluci´on de la ecuaci´on en su intervalo de definici´on, x0 ∈ I y: ϕ (x0 ) = y0 , ϕ0 (x0 ) = y1 , . . . , ϕ(n−1) (x0 ) = yn−1 Observaci´ on 2.3.2. Para mayor claridad de los conceptos los teoremas ser´an expuestos para ecuaciones de orden 2 y notar que la extensi´on se hace por inducci´on sobre n. As´ı, los resultados se indicar´an para orden 2 u orden n seg´ un convenga. Teorema 2.3.1. Sean I ⊆ R un intervalo abierto y p0 , p1 dos funciones continuas sobre I. Considere el operador diferencial: L (y) = y 00 + p1 y 0 + p0 y 148

Apuntes Mat023 (versi´on preliminar actualizada 23-05-2014)

Si x0 ∈ I e y0 , y1 son n´ umeros reales cualesquiera, entonces existe una u ´nica funci´on f : I → R que es soluci´on de la ecuaci´on diferencial: L (y) = 0 y que satisface las condiciones iniciales: f (x0 ) = y0



f 0 (x0 ) = y1

Teorema 2.3.2. Consideremos el operador diferencial L (y) = y 00 + p1 y 0 + p0 y, con p0 y p1 funciones continuas sobre un intervalo abierto I ⊆ R. Suponga que u1 y u2 son dos funciones no nulas, linealmente independientes sobre I, y que son soluci´ on de la ecuaci´ on diferencial L (y) = 0. Entonces, para todo par de constantes c1 y c2 , la combinaci´ on lineal: c1 u1 + c2 u2 es soluci´ on de la ecuaci´on diferencial L (y) = 0 en I. Rec´ıprocamente, si y es una soluci´on de L (y) = 0 en I, entonces existen constantes c1 y c2 tales que: y = c1 u 1 + c2 u 2 Demostraci´on. En primer lugar, note que como L es una transformaci´on lineal, se tiene que: C1 u1 + C2 u2 ∈ ker L luego, C1 u1 + C2 u2 es soluci´on de L (y) = 0. Por otro lado, como u1 y u2 son linealmente independientes en I, bastar´a verificar que: ker L = hu1 , u2 i En efecto, sean f ∈ ker L y x0 ∈ I. Por el Teorema de Unicidad, es suficiente verificar que existen constantes C1 y C2 tales que:   C u (x ) + C u (x ) = f (x ) 1 1 0 2 2 0 0  C1 u0 (x0 ) + C2 u0 (x0 ) = f 0 (x0 ) 1 2 es decir, que las funciones C1 u1 + C2 u2 y f coinciden en las condiciones iniciales. Entonces, para que tales constantes existan, es suficiente a su vez que el determinante: u1 (x0 ) u2 (x0 ) = u1 (x0 ) u02 (x0 ) − u2 (x0 ) u01 (x0 ) 0 0 u1 (x0 ) u2 (x0 ) 149

Apuntes Mat023 (versi´on preliminar actualizada 23-05-2014)

sea no nulo. Consideremos la funci´on W : I ⊆ R → R definida por: u1 (x) u2 (x) = u1 (x) u02 (x) − u2 (x) u01 (x) W (x) = 0 0 u1 (x) u2 (x) Supongamos que W (x) = 0, para todo x ∈ I, entonces:   u1 (x) u02 (x) − u2 (x) u01 (x) d u2 (x) = dx u1 u21 (x) W (x) = u21 (x) = 0 luego, por el Teorema del Valor Medio el cociente u2 /u1 debe ser constante, pues I es un intervalo. Esto contradice la hip´otesis de que u1 y u2 son linealmente independientes sobre I. Por tanto, debe existir al menos un x0 ∈ I tal que W (x0 ) 6= 0, y esto u ´ ltimo implica que existen constantes C1 y C2 tales que: f (x) = C1 u1 (x) + C2 u2 (x) en otras palabras, u1 y u2 forman una base para el n´ ucleo del operador L, y as´ı: dim ker L = 2

Observaci´ on 2.3.3. As´ı, entonces, el operador L asociado a la ecuaci´on diferencial lineal: y (n) + pn−1 (x) y (n−1) + · · · + p0 (x) y = Q (x) tiene n´ ucleo ker L con dimensi´on finita dado por el orden de la ecuaci´on diferencial. Es decir: dim ker L = n vemos entonces que para determinar la soluci´on general de una ecuaci´on diferencial lineal homog´enea, necesitamos una familia de n funciones linealmente independientes si la ecuaci´on es de orden n. Antes de continuar, veamos algunos ejemplos de funciones linealmente independientes: 150

Apuntes Mat023 (versi´on preliminar actualizada 23-05-2014)

Ejemplo 2.3.1. Sean r1 , r2 , . . . , rn ∈ R tales que ri = 6 rj , para todo i 6= j. Considere la familia de n funciones definidas por: ui (x) = eri x ,

∀i = 1, 2, . . . , n

con x ∈ U ⊆ R. Note que si n = 2, entonces u1 y u2 son linealmente independientes, pues: u1 (x) = e(r1 −r2 )x u2 (x) no es constante. Sup´ongase que el resultado es cierto para k funciones exponenciales. Sea: k+1 X

Ci eri x = 0,

∀x ∈ U

(2.8)

i=1

Luego, multiplicando por e−rk+1 x y derivando la ecuaci´on anterior respecto de x, obtenemos: k X

Ci (ri − rk+1 ) e(ri −rk+1 )x = 0

i=1

Los k n´ umeros ri − rk+1 , con 1 ≤ i ≤ k son distintos. Utilizando la hip´otesis de inducci´on, las k exponenciales de la ecuaci´on anterior son linealmente independientes en U . Por tanto, se debe tener que: Ci (ri − rk+1 ) = 0,

∀i = 1, 2, . . . , k

pero ri = 6 rk+1 , para cada i ≤ k, se tiene que Ci = 0 para i ≤ k. Utilizando esto u ´ ltimo junto con la ecuaci´on (2.8) se concluye tambi´en que Ck+1 = 0. Ejemplo 2.3.2. Sea r ∈ R. Las n funciones: ui (x) = xi−1 erx ,

∀i = 1, 2, . . . , n

son linealmente independientes en todo intervalo I ⊆ R. Observaci´ on 2.3.4. Si el cociente entre dos funciones u y v es id´enticamente una constante, entonces u y v son linealmente dependientes. En efecto, suponga que: u ≡ λ, v

λ constante

entonces u = λv. Es decir, u ∈ hvi, y por tanto, no puede ser linealmente independiente. 151

Apuntes Mat023 (versi´on preliminar actualizada 23-05-2014)

Ejemplo 2.3.3. Tres polinomios de primer grado cualesquiera son linealmente dependientes en (−∞, +∞). Ejemplo 2.3.4. Cuatro polinomios de segundo grado cualesquiera son linealmente dependientes en (−∞, +∞). Definici´ on 2.3.2. Diremos que las funciones u1 , u2 , . . . , un forman un sistema fundamental de soluciones de la ecuaci´on diferencial L (y) = 0, de orden n, si: 1. L (ui ) = 0, para todo i = 1, 2, . . . , n. 2. u1 , u2 , . . . , un son linealmente independientes (sobre I). Observaci´ on 2.3.5. Note que un sistema fundamental de soluciones u1 , u2 , . . . , un es una base para el n´ ucleo de L. Es decir: ker L = h{u1 , u2 , . . . , un }i ≤ C n (I) Ejemplo 2.3.5. Considere la ecuaci´on y 00 +y = 0. Verifique que las funciones u1 (x) = cos x y u2 (x) = sin x, forman un sistema fundamental de soluciones de la ecuaci´on diferencial. Soluci´ on. Las funciones sin x y cos x son linealmente independientes y son soluciones de la ecuaci´on y 00 + y = 0, se sigue que ker (D2 + 1) = h{sin x, cos x}i.

El wronskiano Observaci´ on 2.4.1. En el teorema anterior, se demostr´o que el operador diferencial L, asociado a la ecuaci´on diferencial lineal homog´enea de orden 2 siguiente: y 00 + p1 (x) y 0 + p0 (x) y = 0 tiene n´ ucleo: ker L = h{u1 , u2 }i en el cual el determinante:

u1 (x0 ) u2 (x0 ) 0 u1 (x0 ) u02 (x0 )



juega un papel importante en relaci´on con la independencia lineal de las funciones, tenemos la siguiente definici´on: 152

Apuntes Mat023 (versi´on preliminar actualizada 23-05-2014)

Definici´ on 2.4.1. Llamaremos wronskiano de las funciones u1 (x) , u2 (x) , . . . , un (x) a: u (x) u2 (x) ··· un (x) 1 u0 (x) u02 (x) ··· u0n (x) 1 W (x) = .. .. .. .. . . . . (n−1) (n−1) (n−1) u1 (x) u2 (x) · · · un (x) n×n

Se anota tambi´en W [u1 , u2 , . . . , un ] (x). Ejemplo 2.4.1. Calcule el wronskiano de u1 (x) = cos x y u2 (x) = sin x. cos x sin x = cos2 x + sin2 x = 1 Soluci´ on. W (x) = − sin x cos x Teorema 2.4.1. Sean u1 (x) y u2 (x) dos soluciones de la ecuaci´on diferencial: y 00 + p1 (x) y 0 + p0 (x) y = 0

(2.9)

Entonces, el wronskiano: u1 (x) u2 (x) W (x) = u01 (x) u02 (x)



satisface la ecuaci´on diferencial: W 0 + p1 (x) W = 0 Demostraci´on. Note que: 0

W 0 = (u1 u02 − u01 u2 )

= u01 u02 + u1 u002 − u001 u2 − u01 u02 = u1 u002 − u001 u2 luego: W 0 + p1 (x) W = u1 u002 − u001 u2 + p1 (x) {u1 u02 − u01 u2 } = u1 {u002 + p1 (x) u02 } − u2 {u001 + p1 (x) u01 } = u1 {u002 + p1 (x) u02 + p0 (x) u2 } − u2 {u001 + p1 (x) u01 + p0 (x) u1 } = u1 · 0 − u2 · 0 = 0 Por tanto, W 0 + p1 (x) W = 0, como se quer´ıa demostrar. 153

Apuntes Mat023 (versi´on preliminar actualizada 23-05-2014)

Observaci´ on 2.4.2. En vista del teorema anterior, si u1 (x) y u2 (x) son dos soluciones de la ecuaci´on (2.9), la ecuaci´on diferencial asociada al wronskiano: W 0 + p1 (x) W = 0 es de variable separable, luego: Z

dW =− W

Z p1 (x) dx + C

Por tanto: W (x) = W (x0 ) e−

R

p1 (x)dx

para alg´ un x0 ∈ I. Por tanto se tiene el siguiente teorema: Teorema 2.4.2 (F´ormula de Abel para el wronskiano). Sea W = W (x) el wronskiano asociado a dos soluciones linealmente independientes de la ecuaci´on diferencial de segundo orden: y 00 + p1 (x) y 0 + p0 (x) y = 0 entonces: W (x) = W (x0 ) e−

R

p1 (x)dx

Teorema 2.4.3. Sean u1 , u2 , . . . , un soluciones de la ecuaci´on diferencial: y (n) + pn−1 (x) y (n−1) + · · · + p0 (x) y = 0 donde las funciones p1 , p2 , . . . , pn−1 son continuas en un intervalo I ⊆ R. Entonces, u1 , u2 , . . . , un son linealmente independientes sobre I si y solamente si W [u1 , u2 , . . . , un ] (x) = 6 0 en I. Ejemplo 2.4.2. Sean u1 (x) = x y u2 (x) =

1 x

soluciones de la ecuaci´on diferencial:

1 1 y 00 + y 0 − 2 y = 0 x x definida para x > 0. Hallar la soluci´on general. Soluci´ on. Para hallar la soluci´on general, es suficiente probar que u1 (x) y u2 (x) son linealmente independientes. Por el teorema anterior:   1 x x 1 = −2 6= 0 W x, = 1 x 1 − 2 x

154

Apuntes Mat023 (versi´on preliminar actualizada 23-05-2014)

Por tanto: y (x) = C1 x + C2

1 x

Teorema 2.4.4 (F´ormula de Abel para una segunda soluci´on). Sea u1 (x) una soluci´on no trivial de la ecuaci´on diferencial: y 00 + p1 (x) y 0 + p0 (x) y = 0 entonces, una segunda soluci´ on u2 (x), linealmente independiente con u1 (x), est´ a dada por: R Z − p1 (x)dx e dx u2 (x) = u1 (x) u21 (x) Demostraci´on. Como: u1 (x) u2 (x) W (x) = u01 (x) u02 (x)



= u1 u02 − u01 u2 y: W (x) = C e−

R

p1 (x)dx

obtenemos la ecuaci´on diferencial lineal de primer orden en la variable u2 siguiente: u1 u02 − u01 u2 = C e−

R

p1 (x)dx

Dividiendo la ecuaci´on anterior por u21 , se tiene que: u1 u02 − u01 u2 C − R p1 (x)dx = e u21 u21 Por consiguiente:   R d u2 C = 2 e− p1 (x)dx dx u1 u1 integrando y eligiendo valores adecuados para las constantes de integraci´on, se obtiene finalmente que: Z u2 (x) = u1 (x)

R

e− p1 (x)dx dx u21 (x)

Ejemplo 2.4.3. Hallar la soluci´on general de la ecuaci´on de Legendre, con α = 1:  1 − x2 y 00 − 2xy 0 + 2y = 0, 155

|x| < 1

Apuntes Mat023 (versi´on preliminar actualizada 23-05-2014)

Soluci´ on. Notamos que u1 (x) = x es una soluci´on particular de la ecuaci´on diferencial. Luego, de: y 00 −

2x 0 2 y + y=0 1 − x2 1 − x2

y de la f´ormula de Abel, se obtiene que: Z

e

R

2x dx 1−x2

dx x2 Z − ln(1−x2 ) e = x dx x2 Z dx = x 2 x (1 − x2 )  Z  1 1 1 = x + + dx x2 2 (1 − x) 2 (1 + x)

y2 (x) = x

mediante un desarrollo en fracciones parciales del pen´ ultimo integrando. Por tanto: 1 + x 1 −1 y2 (x) = x ln 2 1 − x As´ı, la soluci´on general de la ecuaci´on diferencial es:   x 1 + x −1 y (x) = C1 x + C2 ln 2 1 − x Ejemplo 2.4.4. Hallar la soluci´on general de la ecuaci´on diferencial: xy 00 − 2 (x + 1) y 0 + (x + 2) y = 0 para x > 0, bajo el supuesto de que la ecuaci´on homog´enea tiene una soluci´on de la forma y = emx , con m ∈ R una constante adecuada. Soluci´ on. Por hip´otesis, la ecuaci´on diferencial posee una soluci´on de la forma y = emx . As´ı, al reemplazar en: xy 00 − 2 (x + 1) y 0 + (x + 2) y = 0 obtenemos la ecuaci´on: xm2 emx − 2m (x + 1) emx + (x + 2) emx = 0

156

Apuntes Mat023 (versi´on preliminar actualizada 23-05-2014)

para todo x > 0. Pero: xm2 − 2m (x + 1) + (x + 2) = xm2 − 2m (x + 1) + (x + 1) + 1 = xm2 + (x + 1) (1 − 2m) + 1 = xm2 + (x + 1) (1 − 2m) + 1 − m2 + m2 = (x + 1) m2 + (x + 1) (1 − 2m) + 1 − m2   = (x + 1) m2 − 2m + 1 + 1 − m2  = (x + 1) (m − 1)2 + 1 − m2 entonces:  (x + 1) (m − 1)2 + 1 − m2 = 0 para todo x > 0. Por tanto, m = 1. As´ı, y1 (x) = ex es una soluci´on de la ecuaci´on. Para hallar una segunda soluci´on linealmente independiente con y1 (x) = ex , utilizaremos la f´ormula de Abel. Primeramente escribimos la ecuaci´on en su forma normal:     2 1 00 0 y + 1+ y=0 y −2 1+ x x  entonces, si p1 (x) = −2 1 + x1 , tenemos que: y2 (x) = = = = = =

R

e− p1 (x)dx dx y1 (x) [y1 (x)]2 Z 2 R (1+ 1 )dx x e ex dx e2x Z 2(x+ln x) e x dx e e2x Z 2 2x xe ex dx e2x Z ex x2 dx Z

1 3 x xe 3

por tanto, la soluci´on general de la ecuaci´on diferencial est´a dada por: y (x) = C1 ex + C2 x3 ex

157

Apuntes Mat023 (versi´on preliminar actualizada 23-05-2014)

Ecuaciones diferenciales a coeficientes constantes Definiciones Definici´ on 2.5.1. Una ecuaci´on diferencial lineal a coeficientes constantes es una ecuaci´on de la forma: y (n) + an−1 y (n−1) + an−2 y (n−2) + · · · + a1 y 0 + a0 y = Q (x) en donde a0 , a1 , . . . , an−1 ∈ R y Q es una funci´on continua en R. Las constantes ai ∈ R, i = 0, 1, . . . , n − 1, se llaman coeficientes de la ecuaci´on. Observaci´ on 2.5.1. Comenzamos el estudio de este tipo de ecuaciones con el caso n = 2.

La ecuaci´ on de orden 2 Observaci´ on 2.5.2. Consideremos la ecuaci´on diferencial de segundo orden siguiente: y 00 + ay 0 + by = 0

(2.10)

Supongamos que la ecuaci´on (2.10) posee una soluci´on y = y (x) de la forma y (x) = eλx , con λ ∈ R. Note que: y 0 (x) = λeλx



y 00 (x) = λ2 eλx

Por tanto, reemplazando lo anterior en la ecuaci´on (2.10) obtenemos: λ2 eλx + aλeλx + beλx = 0 As´ı, en vista de la ecuaci´on anterior, las condiciones sobre λ ∈ R de modo que y (x) = eλx sea soluci´on es que: λ2 + aλ + b = 0

(2.11)

Esto es, que λ sea ra´ız de la ecuaci´on de segundo grado anterior. La ecuaci´on (2.11) se llama ecuaci´on caracter´ıstica asociada a la ecuaci´on (2.10). Por tanto, tenemos tres casos dados por ∆ = a2 − 4b, el discriminante de (2.11). En efecto:

158

Apuntes Mat023 (versi´on preliminar actualizada 23-05-2014)

1. Si ∆ = a2 − 4b > 0, entonces la ecuaci´on (2.11), tiene dos soluciones reales λ1 y λ2 distintas, dadas por: λ1 =

−a +



a2 − 4b 2



λ2 =

−a −

√ a2 − 4b 2

las que a su vez definen las funciones: u1 (x) = eλ1 x

u2 (x) = eλ2 x



Luego, como u2 /u1 no es constante, la soluci´on general de la ecuaci´on diferencial lineal homog´enea de orden 2 es: y (x) = C1 eλ1 x + C2 eλ2 x Ejemplo 2.5.1. Resuelva la ecuaci´on diferencial: y 00 + y 0 − 2y = 0 Soluci´ on. En este caso la ecuaci´on caracter´ıstica es λ2 + λ − 2 = 0 ⇔ (λ + 2) (λ − 1) = 0 luego tenemos dos soluciones u1 (x) = e−2x y u1 (x) = ex de y 00 + y 0 − 2y = 0 las cuales son linealmente independientes pues −2x x  e e −2x x = 3e−x W e ,e = −2x x e −2e la soluci´on general de la ecuaci´on ser´a entonces y (x) = c1 ex + c2 e−2x 2. Si ∆ = a2 − 4b = 0, entonces la ecuaci´on (2.11) tiene una ra´ız real λ de multiplicidad 2, dada por: λ=− 159

a 2

Apuntes Mat023 (versi´on preliminar actualizada 23-05-2014)

Luego, u1 (x) = eλx es una soluci´on, pero dim ker L = 2 por tanto, debemos hallar otra soluci´on u2 (x) linealmente independiente con u1 (x), podemos usar la f´ormula de Abel para la segunda soluci´on u2 (x) = e

− a2 x

Z

R

e− adx dx e−ax

a

= xe− 2 x a

se sigue que la segunda soluci´on es u2 (x) = xe− 2 x y as´ı la soluci´on general es y (x) = c1 eλx + c2 xeλx donde λ es la ra´ız repetida de la ecuaci´on caracter´ıstica. Ejemplo 2.5.2. Resuelva la ecuaci´on diferencial: y 00 + 4y 0 + 4y = 0 Soluci´ on. La ecuaci´on caracter´ıstica es λ2 + 4λ + 4 = 0 la cual tiene ra´ız repetida λ = −2 se sigue que la soluci´on general de la ecuaci´on es y (x) = c1 e−2x + c2 xe−2x 3. Si ∆ = a2 − 4b < 0, entonces la ecuaci´on (2.11) no tiene ra´ıces reales. Sin embargo, sea: λ = α + iβ una ra´ız compleja de (2.11). Recordemos el siguiente teorema: Teorema 2.5.1 (F´ormula de Euler). Sea θ ∈ R, entonces: eiθ = cos θ + i sin θ Entonces: eλx = e(α+iβ)x = eαx eiβx = eαx {cos (βx) + i sin (βx)} = eαx cos (βx) + i {eαx sin (βx)} Consideremos ahora: 160

Apuntes Mat023 (versi´on preliminar actualizada 23-05-2014)

Teorema 2.5.2. Sean L (y) = 0 una ecuaci´on diferencial lineal de orden 2 e y (x) = u (x) + iv (x) una soluci´on compleja de la ecuaci´on tal que u = 6 v. Entonces, u (x) y v (x) son dos soluciones reales y linealmente independientes de L (y) = 0. De: eλx = eαx cos (βx) + i eαx sin (βx) se obtiene que: u1 (x) = eαx cos (βx)



u2 (x) = eαx sin (βx)

son dos soluciones reales y linealmente independientes de la ecuaci´on diferencial lineal: y 00 + ay 0 + by = 0 ¯

Observaci´ on 2.5.3. Debemos notar que eλx y eλx son efectivamente funciones linealmente independientes como C espacio vectorial. Ejemplo 2.5.3. Resuelva la ecuaci´on: y 00 + 2y 0 + 5y = 0 Soluci´ on. La ecuaci´on caracter´ıstica es λ2 + 2λ + 5 = 0 la cual tiene ra´ıces λ = −1 ± 2i se sigue que la soluci´on general es y (x) = c1 e−x cos (2x) + c2 e−x sin (2x)

Ahora daremos una segunda mirada a la ecuaci´on de segundo orden, en relaci´on al operador diferencial asociado. En general los operadores diferenciales no conmutan, esto es, si L1 y L2 son dos operadores diferenciales en general no es cierto que L1 L2 = L2 L1 161

Apuntes Mat023 (versi´on preliminar actualizada 23-05-2014)

sin embargo, si los operadores diferenciales son con coeficientes constantes, estos se comportan igual que polinomios. Por ejemplo, considere los operadores diferenciales D − 1 y D + 2 entonces (D − 1) (D + 2) y = (D − 1) (y 0 + 2y) = y 00 + 2y 0 − y 0 − 2y = y 00 + y 0 − 2y =

 D2 + D − 2 y

por otro lado (D + 2) (D − 1) y = (D + 2) (y 0 − y) = y 00 − y 0 + 2y 0 − 2y = y 00 + y 0 − 2y =

 D2 + D − 2 y

se sigue  (D − 1) (D + 2) = (D + 2) (D − 1) = D2 + D − 2 note que si (D − 1) y = 0 entonces (D + 2) (D − 1) y = 0 y as´ı y ∈ ker (D2 + D − 2) en otras palabras una soluci´on de la ecuaci´on y0 − y = 0 tambi´en es soluci´on de y 00 + y 0 − 2y = 0 de manera similar, si (D + 2) y = 0 entonces (D − 1) (D + 2) y = 0 y as´ı y ∈ ker (D2 + D − 2) en otras palabras una soluci´on de la ecuaci´on y 0 + 2y = 0 tambi´en es soluci´on de y 00 + y 0 − 2y = 0 pero la soluciones y 0 − y = 0 son u1 (x) = cex y las soluciones de y 0 + 2y = 0 son u2 (x) = ke−2x (son de primer orden de variables separadas) as´ı podemos obtener las 162

Apuntes Mat023 (versi´on preliminar actualizada 23-05-2014)

soluciones ex y e−2x de la ecuaci´on y 00 + y 0 − 2y = 0 y como son linealmente independientes, se sigue que la soluci´on general de la ecuaci´on es dada por y (x) = c1 ex + c2 e−2x son estas ideas las que utilizaremos para resolver la ecuaci´on diferencial lineal de orden n.

La ecuaci´ on de orden superior Observaci´ on 2.5.4. En general, respecto de la ecuaci´on diferencial: y (n) + an−1 y (n−1) + an−2 y (n−2) + · · · + a1 y 0 + a0 y = Q (x) tenemos: Definici´ on 2.5.2. Sean I ⊆ R un intervalo abierto y L : C (n) (I) → C (I) un operador diferencial de orden n a coeficientes constantes, es decir: L = Dn + an−1 Dn−1 + · · · + a1 D + a0 1

(2.12)

El polinomio caracter´ıstico de la ecuaci´on diferencial L (y) = 0 es el polinomio en R [λ] siguiente: fL (λ) = λn + an−1 λn−1 + · · · a1 λ + a0 Diremos, adem´as, que la ecuaci´ on caracter´ıstica asociada a la ecuaci´on diferencial L (y) = 0 es la ecuaci´on siguiente: fL (λ) = 0 O bien: λn + an−1 λn−1 + · · · a1 λ + a0 = 0 Ejemplo 2.5.4. La ecuaci´on diferencial: y (4) − 3y 00 + y 0 − 3y = 0 tiene polinomio caracter´ıstico: f (λ) = λ4 − 3λ2 + λ − 3 Note que, el operador diferencial asociado es: L = D4 − 3D2 + D − 3 163

Apuntes Mat023 (versi´on preliminar actualizada 23-05-2014)

Ejemplo 2.5.5. La ecuaci´on diferencial: y (5) + 2y (4) − 3y 000 − y 00 − 2y 0 + 3y = 0 tiene polinomio caracter´ıstico: f (λ) = λ5 + 2λ4 − 3λ3 − λ2 − 2λ + 3 Note que:  D5 + 2D4 − 3D3 − D2 − 2D + 3 = D2 + D + 1 (D − 1)2 (D + 3) Teorema 2.5.3. Sean L y S dos operadores a coeficientes constantes con polinomios caracter´ısticos fL y fS , respectivamente, y sea α ∈ R, entonces: 1. L = S, si y solo si, fL = fS 2. fL+S = fL + fS 3. fLS = fL · fS 4. fαL = αfL Observaci´ on 2.5.5. Del teorema anterior se deduce que toda relaci´on algebraica constitu´ıda por sumas, productos y productos por escalares de polinomios caracter´ısticos son tambi´en v´alidas para los operadores L y S. En particular, se tienen las leyes conmutativas: L+S =S+L



LS = SL



α (βL) = β (αL)

Note, adem´as, que si L y S son operadores tales que LS = SL, entonces: ker L + ker S ⊆ ker (LS)

(2.13)

Por otro lado, recordemos que en el conjunto R [x] los elementos irreducibles m´onicos son los polinomios de la forma x − c y x2 + αx + β, con α2 − 4β < 0. Luego, el Teorema ´ Fundamental del Algebra implica el siguiente resultado: Teorema 2.5.4. Sea p (x) ∈ R [x]. Entonces, p (x) se factoriza como p (x) =

r Y i=1

ni

(x − λi ) ·

s Y

x2 + αj x + βj

mj

(2.14)

j=1

donde λi son las ra´ıces reales de p (x) con multiplicidad ni , para cada i = 1, 2, . . . , r, αj2 − 4βj < 0 y mj es la multiplicidad de la ra´ız compleja asociada, para cada j = 1, 2, . . . , s. 164

Apuntes Mat023 (versi´on preliminar actualizada 23-05-2014)

Obtenemos lo siguiente para los operadores diferenciales dados por la factorizaci´on de la ecuaci´on caracter´ıstica en (2.14): 1. ker (D − λ)m =



eλx , xeλx , x2 eλx , . . . , xm−1 eλx



2. ker (D − α)2 + β 2

m

=



eαx cos (βx) , xeαx cos (βx) , . . . , xm−1 eαx cos (βx) ; eαx sin (βx) ,

, xeαx sin (βx) , . . . , xm−1 eαx sin (βx) Por tanto, mediante el uso del wronskiano y la ecuaci´on (2.14), la soluci´on general de la ecuaci´on diferencial est´a dada por las combinaciones lineales (con las constantes indexadas adecuadamente) de los sistemas fundamentales de soluciones de cada factor. Ejemplo 2.5.6. Resuelva la ecuaci´on diferencial: y 000 − y 00 − 8y 0 + 12y = 0 Soluci´ on. Consideremos la ecuaci´on caracter´ıstica: λ3 − λ2 − 8λ + 12 = 0 Como: λ3 − λ2 − 8λ + 12 = (λ + 3) (λ − 2)2 se sigue que ker (D − 2)2 ⊂ ker D3 − D2 − 8D + 12



ker (D + 3) ⊂ ker D3 − D2 − 8D + 12



pero ker (D − 2)2 = h{e2x , xe2x }i y ker (D + 3) = h{e−3x }i luego 

 e2x , xe2x , e−3x ⊂ ker D3 − D2 − 8D + 12

sabemos que el espacio ker (D3 − D2 − 8D + 12) tiene dimensi´on tres luego, la soluci´on general de la ecuaci´on y 000 − y 00 − 8y 0 + 12y = 0 es dada por yG (x) = c1 e2x + c2 xe2x + c3 e−3x 165

Apuntes Mat023 (versi´on preliminar actualizada 23-05-2014)

Ejemplo 2.5.7. Suponga que L (y) = 0 es una ecuaci´on diferencial con operador: L = D5 + 2D4 − 3D3 − D2 − 2D + 3 Hallar la soluci´on general. Soluci´ on. Note que: L = D5 + 2D4 − 3D3 − D2 − 2D + 3  = D2 + D + 1 (D − 1)2 (D + 3) Ahora bien: * ker D2 + D + 1 = 

e−x/2 cos

√ ! 3x ; e−x/2 sin 2



3x 2

!+

ker (D − 1)2 = hex ; xex i y adem´as:

ker (D + 3) = e−3x Por tanto, la soluci´on general de la ecuaci´on L (y) = 0 est´a dada por: √ ! √ ! 3x 3x y (x) = C1 e−x/2 cos + C2 e−x/2 sin + C3 ex + C4 xex + C5 e−3x 2 2 Ejemplo 2.5.8. Hallar la soluci´on general de una ecuaci´on diferencial lineal a coeficientes constantes cuya ecuaci´on caracter´ıstica es: λ5 − 2λ4 + 6λ3 − 9λ2 + 8λ − 4 = 0 sabiendo que y = ex/2 cos

√

3 x 2



es una soluci´on de dicha ecuaci´on.

Soluci´ on. Sea ε una ecuaci´on diferencial lineal a coeficientes constantes homog´enea de la cual se sabe que: λ5 − 2λ4 + 6λ3 − 9λ2 + 8λ − 4 = 0 es la ecuaci´on caracter´ıstica asociada, y que adem´as: √ ! 3 y = ex/2 cos x 2 166

(2.15)

Apuntes Mat023 (versi´on preliminar actualizada 23-05-2014)

x/2

es soluci´on de la ecuaci´on. Ahora bien, como y = e

cos

√

3 x 2



es soluci´on:

√ 3 1 i λ= + 2 2 es una ra´ız de la ecuaci´on caracter´ıstica (2.15), entonces: √ 1 3 λ= − i 2 2 tambi´en es ra´ız, luego la expresi´on: ( √ !) ( 1 3 λ− + i λ− 2 2

√ !) 1 3 − i = λ2 − λ + 1 2 2

factoriza a la ecuaci´on caracter´ıstica. As´ı:   λ5 − 2λ4 + 6λ3 − 9λ2 + 8λ − 4 = λ3 − λ2 + 4λ − 4 λ2 − λ + 1 Notamos, adem´as, que λ = 1 es ra´ız de la ecuaci´on λ3 − λ2 + 4λ − 4 = 0. Entonces, por el teorema del factor, se tiene que:  λ3 − λ2 + 4λ − 4 = λ2 + 4 (λ − 1) entonces λ5 − 2λ4 + 6λ3 − 9λ2 + 8λ − 4 = λ2 − λ + 1



 λ2 + 4 (λ − 1)

Por tanto, la ecuaci´on (2.15) tiene las siguientes ra´ıces: ( ) √ √ 1 3 1 3 + i, − i, 1, 2i, −2i 2 2 2 2 Finalmente, la soluci´on general de la ecuaci´on ε est´a dada por: ( √ ! √ !) 3 3 y (x) = C1 ex + ex/2 C2 cos x + C3 sin x + {C4 cos 2x + C5 sin 2x} 2 2

M´ etodo de variaci´ on de par´ ametros Observaci´ on 2.6.1. En las secciones anteriores obtuvimos m´etodos para encontrar el espacio soluci´on de ecuaciones diferenciales, para los siguientes casos: 167

Apuntes Mat023 (versi´on preliminar actualizada 23-05-2014)

1. Es conocida una soluci´on particular y1 (x) de la ecuaci´on: y 00 + p1 (x) y 0 + p0 (x) y = 0 En particular, para la ecuaci´on anterior, obtuvimos que la soluci´on general est´a dada por: y (x) = C1 y1 (x) + C2 y2 (x) donde: Z y2 (x) = y1 (x)

R

e− p1 (x)dx dx y12 (x)

2. La ecuaci´on diferencial es de la forma: y (n) + an−1 y (n−1) + · · · + a1 y 0 + a0 y = 0 con ai ∈ R, para cada i = 0, 1, . . . , n − 1. En este caso, las soluciones dependen de las ra´ıces del polinomio caracter´ıstico: fL (λ) = λn + an−1 λn−1 + · · · + a1 λ + a0 Por otro lado, dada la estructura lineal de la ecuaci´on L (y) = Q, donde L es el operador diferencial asociado a la ecuaci´on y Q es una funci´on continua, entonces la soluci´on general de la ecuaci´on lineal se descompone como: y = yh + yp donde yh es la soluci´on general de la ecuaci´on L (y) = 0. El objetivo de esta secci´on es hallar una soluci´on particular yp para la ecuaci´on L (y) = Q. Observaci´ on 2.6.2. En este curso, se ven dos de tales m´etodos: el m´etodo de variaci´on de par´ametros y el m´etodo de los anuladores. Como se acostumbra, haremos los razonamientos para n = 2, y luego se presentar´a el resultado para n ≥ 2. Veamos primeramente: Teorema 2.6.1 (Variaci´on de par´ametros). Sean u1 y u2 dos soluciones linealmente independientes de la ecuaci´on: L (y) = y 00 + p1 (x) y 0 + p0 (x) y = 0 168

Apuntes Mat023 (versi´on preliminar actualizada 23-05-2014)

en un intervalo I ⊆ R. Entonces, la ecuaci´on no homog´enea L (y) = Q tiene una soluci´on particular de la forma: yp (x) = C1 (x) u1 (x) + C2 (x) u2 (x) donde: Z C1 (x) = −

u2 (x) Q (x) dx W (x)

Z ∧

C2 (x) =

u1 (x) Q (x) dx W (x)

(2.16)

y W (x) es el wronskiano de u1 (x) y u2 (x).

Demostraci´on. Supongamos que yp (x) = C1 (x) u1 (x) + C2 (x) u2 (x), luego: yp0 = C1 u01 + C2 u02 + (C10 u1 + C20 u2 ) yp00 = C1 u001 + C2 u002 + (C10 u01 + C20 u02 ) + (C10 u1 + C20 u2 )

0

Ahora bien, al evaluar yp en L se obtiene que: L (yp ) = yp00 + p1 (x) yp0 + p0 (x) yp 0

= (C10 u01 + C20 u02 ) + (C10 u1 + C20 u2 ) + p1 (x) (C10 u1 + C20 u2 ) pues todos los t´erminos que contienen a C1 (x) y C2 (x) desaparecen dado que L (u1 ) = L (u2 ) = 0. Como deseamos que L (yp ) = Q, entonces de la u ´ ltima ecuaci´on imponemos que: C10 u1 + C20 u2 = 0

C10 u01 + C20 u02 = Q



As´ı, las funciones C1 (x) y C2 (x), deben satisfacer que:   C 0 (x) u (x) + C 0 (x) u (x) = 1

1

2

2

0

 C 0 (x) u0 (x) + C 0 (x) u0 (x) = Q (x) 1 1 2 2 para cada x ∈ I. Sin embargo, note que el u´ltimo sistema tiene soluci´on si y solo si la matriz de coeficientes de ´el tiene determinante no nulo. Es decir, si: u1 (x) u2 (x) = 6 0 0 u1 (x) u02 (x) 169

Apuntes Mat023 (versi´on preliminar actualizada 23-05-2014)

para cada x ∈ I. Pero el determinante anterior es el wronskiano W (x) de u1 (x) y u2 (x), y como u1 , u2 son linealmente independientes W (x) = 6 0, para cada x ∈ I. Por lo tanto, el sistema tiene u ´nica soluci´on, y adem´as, por la regla de Cramer, obtenemos finalmente: 0 u2 (x) Q (x) u02 (x) u (x) Q (x) =− 2 C10 (x) = W (x) u1 (x) u2 (x) 0 u1 (x) u02 (x) y: 0 C1 (x) =

u1 (x) 0 0 u1 (x) Q (x) u (x) Q (x) = 1 W (x) u1 (x) u2 (x) u01 (x) u02 (x)

Integrando, se obtiene el resultado deseado. Observaci´ on 2.6.3. En base a los razonamientos en la demostraci´on del teorema anterior, tenemos: Teorema 2.6.2. Sean u1 , u2 , . . . , un una familia de n soluciones linealmente independientes de la ecuaci´on lineal de orden n: L (y) = y (n) + pn−1 (x) y (n−1) + · · · + p1 (x) y 0 + p0 (x) y = 0 definida en un intervalo abierto I ⊆ R. Entonces, la ecuaci´on no homog´enea L (y) = Q, tiene una soluci´on particular: yp (x) =

n X

Ci (x) ui (x)

i=1

donde Ci (x), con i = 1, 2, . . . , n son funciones que se obtienen como soluciones del sistema de ecuaciones:   C10 u1 + C20 u2 + · · · + Cn0 un      0 0 0 0 0 0    C1 u1 + C2 u2 + · · · + Cn un C10 u001 + C20 u002 + · · · + Cn0 u00n   ..   .      C 0 u(n−1) + C 0 u(n−1) + · · · + C 0 u(n−1) 1 1 2 2 n n 170

=

0

=

0

=

0 .. .

= Q (x)

(2.17)

Apuntes Mat023 (versi´on preliminar actualizada 23-05-2014)

Ejemplo 2.6.1. Resuelva la ecuaci´on diferencial: y 00 + y = tan x Soluci´ on. Consideremos un intervalo en el que el segundo miembro sea continuo, sea ´este (−a, a), siendo 0 < a < π/2. La ecuaci´on homog´enea tiene soluci´on general: yh (x) = C1 cos x + C2 sin x pues la ecuaci´on caracter´ıstica de dicha ecuaci´on homog´enea es: λ2 + 1 = 0 En particular, notamos que: cos x sin x W (x) = − sin x cos x

= cos2 x + sin2 x = 1

Entonces, en vista de las ecuaciones en (2.16) las funciones C1 (x) y C2 (x) satisfacen: Z sin x tan x C1 (x) = − dx W (x) Z = − sin x tan xdx = sin x − ln |sec x + tan x| y adem´as: Z C2 (x) =

cos x tan x dx W (x)

Z =

cos x tan xdx

= − cos x Por tanto, por el m´etodo de variaci´on de par´ametros, una soluci´on particular yp viene dada por: yp = C1 (x) cos x + C2 (x) sin x = sin x cos x − cos x ln |sec x + tan x| − sin x cos x = − cos x ln |sec x + tan x| 171

Apuntes Mat023 (versi´on preliminar actualizada 23-05-2014)

y la soluci´on general es: y = C1 cos x + C2 sin x + yp = C1 cos x + C2 sin x − cos x ln |sec x + tan x| Soluci´ on. Hallar la soluci´on general de la ecuaci´on diferencial: ex y − 2y + y = (1 − x)2 00

0

Soluci´ on. Considere la ecuaci´on diferencial: y 00 − 2y 0 + y =

ex (1 − x)2

Notamos que la ecuaci´on anterior es una ecuaci´on diferencial lineal a coeficientes constantes no homog´enea. As´ı, si y = y (x) es la soluci´on general, entonces: y (x) = yh (x) + yp (x) donde yh (x) es la soluci´on de la ecuaci´on homog´enea asociada e yp (x) es una soluci´on particular. Consideremos, entonces, la ecuaci´on homog´enea asociada: y 00 − 2y 0 + y = 0 Luego, la ecuaci´on caracter´ıstica: λ2 − 2λ + 1 = 0 tiene ra´ız λ = 1 con multiplicidad 2. Por tanto, la soluci´on homog´enea yh (x) est´a dada por: yh (x) = C1 ex + C2 xex Ahora bien, por el m´etodo de variaci´on de par´ametros, tenemos que la soluci´on particular yp (x) tiene la forma: yp (x) = C1 (x) ex + C2 (x) xex donde C1 (x) y C2 (x) son soluciones del sistema de ecuaciones:   0 C10 (x) ex + C20 (x) xex = ex  C10 (x) ex + C20 (x) (1 + x) ex = 2 (1−x)

172

Apuntes Mat023 (versi´on preliminar actualizada 23-05-2014)

Ahora bien, como el wronskiano: x e xex W (x) = ex (1 + x) ex es no nulo, por la regla de Cramer, se tiene que: 0 xex ex (1−x)2 (1 + x) ex 0 C1 (x) = W (x)



= e2x

=−

x (x − 1)2

y adem´as:

C20

(x) =

x e x e

0 ex (1−x)2

W (x)



=

1 (x − 1)2

As´ı: Z C1 (x) = −

x 1 {ln (x − 1) − x ln (x − 1) + 1} 2 dx = x−1 (x − 1)

y: Z

1 1 2 dx = − x−1 (x − 1) Por tanto, la soluci´on general y (x) de la ecuaci´on diferencial est´a dada por: ex xex y (x) = C1 ex + C2 xex + {ln (x − 1) − x ln (x − 1) + 1} − x−1 x−1 x x x = C1 e + C2 xe − e {ln (x − 1) + 1} C2 (x) =

Observaci´ on 2.6.4. Las integrales que aparecen en las f´ormulas (2.16) o al despejar C1 (x) y C2 (x) del sistema (2.17), son integrales indefinidas, luego las funciones quedan determinadas salvo una constante de integraci´on. Suponga entonces que tenemos: vi (x) = Ci (x) + Ki con Ki ∈ R, i = 1, 2, constantes de integraci´on. Luego, si u1 y u2 son las soluciones linealmente independientes de L (y) = 0, considere: y1 (x) = A1 u1 (x) + A2 u2 (x) + v1 (x) u1 (x) + v2 (x) u2 (x) = (A1 + K1 ) u1 (x) + (A2 + K2 ) u2 (x) + C1 (x) u1 (x) + C2 (x) u2 (x) Entonces: y1 (x) = α1 u1 (x) + α2 u2 (x) + C1 (x) u1 (x) + C2 (x) u2 (x) que era la soluci´on que ya ten´ıamos. 173

Apuntes Mat023 (versi´on preliminar actualizada 23-05-2014)

M´ etodo del anulador Definici´ on 2.7.1. Diremos que el operador diferencial L es un anulador o aniquilador de ϕ si se cumple L (ϕ) = 0. Ejemplo 2.7.1. El operador diferencial L = (D − 1)2 es un anulador de ϕ (x) = 2ex +5xex pues ker (D − 1)2 = h{ex , xex }i, es decir, toda combinaci´on lineal de ex y xex est´a en el n´ ucleo de (D − 1)2 . Notar que (D − 1)2 no es un anulador de x2 pues (D − 1)2 x2



= (D − 1) (D − 1) x2  = (D − 1) 2x − x2



= (2 − 2x) − 2x − x2



= x2 − 4x + 2 6≡ 0 Observaci´ on 2.7.1. Recordemos que ker L + ker L ⊆ ker (LS) para los operadores con coeficientes constantes. 2

Ejemplo 2.7.2. (D2 + 1) es anulador de x sin x y D2 es anulador de x luego D2 (D2 + 1)

2

es un anulador de 5x sin x − 3x Suponga que buscamos resolver la ecuaci´on L (y) = Q donde L es un operador con coeficientes constantes y LQ es un anulador de Q (con coeficientes constantes tambi´en) entonces L (y) = Q ⇒ LQ L (y) = LQ (Q) = 0 luego la soluci´on de L (y) = Q se encuentra entre las soluciones de la ecuaci´on homog´enea LQ L (y) = 0. Ejemplo 2.7.3. Resolver la ecuaci´on diferencial: y 00 − 5y 0 + 6y = xex

174

Apuntes Mat023 (versi´on preliminar actualizada 23-05-2014)

Soluci´ on. Notamos que el operador asociado a la ecuaci´on diferencial es: L = D2 − 5D + 6 = (D − 2) (D − 3) As´ı, la ecuaci´on diferencial puede escribirse como L (y) = Q, con Q (x) = xex . Por otro lado, la ecuaci´on homog´enea L (y) = 0 tiene soluciones linealmente independientes u1 (x) = e2x y u2 (x) = e3x . Sabemos que la soluci´on general de la ecuaci´on es de la forma: y (x) = C1 e2x + C2 e3x + yp (x) donde yp es una soluci´on de la ecuaci´on no homog´enea L (y) = xex . Notamos que: xex ∈ ker (D − 1)2 por tanto, aplicamos el operador diferencial (D − 1)2 a la ecuaci´on diferencial para obtener : (D − 1)2 (D − 2) (D − 3) (y) = (D − 1)2 (xex ) = 0 As´ı, hemos obtenido la ecuaci´on diferencial homog´enea: S (y) = (D − 1)2 (D − 2) (D − 3) (y) = 0 por tanto, la soluci´on general de S (y) = 0 es: yS (x) = aex + bxex + ce2x + de3x donde a, b, c y d son constantes reales. Ahora bien, se deben elegir los valores de a, b, c y d de tal modo que:  L aex + bxex + ce2x + de3x = xex sin embargo, ce2x + de3x ∈ ker L, luego bastar´a elegir a y b tales que: L (aex + bxex ) = xex as´ı, la ecuaci´on: L (aex + bxex ) = (D − 2) (D − 3) (aex + bxex ) = (2a − 3b) ex + 2bxex = xex 175

Apuntes Mat023 (versi´on preliminar actualizada 23-05-2014)

implica que a = 3/4 y b = 1/2. Finalmente: 1 3 yp (x) = ex + xex 4 2 y por tanto: yp

y z }|h { z3 }|1 { 2x 3x y (x) = C1 e + C2 e + ex + xex 4 2

Observaci´ on 2.7.2. Recalcamos que el operador (D − 1)2 se eligi´o de tal modo que: xex ∈ ker (D − 1)2 Luego, al aplicar dicho operador a la ecuaci´on diferencial L (y) = xex , vemos que el t´ermino de la derecha se anula bajo la aplicaci´on de (D − 1)2 . Por este hecho, el m´etodo anterior se llama el m´etodo del anulador. El m´etodo del anulador, funciona correctamente para ecuaciones que son anulables, en el sentido de que son funciones que aparecen como soluciones de ecuaciones diferenciales a coeficientes constantes. En particular, funciones que aparecen como combinaciones lineales de funciones de la forma: xm−1 eαx



xm−1 eαx cos βx



xm−1 eαx sin βx

donde m es un n´ umero natural y α y β son constantes reales. A continuaci´on detallamos algunas de las funciones anteriores con sus respectivos anuladores: Funci´on

Anulador

y = xm−1

Dm

y = eαx

D−α

y = xm−1 eαx

(D − α)m

y = cos βx o bien y = sin βx

D2 + β 2

y = xm−1 cos βx o bien y = xm−1 sin βx

(D2 + β 2 )

y = eαx cos βx o bien y = eαx sin βx

D2 − 2αD + (α2 + β 2 )

y = xm−1 eαx cos βx o bien y = xm−1 eαx sin βx

{D2 − 2αD + (α2 + β 2 )}

m

m

Observaci´ on 2.7.3. Un resultado que utiliza las propiedades lineales del operador diferencial asociado a una ecuaci´on, es el principio de superposici´on, que facilita en algunos casos el c´alculo de la soluci´on particular. Tenemos, entonces: 176

Apuntes Mat023 (versi´on preliminar actualizada 23-05-2014)

Teorema 2.7.1 (Principio de superposici´on). Sean L un operador diferencial no necesariamente a coeficientes constantes y f1 , f2 funciones continuas. Suponga que yi es soluci´on de la ecuaci´on diferencial L (y) = fi , con i = 1, 2. Entonces: y = y1 + y2 es soluci´on de la ecuaci´on L (y) = f1 + f2 . Este principio nos permite separar la busqueda de la soluci´on particular. Ejemplo 2.7.4. Resolver (D − 1) (D − 2) y = ex + x o en otras palabras

d2 y dy − 3 + 2y = ex + x 2 dx dx

Soluci´ on. Sabemos que (D − 1) (ex ) = 0 y D2 (x) = 0 se sigue que (D − 1) D2 (ex + x) = 0 as´ı (D − 1) D2 (D − 1) (D − 2) y = 0 D2 (D − 1)2 (D − 2) = 0 luego yp (x) = c1 + c2 x + c3 ex + c4 xex + c5 e2x y como (D − 1) (D − 2) yp (x) = ex + x

177

Apuntes Mat023 (versi´on preliminar actualizada 23-05-2014)

se sigue   (D − 1) (D − 2) c1 + c2 x + c3 ex + c4 xex + c5 e2x = (D − 1) (D − 2) [c1 + c2 x] + (D − 1) (D − 2) [c4 xex ]  = D2 − 3D + 2 [c1 + c2 x] + (D − 1) (D − 1 − 1) [c4 xex ] = 2c1 − 3c2 + 2c2 x − (D − 1) [c4 xex ] = 2c1 − 3c2 + 2c2 x − (c4 ex + c4 xex − c4 xex ) = 2c1 − 3c2 + 2c2 x − c4 ex de esto obtenemos que las constantes cumplen 2c1 − 3c2 = 0 2c2 = 1 −c4 = 1 se sigue c4 = −1, c2 = 1/2, c1 = 3/4 as´ı yG (x) = (3/4) + (1/2) x − xex + c3 ex + c5 e2x   = [(3/4) + (1/2) x − xex ] + c3 ex + c5 e2x = yp (x) + yh (x) es la soluci´on general. Note que d2 ((3/4) + (1/2) x) dx2 d −3 ((3/4) + (1/2) x) + 2 ((3/4) + (1/2) x) dx = x d2 d x (D − 1) (D − 2) (−xe ) = (−xex ) − 3 (−xex ) + 2 (−xex ) 2 dx dx = 3ex (x + 1) − ex (x + 2) − 2xex

(D − 1) (D − 2) ((3/4) + (1/2) x) =

= ex luego vale el principio de superposici´on. 178

Apuntes Mat023 (versi´on preliminar actualizada 23-05-2014)

Veamos como podemos trabajar, en algunos casos, si no conocemos el anulador: Ejemplo 2.7.5. Hallar la soluci´on general de la ecuaci´on diferencial: y 00 − 3y 0 + 2y = xex+x

2

Soluci´ on. Notamos que: L = D2 − 3D + 2 = (D − 1) (D − 2) Buscamos, primeramente, una soluci´on particular yp . Considere el cambio de variables: u = (D − 2) y

(2.18)

la ecuaci´on queda: (D − 1) u = xex+x

2

Mediante la f´ormula de Leibnitz, obtenemos: Z 1 2 2 −x ue = xex dx + C = ex + C 2 Ahora bien, como buscamos yp , podemos elegir C = 0. As´ı: 1 2 u = ex+x 2 Sustituyendo en (2.18), se obtiene: 1 2 (D − 2) y = ex+x 2 Luego, nuevamente por la f´ormula de Leibnitz: Z 1 2x 2 yp = e ex −x dx 2 Finalmente, la soluci´on general de la ecuaci´on diferencial es: Z 1 2x 2 x 2x y = C1 e + C2 e + e ex −x dx 2 Finalmente, como una aplicaci´on de los m´etodos de variaci´on de par´ametros y del anulador resolveremos la ecuaci´on de Euler. 179

Apuntes Mat023 (versi´on preliminar actualizada 23-05-2014)

Definici´ on 2.7.2. Una ecuaci´ on de Euler es una ecuaci´on de la forma: (ax + b)n y (n) + An−1 (ax + b)n−1 y (n−1) + · · · + A1 (ax + b) y 0 + A0 y = Q (x)

(2.19)

donde ax + b > 0 y A0 , A2 , . . . , An−1 son constantes cualesquiera. Observaci´ on 2.7.4. El m´etodo de resoluci´on de la ecuaci´on de Euler consiste en considerar el cambio de variables: ax + b = et Como se puede observar, el cambio de variable es un cambio en la variable independiente. As´ı, por la regla de la cadena: dy dt dy dy = = ae−t dx dt dx dt      2  d2 y d dy d dt d y dy 2 −2t −t dy = =a e − = ae dx2 dx dx dt dt dx dt2 dt  2      2  3 3 dy d dy d2 y dy d d y dy dy 2 −2t 3 −3t = − −3 2 +2 = ae =a e dx3 dx dt dt dt2 dt dt3 dt dt y as´ı sucesivamente. Reemplazando todas las derivadas de orden superior en la ecuaci´on de Euler en (2.19) se obtiene la siguiente ecuaci´on a coeficientes constantes: ˜ (t) y (n) (t) + Bn−1 y (n−1) (t) + · · · + B1 y 0 (t) + B0 y (t) = Q Ejemplo 2.7.6. Resuelva la ecuaci´on diferencial: x2 y 00 + xy 0 + y = 1 Soluci´ on. Utilizando el cambio de variables x = et , se tiene que:  2  dy d2 y d y dy −t dy −2t =e ∧ =e − dx dt dx dt2 dt al reemplazar en la ecuaci´on original, obtenemos: d2 y +y =1 dt2 luego: y = C1 cos t + C2 sin t + 1 donde yp = 1, se obtiene inmediatamente por medio del m´etodo del anulador. Por tanto: y = C1 cos (ln x) + C2 sin (ln x) + 1 es la soluci´on general de la ecuaci´on diferencial. 180

Apuntes Mat023 (versi´on preliminar actualizada 23-05-2014)

Ejemplo 2.7.7. Resolver las ecuaci´on de Euler x2 y 00 + 2xy 0 − 2y = 0 Soluci´ on. Haciendo el cambio de variables x = et se sigue   2 d y dy 2 00 xy = − dt2 dt dy xy 0 = dt entonces la ecuaci´on se transforma en D (D − 1) y + 2Dy − 2y = 0 donde y = y (et ) entonces  D2 + D − 2 y = 0 ⇔ (D + 2) (D − 1) y = 0 se sigue  y et = c1 e−2t + c2 et se sigue y (x) =

c1 + c2 x x2

En las ecuaciones diferenciales de orden superior (al igual que en las de primer orden) podemos hacer los cambios de variables que consideremos convenientes, sin embargo, cualquier cambio de variables no asegura el poder resolver la ecuaci´on:

Ejemplo 2.7.8. Haciendo el cambio de variables x = t3 resolver la ecuaci´on 9x4/3

 √ d2 y 2/3 dy 3 + 6 x − 9x + 2y = x2/3 2 dx dx

Soluci´ on. Usando el cambio de variables dy dy dt = dx dt dx  2 2 dy d2 y dt dy d2 t = + dx2 dt2 dx dt dx2

181

Apuntes Mat023 (versi´on preliminar actualizada 23-05-2014)

pero dt 1 −2/3 = x dx 3 d2 t −2 −5/3 x = 2 dx 9 se sigue  1 −2 t 3 2    d2 y dy −2 −5 d2 y 1 −2 + t t = dx2 dt2 3 dt 9 dy dy = dx dt



reemplazando 9t

4

d2 y dt2



1 −2 t 3

esto es

2

dy + dt



−2 −5 t 9

! + 6t − 9t

2





dy dt



1 −2 t 3

 d2 y dy dy dy −1 + −2t − 3 + 2y = t2 + 2t−1 2 dt dt dt dt

es decir,

dy d2 y − 3 + 2y = t2 2 dt dt

esta ecuaci´on es (D − 2) (D − 1) y = t2 aplicamos anuladores D3 (D − 2) (D − 1) y = 0 la soluci´on es de la forma y = αe2t + βet + C1 + C2 t + C3 t2 determinamos las constantes C1 , C2 y C3  D2 − 3D + 2 C1 + C2 t + C3 t2 = t2 2C3 − 3C2 − 6C3 t + 2C1 + 2C2 t + 2C3 t2 = t2 obtenemos el sistema 2C3 − 3C2 + 2C1 = 0 −6C3 + 2C2 = 0 2C3 = 1 182



+ 2y = t2

Apuntes Mat023 (versi´on preliminar actualizada 23-05-2014)

que tiene soluci´on C1 = 74 , C2 = 32 , C3 =

1 2

as´ı

y = αe2t + βet +

7 3 1 + t + t2 4 2 2

volvemos a la variable original y la soluci´on es y (x) = αe2

√ 3

x

+ βe

√ 3

x

+

1 7 3√ + 3 x + x2/3 4 2 2

con α, β ∈ R. Ejemplo 2.7.9. Use z = sin (x) para resolver la ecuaci´on y 00 + tan (x) y 0 + cos2 (x) y = sin (x) cos2 (x) Soluci´ on. Las derivadas son con respecto a x la idea es cambiar las derivadas ahora con respecto a z, para ello usamos la regla de la cadena: Notemos que dy dy dx = dz dx dz o bien dy dy dz = dz dx dx reemplazando  dy dy dy = dx = sec x dz (cos x) dx luego d dz



dy dz



d ((sec x) y 0 ) dz     2  dx dy d y dx = sec x tan x + sec x dz dx dx2 dz   sin x 1 dy 1 d2 y = + cos2 x cos x dx cos2 x dx2 =

se sigue 2

cos x



d2 y dz 2

 = tan x

dy d2 y + dx dx2

reemplazando en la ecuaci´on original    d2 y  2 2 cos x + cos x y=0 dz 2 183

Apuntes Mat023 (versi´on preliminar actualizada 23-05-2014)

luego

d2 y +y =0 dz 2

se sigue y (z) = c1 sin z + c2 cos z es decir la soluci´on general de la es y (x) = c1 sin (sin x) + c2 cos (sin x) para c1 , c2 ∈ R

Movimiento vibratorio Observaci´ on 2.8.1. Consideremos un sistema mec´anico compuesto por un resorte de longitud inicial l0 con su extremo superior sujeto firmemente y una masa m atada en su extremo inferior. Nos interesa hallar la posici´on x (t) de la masa en cualquier instante t. M´as precisamente, nos interesa hallar la ecuaci´on que describa el movimiento de la part´ıcula a la cual se le ha dado un desplazamiento x0 y una velocidad inicial v0 . Para ello consideramos los siguientes supuestos: 1. La masa se mueve a lo largo de la vertical que pasa por el centro de gravedad de la masa y en el sistema de referencia x = 0 indica la posici´on de reposo de la masa en el resorte. 2. En cualquier tiempo t la magnitud de la fuerza ejercida sobre la masa es proporcional a la diferencia entre la longitud l del resorte y su posici´on de equilibrio l0 . La constante positiva de proporcionalidad k se llama constante del resorte, y el principio anterior se conoce como la ley de Hooke. Recordemos que la segunda ley del movimiento de Newton establece que la sumatoria de fuerzas que act´ ua sobre la part´ıcula es igual a la variaci´on instant´anea del momemtum mv respecto del tiempo, pero como la masa m es constante, se tiene: X

Fi = m a

i

donde a (t) =

d2 x(t) . dt2

184

Apuntes Mat023 (versi´on preliminar actualizada 23-05-2014)

Suponemos, por ahora, que la u ´ nica fuerza que act´ ua sobre la part´ıcula es la fuerza restauradora inducida por el resorte, es decir, no se consideran las fuerzas de fricci´on, por ejemplo. Por la ley de Hooke, tenemos que: m

d2 x = −kx dt2

y el signo negativo es debido a que la fuerza restauradora del resorte se opone al movimiento. Asumiendo, que la posici´on inicial (es decir, en t = 0) de la masa es x0 y que su velocidad inicial es v0 , obtenemos el siguiente problema de valor inicial: x00 +

k x = 0, m

x (0) = x0 ,

x0 (0) = v0

(2.20)

La ecuaci´on caracter´ıstica de la ecuaci´on diferencial anterior es: λ2 +

k =0 m

As´ı: λ = ± ω0 en donde ω0 =

p k/m, luego: x (t) = C1 cos ω0 t + C2 sin ω0 t

Usando las condiciones iniciales, obtenemos que C1 = x0 y C2 = v0 /ω0 . Por tanto: x (t) = x0 cos ω0 t + (v0 /ω0 ) sin ω0 t = A sin (ω0 t + φ) donde A =

q

x20 + (v0 /ω0 )2 y φ es tal que tan φ = x0 ω0 /v0 .

Definici´ on 2.8.1. El movimiento de la masa m bajo las condiciones anteriores, se denomina movimiento arm´ onico simple. La frecuencia natural f del movimiento es el n´ umero de oscilaciones completas por unidad de tiempo: f=

ω0 2π

Ejemplo 2.8.1. Considere un resorte, sujeto en su extremo superior, que sostiene una pesa de 10 libras en su extremo inferior. Suponga que la pesa estira el resorte 6 pulgadas. 185

Apuntes Mat023 (versi´on preliminar actualizada 23-05-2014)

Halle la ecuaci´on del movimiento de la pesa si ´esta se lleva a una posici´on de 4 pulgadas por debajo de su punto de equilibrio y se suelta. Calcule la ecuaci´on del movimiento su amplitud y la frecuencia natural. Ayuda: Considere que 1 [pie] = 12 [pulgadas] y que g = 32

h

pie seg2

i

.

Observaci´ on 2.8.2. En los supuestos que dan origen al movimiento arm´onico simple no se consideran las fuerzas, que en una situaci´on m´as realista, act´ uan sobre la masa. Considerando por ejemplo resistencia del medio, ya sea por acci´on del aire o realizaci´on de la oscilaci´on en un medio viscoso, es que se tienen las vibraciones amortiguadas. El supuesto es simple, la fuerza amortiguadora del movimiento se presupone proporcional a la velocidad del movimiento, as´ı el problema de valor inicial de la ecuaci´on (2.20) queda como: x00 = −

c k x − x0 m m

(2.21)

con las condiciones iniciales x (0) = x0 y x0 (0) = v0 . La ecuaci´on caracter´ıstica de la ecuaci´on (2.21) es: λ2 +

k c λ+ =0 m m

cuya soluci´on es: λ=

−c ±

√ c2 − 4km 2m

Por tanto, el comportamiento del movimiento depende del valor que tome el discriminante: ∆ = c2 − 4km Tenemos, por tanto, tres casos: Caso 2.8.1 (c2 − 4km > 0). La soluci´on de la ecuaci´on (2.21) en este caso est´a dada por: √ √     −c + c2 − 4km −c − c2 − 4km x (t) = C1 exp + C2 exp 2m 2m √ Note que 4km < c2 , lo que implica que c2 − 4km < c, y entonces: −c +



c2 − 4km < 0



−c −

As´ı, en este caso tenemos que: l´ım x (t) = 0

t→∞

186

√ c2 − 4km < 0

Apuntes Mat023 (versi´on preliminar actualizada 23-05-2014)

Caso 2.8.2 (c2 − 4km = 0). La soluci´on de la ecuaci´on (2.21) es: x (t) = (C1 + C2 t) e(−c/2m)t y al igual que en el caso anterior: l´ım x (t) = 0

t→∞

Observaci´ on 2.8.3. Entonces, se observa por la forma de las soluciones que si: c2 − 4km ≥ 0 no se producen oscilaciones en el sistema mec´anico masa-resorte considerando amortiguaci´on. Este tipo de movimiento t´ıpicamente ocurre en un medio viscoso como agua o aceite. Caso 2.8.3 (c2 − 4km < 0). Como (−c/2m) 6= 0, obtenemos: √ √   c  4km − c2 4km − c2 t + C2 sin t x (t) = exp − C1 cos 2m 2m 2m Observaci´ on 2.8.4. Finalmente, podemos suponer una fuerza g = g (t) aplicada sobre extremo superior del resorte (que inicialmente se supuso sujeto firmemente, esto es con g ≡ 0) en la direcci´on del movimiento de la masa m. El movimiento de la masa obtenido bajo este supuesto se conoce como vibraci´ on forzada. Una vibraci´on forzada tambi´en puede contener fuerzas amortiguadoras. Este es el caso m´as general del movimiento vibratorio, el problema de valor inicial queda en su forma m´as general como: x00 +

c 0 k g (t) x + x= m m m

(2.22)

con x (0) = x0 e x0 (0) = v0 . Ejemplo 2.8.2. Considere un resorte que en su extremo superior est´a sometido a una fuerza externa: f (t) =

10 sin t 32

y sostiene, en su parte inferior una masa de 10 libras. Suponga que la masa estira el resorte 6 pulgadas. Hallar la ecuaci´on del movimiento de la masa si ´esta se lleva a una posici´on de 4 pulgadas por debajo del punto de equilibrio y se suelta.

187

Apuntes Mat023 (versi´on preliminar actualizada 23-05-2014)

Observaci´ on 2.8.5. Un caso muy importante es cuando g es una funci´on peri´odica, es decir, supongamos que: g (t) = F0 sin ωt As´ı, la ecuaci´on (2.22) queda: x00 +

c 0 k F0 x + x= sin ωt m m m

(2.23)

Sabemos que el anulador de g (t) = F0 sin ωt es el operador: D2 + ω 2 k Aplicando este u ´ltimo operador al operador D2 + mc D + m , obtenemos:    F0  k c 2 2 2 · x = D2 + ω 2 · sin ωt = 0 D +ω D + D+ m m m

Por tanto, la ecuaci´on (2.23) tiene una soluci´on particular de la forma: xp = b1 cos ωt + b2 sin ωt donde b1 y b2 son soluciones del sistema:   (ω 2 − ω 2 ) b + cω b = 1 0 m 2  − cω b1 + (ω 2 − ω 2 ) b2 = 0 m donde ω0 =

q

k m

(2.24)

0 F0 m

. As´ı: b1 = − b2 =

F0 cω 2

m2 (ω02 − ω 2 ) + (cω)2 F0 m (ω02 − ω 2 ) 2

m2 (ω02 − ω 2 ) + (cω)2

As´ı, a partir de lo anterior y mediante el procedimiento de transformaci´on a sinusoide de la ecuaci´on (2.24) encontramos que: xp = A sin (ωt + φ) donde la amplitud A est´a dada por: F0 /k A = s 2  2 2  1 − ωω0 + 2 cc0 ωω0 188

Apuntes Mat023 (versi´on preliminar actualizada 23-05-2014)

y el ´angulo de fase φ satisface:

2 cc0 tan φ =  2 ω ω0

−1

con c0 = 2mω0 . Notamos que de la ecuaci´on que define la amplitud los coeficientes importantes son: 1. c/c0 llamado el cociente de amortiguaci´ on, y 2. ω/ω0 llamado el cociente de frecuencia del movimiento. Se observa que el comportamiento de la soluci´on particular xp depende entonces del cociente de amortiguaci´on si ´este es cero o despreciable, entonces tenemos el movimiento arm´onico simple: x00 + ω02 x = 0 y por tanto, hay dos casos para el cociente de frecuencia: Caso 2.8.4 (ω 2 6= ω02 ). En este caso se tiene: x (t) = C1 cos ω0 t + C2 sin ω0 t +

F0 /k sin ωt 1 − (ω/ω0 )2

donde: C 1 = x0



C2 =

(F0 /k) (ω/ω0 ) v0 − ω0 1 − (ω/ω0 )2

Caso 2.8.5 (ω 2 = ω02 ). En este caso debemos buscar una soluci´on particular de la forma: xp = b1 t cos ωt + b2 t sin ωt pues la ecuaci´on (2.24) se anula con el operador D2 + ω 2 , luego el anulador a considerar es 2

(D2 + ω 2 ) . As´ı, luego de los c´alculo habituales del m´etodo del anulador se obtiene: b1 = −

F0 2mω



b2 = 0

Por tanto, la soluci´on general tiene la forma: x (t) = C1 cos ωt + C2 sin ωt −

F0 t cos ωt 2mω

Observaci´ on 2.8.6. Observe en el caso anterior que al aumenta t, las vibraciones causadas por el u ´ ltimo t´ermino de la ecuaci´on anterior aumentar´ıan sin cota. En este caso, se dice que la fuerza externa est´a en resonancia con la masa en vibraci´on. 189

Apuntes Mat023 (versi´on preliminar actualizada 23-05-2014)

Ejercicios del cap´ıtulo 1. En los problemas siguientes se muestra que el principio de superposici´on generalmente no se cumple para ecuaciones no lineales a) Muestre que y = 1/x es soluci´on de la ecuaci´on y 0 + y 2 = 0 pero si c 6= 0 y c 6= 1 entonces y = c (1/x) no es soluci´on. √ b) Muestre que φ1 (x) = 1 e φ2 (x) = x son soluciones de yy 00 + (y 0 )2 = 0 pero la suma no es soluci´on. 2. Busque una soluci´on polinomial de la ecuaci´on de Legendre  1 − x2 y 00 − 2xy 0 + 2y = 0 y usando la f´ormula de Abel determine la soluci´on general de esta ecuaci´on. 3. Verificar que φ1 (x) = x−1/2 cos x es soluci´on (para x > 0) de la ecuaci´on de Bessel de orden

1 2

  1 2 y=0 x y + xy + x − 4 y obtener su soluci´on general. 2 00

0

4. Muestre que y1 (x) = x + 1 es soluci´on de la ecuaci´on  1 − 2x − x2 y 00 + 2 (1 + x) y 0 − 2y = 0 y resolverla completamente (encontrar soluci´on general). 5. Considere la ecuaci´on diferencial: y 00 + P1 (x) y 0 + P2 (x) y = Q (x)

(2.25)

y sean u1 (x) , u2 (x) soluciones linealmente independientes de la ecuaci´on homog´enea asociada. Pruebe que la funci´on: f (x) = C1 (x) u1 (x) + C2 (x) u2 (x) es una soluci´on particular de la ecuaci´on (2.25), si se satisface el sistema de ecuaciones:   C 0 (x) u (x) + C 0 (x) u (x) = 0 1 2 1 2  C 0 (x) u0 (x) + C 0 (x) u0 (x) = Q (x) 1

1

2

190

2

Apuntes Mat023 (versi´on preliminar actualizada 23-05-2014)

6. Compruebe que: 1 y= ω

Z

x

f (t) sin ω (x − t) dt 0

es una soluci´on particular de y 00 + ω 2 y = f (x). 7. Suponga que ex y e−x son soluciones de una ecuaci´on diferencial lineal ordinaria homog´enea, muestre que las funciones sinh x y cosh x tambi´en son soluciones de esa ecuaci´on. 8. Muestre que la soluci´on particular de una EDO lineal de segundo orden no homog´enea y 00 + a1 (x) y 0 + a0 (x) y = f (x) se puede escribir en la forma Z

x

yp (x) =

G (x, t) f (t) dt x0

donde G es una funci´on adecuada. 9. Hallar la soluci´on general de d4 y dx4 00

d2 y dx2 0

1) y 000 + 3y 00 − y 0 − 3y = 0

2) y (4) + 16y = 0

4) y 00 − 36y = 0

5)

y 00 + 2y 0 − 3y = 0

6) y + 8y + 16y = 0

7) y 0000 + 4y 00 + 3y = 0

8)

y 00 − 4y 0 + 5y = 0

9) y 000 − 5y 00 + 3y 0 + 9y = 0

3)

10. Resuelva la ecuaci´on diferencial sujeta a las condiciones indicadas. 0

a) y 00 + 16y = 0 ; y(0) = 2, y (0) = −2 b) y 00 + 6y 0 + 25y = 0 ; y(0) = 2, y 0 (0) = 6 c) y 00 − 4y = 0 ; y(0) = 4, y 0 (0) = 16 d ) y 000 + 2y 00 − 5y 0 − 6y = 0 ; y(1) = 0, y 0 (1) = 0, y 00 (1) = 1 11. Resuelva las siguientes ecuaciones por anuladores. a) y 00 + y 0 = 2t + sin t b) y 00 + 4y 0 + 2y = te−2t 191

+

+y =0

Apuntes Mat023 (versi´on preliminar actualizada 23-05-2014) √ c) y 00 − 4y 0 + 5y = 4 + sin( 7t) d ) y 00 + 3y = x2 e3x e) y 00 − 2y 0 + 2y = e2x (cos x − 3 sin x) f ) y 000 − y 00 − 4y 0 + 4y = 5 − ex + e2x g) y 000 − 2y 00 + 2y 0 − y = ex + cos x 12. Resuelva la ecuaci´on diferencial sujeta a las condiciones indicadas. a) 2y 00 + 3y 0 − 2y = 14x2 − 4x − 11 , y(0) = 0, y 0 (0) = 0 b) y 00 + 4y 0 + 4y = (3 + x)e−2x , y(0) = 2, y 0 (0) = 5 c) y 00 − 2y 0 + 2y = 2x − 2 , y(π) = 0, y 0 (π) = 0 d2 x + ω 2 x = F0 cos γt , x(0) = 0, x0 (0) = 1 dt2 d2 x e) + ω 2 x = F0 sin ωt , x(0) = 0, x0 (0) = 0 dt2

d)

13. Hallar una EDO lineal con coeficientes constantes que tenga por soluci´on general: 1) y (x) = c1 xex + c2 ex + x + 1 √  √  √  √  2) y (x) = c1 e2x cos 3x + c2 e2x sin 3x + c3 xe2x cos 3x + c4 xe2x sin 3x + ex

14. Encontrar un anulador de 1) f (x) = x cos 2x +

π 3



2

2) f (x) = (x2 + 1) ex + x sinh (x) √

2

3) f (x) = (x2 − 1) (1 + sin x) e

3x

15. Encontrar la soluci´on general de 1) x2 y 00 + xy 0 + 9y = 0 2) x2 y 00 + xy 0 − p2 y = 0

con p ∈ R

16. Encontrar la soluci´on general de

192

Apuntes Mat023 (versi´on preliminar actualizada 23-05-2014)

00

0

1) x2 y 00 + xy 0 − 9y = x2 + 1

2)

x2 y + xy − 9y = x3 + 1

3) x2 y 00 + 4xy 0 + 2y = 2 ln x

4)

x2 y + xy + 9y = sin(ln x3 )

6)

x2 y 00 − xy 0 + 2y = 16

8)

y 00 − x2 y 0 +

5)

x3 y 000 + 4x2 y 00 + xy 0 − y = x

7) y 00 + x1 y 0 −

1 y x2

=

1 x2 +x3

9) x4 y 00 + x3 y 0 − 4x2 y = 1

00

0

2 y x2 2 00

=

ln x x 0

10) x3 y 000 + 6x y + 4xy − 4y = x

17. Encontrar una soluci´on general de cada una de las ecuaciones diferenciales, usando el m´etodo de variaci´on de par´ametros. a) y 000 − y 00 + y 0 − y = 4xex b) y 000 − y 0 = sin x c) y 000 − 2y 00 = 4(x + 1) d ) y 000 − 3y 00 − y 0 + 3y = 1 + ex e) y 000 − 7y 0 + 6y = 2 sin x f ) 3y 00 − 6y 0 + 30y = ex tan 3x ex g) y 00 − 2y 0 + y = 4x2 − 3 + x 000 0 −x h) y − 2y − 4y = 2e sec x (puede dejar integrales finales sin calcular) 18. Una soluci´on y = u(x) de la ecuaci´on diferencial y 00 − 3y 0 − 4y = 0 intersecta a una soluci´on de la ecuaci´on diferencial y 00 + 4y 0 − 5y = 0 en el origen. Determinar las funciones u y v si las dos soluciones tienen la misma pendiente en el origen y si (v(x))4 5 = . x→∞ u(x) 6 l´ım

19. Sean y1 = u(x) y y2 = xu(x) dos soluciones de la ecuaci´on y 00 + 4xy 0 + p(x)y = 0. Si u(0) = 1, determine u(x) y p(x) expl´ıcitamente. 20. Se sabe que para x > 0 la ecuaci´on homog´enea asociada a √  4xy 00 + 2 − 8 x y 0 − 5y = 1 tiene una soluci´on y1 (x) = e− determine la soluci´on general. 193



x

Apuntes Mat023 (versi´on preliminar actualizada 23-05-2014)

21. Determine la soluci´on general de la ecuaci´on diferencial (2x − 1)2

√ d2 y dy − 8y = 2x − 1 + 4 (2x − 1) 2 dx dx

Ind.: Considere el cambio de variables u = 2x − 1. 22. Dada la ecuaci´on diferencial (x − a)

d2 y dy + a2 y = a (x − 1)2 ex −x 2 dx dx

determine el valor del par´ametro de forma que ex sea una soluci´on de la homog´enea asociada y determine la soluci´on general de la ecuaci´on no homog´enea para tales valores del par´ametro. 23. Use z = sin (x) para resolver la ecuaci´on y 00 + tan (x) y 0 + cos2 (x) y = sin (x) cos2 (x) 24. Considere la ecuaci´on diferencial x2 y 00 − 2xy 0 + 2y = 6 con condiciones iniciales y (0) = 3, y 0 (0) = 1 tiene soluciones yc (x) = cx2 + x + 3 para todo c ∈ R, este problema no tiene soluci´on u ´nica, ¿contradice esto el teorema de existencia y unicidad.? 25. Resolver (xD + 1) (D − x) (xD) y = x donde D =

d . dx

26. Un cuerpo con masa m =

1 2

kilogramo (kg) est´a unido en el extremo de un resorte

estirado 2 metros (m) debido a una fuerza de 100 newtons (N) y es puesto en movimiento a partir de la posici´on inicial x0 = 1 (m) y velocidad inicial v0 = 5 (m/s). (N´otese que estas condiciones iniciales indican que el cuerpo se desplaza a la derecha y a la izquierda en el tiempo t = 0). Encu´entrese la funci´on de la posici´on del cuerpo, as´ı como su amplitud, frecuencia, periodo de oscilaci´on y el tiempo de retardo de su movimiento. 194

Apuntes Mat023 (versi´on preliminar actualizada 23-05-2014)

27. Determine el periodo y la frecuencia del movimiento arm´onico simple de una masa de 4 kg unida al extremo de un resorte con constante de 16 N/m. 28. Establezca el periodo y la frecuencia del movimiento arm´onico simple de un cuerpo con una masa de 0.75 kg unida al extremo de un resorte con constante de 48 N/m. 29. Un cuerpo con masa de 250 g est´a unido al extremo de un resorte estirado 25 cm por una fuerza de 9 N. En el tiempo t0 el cuerpo es movido 1 m a la derecha, estirando el resorte y aplicando un movimiento con una velocidad inicial de 5 m/s a la izquierda. (a) Encuentre x(t). (b) Obtenga la amplitud y el periodo de movimiento del cuerpo. 30. Asuma que la ecuaci´on diferencial de un p´endulo simple de longitud L es Lθ00 +gθ = 0, donde g = GM/R2 es la aceleraci´on gravitacional en el lugar donde ´este se encuentra (a una distancia R del centro de la Tierra; M significa la masa de la Tierra). Dos p´endulos de longitudes L1 y L2 —ubicados a una distancia R1 y R2 respecto del centro de la Tierra— tienen periodos p1 y p2 . Muestre que √ R1 L1 p1 √ = p2 R2 L2

195

Cap´ıtulo 3 : Sistemas de ecuaciones diferenciales

Definiciones Considere los dos tanques que se ilustran en la figura. Suponga que el tanque A contiene 50 galones de agua en los que hay disueltas 25 libras de sal. Suponga que el tanque B contiene 50 galones de agua pura. A los tanques entra y sale l´ıquido como se indica en la figura; se supone que tanto la mezcla intercambiada entre los tanques como el l´ıquido bombeado hacia fuera del tanque B est´an bien mezclados. Se desea construir un modelo matem´atico que describa la cantidad de libras x1 (t) y x2 (t) de sal en los tanques A y B respectivamente en el tiempo t.

se tiene              gal lb gal x2 lb gal x1 lb 3 ·0 +1 · −4 · min gal min 50 gal min 50 gal 2 x2 = − x1 + 25 50

dx1 = dt

y dx2 x1 x2 x2 = 4 −3 −1 dt 50 50 50 2 2 = x1 − x2 25 25 196

Apuntes Mat023 (versi´on preliminar actualizada 23-05-2014)

as´ı obtenemos el sistema dx1 2 1 = − x1 + x2 dt 25 50 2 2 dx2 = x1 − x2 dt 25 25 y las condiciones x1 (0) = 25 y x2 (t) = 0 Un sistema de ecuaciones de la forma dX = A (t) X (t) + B (t) dt esto es 

x01 (t)

  x0 (t)  2  .  ..  x0n (t)





a11 (t) a12 (t) · · · a1n (t)

    a (t) a (t) · · · a (t) 22 2n   21 = . . .. . .. .. ..   .   an1 (t) an2 (t) · · · ann (t)



x1 (t)

   x (t)  2  .   ..  xn (t)

b1 (t)



    b (t)   2 + .   ..   bn (t)

     





donde X (t) = (x1 (t) , x2 (t) , . . . , xn (t))T , A (t) = (aij (t)) es una matriz de orden n × n y B (t) = (b1 (t) , b2 (t) , . . . , bn (t))T es llamado sistema de ecuaciones diferenciales lineal de primer orden (se supone que las funciones aij (t) y bi (t) son continuas en un intervalo abierto I). Si adicionalmente se debe cumplir X (t0 ) = X0 para t0 ∈ I, X0 ∈ Rn entonces tenemos el P.V.I dX = A (t) X (t) + B (t) dt X (t0 ) = X0 Ejemplo 3.1.1. Son sistemas de ecuaciones lineales: dx dt

=

dy dt

= −x + 2y + cos t

dx dt

= et x + e−t y + t

dy dt

= −x + (sin t) y

x + 2y + et

1.

2.

197

Apuntes Mat023 (versi´on preliminar actualizada 23-05-2014)

3.

dx dt

=

dy dt

= −x + 2y + 3z

dz dt

=

x+y+z

2x + 3y + z

mientras que el sistema dx = x2 + e t y 2 dt p dy = cos t + x2 + y 2 dt no es lineal.

Teorema 3.1.1. Si las funciones aij (t) y bi (t) son continuas en el abierto I ⊆ R, t0 ∈ I, X0 ∈ Rn entonces el problema de valores iniciales dX = A (t) X (t) + B (t) dt X (t0 ) = X0 tiene soluci´on u ´nica.

Ejemplo 3.1.2. Resolver el sistema dx1 = x2 dt dx2 = −x1 dt con x1 (0) = 1, x2 (0) = 0. Soluci´ on. Note que derivando la primera ecuaci´on respecto a t obtenemos d 2 x1 dx2 = = −x1 2 dt dt de donde x1 debe cumplir

d 2 x1 + x1 = 0 dt2 198

Apuntes Mat023 (versi´on preliminar actualizada 23-05-2014)

as´ı x1 (t) = c1 cos t + c2 sin t para algunas constantes c1 y c2 ahora bien, x1 (0) = 1 = c1 se sigue x1 (t) = cos t + c2 sin t adem´as x2 (t) =

dx1 = − sin t + c2 cos t dt

luego 0 = x2 (0) = c2 se sigue x1 (t) = cos t x2 (t) = − sin t

Observaci´ on 3.1.1. Toda ecuaci´on diferencial ordinaria lineal de orden n puede ser transformada en un sistema de ecuaciones lineales, en efecto si tenemos y (n) + an−1 (t) y (n−1) + · · · + a1 (t) y 0 + a0 (t) y = h (t) poniendo x1 (t) = y (t) x2 (t) = y 0 (t) .. . xn (t) = y (n−1) (t) entonces 

x1

  x  2 d   ... dt    xn−1  xn





0

1

0

0

···

0



        =        

0 .. .

0 .. .

1 .. .

0 .. .

··· .. .

0 .. .

0

0

0

0

..

1

  x  2  .   ..     xn−1  xn

.

−a0 −a1 · · · · · · · · · −an−1

199

x1





0



        +        

0 .. .

        

0 h (t)

Apuntes Mat023 (versi´on preliminar actualizada 23-05-2014)

Ejemplo 3.1.3. Considere la ecuaci´on d2 x dx + 2x = et + (cos t) 2 dt dt esta se transforma en el sistema        1 x 0 d  x1   0  1  +   = dt x2 −2 − cos t x2 et donde x1 (t) = x (t) y x2 (t) = x0 (t) La teor´ıa de los sistemas lineales es similar a la que hemos visto hasta ahora en el caso de una ecuaci´on. La ecuaci´on Homog´enea asociada a dX = A (t) X (t) + B (t) dt es dX = A (t) X (t) dt su conjunto soluci´on  S=

dX = A (t) X (t) X (t) ∈ C (I, R ) : dt 1

n



es un espacio vectorial de dimensi´on n (si el sistema es de orden n) y la soluci´on de la no homog´enea ser´a una soluci´on particular m´as una combinaci´on lineal arbitraria de elementos de la base de S.

Ecuaci´ on con coeficientes constantes Suponga que trabajamos con el sistema dX = AX dt donde A ∈ Mn×n (R) (coeficientes constantes). Teorema 3.2.1. Si λ es un valor propio de A con vector propio v entonces x (t) = veλt es soluci´on de

dX dt

= AX.

200

Apuntes Mat023 (versi´on preliminar actualizada 23-05-2014)

Matriz A diagonalizable Si A es una matriz diagonalizable entonces P −1 AP = D donde D es una matriz diagonal, la ecuaci´on dX = AX dt la podemos ver como dX = P DP −1 X dt multiplicando la ecuaci´on por P −1 por la izquierda  d P −1 X = DP −1 X dt cambiamos la variable Y = P −1 X nos queda dY = DY dt que es un sistema de la forma dyi = λi yi para i = 1, 2, . . . , n dt tiene soluciones yi (t) = ci eλi t luego la soluci´on es X (t) = P Y (t) =

n X

ci vi eλi t

i=1

donde vi son los vectores propios asociados a los valores propios correspondientes. Observaci´ on 3.2.1. Si el valor propio λi es complejo vi eλi t y vi eλi t aparecer´an como   soluciones, al buscar soluciones reales se tiene que Re vi eλi t y Im vi eλi t son soluciones l.i. reales correspondientes a combinaciones lineales de las dos complejas luego generan lo mismo. 201

Apuntes Mat023 (versi´on preliminar actualizada 23-05-2014)

Ejemplo 3.2.1. Resolver 

x1





−1

1

  d   x2  =  1   dt  x3 1

1



x1



   x2  −1 1    1 1 x3

Soluci´ on. Primero buscamos el polinomio caracter´ıstico de la matriz   −1 1 1    A= 1 −1 1   1 1 1 esta dado por −1 − λ 1 1 PA (λ) = |A − λI| = 1 −1 − λ 1 1 1 1−λ

= (λ − 2) (λ + 2) (λ + 1)

luego tiene 3 valores propios distintos y es una matriz de orden 3 luego es diagonalizable, ahora buscamos los vectores propios:      * −1 + * −1 + * 21       −1  ↔ −1,  1  ↔ −2,  1      2 1 1 0 as´ı la soluci´on del sistema es      1 −1 −1   −t   −2t  2     1 X (t) = c1   −1  e + c2  1  e + c3  2 1 0 1





 2t  e =   

 +   

1 c e2t 2 3 c2 e−2t

x1





5

2

2



x1



    d   x2  =  2 2 −4   x2      dt  x3 2 −4 2 x3 Si 

5

2

 A=  2

2

2



 −4   2 −4 2 202

− c2 e−2t − c1 e−t



 − c1 e−t + 21 c3 e2t   −t 2t c1 e + c3 e

Ejemplo 3.2.2. Resolver 

↔2

Apuntes Mat023 (versi´on preliminar actualizada 23-05-2014)

entonces es diagonalizable por ser una matriz sim´etrica 5−λ 2 2 2 |A − λI| = 2 2 − λ −4 = − (λ + 3) (λ − 6) 2 −4 2 − λ luego hay 1 valor propio de multiplicidad 2 buscamos los vectores propios       * 2 * − 21 + 2 +        1  ↔ −3,  1  ,  0  ↔ 6       1 0 1 se sigue que la soluci´on del sistema es        2c2 e6t − 21 c1 e−3t + 2c3 e6t 2 2 − 21   6t    6t  −3t        X (t) = c1  c1 e−3t + c2 e6t  1  e + c2  1  e + c3  0  e =  1 0 1 c1 e−3t + c3 e6t

   

Ejemplo 3.2.3. Resolver 

x1





0

2 0



x1



    d   x2  =  −2 0 0   x2      dt  x3 0 0 1 x3 Soluci´ on. Si 

0

2 0



   A= −2 0 0   0 0 1 entonces la ecuaci´on caracter´ıstica es (λ − 1) (λ2 + 4) = 0 de donde tenemos que la matriz es diagonalizable, 3 valores propios distintos aunque hay dos complejos, veamos los espacios propios asociados       * −i + * 0 + * i +        0  ↔ 1,  1  ↔ −2i,  1  ↔ 2i       1 0 0

203

Apuntes Mat023 (versi´on preliminar actualizada 23-05-2014)

el resultado nos dice que 



i



−i



  −2it    1 e  1  e2it y     0 0 son soluciones pero buscamos soluciones reales y para     i   −2it    1  e  y Im  Re      0

ello   i  −2it   1  e  0

generan lo mismo que las dos anteriores     i i   −2it    1 e  =     1  (cos (2t) − i sen (2t)) 0 0   i cos (2t) + sen (2t)    =   cos (2t) − i sen (2t)  0    cos (2t) sen (2t)      =   cos (2t)  + i  − sen (2t) 0 0

   

se sigue 



i





sen (2t)



   −2it        Re   1  e  =  cos (2t)  0 0 y 

i







cos (2t)



  −2it         Im   1  e  =  − sen (2t)  0 0 luego la soluci´on  0  X (t) = c1   0 1

es 



sin (2t)





cos (2t)





c3 cos 2t + c2 sin 2t



 t        e + c2  cos (2t)  + c3  − sin (2t)  =  c2 cos 2t − c3 sin 2t         t 0 0 c1 e 204

Apuntes Mat023 (versi´on preliminar actualizada 23-05-2014)

Si la matriz no es diagonalizable no hemos desarrollado una teor´ıa para poder resolverla (m´as adelante resolveremos estos sistemas utilizando la transformada de Laplace) sin embargo podemos enfrentar directamente el sistema a trav´es de eliminaci´on, considere el siguiente ejemplo: Ejemplo 3.2.4. Resolver el sistema      d  x   −2 −1   x  = dt y 1 −4 y Soluci´ on. En este caso el polinomio es −2 − λ −1 |A − λI| = 1 −4 − λ

= λ2 + 6λ + 9 = (λ + 3)2

se sigue que hay un solo valor propio de multiplicidad algebraica 2, Si calculamos vemos que la multiplicidad geom´etrica (dimensi´on del espacio propio asociado) es 1 luego la matriz no es diagonalizable * Wλ=−3 =



1 1

+ 

Lo resolveremos directamente x0 = −2x − y y 0 = x − 4y si derivamos la primera ecuaci´on obtenemos x00 = −2x0 − y 0 pero de la segunda ecuaci´on sabemos y 0 = x − 4y reemplazando x00 = −2x0 − (x − 4y) = 4y − x − 2x0 205

Apuntes Mat023 (versi´on preliminar actualizada 23-05-2014)

pero de la primera ecuaci´on y = −x0 − 2x as´ı x00 = 4 (−x0 − 2x) − x − 2x0 = −9x − 6x0 as´ı x00 + 6x0 + 9x = 0 ⇔ (D + 3)2 x = 0 luego x (t) = c1 e−3t + c2 te−3t adem´as y (t) = −x0 − 2x   d c1 e−3t + c2 te−3t − 2 c1 e−3t + c2 te−3t = − dt −3t = e (c1 − c2 + tc2 ) = c1 e−3t + c2 (−1 + t) e−3t luego  X (t) = 

x (t)





=

c1 e

−3t

+ c2 te

−3t

y (t) c1 e−3t + c2 (−1 + t) e−3t     1 t  e−3t = c1   e−3t + c2  1 −1 + t

 

es la soluci´on. Note que la estructura de la soluci´on es similar a las ecuaciones con ra´ız repetida.

Variaci´ on de par´ ametros en sistemas Suponga que queremos determinar la soluci´on particular de la ecuaci´on dX = AX + B (t) dt 206

Apuntes Mat023 (versi´on preliminar actualizada 23-05-2014)

donde la soluci´on de la homog´enea es Xh (t) = c1 X1 (t) + c2 X2 (t) + · · · + cn Xn (t) entonces proponemos una soluci´on particular de la forma Xp (t) =

n X

ci (t) Xi (t)

i=1

se sigue que Xp0

(t) = =

n X i=1 n X

c0i

(t) Xi (t) +

c0i (t) Xi (t) +

i=1

=

n X

n X i=1 n X

ci (t) Xi0 (t) ci (t) AXi (t)

i=1

c0i (t) Xi (t) + AXp (t)

i=1

entonces

n X

c0i (t) Xi (t) = B (t)

i=1

se sigue X1 (t) X2 (t) · · · Xi−1 (t) B (t) Xi+1 (t) · · · Xn (t) c0i (t) = X1 (t) X2 (t) · · · Xi−1 (t) Xi (t) Xi+1 (t) · · · Xn (t) es un cociente de determinantes por la regla de Cramer.

Ejemplo 3.3.1. Determine la soluci´on general del sistema        dx 2t 1 −3 x e  dt  =    +   dy 4t −3 1 y e dt Soluci´ on. Primero resolvemos el sistema homog´eneo, la matriz   1 −3   −3 1

207

Apuntes Mat023 (versi´on preliminar actualizada 23-05-2014)

tiene valores propios −2, 4 y *   =  *   = 

Wλ=−2

Wλ=4

 + 1   1  + −1    1

se sigue que la soluci´on general del sistema homog´eneo es     1 −1  e4t para α, β ∈ R Xh (t) = α   e−2t + β  1 1 para determinar la solkuci´on particular usamos variaci´on de par´ametros, existe una soluci´on de la forma

 Xp (t) = C1 (t) 

1 1





 e−2t + C2 (t) 

−1 1

  e4t

donde 2t 4t e −e 4t 4t e e e6t + e8t = C10 (t) = 2e2t e−2t −e4t −2t 4t e e 1 4t 1 6t e + e 2 2

= se sigue Z  C1 (t) =

 1 1 1 4t 1 6t e + e dt = e4t + e6t 2 2 8 12

por otro lado −2t 2t e e −2t 4t e e

C20 (t) = e−2t −e4t −2t e4t e =

1 1 −2t − e 2 2 208



e2t − 1 = 2e2t

Apuntes Mat023 (versi´on preliminar actualizada 23-05-2014)

se sigue  1 1 −2t − e dt C2 (t) = 2 2 1 1 = t + e−2t 2 4 Z 

as´ı  Xp (t) = C1 (t) 

1 1





 e−2t + C2 (t)  

−1



1

  e4t 



1 −1 1 4t 1 1 1  e4t e + e6t   e−2t + t + e−2t  8 12 2 4 1 1      1 2t 1 4t 1 2t 1 4t e + e te + e − 12 2 4 + =  8   1 4t 1 2t 1 4t 1 2t e + 12 e te + 4 e 8 2   1 2t 1 4t 1 4t 1 2t e + e − te − e 12 2 4  =  8 1 2t 1 4t 1 4t 1 2t e + e + te + e 8 12 2 4   1 2t 1 2t 1 4t 1 4t e − 4 e + 12 e − 2 te  =  8 1 2t 1 2t 1 4t 1 4t e + e + e + te 8 4 12 2 







=

la soluci´on general es  XG = α 

1 1





 e−2t + β 

−1 1





 e4t + 

1 − 81 e2t + 12 − 12 t e4t  3 2t 1 e + 12 + 2t e4t 8



para α, β ∈ R

Ejemplo 3.3.2. Resolver el sistema      

dx dt dy dt dz dt

1 −4 0

  = 4   0

1 0



x





1



        0   y  +  0  2 z t

Soluci´ on. Este sistema tiene la forma dX = AX + B (t) dt 209

 

Apuntes Mat023 (versi´on preliminar actualizada 23-05-2014)

donde 

1





1 −4 0

    yA= 4 B (t) =  0    t 0 primero resolvemos el sistema homog´eneo dX = AX dt es decir    dx 1 −4 0 dt    dy     dt  =  4 1 0 dz 0 0 2 dt

 0   2

1 0





x



   y    z

buscamos los valores y vectores propios asociados, en este caso son:             −i 0       i        0  ↔ 2,  1  ↔ 1 − 4i,  1  ↔ 1 + 4i                      0 0 1 se sigue que las soluci´on general del sistema homog´eneo es de la forma       −i −i 0  (1−4i)t      2t   + c3 Im  1   Xh (t) = c1     0  e + c2 Re  1  e 0 0 1



 (1−4i)t  e   

donde 

−i





−i



     1  e(1−4i)t =  1  et e−4ti     0 0   −i   t  e (cos 4t − i sin 4t) =  1   0   −iet cos 4t − et sin 4t   t  =  e (cos 4t − i sin 4t)   0    − sin 4t − cos 4t    t  = et   cos 4t  + ie  − sin 4t 0 0 210



   

Apuntes Mat023 (versi´on preliminar actualizada 23-05-2014)

entonces 

0





− sin 4t





− cos 4t



  2t      e + c2 et  cos 4t  + c3 et  − sin 4t  Xh (t) = c1  0       1 0 0 ahora buscamos una soluci´on particular de la ecuaci´on no homog´enea dX = AX + B (t) dt de la forma Xp (t) = c1 (t) X1 (t) + c2 (t) X2 (t) + c3 (t) X3 (t) (mediante variaci´on de par´ametros) donde    0 −et sin 4t      t X1 (t) =   0  , X2 (t) =  e cos 4t e2t 0 se sigue las funciones c1 , c2 y c3 deben cumplir  0 −et sin 4t −et cos 4t   0 et cos 4t −et sin 4t  e2t 0 0





−et cos 4t

    y X3 (t) =  −et sin 4t     0



c01 (t)





1



      c0 (t)  =  0      2 0 t c3 (t)

sistema que podemos resolver por ejemplo mediante determinantes: 1 −et sin 4t −et cos 4t 0 et cos 4t −et sin 4t t 0 0 te2t c01 (t) = = 4t = te−2t 0 −et sin 4t −et cos 4t e 0 et cos 4t −et sin 4t 2t e 0 0 :

0 1 −et cos 4t 0 0 −et sin 4t 2t e t 0



c02 (t) = 0 −et sin 4t −et cos 4t 0 et cos 4t −et sin 4t 2t e 0 0 211



=

−e3t sin 4t = −e−t sin 4t e4t

Apuntes Mat023 (versi´on preliminar actualizada 23-05-2014)

y 0 −et sin 4t 1 0 et cos 4t 0 2t e 0 t

c03 (t) = 0 −et sin 4t −et cos 4t 0 et cos 4t −et sin 4t 2t e 0 0

=

−e3t (cos 4t) = − (cos 4t) e−t 4t e

integramos : Z

1 c1 (t) = te−2t dt = − e−2t (2t + 1) 4 Z 1 c2 (t) = −e−t sin 4tdt = e−t (4 cos 4t + sin 4t) 17 Z 1 c3 (t) = − (cos 4t) e−t )dt = e−t (cos 4t − 4 sin 4t) 17 entonces la soluci´on particular es 

0





−et sin 4t



       1 −t 1 −2t   e (4 cos 4t + sin 4t)  Xp (t) = − e (2t + 1)  0  + et cos 4t    4 17 2t e 0   −et cos 4t   1 + e−t (cos 4t − 4 sin 4t)  −et sin 4t    17 0   1 − 17   4  =    17 1 1 −2t − 4 

212

Apuntes Mat023 (versi´on preliminar actualizada 23-05-2014)

verifiquemos que es correcto 

0



   Xp0 (t) =   0  − 21  1 −4  =   4 1 0 0  −1  =  0  −t − 21

0





1 − 17

 4  0   17 − 12 t − 2    1    + 0     t

1 4



1



   + 0     t

se sigue que la soluci´on del sistema es X (t) = Xh (t) + Xp (t)    0 − sin 4t   2t  t  = c1   0  e + c2 e  cos 4t 1 0





− cos 4t







1 − 17

    4  + c3 et  − sin 4t  +      17 − 21 t − 0

1 4

  

An´ alisis cualitativo de sistemas Vamos a analizar el comportamiento de los sistemas de orden 2 de la forma dx = ax + by dt dy = cx + dy dt donde a, b, c y d son constantes, bas´andonos en los valores propios de la matriz asociada al sistema

 A=

a b c d

 

la cual supondremos diagonalizable y adicionalmente que det (A) = ad − bc 6= 0 de esta forma la u´nica soluci´on constante del sistema corresponde al origen, esta soluci´on es llamada soluci´on de equilibrio, las dem´as soluciones ser´an entonces curvas param´etricas (x (t) , y (t)) y seg´ un su comportamiento clasificaremos la soluci´on de equilibrio. 213

Apuntes Mat023 (versi´on preliminar actualizada 23-05-2014)

Valores propios reales y distintos (no nulos) 1. Punto silla: En el caso en que los valores propios son reales y distintos con signos contrarios, el origen se dice que es un punto de silla. Considere el sistema      x −1 1 x d      = dt y 3 1 y En este caso la matriz de coeficientes es  A=

−1 1 3

1

 

la cual tiene determinante distinto de cero. Buscamos sus valores propios y vectores propios asociados, en este caso son:       −1   1    ↔ −2,  3  ↔ 2    1  1 luego las soluciones del sistema son de la forma    X (t) = c1 

1 3

1

 e2t + c2 

−1 1

  e−2t

Note que si c1 = 1 y c2 = 0 obtenemos una soluci´on recta   X1 (t) = 

1 3

1

 e2t

en esta soluci´on x (t) = 13 e2t y y (t) = e2t se sigue que los puntos de esta curva est´an sobre la recta

y e2t = 1 2t = 3 x e 3 es decir y = 3x y se alejan delorigen  cuando t → +∞ por el primer cuadrante. De manera similar, −X1 (t) = − 

1 3

 e2t es la curva param´etrica que esta sobre la

1 recta y = 3x y si t → +∞ la soluci´on se aleja del origen por el tercer cuadrante. Existe otra soluci´on recta, al poner c1 = 0 y c2 = 1 se obtiene   −1  e−2t X2 (t) =  1 214

Apuntes Mat023 (versi´on preliminar actualizada 23-05-2014)

es decir la curva param´etrica x = −e−2t y = e−2t esta curva esta sobre la recta y = −1 es decir y = −x x note que cuando t → +∞ los puntos se acercan al origen sobre tal recta por el segundo cuadrante. Note que −X2 (t) tambi´en es soluci´on, en este caso x = e−2t y = −e−2t esta curva tambi´en esta sobre la recta y = −x los puntos se acercan al origen pero por el cuarto cuadrante. Las soluci´on general es    X (t) = c1   note que si t → +∞ entonces c2   c1 

1 3

1 3

1

 e2t + c2 

−1 1

−1 1

  e−2t

  e−2t ≈ 0 y la soluci´on se comporta como

  e2t esto es, cuando el tiempo avanza la curva soluci´on se acerca a la recta

1 el comportamiento cuando y = 3x (el cuadrante depende de c1 ) y sianalizamos  −1  e−2t esto se interpreta como que t → −∞ las curvas soluciones tienden a c2  1 las soluciones parten cercana a esa recta y se aproximan a y = 3x. Recuerde que por teorema de existencia y unicidad dos soluciones distintas no se pueden cortar.

215

Apuntes Mat023 (versi´on preliminar actualizada 23-05-2014)

Grafiquemos algunas soluciones y el campo de direcciones asociado a este sistema

2. Sumidero: Si los valores propios son reales y distintos, ambos negativos, el origen es llamado sumidero. Considere el sistema      6 2 d  x   −5 5   x  = 2 dt y − 95 y 5 en este caso la matriz de coeficientes es  A=

− 65

2 5

2 5

− 95

 

que tiene vectores y valores propios asociados       2   −1    ↔ −1,  2  ↔ −2   1   1 luego las soluciones son de la forma     1 2 − X (t) = c1   e−t + c2  2  e−2t 1 1 216

Apuntes Mat023 (versi´on preliminar actualizada 23-05-2014)

grafiquemos el campo de direcciones y diagrama de fase asociados. Note que en este caso tenemos dos soluciones rectas y que las soluciones tienden al origen si t → +∞

3. Fuente: Si los valores propios son distintos pero ambos positivos entonces el origen es llamado fuente (las soluciones se alejan del origen) Considere el sistema      x 0 6 x d      = dt y −1 5 y en este caso la matriz de coeficientes es  A=

0

6

−1 5

 

que tiene vectores y valores propios        3   2    ↔ 2,   ↔ 3  1   1  217

Apuntes Mat023 (versi´on preliminar actualizada 23-05-2014)

luego las soluciones son de la forma  X (t) = c1 

3 1





 e2t + c2 

2 1

  e3t

note que si t → ∞ entonces kX (t)k → ∞ se sigue que las soluciones se alejan del origen y tenemos dos soluciones rectas.

Valores propios complejos 1. Sumidero espiral: En el caso en que los valores propios son complejos y la parte real del valor propio es negativa, el origen es un sumidero espiral. Considere el sistema      2 x d  x   1   = dt y −4 −3 y en este caso la matriz asociada es  A=

1

2

−4 −3

218

 

Apuntes Mat023 (versi´on preliminar actualizada 23-05-2014)

tiene por vectores y valores propios      −1 − 1i   −1 + 1i   2 2  ↔ −1 − 2i,  2 2  ↔ −1 + 2i     1 1 se sigue que la soluci´on del sistema esta generada por       1 1 1 1 − + i − + i Re  2 2  e(−1−2i)t  e Im  2 2  e(−1−2i)t  1 1 entonces  X (t) = c1 e−t 

1 2

1 2

sin 2t − cos 2t cos 2t





 + c2 e−t 

1 2

1 2

sin 2t + cos 2t − sin 2t

 

ahora el campo de vectores y algunas curvas (note que en este caso no hay soluciones rectas) las soluciones se acercan al origen

2. Fuente espiral: En el caso en que los valores propios son complejos con parte real positiva las soluciones se alejan del origen y este es llamado fuente espiral. Considere 219

Apuntes Mat023 (versi´on preliminar actualizada 23-05-2014)

el sistema



x





4

3

4

3



x



d      = dt y −6 −2 y en este caso la matriz asociada es  A=

−6 −2

 

los valores y vectores propios son      −1 − 1i   −1 + 1i   2 2  ↔ 1 − 3i,  2 2  ↔ 1 + 3i     1 1 luego la soluci´on es generada por       1 1 1 1 − + i − + i Re  2 2  e(1−3i)t  e Im  2 2  e(1−3i)t  1 1 es decir las soluciones son de la forma    1 1 sin 3t − cos 3t 2  + c2 et  X (t) = c1 et  2 cos 3t

1 2

sin 3t + 21 cos 3t − sin 3t

 

grafiquemos el campo de direcciones y algunas soluciones el comportamiento es el

220

Apuntes Mat023 (versi´on preliminar actualizada 23-05-2014)

mismo pero las soluciones se alejan del origen. (no tenemos soluciones rectas)

3. Centro: Cuando los valores propios son complejos con parte real igual a cero el origen es llamado centro, en este caso las soluciones no se acercan al origen sino que se mantienen alrededor de ´el, el origen es estable en este caso pero no asint´oticamente estable. Considere el sistema      3 x d  x   3   = dt y −6 −3 y en este caso la matriz asociada es  A=

3

3

−6 −3

 

que tiene por valores y vectores propios      −1 − 1i   −1 + 1i   2 2  ↔ −3i,  2 2  ↔ 3i     1 1

221

Apuntes Mat023 (versi´on preliminar actualizada 23-05-2014)

luego la soluci´on del sistema es generada por       1 1 1 1 − + i − + i Re  2 2  e(−3i)t  e Im  2 2  e(−3i)t  1 1 es decir las soluciones son de la forma    1 1 sin 3t − cos 3t 2  + c2  X (t) = c1  2 cos 3t

1 2

sin 3t + 21 cos 3t − sin 3t

 

grafiquemos el campo de vectores y algunas curvas soluci´on.

Valores propios repetidos (no nulos) 1. Fuente: Cuando hay solo un valor propio y es positivo el origen es llamado fuente, las soluciones se alejan del origen. Considere el sistema      d  x   2 0  x  = dt y 0 2 y 222

Apuntes Mat023 (versi´on preliminar actualizada 23-05-2014)

en este caso la matriz asociada es 



2 0

A=



0 2

y el valor propio es λ = 2 se ve que las soluciones son de la forma     1 0 X (t) = c1   e2t + c2   e2t 0 1 es decir



2t

c1 e

X (t) = 

2t

c2 e

 

las soluciones son todas rectas que se alejan del origen. 2. Sumidero: Si hay solo un valor propio y es negativo las curvas se acercan al origen y este es llamado sumidero. Considere el sistema      d  x   −2 0   x  = dt y 0 −2 y en este caso la matriz asociada es  A=

−2

0

0

−2

 

los valores propios son iguales a −3 pero la matriz no es diagonalizable, las soluciones son de la forma

 X (t) = 

c1 e−2t



c2 e−2t



son rectas que se acercan al origen.

Teorema 3.4.1. Para un sistema lineal X0 = AX en el cual det (A) = 6 0, sea X = X (t) la soluci´on que satisface X (0) = X0 con X0 6= 0. 1. l´ımt→+∞ X (t) = 0 si y solo si los valores propios de A tienen partes reales negativas. 223

Apuntes Mat023 (versi´on preliminar actualizada 23-05-2014)

2. X (t) es peri´odica si y solo si las valores propios de A son imaginarios puros.

Ejemplo 3.4.1. Sea α ∈ R− {1, 2}. Considere el sistema de ecuaciones      α−2 x d  x   α   = dt y 1−α α y 1. Clasificar la soluci´on de equilibrio (silla, atractor, repulsor, etc.) para los distintos valores de α. Primero notemos que α α−2 α 1−α

= 2α2 − 3α + 2 > 0

para todo α ∈ R, luego la u ´ nica soluci´on de equilibrio es el origen. Para clasificar calculamos los valores propios λ2 − 2αλ + 2α2 − 3α + 2



luego p 4α2 − 4 (2α2 − 3α + 2) λ = 2 p 2 2α ± 4α − 4 (2α2 − 3α + 2) = 2 p = α ± − (α − 1) (α − 2) 2α ±

a) Si − (α − 1) (α − 2) > 0 es decir α ∈ ]1, 2[ tenemos soluciones reales y notemos que el producto de las dos es 2α2 − 3α + 2 > 0 luego las dos son positivas o negativas, α−

p − (α − 1) (α − 2) > 0 ⇔

(para α > 0)

α2 > − (α − 1) (α − 2) ⇔ α2 + (α − 1) (α − 2) > 0 224

Apuntes Mat023 (versi´on preliminar actualizada 23-05-2014)

lo que tiene soluci´on todo R+ pero por la restricci´on se sigue α ∈ ]1, 2[ . As´ı, para α ∈ ]1, 2[ tenemos dos valores propios reales y positivos, el punto es un repulsor. b) Si − (α − 1) (α − 2) < 0 esto es α ∈ ]−∞, 1[ ∪ ]2, +∞[ tenemos ra´ıces complejas conjugadas. Para α = 0 tenemos un centro, para α < 0 tenemos un atractor espiral y para α ∈ ]0, 1[ ∪ ]2, +∞[ tenemos un repulsor espiral. 2. Para α =

3 2

determine la soluci´on general del sistema y bosquejar el diagrama de

fases. si α =

3 2

entonces λ=

3 1 ± 2 2

es decir, los valores propios son 2, 1 buscamos los espacios propios     1 3 3 3 −2 −2 2 = 2   2 1 3 3 1 − 32 − 2 2 2 se sigue Wλ=1

* +  1    =  1 

y Wλ=2 la soluci´on general es  

x (t) y (t)



 + *  −1    =   1



 = C1 

1 1

225





 et + C2 

−1 1

  e2t

Apuntes Mat023 (versi´on preliminar actualizada 23-05-2014)

el diagrama de fases es

Ejercicios del cap´ıtulo 1. Escribir los siguientes sistemas en forma matricial a)

c)

dx dt dy dt

=

3x − 2y

=

4x + 8y

dx dt dy dt dz dt

=

3x − y + 3z

=

4x + 3y + z

=

2x + y − z

b)

d)

dx dt dy dt

= x − 2ty + t2

dx dt dy dt dz dt

= et x − cos ty + 3tz + cos t

=

4t2 x + 8y + sin t

= x + et =

2x + y − z − 3t + 1

2. Escribir los siguientes sistemas sin usar matrices        x −2 1 x sin t d   =    +   a) dt y t 0 y et      −1 + t −1 1 1 −1        t  b) X0 =  1 1 2   0  X+  0  e +  −1 −1 1 −1 −1

  t  e sin t 

3. En los siguientes ejercicios verificar si el vector X dado es soluci´on del sistema     dx = 3x − 4y 1 dt a) X =   e−5t de dy  2 = 4x − 7y dt 226

Apuntes Mat023 (versi´on preliminar actualizada 23-05-2014)





   

dx = dt   dy  b) X =   6  de  dt =   dz = 13 dt   sin t   1 1  c) X =   − 2 sin t − 2 cos t  − sin t + cos t

1

x + 2y + z 6x − y −x − 2y − z    1 0 1    0  de X =  1 1 0    −2 0 −1

  X 

4. Determine si los vectores dados son linealmente independientes       1 2 8  et y X2 =   et +   tet a) X1 =  −1 6 −8       2 1 1       , X2 =  −2  e−4t y X3 =  3  e3t b) X1 =  6       −2 −1 −13  5. Muestre que la funci´on Xp =  sistema

 X0 = 

1 1 2





1

 et + 

−1 



1

1 −1

 X− 

6. Demostrar que la soluci´on general del sistema  0 6 0  X0 =   1 0 1 1 1 0

  tet es soluci´on particular del

1 7

  et

  X 

es de la forma 

6





−3





2



    −t    e + c2  1  e−2t + c2  1  e3t X =c1  −1       1 −5 1 7. Determinar la soluci´on general de los sistemas:

227

Apuntes Mat023 (versi´on preliminar actualizada 23-05-2014)

a)

c)

e)

dx dt dy dt

= x + 2y

dx dt dy dt dz dt

= x+y−z

dx dt dy dt

=

6x − y

=

5x + 2y

=

=

b)

4x + 3y d)

2y

= y−z f)

dx dt dy dt

=

dx dt dy dt dz dt

=

2x − 7y

=

5x + 10y + 4z

=

5y + 2z

dx dt dy dt dz dt

=

2x + y + 2z

=

3x + 6z

2x + 2y

= x + 3y

= −4x − 3z

8. Resolver los siguientes sistemas usando variaci´on de par´ametros a)

c)

e)

dx dt dy dt

=

3x − 3y + 4

=

2x − 2y − 1

dx dt dy dt dz dt

= x+y−z+t

dx dt dy dt

= x + 2y + et csc t

=

b)

d)

2y + t

dx dt dy dt

dx dt dy dt

=

2x − y

=

3x − 2y + 4t

= −y + sec t = x

= y−z+t f)

= − 21 x + y + et sec t

dx dt dy dt dz dt

3x − y − z

=

= x+y−z+t = x − y + z + 2et

9. Sea X = X (t) la soluci´on al sistema de ecuaciones x0 = αx − βy y 0 = βx + αy que satisface la condici´on inicial X (0) = X0 . Determine condiciones sobre α, β que aseguren  l´ım X (t) = 

t→+∞

228

0 0

 

Apuntes Mat023 (versi´on preliminar actualizada 23-05-2014)

10. Determine condiciones sobre µ tal que (0, 0) sea un centro para el sistema lineal x0 = −µx + y y 0 = −x + µy 11. Sea A ∈ Mn×n (R) defina eAt = I + At + A2 =

∞ X

Ak

k=0

 a) Calcular eAt para A = 

a c 0 b

t3 t2 + A3 + · · · 2! 3!

tk k!

 

b) Muestre que si A = PDP−1 con D =diag(λ1 , λ2 , . . . , λn ) diagonal entonces etA = P diag eλ1 t , eλ2 t , . . . , eλn t



P−1

c) Muestre que la soluci´on general de 

4

3



d X X = dt −2 −1 es

 X =etA  

donde A = 

4

3

−2 −1

c1 c2

 

 

d ) Conjeturar una f´ormula para la soluci´on general de X0 = AX + B (t) en t´erminos de etA . 12. Clasificar la soluci´on de equilibrio de los siguientes sistemas y bosquejar el diagrama de fases: 229

Apuntes Mat023 (versi´on preliminar actualizada 23-05-2014)

a)

c)

e)

dx dt dy dt

= −x − 5y

dx dt dy dt

= −x + 2y

dx dt dy dt

= −3x + y

b)

= x + 3y d)

= x − 4y

= −2x − y

f)

dx dt dy dt

=

4x + 3y

=

2x − 3y

dx dt dy dt

= x + 4y

dx dt dy dt

=

2x + y

=

3x + 6y

= −x + y

230

Cap´ıtulo 4 : Transformaci´ on integral de Laplace

Definiciones y teoremas fundamentales Observaci´ on 4.1.1. Las ecuaciones diferenciales describen formas en las cuales ciertas cantidades cambian respecto al tiempo, cantidades tales como la corriente en un circuito el´ectrico, las oscilaciones de una membrana, flujos de calor, etc. esas ecuaciones generalmente est´an acompa˜ nadas de condiciones iniciales que describen el estado del sistema en el tiempo t = 0. Una herramienta muy poderosa para resolver este tipo de problemas es la transformada de Laplace la cual transforma la ecuaci´on diferencial original en una ecuaci´on algebraica la cual puede ser transformada nuevamente en la soluci´on del problema original. Esta t´ecnica es conocida como “el m´etodo de la transformada de Laplace”. La clave del proceso anterior radica en la f´ormula de integraci´on por partes. Si f, g ∈ C 1 recordemos que la f´ormula de integraci´on por partes corresponde a: Z b b Z b 0 f (x) g (x) dx = f (x) g (x) − f 0 (x) g (x) dx a

a

a

y se puede interpretar del modo siguiente: En un producto de funciones bajo el s´ımbolo de la integral es posible intercambiar la derivada presente en una funci´on. Sin embargo, tal procedimiento implica al final el tener que evaluar el producto de tales funciones en los extremos del intervalo y trabajar con una nueva derivada. La idea, entonces, es trabajar con una funci´on que tenga un buen comportamiento de la derivada y la evaluaci´on en los extremos del intervalo de integraci´on, esto se puede lograr mediante una funci´on que no cambie esencialmente al derivarla i.e. una exponencial y que se pueda anular en los extremos. Lo anterior motiva la siguiente definici´on: Definici´ on 4.1.1. Una funci´on f : [0, +∞) → R se dice localmente integrable si para RT cada T > 0, la integral 0 f (t) dt existe. Llamaremos transformaci´ on integral de Laplace o simplemente la transformada de Laplace a la funci´on L [f (t)] : D ⊆ R → R definida mediante la integral impropia: Z s → L [f (t)] (s) =



f (t) e−st dt

0

para todos los valores de s ∈ R para los cuales la integral sea convergente. El conjunto de 231

Apuntes Mat023 (versi´on preliminar actualizada 23-05-2014)

los valores de s ∈ R para los cuales la integral converge se llamar´a dominio de L [f (t)] y se denotar´a por D (L [f ]). Ejemplo 4.1.1. Sea f : R → R, t → f (t) = 1. Calcular su transformada de Laplace. Soluci´ on. Note que: Z

∞ −st

1e

Z

T

e−st dt T →∞ 0   1 −st T = l´ım − e T →∞ s 0 1 = s

dt =

0

l´ım

para cada s > 0. Por tanto: 1 L [1] (s) = , s

s>0

Ejemplo 4.1.2. Sea f (t) = eat , con a una constante real. Calcular su transformada de Laplace Soluci´ on. Por definici´on: at

Z



Z

at −st

T

dt = l´ım e(a−s)t dt T →∞ 0 0 1 (a−s)t T e = l´ım T →∞ a − s 0  1 e(a−s)T − 1 = l´ım T →∞ a − s  1 1 as´ı, si a − s < 0, entonces l´ımT →∞ a−s e(a−s)T − 1 = s−a , luego: 

L e



(s) =

e e

  L eat (s) =

1 s−a

Para s > a

Observaci´ on 4.1.2. Con respecto al ejemplo anterior, dos observaciones deben hacerse. En primer lugar, respecto a lo notacional, en lo sucesivo escribiremos: T ∞ l´ım f (t) = f (t)

T →∞

0

0

para evitar anotar cada vez que se calcule una transformada de Laplace, el l´ımite relacionado con la integral impropia. En segundo lugar, notamos que D (L [eat ]) = ]a, +∞[, luego la transformada no necesariamente estar´a definida para todos los valores de s ∈ R. 232

Apuntes Mat023 (versi´on preliminar actualizada 23-05-2014)

Ejemplo 4.1.3. Calcule, si acaso existe, la transformada de Laplace de f (t) = 1t , con t > 0. Soluci´ on. Sea t > 0. Luego: Z ∞ 0

e−st dt = t

Z 0

1

e−st dt + t

Z



1

e−st dt t

pero si 0 < t ≤ 1 y s > 0, entonces e−s ≤ e−st y la integral: Z 1 −s e dt t 0 diverge. Por tanto, la transformada de Laplace de f (t) = 1/t no existe para ning´ un valor de s ∈ R. En vista del ejemplo anterior, trataremos de dar respuesta a la siguiente pregunta: ¿Para qu´e tipo de funciones podemos asegurar la existencia de la transformada de Laplace? Definici´ on 4.1.2. Sean f una funci´on real. Anotaremos:  f a+ = l´ım+ f (t)



t→a

 f a− = l´ım− f (t) t→a

si acaso estos l´ımites existen. Diremos que una funci´on f tiene una discontinuidad de salto en a si:   f a+ − f a− < ∞ o bien si f (a+ ) = f (a− ), pero estos valores son distintos de f (a) (puede ser incluso que f (a) no est´e definida). Una funci´on f : [0, +∞) → R se dice seccionalmente continua si para cada T > 0 la funci´on f |[0,T ] es continua y tiene a lo sumo un n´ umero finito de discontinuidades de salto. Ejemplo 4.1.4. La funci´on   1/t t 6= 0 f (t) =  0 t=0 no es seccionalmente continua en [0, +∞), lo mismo para   cos (1/t) t > 0 f (t) =  0 t≤0 mientras que g (t) = btc si es seccionalmente continua. 233

Apuntes Mat023 (versi´on preliminar actualizada 23-05-2014)

Definici´ on 4.1.3. Diremos que una funci´on f : [0, +∞) → R es de orden exponencial β si existen constantes M > 0, β ∈ R y un valor t0 ≥ 0 tales que: |f (t)| ≤ M eβt para todo t ≥ t0 . Ejemplo 4.1.5. Las siguientes funciones son seccionalmente continuas y de orden exponencial: 1. f (t) = sinh t, pues: et − e−t et < 2 2 P tn 2. f (t) = tn . En efecto, sabemos que et = ∞ n=0 n! . Luego, para t > 0 , sinh t =

tn n!

< et . Por

tanto: tn ≤ n! et Teorema 4.1.1 (Existencia). Sea f : [0, +∞) → R una funci´on seccionalmente continua y de orden exponencial β entonces la transformada de Laplace L [f (t)] (s) existe para s > β. Demostraci´on. En primer lugar note que, como f : [0, +∞) → R es seccionalmente continua, la funci´on t 7→ f (t) e−st es localmente integrable en [0, +∞). Por otro lado, por la desigualdad triangular tenemos que: Z ∞ Z t0 Z −st −st ≤ + f (t) e dt f (t) e dt 0

0



|f (t)| e−st dt

t0

R t Note que 0 0 f (t) e−st dt < ∞ y adem´as, como f : [0, +∞) → R es de orden exponencial, existen constantes M, β > 0 y un valor t0 > 0 tales que: |f (t)| ≤ M eβt para todo t ≥ t0 . Luego: Z ∞ Z −st |f (t)| e dt ≤ M t0

∞ βt −st

e e

Z



dt = M

t0

t0

e(β−s)t dt =

M e(β−s) ∞ β − s t0

en donde el u ´ltimo t´ermino converge si y solo si β − s < 0. Por tanto, como: Z ∞ Z ∞ −st −st f (t) e dt < f (t) e dt 0

0

se tiene que L {f (t)} (s) existe para cada s > β. 234

Apuntes Mat023 (versi´on preliminar actualizada 23-05-2014)

t2



t2

Observaci´ on 4.1.3. La funci´on f (t) = 2te cos e



es continua en [0, +∞[ pero no de

orden exponencial sin embargo su transformada de Laplace existe para s > 0, por otro lado la funci´on g (t) =

1 √ t

no es seccionalmente continua en [0, +∞[ sin embargo posee

transformada de Laplace. Teorema 4.1.2 (Linealidad de la transformada). Sean f, g : [0, +∞) → R dos funciones localmente integrables y α ∈ R, entonces para cada s ∈ D (L [f ]) ∩ D (L [g]) se tiene que: L [αf + g] (s) = αL [f ] (s) + L [g] (s)

C´ alculo de transformadas Observaci´ on 4.2.1. En esta secci´on haremos el c´alculo de transformadas de Laplace b´asicas, para as´ı obtener una tabla elemental de transformadas. Ejemplo 4.2.1. Transformadas de f (t) = sin ωt y f (t) = cos ωt. Soluci´ on. Integrando por partes obtenemos Z α cos (βx) eαx β sin (βx) eαx eαx cos (βx) dx = + +C α2 + β 2 α2 + β 2 Z β cos (βx) eαx α sin (βx) eαx eαx sin (βx) dx = − + +C α2 + β 2 α2 + β 2 luego ∞

Z

sin (ωt) e−st dt

L [sin ωt] (s) = 0

+∞ ω cos (ωt) e−st s sin (ωt) e−st = − − s2 + ω 2 s2 + ω 2 0 ω = 2 s + ω2 si s > 0. De manera similar Z L [cos ωt] (s) =



cos (ωt) e−st dt

0

+∞ −s cos (ωt) e−st ω sin (ωt) e−st = + s2 + ω 2 s2 + ω 2 0 s = 2 s + ω2 para s > 0. 235

Apuntes Mat023 (versi´on preliminar actualizada 23-05-2014)

Ejemplo 4.2.2. Usemos variable compleja para calcular las mismas transformadas: Por la f´ormula de Euler: eiωt = cos ωt + i sin ωt y la linealidad de la transformada, se obtiene que: cos ωt =

eiωt + e−iωt 2

sin ωt =

eiωt − e−iωt 2i

y:

Entonces:  eiωt + e−iωt L (s) 2      1 L eiωt (s) + L e−iωt (s) 2  1 1 1 + 2 s − iω s + iω   1 s + iω + s − iω 2 s2 − (iω)2 s 2 s + ω2 

L [cos ωt] (s) = = = = =

para s > 0. An´alogamente, se obtiene que: L [sin ωt] (s) =

s2

ω , + ω2

s>0

Ejemplo 4.2.3. Transformadas de f (t) = sinh ωt y f (t) = cosh ωt. Sabemos que: cosh ωt =

eωt + e−ωt 2



sinh ωt =

eωt − eωt 2

Por la linealidad de la transformada de Laplace se obtiene que: L [cosh ωt] (s) =

s , − ω2

s > |ω|

ω , s2 − ω 2

s > |ω|

s2

y: L [sinh ωt] (s) =

236

Apuntes Mat023 (versi´on preliminar actualizada 23-05-2014)

Ejemplo 4.2.4. Transformada de f (t) = t. Notemos que: Z ∞ Z 0 1 ∞ −st te dt = − t e−st dt s 0 0   ∞ Z ∞ 1 −st −st e dt = − te − s 0 0   1 1 = − 0− s s 1 = 2 s para todo s > 0. Por tanto: L [t] (s) =

1 , s2

s>0

usando integraci´on por partes, es f´acil mostrar que L [tn ] (s) =

n! sn+1

,

s>0

Observaci´ on 4.2.2. Resumimos las funciones con sus respectivas transformadas de Laplace y sus dominios en la siguiente tabla: f (t)

1 t eαt cos ωt sin ωt cosh ωt sinh ωt tn

L [f (t)] (s) 1 s 1 s2

Dominio

s>0 s>0

1 s−α s 2 s + ω2 ω s2 + ω 2 s 2 s − ω2 ω 2 s − ω2 n! n+1 s

237

s>α s>0 s>0 s > |ω| s > |ω| s>0

Apuntes Mat023 (versi´on preliminar actualizada 23-05-2014)

Ejemplo 4.2.5. Calcule la transformada de Laplace de la funci´on: f (t) = 3 − 2t + 4 cos 2t Soluci´ on. Por la linealidad de la transformada y la tabla anterior, se obtiene: L [f (t)] (s) = L [3 − 2t + 4 cos 2t] (s) = 3L [1] (s) − 2L [t] (s) + 4L [cos 2t] (s) 3 2 4s = − 2+ 2 s s s +4 Ejemplo 4.2.6. si f (t) = a0 + a1 t + · · · + an tn entonces L [f (t)] (s) = =

n X k=0 n X k=0

  ak L tk (s) ak

k! sk+1

Teorema 4.2.1. Si f es seccionalmente continua en [0, +∞) y de orden exponencial β entonces l´ım L [f (t)] (s) = 0

s→+∞

El resultado anterior se debe a la desigualdad Z ∞ −st ≤ M e f (t) dt s−β 0 tomando s → +∞ se obtiene el resultado.

Observaci´ on 4.2.3. Las funciones

s−1 es , s+1 s

no pueden ser transformadas de una funci´on

seccionalmente continua de orden exponencial.

Primer Teorema de la Traslaci´ on Observaci´ on 4.3.1. El siguiente resultado nos permite calcular la transformada de Laplace de eαt f (t) cuando es conocida la transformada de Laplace de f (t), en este resultado se pone de manifiesto la primera propiedad de la traslaci´on de la transformada de Laplace. En efecto, tenemos el siguiente teorema: 238

Apuntes Mat023 (versi´on preliminar actualizada 23-05-2014)

Teorema 4.3.1. Suponga que F (s) = L [f (t)] (s) existe para cada s > β. Si α ∈ R, entonces:   L eαt f (t) (s) = F (s − α) para todo s > α + β. Ejemplo 4.3.1. Calcule L [e−3t cos 2t] (s) Soluci´ on. Notamos que F (s) = L [cos 2t] (s) =

s . s2 +4

  L e−3t sin 2t (s) = F (s + 3) =

Luego: s+3 (s + 3)2 + 4

Observaci´ on 4.3.2. La primera propiedad de traslaci´on de la transformada de Laplace, deja nuestra tabla b´asica de transformadas como sigue: L [f (t)] (s)

f (t)

eαt tn eαt cos ωt eαt sin ωt eαt cosh ωt eαt sinh ωt

Dominio

n! (s − α)n+1 s−α (s − α)2 + ω 2 ω (s − α)2 + ω 2 s−α (s − α)2 − ω 2 ω (s − α)2 − ω 2

s>α s>α s>α s > α + |ω| s > α + |ω|

Transformada de la derivada Observaci´ on 4.4.1. El origen y el objetivo de la transformada de Laplace es la resoluci´on de ecuaciones diferenciales. Para lograr lo anterior, debemos calcular la transformada de Laplace de la derivada de una funci´on. Entonces, sea y = y (t) una funci´on derivable, note que: Z

∞ 0

y (t) e 0

−st

∞ Z ∞ 0 dt = y (t) e − y (t) e−st dt 0 Z ∞ 0 = y (0) + s y (t) e−st dt −st

0

239

Apuntes Mat023 (versi´on preliminar actualizada 23-05-2014)

bajo el supuesto que: l´ım y (t) e−st = 0

t→∞

As´ı, si y = y (t) es una funci´on derivable y de orden exponencial, se tiene que y 0 (t) existe para cada t ≥ 0 y: |y (t)| ≤ M eβt ,

t ≥ t0

Entonces: −M e(β−s)t ≤ y (t) e−st ≤ M e(β−s)t donde los extremos convergen a cero si β − s < 0, cuando t → ∞. As´ı, bajo las condiciones anteriores se obtiene que: L [y 0 (t)] (s) = sL [y (t)] (s) − y 0+



Por tanto, tenemos el siguiente teorema: Teorema 4.4.1. Suponga que f es continua en ]0, ∞[ y de orden exponencial β, supongamos adem´as que f 0 es seccionalmente continua en [0, +∞) entonces: L [f 0 (t)] (s) = sL [f (t)] (s) − f 0+



Corolario 4.4.1. Suponga que f (t) , f 0 (t) , · · · , f (n−1) (t) son continuas en ]0, ∞[ y de orden exponencial, suponga tambi´en que f (n) (t) es seccionalmente continua en [0, +∞). Entonces      L f (n) (t) (s) = sn L [f (t)] (s) − sn−1 f 0+ − sn−2 f 0 0+ − · · · − f (n−1) 0+ Ejemplo 4.4.1. Calcule la transformada de Laplace de f (t) = tn , para n ≥ 2. Soluci´ on. Note que: f 0 (t) = ntn−1 as´ı, por la transformada de la derivada, se tiene que:    L ntn−1 (s) = L [f 0 (t)] (s) = sL [f (t)] (s) − f 0+ = sL [tn ] (s) as´ı: L [tn ] (s) =

n  n−1  L t (s) , s 240

n≥2

Apuntes Mat023 (versi´on preliminar actualizada 23-05-2014)

entonces: n

L [t ] (s) =

  

1 s2 n−1

n L [t s

n=1

s>0

] (s) n > 1

se sigue: L [tn ] (s) =

n! sn+1

,

s>0

Ejemplo 4.4.2. Como aplicaci´on de la transformada de la derivada, calcular la transformada de Laplace de sin2 (αt) Soluci´ on. Derivando f 0 (t) = 2α sin (αt) cos (αt) = α sin (2αt) luego sL [f (t)] (s) − f (0) = L [α sin (2αt)] (s) as´ı sL [f (t)] (s) = finalmente L [f (t)] (s) =

2α2 s2 + 4α2

2α2 s (s2 + 4α2 )

Ejemplo 4.4.3. Sea f : [0, +∞) → R una funci´on continua. Se define: Z t f (u) du F (t) = 0

Calcule L [F (t)] (s). Soluci´ on. Como f es continua, por el Teorema Fundamental del C´alculo, F es derivable con: F 0 (t) = f (t) Entonces, por la transformada de la derivada tenemos que: L (F 0 (t)) (s) = sL [F (t)] (s) − F 0+ As´ı: Z L 0

t



 1 f (t) dt (s) = L [F (t)] (s) = L [f (t)] (s) s

pues F (0) = 0. 241

Apuntes Mat023 (versi´on preliminar actualizada 23-05-2014)

Teorema 4.4.2. Sea n ∈ N, suponga que f es seccionalmente continua en [0, +∞) de orden exponencial β y que F (s) = L [f (t)] (s) entonces: L [tn f (t)] (s) = (−1)n

dn F (s) dsn

(4.1)

para s > β.

Observaci´ on 4.4.2. El siguiente teorema, habla sobre la inyectividad de la transformaci´on de Laplace, lo que nos permite hablar de inversa.

Teorema 4.4.3. Dos funciones con la misma transformada de Laplace no pueden diferir en todo un intervalo de longitud positiva. Es decir, si L [f (t)] (s) = L [g (t)] (s) entonces f (t) = g (t), para cada t tal que f y g simult´aneamente sean continuas. Observaci´ on 4.4.3. Funciones continuas diferentes tienen transformadas de Laplace diferentes. Definici´ on 4.4.1. Sea F : [0, +∞) → R una funci´on tal que l´ıms→∞ F (s) = 0. Diremos que una funci´on f : [0, +∞) → R, para la cual existe su transformada de Laplace, es una transformada de Laplace inversa de F si satisface: L [f (t)] (s) = F (s) Por el Teorema de Lerch, tal transformada inversa est´a u´nicamente determinada s´olo sobre sus puntos de continuidad. Abusando del lenguaje, se escribe: f (t) = L−1 [F (s)] (t) Observaci´ on 4.4.4. El operador L−1 tambi´en es lineal. Es decir: L−1 [αf + g] = αL−1 [f ] + L−1 [g] Ejemplo 4.4.4. Calcule L−1



s+9 s2 +6s+13



(t).

242

Apuntes Mat023 (versi´on preliminar actualizada 23-05-2014)

Soluci´ on. Observe que: s2

s+9 s+9 = + 6s + 13 (s + 3)2 + 4 s+3 6 = + 2 (s + 3) + 4 (s + 3)2 + 4

Entonces: L

−1



     s+3 s+9 2 −1 −1 (t) = L (t) + 3L (t) s2 + 6s + 13 (s + 3)2 + 4 (s + 3)2 + 4 = e−3t cos 2t + 3e−3t sin 2t

Ejemplo 4.4.5. Calcular L

−1



 s (s) (s + 1) (s + 2) (s + 3)

Soluci´ on. Usando la t´ecnica de fracciones parciales 2 1 3 s = − − (s + 1) (s + 2) (s + 3) s + 2 2 (s + 1) 2 (s + 3) luego 

 s L (s) (s + 1) (s + 2) (s + 3)   1 3 2 −1 − − = L (s) s + 2 2 (s + 1) 2 (s + 3) 3 1 = 2e−2t − e−t − e−3t 2 2 −1

Ejemplo 4.4.6. Calcular L

−1

   s+1 ln (s) s+2

Soluci´ on. Queremos determinar f (t) tal que  L [f (t)] (s) = ln

s+1 s+2



notamos que L [f (t)] (s) = ln (s + 1) − ln (s + 2) derivando L [f (t)]0 (s) =

1 1 − s+1 s+2

243

Apuntes Mat023 (versi´on preliminar actualizada 23-05-2014)

pero L [f (t)]0 (s) = L [−tf (t)] (s) se sigue   L [−tf (t)] (s) = L e−t − e−2t (s) as´ı −tf (t) = e−t − e−2t de donde obtenemos

e−2t − e−t f (t) = t

Ahora veremos algunos ejemplos de como funciona el m´etodo de la transformada de Laplace para resolver ecuaciones: Ejemplo 4.4.7. Resuelva la ecuaci´on diferencial: ty 00 − ty 0 − y = 0,

y (0) = 0,

y 0 (0) = 3

(4.2)

Soluci´ on. Sea Y (s) = L [ y (t)] (s). Aplicando la transformada de Laplace a la ecuaci´on (4.2), obtenemos: L [ty 00 ] (s) − L [ty 0 ] (s) − L [y] (s) = 0 pero: L [ty 00 ] (s) = −

d  2 s L [y] (s) − sy (0) − y 0 (0) = −s2 Y 0 − 2sY ds

y L [ty 0 ] (s) = −

d {sL [y] (s) − y (0)} = −sY 0 − Y ds

reemplazando estas expresiones en la ecuaci´on original obtenemos: −s2 Y 0 − 2sY + sY 0 + Y − Y = 0 rs decir: s (s − 1) Y 0 + 2sY = 0 entonces: Y0+

2 Y =0 s−1

244

Apuntes Mat023 (versi´on preliminar actualizada 23-05-2014)

la cual es una ecuaci´on de variable separable. Separando, entonces, las variables, obtenemos: dY 2 =− ds Y s−1 de donde se sigue que: Y (s) =

C (s − 1)2

se sigue que L [y (t)] (s) = Y (s) =

  C t 2 = L Cte (s) (s − 1)

usando el teorema de Lerch, se sigue y (t) = Ctet (note que se busca una funci´on derivable, luego continua). Ahora bien, como y 0 (0) = 3, se obtiene que C = 3. Por tanto, la soluci´on de la ecuaci´on diferencial es: y (t) = 3tet Ejemplo 4.4.8. Utilizando la transformada de Laplace, resuelva la siguiente ecuaci´on diferencial: y 00 − 4y = 0,

y (0) = 1,

y 0 (0) = 2

Soluci´ on. Aplicando la transformada de Laplace s2 L [y] (s) − sy (0) − y 0 (0) − 4L [y] (s) = 0 reemplazando los valores iniciales  s2 − 4 L [y] (s) = s + 2 as´ı L [y] (s) =

 2t  s+2 1 = = L e (s) s2 − 4 s−2

finalmente la soluci´on de la ecuaci´on es: y (t) = e2t

245

Apuntes Mat023 (versi´on preliminar actualizada 23-05-2014)

Funciones escalonadas y Segundo Teorema de la Traslaci´ on Observaci´ on 4.5.1. Cuando se estudio el movimiento vibratorio en el caso m´as general, se obtuvo una ecuaci´on diferencial de la forma: x00 +

k c 0 x + x = F (t) m m

para constantes f´ısicas c, k y m. La funci´on forzadora F (t), que actuaba como fuerza externa al sistema, se supuso continua. En las aplicaciones, no siempre se puede pedir tal condici´on; muchas veces, tal funci´on es una funci´on escalonada o con discontinuidades de salto. Definici´ on 4.5.1. La funci´ on de Heaviside con salto en 0 se define como la funci´on:   1 t≥0 H (t) =  0 t 0 est´a dada por Ha (t) = H (t − a), con t ∈ R. Observaci´ on 4.5.2. Para las funciones anteriores, se tiene: Z ∞ 1 L [H (t)] (s) = H (t) e−st dt = , s 0 y para todo a > 0: Z



L [Ha (t)] (s) = Z0 ∞ =

Ha (t) e−st dt e−st dt

a

e−as = s para s > 0. Observaci´ on 4.5.3. Note que si a < b la funci´on: f (t) = Ha (t) − Hb (t) corresponde a la funci´on   1 t ∈ [a, b[ f (t) =  0 t ∈ [a, b[c 246

s>0

Apuntes Mat023 (versi´on preliminar actualizada 23-05-2014)

Observaci´ on 4.5.4. Para introducir la segunda propiedad de traslaci´on de la transformada de Laplace, requerimos truncar una funci´on. M´as precisamente, consideremos f : R → R una funci´on y a > 0. Entonces, Ha fa : R → R es la funci´on definida como:   f (t − a) t ≥ a (Ha fa ) (t) =  0 t 0 y f una funci´on con transformada de Laplace, entonces: L [(Ha fa ) (t)] (s) = e−as L [f (t)] (s) Ejemplo 4.5.1. L [sin a (t − b) H (t − b)] (s) = e−bs L [sin at] (s) = Ejemplo 4.5.2. Calcule L [f (t)] (s), si:   et f (t) =  et + cos t

0 ≤ t < 2π t > 2π

Soluci´ on. Note que f (t) = et + H (t − 2π) (cos t) = et + H (t − 2π) (cos (t − 2π + 2π)) = et + H (t − 2π) (cos (t − 2π)) as´ı L [f (t)] (s) =

1 se−2πs + 2 s−1 s +1

Ejemplo 4.5.3. Calcule: −1

L



 1 − e−πs/2 (t) 1 + s2

247

ae−bs . s2 +a2

Apuntes Mat023 (versi´on preliminar actualizada 23-05-2014)

Soluci´ on. Notamos que 1 − e−πs/2 1 1 − e−πs/2 2 = 2 2 1+s s +1 s +1 −πs/2 = L [sin t] (s) − e L [sin t] (s)  h  π i π sin t − (s) = L [sin t] (s) − L H t − 2 2 h    i π π = L sin t − H t − sin t − (s) 2 2 luego −1



L

  1 − e−πs/2 π (t) = sin t + H t − cos t 1 + s2 2

Ejemplo 4.5.4. Resolver el P.V.I. dado por y 00 − 2y = f (t) , y (0) = y 0 (0) = 0 donde

  t 0≤t1

Soluci´ on. Usando la funci´on salto podemos escribir la ecuaci´on como y 00 − 2y = t + H (t − 1) t2 − t



para poder usar el segundo teorema de la traslaci´on manipulamos la funci´on t2 − t = (t − 1 + 1)2 − (t − 1 + 1) = (t − 1)2 + 2 (t − 1) + 1 − (t − 1) − 1 = (t − 1)2 + (t − 1) as´ı y 00 − 2y = t + H (t − 1) (t − 1)2 + H (t − 1) (t − 1) se sigue  1 2 1 s2 − 2 L [y] (s) = 2 + e−s 3 + e−s 2 s s s luego L [y] (s) =

1 s2

(s2

− 2)

+ e−s

2 s3

248

(s2

− 2)

+ e−s

1 s2

(s2

− 2)

Apuntes Mat023 (versi´on preliminar actualizada 23-05-2014)

usando fracciones parciales 1

1 = s2 (s2 − 2) 2



1 1 − s2 − 2 s2



y 2 s3

(s2

− 2)

=

1 s 1 1 − − 3 2 2 s − 2 2s s

luego 1

 1 1 − s2 − 2 s2  √  t  1 (s) = L √ sinh 2t − 2 2 2

1 = 2 2 s (s − 2) 2



y 2 s3

(s2

1 s 1 1 − − 3 2 − 2) 2 s − 2 2s s  √  1 t2  1 = L cosh 2t − − (s) 2 2 2 =

se sigue L [y] (s) =

1

+ e−s

2

+ e−s

1

s2 (s2 − 2) s3 (s2 − 2) s2 (s2 − 2)     √ 1 t = L √ sinh 2t − (s) 2 2 2  √  1 t2  1 −s +e L cosh 2t − − (s) 2 2 2  √  t  1 −s +e L √ sinh 2t − (s) 2 2 2  √  t  1 (s) = L √ sinh 2t − 2 2 2 " !# √  1 (t − 1)2 1 +L H (t − 1) cosh 2 (t − 1) − − (s) 2 2 2   √  (t − 1)  1 2 (t − 1) − (s) +L H (t − 1) √ sinh 2 2 2

la soluci´on es entonces √  t 1 √ y (t) = sinh 2t − 2 2 2 ! 2     √ √ 1 1 (t − 1) 1 (t − 1) +H (t − 1) cosh 2 (t − 1) − − + √ sinh 2 (t − 1) − 2 2 2 2 2 2 249

Apuntes Mat023 (versi´on preliminar actualizada 23-05-2014)

La Transformada de integrales de convoluci´ on Definici´ on 4.6.1. Sean f y g funciones seccionalmente continuas en [0, +∞). Se define la convoluci´ on de f y g como la funci´on: t

Z (f ∗ g) (t) =

f (t − u) g (u) du 0

Ejemplo 4.6.1. La convoluci´on entre f (t) = t y g (t) = sin t, est´a dada por: Z t t ∗ sin t = (t − u) sin u du 0 Z t Z t = t sin u du − u sin u du 0

0

= t − t cos t − sin t + t cos t = t − sin t Ejemplo 4.6.2. Calcular t ∗ t2 Soluci´ on. t∗t

2

Z =

t

(t − u) u2 du

0

1 4 = t 12 Observaci´ on 4.6.1. Dentro de las propiedades algebraicas del producto de convoluci´on, destacaremos la propiedad de conmutatividad. En efecto, observe que: Z t (f ∗ g) (t) = f (t − u) g (u) du 0 Z 0 = − f (z) g (t − z) dz ←− z = t − u t Z t = g (t − z) f (z) dz 0

= (g ∗ f ) (t) otras de sus propiedades b´asicas son: 1. α (f ∗ g) = (αf ) ∗ g = f ∗ (αg) para α constante. 2. f ∗ (g ∗ h) = (f ∗ g) ∗ h 250

Apuntes Mat023 (versi´on preliminar actualizada 23-05-2014)

3. f ∗ (g + h) = (f ∗ g) + (f ∗ h) Teorema 4.6.1. Sean f y g funciones seccionalmente continuas en [0, +∞) y de orden exponencial β entonces: L [(f ∗ g) (t)] (s) = L [f (t)] (s) · L [g (t)] (s) o bien L−1 [L [f ] (s) L [g] (s)] (t) = (f ∗ g) (t) Observaci´ on 4.6.2. El resultado anterior, desde una perspectiva pr´actica se utiliza diciendo que la transformada de Laplace inversa de un producto de transformadas es la convoluci´on. Ejemplo 4.6.3. Calcule: ( L−1

s+1  2 (s − 2) (s + 1)2 + 32

)

Soluci´ on. Note que: s+1 1 s+1 = · 2 2 (s − 2) (s + 1) + 32 (s − 2) (s + 1)2 + 32 2

Luego: " L−1

#     s+1 1 s+1 −1 −1  (t) = L (s) (t) ∗ L (s − 2)2 (s + 1)2 + 32 (s − 2)2 (s + 1)2 + 32 = te2t ∗ e−t cos (3t) Z t = (t − u) e2(t−u) e−u cos (3u) du 0

1 2t 1 = te − e−t sin 3t 6 18 Ejemplo 4.6.4. Calcular L

−1



1 (s − 2) (s + 2) (s − 1)

 (t)

Soluci´ on. Esto lo podemos calcular con fracciones parciales pero 1 1 1 1 = (s − 2) (s + 2) (s − 1) s−2s+2s−1   = L e2t ∗ e−2t ∗ et (s) 251

Apuntes Mat023 (versi´on preliminar actualizada 23-05-2014)

donde 2t

e ∗e

−2t

t

Z = 0

e−2t 1 e2(t−u) e−2u du = e2t − 4 4

y 2t

−2t

e ∗e

t

∗e

t

1 2u e−2u e e − = 4 4 0 −2t e 1 1 = − et + e2t 12 3 4 Z

t−u



se sigue −1



L

1 (s − 2) (s + 2) (s − 1)

 (t) =

 du

e−2t 1 t 1 2t − e + e 12 3 4

Definici´ on 4.6.2. Sean f (t) , g (t) funciones continuas. Llamaremos ecuaci´ on de Volterra a una ecuaci´on de la forma: Z

t

g (t − u) x (u) du

x (t) = f (t) + 0

Observaci´ on 4.6.3. Note que, mediante el producto de convoluci´on, podemos escribir la ecuaci´on anterior como: x (t) = f (t) + (g ∗ x) (t) Ahora bien, si ponemos X (s) = L [x (t)] (s) , F (s) = L [f (t)] (s) y G (s) = L [g (t)] (s) y aplicamos luego la transformada de Laplace a la ecuaci´on anterior, obtenemos: X (s) = F (s) + G (s) X (s) de donde se obtiene finalmente: X (s) =

F (s) 1 − G (s)

de donde podemos obtener x (t) = L

−1



 F (s) (t) 1 − G (s)

Ejemplo 4.6.5. Resuelva la ecuaci´on integral: Z t 2 x (t) = t + sin (t − u) x (u) du 0

252

Apuntes Mat023 (versi´on preliminar actualizada 23-05-2014)

Soluci´ on. Supongamos que X (s) = L [x (t)] (s). Entonces, aplicando la transformada de Laplace a la ecuaci´on anterior, tenemos: X (s) =

2 1 X (s) + s3 s2 + 1

Es decir: 2 2 + s3 s5 2 2 4! = 3+ s 4! s5

X (s) =

Por lo tanto: 

   2 2 −1 4! x (s) = L (t) + L (t) s3 4! s5 t4 = t2 + 12 −1

Ejemplo 4.6.6. Resolver el sistema de ecuaciones x0 = x − y + t y 0 = x + y + et con las condiciones iniciales x (0) = y (0) = 0. Soluci´ on. Pongamos X (s) = L [x (t)] (s) y Y (s) = L [y (t)] (s) entonces sX = X − Y +

1 s2

sY

1 s−1

= X +Y +

luego 2s − 1 − 4s3 + 3s4 − s5 s − 12 1 1 = 2 − − 2 s − 2s + 2 s − 1 2s s2 + 1 = 2s2 − 2s3 + s4 1 s − 32 1 1 = − 22 + 2 2s s − 2s + 2 2s

X =

Y

2s2

253

Apuntes Mat023 (versi´on preliminar actualizada 23-05-2014)

de donde obtenemos 1 1 x (t) = et cos t + et sin t − et − t 2 2 1 1 1 t y (t) = t + − e (cos t − 2 sin t) 2 2 2 Observaci´ on 4.6.4. Como ap´endice calcularemos la transformada de la “funci´on” delta de Dirac. Muy informalmente, la funci´on delta de Dirac se describe como una funci´on que es cero en todas partes excepto en t = 0 y cumple adem´as con: Z ∞ δ (t) dt = 1 −∞

Claramente, ninguna funci´on real puede satisfacer tales condiciones. Sin embargo, considere la familia de funciones: fε (t) =

 

1 2ε

,

−ε < t < ε

 0

,

|t| > ε

Note que, para cada ε > 0 las funciones fε (t) son seccionalmente continuas, y adem´as: Z ∞ Z ε fε (t) dt = fε (t) dt = 1 −∞

−ε

De este modo, podemos pensar a δ como: δ (t) = l´ım+ fε (t) ε→0

Definici´ on 4.6.3. Sea a > 0. La funci´on delta de Dirac con salto en t = a se define como: δa (t) = δ (t − a) Observaci´ on 4.6.5. Notemos que 1 1 fε (t − a) = H (t − a + ε) − H (t − a − ε) 2ε 2ε as´ı e(−a+ε)s − e−(ε+a)s L [fε (t − a)] (s) = 2εs se sigue L [δ (t − a)] (s) = l´ım L [fε (t − a)] (s) ε→0

eεs − e−εs ε→0 2εs εs se + se−εs −as = e l´ım ε→0 2s −as = e = e−as l´ım

254

Apuntes Mat023 (versi´on preliminar actualizada 23-05-2014)

Teorema 4.6.2. Sea a > 0, entonces: L [δa (t)] (s) = e−as

s>0

y L [δ (t)] (s) = 1

Ejemplo 4.6.7. Resolver el P.V.I. y 00 − y = δ2 (t) y (0) = 0 y 0 (0) = 0 Soluci´ on. Aplicando la transformada de Laplace  s2 − 1 L [y] (s) = e−2s as´ı e−2s = e−2s L [sinh (t)] (s) s2 − 1 = L [H (t − 2) sinh (t − 2)] (s)

L [y] (s) =

as´ı y (t) = H (t − 2) sinh (t − 2) Observaci´ on 4.6.6. Una aplicaci´on interesante es estudiar la ecuaci´on diferencial: L

Q dI + R I + = δa (t) dt C

que es la ecuaci´on diferencial asociada a un circuito RLC en serie. Es decir, un circuito el´ectrico compuesto por una resistencia R [Ω], un capacitor C [F], un inductor L [H] y una fuente de voltaje modelada por la funci´on delta de Dirac como funci´on forzadora. Naturalmente, las herramientas de las ecuaciones diferenciales a coeficientes constantes no bastan para resolver este sistema.

255

Apuntes Mat023 (versi´on preliminar actualizada 23-05-2014)

Ejercicios del cap´ıtulo 1. Sean α, β > 0. Encontrar la transformada de Laplace de: 1)

sin (t + α) + cos2 (βt)

2)

 π (αt + β) e5t + t cos 2t + 4

3)

t2 cos t + 2e−3t sin (2t)

4)

t3 e2t cos (6t) + 7e−4t sin t cos2 t

5)

te

2t

Z

t

 u

0

7)

9)

 d 3u e cos u du du

6)

f (t) =

Z

8)

sinh [2 (t − u)] cosh (2u) du 0

et

si

4π < t

t

Z

3

12)

t

eu−t cos (3u) du

0

2. Si ω > 0, calcular las transformadas de: Z t sin ωt sin (ωu) 1) 2) du t u 0

7)

1 − cosh ωt t 2 erf (t) = √ π

cos2 (u) du

t

10)

0

4)

t

  cos t si 0 ≤ t < 2π f (t) =  sin t si t > 2π

Z

(t − u) sin u du

11)

t

0 ≤ t ≤ 2π

sin t si 2π < t ≤ 4π

  

Z 3

  t si t < 2 f (t) =  0 si t > 2     cos t si

2

Z

t

5) 0

Z 0

t

2

e−u du

8)

∞ X n=0

256

1 − cos (ωu) du u

H (t − nω)

3)

cos ωt − 1 t Z

6) 0

t

1 − cosh (ωu) du u

Apuntes Mat023 (versi´on preliminar actualizada 23-05-2014)

3. Encontrar la transformada inversa de las siguientes funciones: s+1 3 2 3) 2) 1) 3 s (s + 1) (s + 1) s (s + 4)2 4)

2s s (s − 7)

5)

s s2 + 6s + 25

6)

7)

e−s (s2 + 1)2

8)

e−3s (s − 1) (s + 2)

9)

10)

s2 s3 + 2 3

13) e

−2s



1 1 + s2 s4

16)

e−3s s2 − 9

19)

1 (s2 + 1)3

11)

n X e−ks k=1



s

14)

17)

s 2 (s + 3) (s2 + 1)

20)

ln

s s+1

s3

1 + 23 

  1 arctan s



2s (s2 + 4)2

s2 + 1 s (s − 3)

12)

ln

15)

e−πs s s2 + 3

18)

1 arctan s

21)

  52 ln 1 + 2 s





  1 s

4. Suponga que α, β ∈ R, n, m ∈ N. Calcular expl´ıcitamente las siguientes convoluciones: a) exp (αt) ∗ exp (βt) b) tn ∗ exp (αt) c) tn ∗ tm d ) exp (αt) ∗ sin (βt) e) exp (αt) ∗ cos (βt) f ) sin (αt) ∗ sin (βt) g) sin (αt) ∗ cos (βt) h) cos (αt) ∗ cos (βt) i ) tn ∗ cos (αt) 257

Apuntes Mat023 (versi´on preliminar actualizada 23-05-2014)

j ) tn ∗ sin (αt) 5. Escriba las transformadas inversas de las siguientes funciones en t´erminos de convoluciones: s3 (s2 + 4) (s2 + 9) (s2 + 16) s b) (s − 2) (s + 2) (s − 1)       s+1 1 1 c) ln ln 1 + 2 arctan s+2 s s

a)

6. Resuelva las siguientes ecuaciones integrales o integro-diferenciales. −t

a) x (t) = e

t

Z

cos (t − u) x (u) du

−2 0

Z 1 t (t − u)3 x (u) du b) x (t) = t + 6 0 Z t sin (t − u) x (u) du c) x (t) = t + 0

Z

t

cos (t − u) x (u) du

d ) x (t) = sin t + 0 0

Z

t

x (u) du = f (t) donde

e) x + 2x + 0

  cos t si 0 ≤ t < 2π f (t) =  sin t si t > 2π con la condici´on inicial x (0) = 1 Z t f) sin (t − u) y (u) du = y (t) + et cos (3t) 0

7. Suponga que f, f 0 : [0, +∞[ → R Z sonlocalmente integrables y de orden exponen t Z u df cial: Calcular la transformada de f (v) dv du y tet en t´erminos de la dt 0 0 transformada de f . 8. La funci´on Gamma est´a definida por: Z ∞ Γ (x) = e−u ux−1 du, 0

258

x>0

Apuntes Mat023 (versi´on preliminar actualizada 23-05-2014)

a) Demuestre que Γ (x + 1) =

R∞ 0

e−u ux dx.

b) Integrando por partes, muestre que: Γ (x + 1) = xΓ (x) c) Demuestre que Γ (1) = 1 d ) Usando los resultados de (b) y (c), pruebe que si n es un entero positivo, entonces: Γ (n + 1) = n! e) Calcule, usando la sustituci´on u = st en la parte (a), demuestre que: L [tx ] (s) = f ) Se sabe que Γ n o 1) L √1t √ t 2) L  5/2 3) L t

1 2



=



Γ (x + 1) , sx+1

s > 0,

x > −1

π. Calcule:

9. Resuelva las siguientes ecuaciones diferenciales mediante Transformada de Laplace: a) y 00 + y = 0, b) y 00 − 4y = 0,

y (0) = 1,

y 0 (0) = 3

y (0) = 2,

c) y 00 + 4y 0 + 4y = 0,

f ) y (4) − y = 0,

y 0 (0) = 2

y (0) = −1,

d ) y 00 + 2y 0 + 2y = H (t − 3) , e) y 000 + y = 0,

y 0 (0) = −5

y (0) = y 0 (0) = 0

y (0) = y 00 (0) = 1,

y 0 (0) = −1

y (0) = y 00 (0) = 0,

g) y (4) − y = sinh t,

y 0 (0) = y 000 (0) = 1

y (0) = y 00 (0) = 0,

y 0 (0) = y 000 (0) = 1

10. Resolver los siguientes problemas de valores iniciales

259

Apuntes Mat023 (versi´on preliminar actualizada 23-05-2014)

a) y 00 − 4y = f (t) donde

f (t) =

   

t2

si 0 ≤ t < 1

1 − t si 1 < t < 3

  

et

3 0 entonces Z T 1 L [f (t)] (s) = e−st f (t) dt 1 − e−sT 0 RT R +∞ Ind.: L [f (t)] (s) = 0 f (t) e−st dt + T f (t) e−st dt y cambie variable. 17. Calcular la transformada de las funciones f (t) = |sin ωt| y f (t) = t − btc 18. Calcular las transformadas de las funciones de la figura

262

Cap´ıtulo 5 : Series de Fourier

Definiciones El espacio de las funciones continuas en un intervalo [a, b] a valores reales representado por C [a, b], es un espacio de dimensi´on infinita, pues para todo m ∈ N se cumple 

1.x.x2 , . . . , xm ⊂ C [a, b]

y el conjunto {1.x.x2 , . . . , xm } es linealmente independiente. Sabemos que si un espacio V tiene una base B = {v1 , v2 , . . . , vm } entonces todo elemento del espacio se puede escribir como combinaci´on lineal, esto es, v=

m X

α j vj

j=1

al ser un espacio de dimensi´on infinita, debemos esperar que los elementos sean mas complicados que un polinomio, por ejemplo sabemos que ex no es un polinomio. Es natural preguntarse entonces, si todos los elementos se pueden escribir como una “combinaci´on lineal infinita” de los elementos de este conjunto, esto es, una serie de la forma f (x) =

+∞ X

ak xk = a0 + a1 x + a2 x2 + · · ·

k=0

lo que llamamos serie de Taylor. Sabemos de Mat022 que las funciones que se pueden escribir de esta forma son funciones de clase C ∞ en su intervalo de convergencia, luego no es de esperar que toda funci´on continua se pueda escribir como una serie de Taylor ¿existir´a otra forma de escribir estas funciones en forma de serie? La respuesta a esta pregunta es afirmativa y viene dada por las series de Fourier, en alg´ un sentido que vamos a especificar en breve, toda funci´on continua f ∈ C [a, b] se puede escribir en la forma   X   +∞ +∞ a0 X 2nπx 2nπx + an cos + bn sin 2 b−a b−a n=1 n=1 para elecciones adecuadas de las constantes a0 , an y bn .

El espacio SC [a, b] Definici´ on 5.1.1. Una funci´on a valores reales f es llamada funci´on seccionalmente continua en [a, b] si: 263

Apuntes Mat023 (versi´on preliminar actualizada 23-05-2014)

1. f esta definida y es continua en todos los puntos de [a, b] salvo quiz´as un n´ umero finito de ellos. 2. Los l´ımites f x+ 0



f x− 0



=

l´ım f (x)

x→x+ 0

=

l´ım f (x)

x→x− 0

existen en cada punto de [a, b] (note que en los extremos solo un l´ımite es relevante).

Z Observaci´ on 5.1.1. Si f ∈ SC [a, b] entonces

b

f (x) dx existe y es independiente de a

los valores (si los toma) de la funci´on en los puntos de discontinuidad. En particular, si f, g ∈ SC [a, b] son iguales salvo en los puntos de discontinuidad entonces Z b Z b f (x) dx = g (x) dx a

a

diremos entonces que dos funciones en SC [a, b] son iguales si f (x) = g (x) salvo quiz´as en los puntos de discontinuidad. Observaci´ on 5.1.2. Si α ∈ R, f, g ∈ SC [a, b] entonces αf , f + g y f g son funciones en SC [a, b]. Observaci´ on 5.1.3. C [a, b] ⊆ SC [a, b].

Definici´ on 5.1.2. Sea V un espacio vectorial real. Diremos que la funci´on h·, ·i : V ×V −→ R es un producto interior si cumple: 1. ∀f ∈ V , hf, f i ≥ 0, Adem´as hf, f i = 0 ⇔ f = 0 2. ∀f, g ∈ V , hf, gi = hg, f i 3. ∀α ∈ R, ∀f ∈ V , hαf, gi = α hf, gi 4. ∀f, g, h ∈ V , hf + g, hi = hf, hi + hg, hi

264

Apuntes Mat023 (versi´on preliminar actualizada 23-05-2014)

Ejemplo 5.1.1. En V = Rn , la funci´on h·, ·i : V × V −→ R definida por < x, y >=

n X

xi y i

i=1

es un producto interior.

Ejemplo 5.1.2. En SC[a, b] la funci´on h·, ·i : SC[a, b] × SC[a, b] −→ R definida por Z (f, g) → hf, gi =

b

f (x)g(x) dx a

es un producto interior. Llamaremos a este producto, el producto usual en SC[a, b]. Ejemplo 5.1.3. En C[a, b], si ρ ∈ C [a, b] es tal que ρ (x) > 0 para x ∈ [a, b] la funci´on h·, ·i : C[a, b] × C[a, b] −→ R definida por Z

b

(f, g) → hf, gi =

ρ (x) f (x)g(x) dx a

es un producto interior. Este producto es llamado producto con peso ρ.

En todo espacio vectorial con producto interior es posible definir una norma k · k : V −→ R como kf k =

p hf, f i

y esta a su vez, nos permite definir una funci´on distancia en el espacio V mediante la f´ormula d(f, g) = kf − gk Teorema 5.1.1. Sea (V, h·, ·i) un espacio vectorial real con producto interior, para toda f, g ∈ V se tiene |hf, gi| ≤

p

p hf, f i hg, gi

265

Apuntes Mat023 (versi´on preliminar actualizada 23-05-2014)

Ejemplo 5.1.4. Si f (x) = ex , g(x) = e−x son funciones en C [0, 1], calcular d (f, g), con la distancia inducida por el producto usual del ejemplo. Tenemos p hf − g, f − gi s Z 1 = (f (x) − g (x))2 dx

d (f, g) =

0

s Z

1

(ex − e−x )2 dx

= =



0

sinh 2 − 2

Proposici´ on 5.1.1. Propiedades b´asicas del producto interior h·, ·i en un espacio vectorial real V : 1. ∀f, g ∈ V, ∀α ∈ R, hf, αgi = α hf, gi 2. ∀f, g, h ∈ V, hf, g + hi = hf, gi + hf, hi 3. ∀fk , gj ∈ V, con k = 1, 2, . . . , n y j = 1, 2, . . . , m y ∀αk , βj ∈ R, con k = 1, 2, . . . , n y j = 1, 2, . . . , m * n X k=1

αk fk ,

m X

+ βj gj

j=1

=

n X m X

αk βj hfk , gj i

k=1 j=1

Definici´ on 5.1.3. Sea V un espacio con producto interior < ·, · >. 1. Diremos que f y g en V son ortogonales si < f, g >= 0

2. Diremos que {fk }k∈I es un conjunto ortogonal si < fk , fj >= 0, para k 6= j

3. Diremos que {fk }k∈I es un conjunto ortonormal si es ortogonal y la norma de cada elemento es 1

266

Apuntes Mat023 (versi´on preliminar actualizada 23-05-2014)

 Ejemplo 5.1.5. En SC[0, L], la familia sin

nπx L



, n ∈ N es ortogonal.

En efecto, si n 6= m entonces Z  nπx   mπx  sin sin dx L L   L (m − n) πx (m + n) πx = (m + n) sin + (n − m) sin 2π (m2 − n2 ) L L (usar prostaf´eresis) luego Z L  nπx   mπx  sin sin dx L L 0   L L (m − n) πx (m + n) πx = (m + n) sin + (n − m) sin 2π (m2 − n2 ) L L  0 L (m + n) πL (m − n) πL = + (n − m) sin (m + n) sin 2π (m2 − n2 ) L L L ((m + n) sin [(m − n) π] + (n − m) sin [(m + n) π]) = 2π (m2 − n2 ) = 0 Ejemplo 5.1.6. En SC[a, b] la familia             2πx 2πx 4πx 4πx 6πx 6πx {1, cos , sin , cos , sin , cos , sin ,...} b−a b−a b−a b−a b−a b−a =       2nπx 2nπx 1, cos , sin con n ∈ N b−a b−a es una familia ortogonal.

Ejemplo 5.1.7. Si (V, h·, ·i) es un espacio con producto interior y {u1 , u2 , u3 } ⊆ V es un conjunto ortonormal, determine la norma del vector u1 − u2 + u3 Soluci´ on. Usando las propiedades del producto punto y la ortogonalidad ku1 − u2 + u3 k2 = hu1 − u2 + u3 , u1 − u2 + u3 i = ku1 k2 + ku2 k2 + ku3 k2 = 3 267

Apuntes Mat023 (versi´on preliminar actualizada 23-05-2014)

(los vectores tienen norma 1) entonces √ ku1 − u2 + u3 k = 3 n o √ x , sin √ 2x Ejemplo 5.1.8. Muestre que el conjunto B = √12π , cos es ortonormal en C [−π, π] π π y calcular Z

π

−π



1 cos x sin 2x √ − √ + √ π π 2π

!1/2

2 dx

Soluci´ on. 

1 cos x √ , √ π 2π



Z

π

Z

π

cos x √ dx 2π −π Z π 1 = √ cos xdx 2π −π π 1 = √ sin (x) = 0 2π −π

=

y 

1 sin x √ , √ π 2π

 =

−π

sin 2x √ dx = 0 2π

(funci´on impar)  Z π sin 2x cos x sin 2x cos x √ , √ dx = 0 = π π π −π funci´on impar. Es f´acil ver que √

1 1

√ = √ k1k = √2π = 1

2π 2π 2π

Z π 1/2

sin 2x 2

√ = √1 sin 2xdx

π π −π  Z π 1/2 1 1 − cos 4x = √ 2 dx 2 π 0 1 √ 1/2 = √ π =1 π

Z π 1/2

cos x 1 2

√ = √ cos xdx

π π −π  Z π 1/2 1 2 = √ 2 cos xdx π 0  Z π 1/2 1 + cos 2x 1 = √ 2 dx 2 π 0 = 1 

268

Apuntes Mat023 (versi´on preliminar actualizada 23-05-2014)

usando el ejercicio anterior 2 !1/2 Z π √ 1 cos x sin 2x √ − √ + √ dx = 3 π π 2π −π

Teorema de la mejor aproximaci´ on Ahora vamos a analizar cuando tenemos familia ortonormales como se logra la mejor aproximaci´on de un elemento en un espacio dado. La idea es obtener una aproximaci´on con distancia cero. Teorema 5.1.2. Sea {fk }N k=1 un conjunto ortogonal en V . Si λk ∈ R, ∀k = 1, . . . N entonces

2 N N

X

X

λ f = |λk |2 kfk k2

k k

k=1

k=1

Demostraci´on.

N

2 + * N N

X

X X

λk f k = λk f k , λj f j

k=1

j=1

k=1

=

N X N X

λk λj hfk , fj i

k=1 j=1

=

N X

λ2k kfk k2

k=1

Note que si el conjunto {fk }N k=1 es ortogonal (y funciones no nulas) en V entonces es linealmente independiente N X

λk fk = θV

k=1

implica 0 = hθV , fi i =

* N X k=1

=

N X

λk hfk , fi i

k=1

= λi hfi , fi i 269

+ λk fk , fi

Apuntes Mat023 (versi´on preliminar actualizada 23-05-2014)

de donde obtenemos λi = 0. Sea f ∈ V dado, ¿Cu´al es la mejor aproximaci´on que podemos hacer en V mediante un elemento de de la forma

N X

λk fk

k=1

esto es un elemento del subespacio generado por {f1 , f2 , ..., fN } S = h{f1 , f2 , ..., fN }i donde {fk } es un conjunto ortonormal.

d f,

N X

!2 λk f k

k=1

2 N

X

= f − λk f k

k=1

= kf k2 − 2 = kf k2 − 2 = kf k2 −

N X k=1 N X

λk hf, fk i + λk hf, fk i +

N X k=1 N X

k=1 N X

N X

k=1

k=1

d f,

!

hf, fk i2 +

(hf, fk i − λk )2

λk f k

v u N u X 2 t ≥ kf k − hf, fk i2

k=1

k=1

luego el m´ınimo valor de esta distancia corresponde a v u N u X tkf k2 − hf, fk i2 k=1

que se alcanza en la combinaci´on lineal de la forma N X

hf, fk i fk

k=1

se sigue que

270

|λk |2

k=1

se sigue, independiente de los escalares λ1 , λ2 , . . . , λN N X

|λk |2 kfk k2

Apuntes Mat023 (versi´on preliminar actualizada 23-05-2014)

N X

hf, fk i fk

k=1

es la mejor aproximaci´on de f ∈ V en el espacio. S = h{f1 , f2 , ..., fN }i y

m´ın

(λ1 ,...,λN )∈Rn

d f,

N X

! λk fk

v u N u X = tkf k2 − < f, fk >2

k=1

k=1

Teorema 5.1.3. Sea {fk }N k=1 un conjunto ortonormal en un espacio vectorial V con producto interior, f ∈ V y S = h{f1 , f2 , . . . , fN }i se tiene d (f, S) =

m´ın

(λ1 ,...,λN )∈Rn

d f,

N X

! λk f k

k=1

v u N u X 2 t = kf k − hf, fk i2 k=1

y esta distancia m´ınima se logra para λi = hf, fi i con i = 1, 2, . . . , N . Es decir, el elemento de S a menor distancia de f es N X

hf, fk i fk

k=1

de este resultado obtenemos:

Teorema 5.1.4 (Desigualdad de Bessel). Si {fk }N k=1 es un conjunto ortonormal en V entonces ∀f ∈ V N X

hf, fk i2 ≤ kf k2

k=1

Demostraci´on. De los c´alculos anteriores !2 N N N X X X 0 ≤ d f, λk fk = kf k2 − hf, fk i2 + (hf, fk i − λk )2 k=1

k=1

271

k=1

Apuntes Mat023 (versi´on preliminar actualizada 23-05-2014)

si tomamos hf, fk i − λk = 0 entonces 2

0 ≤ kf k −

N X

hf, fk i2

k=1

que es la desigualdad deseada. Teorema 5.1.5. Si {fk }∞ k=1 es un conjunto ortonormal en V entonces para cada f en V se cumple l´ım hf, fk i = 0

k→∞

Demostraci´on. Por la desigualdad de Bessel se tiene ∀N ∈ N N X

hf, fk i2 ≤ kf k2

k=1

luego la serie

P+∞

k=1

hf, fk i2 converge (sucesi´on de sumas parciales creciente y acotada

superiormente) luego l´ım hf, fk i = 0

k→∞

Ejemplo 5.1.9. Muestre que el conjunto

n

√1 , sin √x π 2π

o

es ortonormal en C [−π, π] y deter-

minar el elemento de la forma sin x α √ +β √ π 2π (con α, β ∈ R) mas cercano a f (x) = x. ¿Cu´al es la m´ınima distancia?. Soluci´ on. Primero calculamos las normas

1/2 Z π

1 1

√ = dx

2π −π 2π  Z π 1/2 1 = dx 2π −π  Z π 1/2 1 = dx 2π −π = (1)1/2 = 1 272

Apuntes Mat023 (versi´on preliminar actualizada 23-05-2014)

y

Z π 1/2

sin x sin2 x

√ = dx

π π −π  Z π 1/2 1 1 − cos 2x = dx π −π 2  1/2 1 π = π = 1 que son ortogonales es f´acil 

1 sin x √ , √ π 2π



1 = √ 2π = 0

Z

π

sin xdx −π

(por imparidad). El elemento mas cercano es     1 sin x sin x 1 g (x) = √ , x √ + √ , x √ π π 2π 2π donde  Z π 1 x √ ,x √ dx = 0 = 2π 2π   Z−π π √ x sin x sin x √ ,x √ dx = 2 π = π π −π 

as´ı g (x) = 2 sin x y la distancia m´ınima es

q √ 2 d = kxk2 − 02 − 2 π

donde 2

kxk = entonces

Z

2 x2 dx = π 3 3 −π

r d=

Observaci´ on 5.1.4. El conjunto ( cos 1 B= , k1k cos

π

2 3 π − 4π 3

2kπx b−a  , 2kπx b−a



sin

sin

273

2kπx b−a  2kπx b−a

)



con k ∈ N

Apuntes Mat023 (versi´on preliminar actualizada 23-05-2014)

es un conjunto ortonormal en SC[a, b], notemos que Z

1/2

b

dx

=

p (b − a)

a

Z

b

cos2

a



2kπx b−a



! 1 + cos 4kπx b−a dx 2 a     Z b−a 1 b 4kπx + cos dx 2 2 a b−a      b b−a 1 4kπx b − a + sin 2 2 b−a 4kπ a          b−a 1 4kπb 4kπa b−a + sin − sin 2 2 b−a b−a 4kπ   b−a 2 Z

dx = = = = =

b

donde hemos usado la f´ormula de prostaf´eresis     x−y x+y sin (x) − sin (y) = −2 sin cos 2 2 para ver  sin

4kπb b−a



 − sin

4kπa b−a

 =0

ahora bien Z

b 2

sin a



2kπx b−a



Z b



2kπx dx = 1 − cos b−a a   b−a = (b − a) − 2   b−a = 2 2

as´ı √ k1k = b−a r

 

b−a

cos 2kπx =

b−a 2 r

 

b−a

sin 2kπx =

b−a 2

274

 dx

Apuntes Mat023 (versi´on preliminar actualizada 23-05-2014)

Definici´ on 5.1.4. Sea f ∈ SC [a, b]. Llamaremos serie de Fourier de f a la serie de la forma



a0 X + ak cos 2 k=1



 X   ∞ 2kπ 2kπ x + x bk sin b−a b − a k=1

donde los coeficientes son dados por la mejor aproximaci´on.

Calculemos los coeficientes en forma expl´ıcita:

= = = =

hf (x) , 1i *   X   + ∞ ∞ a0 X 2kπ 2kπ + ak cos x + bk sin x ,1 2 b−a b−a k=1 k=1 *∞   + *X   + ∞ Da E X 2kπ 2kπ 0 ,1 + ak cos x ,1 + bk sin x ,1 2 b−a b−a k=1 k=1       X   ∞ ∞ X 2kπ 2kπ a0 ak cos bk sin h1, 1i + x ,1 + x ,1 2 b − a b − a k=1 k=1 a0 (b − a) 2

esto es 2 a0 = b−a

Z

b

f (x) dx a

de manera similar         2nπ 2nπ 2nπ x = an cos x , cos x f, cos b−a b−a b−a as´ı an

 2nπ f, cos b−a x   =

2nπ 2nπ cos b−a x , cos b−a x   Z b 2 2nπ = f (x) cos x dx para n ∈ N b−a a b−a

y  2nπ f, sin b−a x   =

2nπ 2nπ sin b−a x , sin b−a x   Z b 2 2nπ = f (x) sin x dx para n ∈ N b−a a b−a

bn

275

Apuntes Mat023 (versi´on preliminar actualizada 23-05-2014)

Teorema 5.1.6. Si f ∈ SC [a, b] entonces

 X !   N N

a0 X 2kπ 2kπ

+ x + x l´ım f (x) − ak cos bk sin

=0 N →+∞

2 b−a b−a k=1

k=1

si a0 , ak y bk corresponden a los coeficientes de Fourier de f .

Podemos entonces escribir ∞

a0 X + ak cos f (x) = 2 k=1



 X   ∞ 2kπ 2kπ x + x bk sin b−a b − a k=1

pero la convergencia de la serie no es necesariamente al evaluar en los puntos, sino en el sentido de la distancia definida.

Ejemplo 5.1.10. Determinar la serie de Fourier de f (x) = x en C [−π, π] Soluci´ on. En este intervalo la serie tiene la forma ∞ ∞ X a0 X + ak cos (kx) + bk sin (kx) 2 k=1 k=1

note que al ser una funci´on impar a0 = ak = 0 y Z 1 π bk = x sin (kx) dx π −π Z 2 π = x sin (kx) dx π 0 2 = ((sin kx − kx cos kx)|π0 ) 2 πk  2  k = −k (−1) πk 2 2 (−1)k+1 = πk as´ı x=

∞ X k=1

2 (−1)k+1 πk

276

! sin (kx)

Apuntes Mat023 (versi´on preliminar actualizada 23-05-2014)

en media. Veamos algunos gr´aficos de la aproximaci´on

N X

2 (−1)k+1 πk

k=1

P3

k=1



2(−1)k+1 πk



P15  2(−1)k+1  k=1

πk



! sin (kx)

sin (kx)

P6



sin (kx)

sin (kx)

P60  2(−1)k+1 

sin (kx)

k=1

2(−1)k+1 πk

k=1

πk

Ejemplo 5.1.11. Determinar la serie de Fourier de f (x) = ex para x ∈ [−1, 1] . Soluci´ on. La serie de Fourier de f en este intervalo es de la forma   X   ∞ ∞ a0 X 2nπx 2nπx + an cos + an sin 2 2 2 n=1 n=1 ∞ ∞ X a0 X = + an cos (nπx) + an sin (nπx) 2 n=1 n=1

277

Apuntes Mat023 (versi´on preliminar actualizada 23-05-2014)

donde 2 = 2 Z =

a0 an

1

Z

ex dx = 2 sinh 1

−1 1

ex cos (nπx) dx =

−1

(2 sinh 1) (−1)n para n ∈ N π 2 n2 + 1

y Z

1

bn =

ex sin (nπx) dx =

−1

(2nπ sinh 1) (−1)n+1 para n ∈ N π 2 n2 + 1

se sigue x

e

= (sinh 1) +

∞ X (2 sinh 1) (−1)n n=1

π 2 n2 + 1

cos (nπx) +

∞ X (2nπ sinh 1) (−1)n+1 n=1

π 2 n2 + 1

∞ ∞ X X 2 (−1)n 2nπ (−1)n+1 = (sinh 1) 1 + cos (nπx) + sin (nπx) π 2 n2 + 1 π 2 n2 + 1 n=1 n=1

sin (nπx)

!

as´ı por ejemplo, la gr´afica de la aproximaci´on ! 10 10 X X 2nπ (−1)n+1 2 (−1)n cos (nπx) + sin (nπx) (sinh 1) 1 + 2 n2 + 1 2 n2 + 1 π π n=1 n=1 es:

en rojo esta la gr´afica de la exponencial. Teorema 5.1.7 (Identidad de Parseval). Si f ∈ SC [a, b] entonces Z b ∞ ∞ 2 a20 X 2 X 2 2 f (x) dx = + an + bn b−a a 2 n=1 n=1 donde 2 an = b−a

Z

b

 f (x) cos

a

 2nπ x dx para n ∈ N∪ {0} b−a

278

Apuntes Mat023 (versi´on preliminar actualizada 23-05-2014)

y 2 bn = b−a

Z

b

 f (x) sin

a

 2nπ x dx para n ∈ N b−a

Ejemplo 5.1.1. Calcular la serie de Fourier de f (x) = |x| para x ∈ [−1, 1] y usando Parseval calcular

∞ X n=1

1 (2n + 1)4

Convergencia Puntual de series de Fourier Si bien hemos enunciado que la serie de Fourier converge en norma la funci´on dada, se presenta la siguiente situaci´on: Determinemos la serie de Fourier de   1 si x ∈ [0, π] f (x) =  −1 si x ∈ [−π, 0[ se tiene: a0 an bn

Z π 1 = √ f (x) dx = 0 (la funci´on es impar) 2π −π Z π 1 f (x) cos (nx) dx = 0 (la funci´on f (x) cos (nx) es impar) para cada n ∈ N = √ π −π Z π 1 1 = √ f (x) sin (nx) dx = − √ (2 (−1)n − 2) para n ∈ N π −π πn

se sigue que ∞  X

 1 sin (nx) n √ f (x) = − √ (2 (−1) − 2) (en media) πn π n=1  ∞  1 2X n = − ((−1) − 1) sin (nx) (en media) π n=1 n pero f (0) = 1 pero si evaluamos

 ∞  2X 1 n − ((−1) − 1) sin (nx) π n=1 n

279

Apuntes Mat023 (versi´on preliminar actualizada 23-05-2014)

en x = 0 nos queda

 ∞  1 2X n − ((−1) − 1) sin (n0) = 0 π n=1 n

luego el valor al cual converge la serie no es el valor de la funci´on en el punto. Como se ve en las siguiente gr´aficas, los combinaciones trigonom´etricas  N  2X 1 n − ((−1) − 1) sin (nx) π n=1 n cuando N crece se va pareciendo m´as a la gr´afica pero todos estas funciones pasan por el 0. El siguiente teorema nos dice a que valor converge la serie de Fourier ∞

a0 X + ak cos 2 k=1



 X   ∞ 2kπ 2kπ x + bk sin x b−a b−a k=1

de f ∈ SC [a, b] cuando evaluamos en un x0 ∈ [a, b]. Teorema 5.2.1 (Convergencia puntual). Sea f, f 0 ∈ SC [a, b] entonces la serie de Fourier converge en todos los puntos del intervalo [a, b], adem´as si x0 ∈ [a, b] la serie de Fourier converge a   − f x+ 0 + f x0 2 cuando x0 ∈ ]a, b[ y a

f (b− ) + f (a+ ) 2

si x0 = a o x0 = b.

Por convergencia puntual se entiende l´ım

N →∞

   ! N  a0 X 2kπ 2kπ = + ak cos x0 + bk sin x0 2 b−a b−a k=1

l´ım SN (x0 )

N →∞

  − f x+ 0 + f x0 = 2 (no es la convergencia en norma, la cual esta garantizada sin mirar la derivada).

280

Apuntes Mat023 (versi´on preliminar actualizada 23-05-2014)

Ejemplo 5.2.1. Sabemos que ! ∞ ∞ X X 2nπ (−1)n+1 2 (−1)n cos (nπx) + sin (nπx) (en media) e = (sinh 1) 1 + π 2 n2 + 1 π 2 n2 + 1 n=1 n=1 x

Usando esa serie, determinar el valor de ∞ X (−1)n π 2 n2 + 1 n=1

Soluci´ on. Como ! ∞ ∞ n n+1 X X 2 (−1) 2nπ (−1) ex = (sinh 1) 1 + cos (nπx) + sin (nπx) 2 n2 + 1 2 n2 + 1 π π n=1 n=1 evaluando en x = 0 que es un punto de continuidad se obtiene ! +∞ X 2 (−1)n 1 = (sinh 1) 1 + π 2 n2 + 1 n=1 as´ı 1 2



 X +∞ (−1)n 1 −1 = sinh 1 π 2 n2 + 1 n=1

Note que si f, f 0 ∈ SC [a, b] la serie de Fourier    X  ∞ ∞ 2kπ 2kπ a0 X ak cos bk sin + x + x 2 b − a b − a k=1 k=1 converge en cada punto del intervalo, si denotamos por   X   ∞ ∞ a0 X 2kπ 2kπ g (x) = + ak cos x + bk sin x 2 b−a b−a k=1 k=1 del resultado anterior se tiene que g (x0 ) = f (x0 ) en los puntos en los cuales f es continua,   2kπ 2kπ pero se obtiene una propiedad adicional, dada que las funciones 1, cos b−a x , sin b−a x son peri´odicas de periodo (b − a) se sigue que   X   ∞ ∞ a0 X 2kπ 2kπ g (x) = + x + x ak cos bk sin 2 b − a b − a k=1 k=1 es una funci´on peri´odica de periodo (b − a) y as´ı g esta bien definida en todo R cumpliendo g

:

R→R

x → g (x) = fe(x) 281

Apuntes Mat023 (versi´on preliminar actualizada 23-05-2014)

donde fe : R → R es una funci´on peri´odica de periodo (b − a) tal que   + − f x + f x 0 0 para x0 ∈ ]a, b[ fe(x0 ) = 2 f (b− ) + f (a+ ) = para x0 = a, b 2 Ejemplo 5.2.2. La serie de Fourier de   1 si x ∈ [0, π] f (x) =  −1 si x ∈ [−π, 0[ es

 ∞  2X 1 n − ((−1) − 1) sin (nx) π n=1 n

entonces  ∞  2X 1 n 1 = − ((−1) − 1) sin (nx) para x ∈ ]0, π[ π n=1 n   ∞ 2X 1 n −1 = − ((−1) − 1) sin (nx) para x ∈ ]−π, 0[ π n=1 n 1 + (−1) = 0 para x = 0, −π, π 2 y por periodicidad podemos decir por ejemplo que  ∞    2X 1 π  n − ((−1) − 1) sin n 2π + π n=1 n 4   ∞  πn  1 2X = − ((−1)n − 1) sin π n=1 n 4 = 1 Ejemplo 5.2.3. Determinar la serie de Fourier de f ∈ C [−2, 2] definida por   1 si 0 ≤ x ≤ 2 f (x) =  −1 si −2 ≤ x < 0 Determine el valor al cual converge la serie en x = 6 y bosquejar un gr´afico de la serie poniendo especial cuidado en los puntos de discontinuidad.

282

Apuntes Mat023 (versi´on preliminar actualizada 23-05-2014)

Soluci´ on. La funci´on es impar, esto permitir´a simplifica los c´alculos, la serie de Fourier tiene la forma ∞

 X   ∞ 2nπx 2nπx + bn sin 4 4 n=1 ∞ ∞  nπx  X  nπx  a0 X = + an cos + bn sin 2 2 2 n=1 n=1

a0 X F (x) = + an cos 2 n=1



donde a0 an

Z 2 2 = f (x) dx = 0 4 −2 Z  nπx  1 2 f (x) cos dx = 0 = 2 −2 2

y bn

Z  nπx  1 2 = f (x) sin dx 2 −2 2 Z 2  nπx  sin = dx 2 0  nπx  2 2 cos = − nπ 2 0 2 = − (cos (nπ) − 1) nπ (−1)n − 1 = −2 nπ n+1 (−1) +1 = 2 nπ

as´ı F (x) = 2

∞ X n=1

(−1)n+1 + 1 nπ

! sin

 nπx  2

por los teoremas vistos en clases, la serie es peri´odica de periodo 4 se sigue que F (6) = F (2) = la gr´afica de la serie es:

283

1 + (−1) =0 2

Apuntes Mat023 (versi´on preliminar actualizada 23-05-2014)

1.

Series de Fourier de senos y cosenos

Definici´ on 5.3.1. Si f : [0, L] → R es una funci´on en SC [0, L], llamaremos: 1. Extensi´on par de f a la funci´on   f (x) si x ∈ [0, L] fp (x) =  f (−x) si x ∈ [−L, 0] 2. Extensi´on impar de f a la funci´on   f (x) si x ∈ [0, L] fI (x) =  −f (−x) si x ∈ [−L, 0[

Observaci´ on 5.3.1. fp ∈ SC [−L, L] es una funci´on par tal que fp (x) = f (x) para x ∈ [0, L] y fI (x) ∈ SC [−L, L] es una funci´on impar tal que fI (x) = f (x) para x ∈ [0, L] Al desarrollar en serie de Fourier la funci´on fp en [−L, L] se tiene   X   ∞ ∞ a0 X kπx kπx fp (x) = + ak cos + bk sin 2 L L k=1 k=1 284

Apuntes Mat023 (versi´on preliminar actualizada 23-05-2014)

donde Z 1 L a0 = fp (x) dx L −L   Z kπx 1 L fp (x) cos dx para k ∈ N ak = L −L L   Z 1 L kπx bk = fp (x) sin dx para k ∈ N L −L L  notemos que al ser fp una funci´on par fp (x) cos kπx es par y fp (x) sin L luego a0

Z Z 1 L 2 L = fp (x) dx = fp (x) dx L −L L 0 Z 2 L f (x) dx = L 0

y ak

  Z 1 L kπx = fp (x) cos dx para k ∈ N L −L L   Z kπx 2 L fp (x) cos dx para k ∈ N = L 0 L   Z 2 L kπx = f (x) cos dx para k ∈ N L 0 L

y bk

1 = L = 0

Z

L

 fp (x) sin

−L

kπx L

 dx para k ∈ N

as´ı la serie es de la forma ∞

a0 X fp (x) = + ak cos 2 k=1



kπx L



con a0 ak

Z 2 L = f (x) dx L 0   Z 2 L kπx = f (x) cos dx para k ∈ N L 0 L 285

kπx L



es impar,

Apuntes Mat023 (versi´on preliminar actualizada 23-05-2014)

de manera similar, si al desarrollar en serie de Fourier la funci´on fI en [−L, L] se tiene  X    ∞ ∞ a0 X kπx kπx + + fI (x) = ak cos bk sin 2 L L k=1 k=1 donde Z 1 L a0 = fI (x) dx L −L   Z 1 L kπx ak = fI (x) cos dx para k ∈ N L −L L   Z 1 L kπx bk = fI (x) sin dx para k ∈ N L −L L  notemos que al ser fI una funci´on impar fI (x) cos kπx es impar y fI (x) sin L

kπx L



es par,

luego 1 a0 = L

Z

L

fI (x) dx = 0 −L

y ak

  Z 1 L kπx = fI (x) cos dx para k ∈ N L −L L = 0 para k ∈ N

y bk

  Z kπx 1 L fI (x) sin = dx para k ∈ N L −L L   Z kπx 2 L fI (x) sin dx para k ∈ N = L 0 L   Z kπx 2 L f (x) sin dx para k ∈ N = L 0 L

as´ı la serie es de la forma fI (x) =

∞ X

 bk sin

k=1

con 2 bk = L

Z

L

 f (x) sin

0

kπx L

kπx L



 dx para k ∈ N

como fp (x) = fI (x) = f (x) para x ∈ [0, L] se obtiene que es posible desarrollar f ∈ SC [0, L] en series de la forma ∞

a0 X f (x) = + ak cos 2 k=1 286



kπx L



Apuntes Mat023 (versi´on preliminar actualizada 23-05-2014)

donde a0 ak

Z 2 L f (x) dx = L 0   Z 2 L kπx = f (x) cos dx para k ∈ N L 0 L

llamada serie cosenoidal de f o serie de cosenos de f y   ∞ X kπx f (x) = bk sin L k=1 donde

  Z 2 L kπx bk = f (x) sin dx para k ∈ N L 0 L llamada serie senoidal de f o serie de senos de f . Ejemplo 5.3.1. Obtener la serie senoidal de f (x) = cos x en [0, π]. Soluci´ on. Hacemos uso de la extensi´on impar de f al intervalo [−π, π] entonces   cos x si x ∈ [0, π]   fI (x) =    − cos (−x) si x ∈ [−π, 0[ entonces

∞ ∞ X a0 X + an cos (nx) + bn sin (nx) fI (x) = 2 n=1 n=1

donde a0 an bn

Z 1 π = fI (x) = 0 π −π Z π 1 = fI (x) cos (nx) dx = 0 π −π Z 2 π = cos x sin (nx) dx π 0

si n = 1 entonces 2 b1 = π

Z

π

cos x sin xdx = 0 0

287

Apuntes Mat023 (versi´on preliminar actualizada 23-05-2014)

pero sin (x + nx) = sin x cos nx + sin nx cos x sin (x − nx) = sin x cos nx − sin nx cos x as´ı sin (x + nx) − sin (x − nx) = 2 cos x sin nx luego cos ((n + 1) x) cos ((1 − n) x) + =2 − n+1 1−n

Z cos x sin nxdx

luego   π cos ((n + 1) x) cos ((1 − n) x) + − n+1 1−n   0  1 1 cos ((n + 1) π) cos ((1 − n) π) + + − − − n+1 1−n n+1 1−n (−1)n (−1)n 2n + + 2 n+1 n−1 n −1 n ((−1) + 1) 2n n2 − 1

Z = 2 Z

cos x sin nxdx 0

Z

π

= 2

cos x sin nxdx 0

Z = 2

as´ı en [0, π] obtenemos la representaci´on  X  +∞  2 ((−1)n + 1) n cos x = sin (nx) π n=2 n2 − 1

288

π

= 2

 X  +∞  2 ((−1)n + 1) n fI (x) = sin (nx) π n=2 n2 − 1

 X  100  2 ((−1)n + 1) n sin (nx) π n=2 n2 − 1

cos x sin nxdx 0

se sigue

la gr´afica de la aproximaci´on (en verde)

π

π

cos x sin nxdx 0

Apuntes Mat023 (versi´on preliminar actualizada 23-05-2014)

es

en rojo esta la gr´afica de coseno en el intervalo deseado.

Ejemplo 5.3.2. Obtener la serie cosenoidal de la funci´on f (x) = x para x ∈ [0, 4] . Soluci´ on. En este caso hacemos uso de la extensi´on par de f al intervalo [−4, 4] se tiene   si x ∈ [0, 4]   x fp (x) =    (−x) si x ∈ [−4, 0[ entonces



a0 X fp (x) = + an cos 2 n=1



2nπx 8

 +

∞ X n=1

donde a0

Z 2 4 = fp (x) dx 8 −4 Z 1 4 = xdx = 4 2 0 289

 an sin

2nπx 8



Apuntes Mat023 (versi´on preliminar actualizada 23-05-2014)

an

Z  nπx  1 4 = fp (x) cos dx 4 −4 4 Z  nπx  1 4 = dx x cos 2 0 4 8 ((−1)n − 1) = π 2 n2

y Z

1 bn = 4

4

fp (x) sin

 nπx  4

−4

se sigue fp (x) = 2 +

dx = 0

 ∞  X 8 ((−1)n − 1) π 2 n2

n=1

cos

 nπx  4

en particular en [0, 4] se tiene x=2+8

 ∞  X ((−1)n − 1) π 2 n2

n=1

cos

 nπx  4

Ejemplo 5.3.3. Sea f : [0, π] → R definida por f (x) = x (π − x) 1. Encontrar una funci´on F peri´odica y par tal que F (x) = f (x) para x ∈ [0, π] y desarrollar en serie de Fourier F . 2. Encontrar una funci´on G peri´odica e impar tal que G (x) = f (x) para x ∈ [0, π] y desarrollar en serie de Fourier G. 3. Use los puntos anteriores para probar ∞ X (−1)n+1 n=1

n2

∞ π 2 X (−1)n+1 π3 = y = 12 n=1 (2n − 1)3 32

Soluci´ on. Por punto pero calculando las sumas pedidas. 1. F es la serie de cosenos ∞

a0 X + an cos (nx) 2 n=1

F (x) = donde 2 a0 = π

Z

π

x (π − x) dx = 0

290

π2 3

Apuntes Mat023 (versi´on preliminar actualizada 23-05-2014)

2 an = π

Z

π

0

2 ((−1)n + 1) x (π − x) cos (nx) dx = − n2

as´ı F (x) =

∞ X π2 ((−1)n + 1) −2 cos (nx) 2 6 n n=1

si consideramos la suma sobre los pares (en los impares los coeficientes son nulos) ∞ X 2 π2 −2 F (x) = 2 cos (2nx) 6 (2n) n=1 ∞

π 2 X cos (2nx) = − 6 n2 n=1 evaluando en

π 2

obtenemos ∞ π   π π 2 X cos 2n π− = − 2 2 6 n2 n=1

π 2





π 2 X (−1)n = − 6 n2 n=1 esto es ∞ X (−1)n+1 π2 π2 − = 4 6 n2 n=1 ∞ X π2 (−1)n+1 = 12 n2 n=1

2. G es la serie senoidal G (x) =

∞ X

bn sin (nx)

n=1

donde bn

2 = π

Z

π

x (π − x) sin (nx) dx 0

2 (2 (−1)n − 2) 3 πn −4 ((−1)n − 1) = πn3 = −

as´ı G (x) =

 ∞  X −4 ((−1)n − 1) πn3

n=1

291

sin (nx)

Apuntes Mat023 (versi´on preliminar actualizada 23-05-2014)

note que ahora en los pares los coeficientes son nulos, al considerar en los impares tenemos G (x) = evaluando en

π 2

∞ 8 X sin ((2n − 1) x) π n=1 (2n − 1)3

tenemos  ∞ π   π 8 X sin (2n − 1) π2 π− = 2 2 π n=1 (2n − 1)3 ∞ 8 X (−1)n+1 = π n=1 (2n − 1)3

pues  π sin (2n − 1) = (−1)n+1 2 as´ı

∞ π2 8 X (−1)n+1 = 4 π n=1 (2n − 1)3

de donde obtenemos



π 3 X (−1)n+1 = 32 n=1 (2n − 1)3

Ejemplo 5.3.4. Sea f (x) = exp (− [x]) definida en [0, 2]. Obtenga su serie de Fourier de cosenos y use la serie para calcular el valor de ∞ X (−1)n−1 n=1

2n − 1

Obs.: [x] es la parte entera de x. Soluci´ on. Tenemos que encontrar la serie de cosenos, usamos la extensi´on par ∞

a0 X fp (x) = + an cos 2 n=1

292



2πnx 4



Apuntes Mat023 (versi´on preliminar actualizada 23-05-2014)

donde a0

Z 2 2 fp (x) dx = 4 −2 Z 2 f (x) dx = 0 Z 2 exp (− [x]) dx = 0 Z 2 Z 1 1dx + edx = = e

0 −1

1

+1

y an

  Z 2 2 2πnx = fp (x) cos dx 4 −2 4 Z 2  πnx  exp (− [x]) cos dx = 2 0 Z 1 Z 2  πnx   πnx  −1 cos = e cos dx + dx 2 2 0 1   1 2 1 1 −1 −1 2e sin πn − 2e sin πn + sin πn = πn 2 πn 2  2  πn  −1 = − sin e −1 πn 2

luego



 πnx   e−1 + 1 X 2  πn  −1 fp (x) = + sin 1−e cos 2 πn 2 2 n=1 evaluando en x = 0 se tiene ∞

1= de donde obtenemos

luego

luego

 e−1 + 1 X 2  πn  + sin 1 − e−1 2 πn 2 n=1 ∞

e−1 + 1 2 (1 − e−1 ) X sin πn 2 1− = 2 π n n=1  ∞ X π 12 − 12 e−1 sin πn 2 = 2 (1 − e−1 ) n n=1 ∞

π X sin πn 2 = 4 n n=1 293

Apuntes Mat023 (versi´on preliminar actualizada 23-05-2014)

es cero, se sigue si n es par sin πn 2 ∞

π(2n−1)

π X sin 2 = 4 2n − 1 n=1 pero sin es decir

π (2n − 1) = (−1)n+1 2 ∞

π X (−1)n+1 = 4 2n − 1 n=1

Derivaci´ on e integraci´ on de Series de Fourier Si para x ∈ ]−π, π[ x=

∞ X 2 (−1)k+1

k

k=1

es cierto que 1=

∞ X

sin (kx)

2 (−1)k+1 cos (kx)

k=1

para x ∈ ]−π, π[? En otras palabras ¿es posible derivar t´ermino a t´ermino una serie de Fourier y asegurar que converge a la derivada de la funci´on?, la respuesta es no, ya que 2 (−1)k+1 no corresponde al coeficiente de Fourier de la funci´on, el cual tiene que cumplir l´ım Ck = 0

k→∞

Para que sea v´alida la derivaci´on t´ermino a t´ermino tenemos el siguiente resultado: Teorema 5.4.1. Si f ∈ C [a, b] , f (a) = f (b) y f 0 ∈ SC [a, b] entonces la serie de Fourier para f 0 puede ser obtenida derivando t´ermino a t´ermino la serie de f . La serie obtenida converge puntualmente a f 0 (x) en los puntos en los cuales f 00 existe.

para la integraci´on de las series tenemos el siguiente resultado: Teorema 5.4.2. Si f ∈ SC [a, b] y     ∞  a0 X 2nπx 2nπx f (x) = + ak cos + bk sin (media) 2 b − a b − a n=1 294

Apuntes Mat023 (versi´on preliminar actualizada 23-05-2014)

entonces Z

x

(x − a) 2 ∞ X Z + ak

f (t) dt = a0 a

x



 2nπt cos dt b−a a n=1   Z x ∞ X 2nπt sin + bk dt (media) b − a a n=1 (Esto es, es posible integrar la serie t´ermino a t´ermino)

Ejercicios del cap´ıtulo 1. Suponga que x (L − x) =

∞ X k=1

 ak sin

kπx L



para x ∈ [0, L]. Haciendo uso de la propiedad de ortogonalidad de la familia   sin kπx en C[0, L] encontrar los coeficientes de la serie. L k∈N 2. Muestre que en C[a, b] = V se cumple a) ∀f, g ∈ V kf − gk2 + kf + gk2 = 2 kf k2 + 2 kgk2 b) Si hf, gi = hf, hi para todo f ∈ V entonces g = h. 3. Muestre que para todo f ∈ C [−π, π] Z π l´ım sin (nx) f (x) dx = 0 n→∞

−π

Ind.: Identidad de Parseval. 4. (Calculando Productos) Calcular para n, m ∈ N Z π Z π sin (mx) sin (nx) dx, sin (nx) dx −π −π Z π Z π cos (nx) dx, cos (nx) cos (mx) dx −π −π Z π cos (nx) sin (mx) dx −π

295

Apuntes Mat023 (versi´on preliminar actualizada 23-05-2014)

¿Qu´e dicen estos c´alculos del conjunto B = {1, cos (nx) , sin (nx) con n ∈ N} en SC[−π, π]? 5. (De lo general a lo particular) En SC [a, b] hemos definido el producto interior Z

b

hf, gi =

f (x) g (x) dx a

y enunciado que el conjunto       2nπx 2nπx , sin con n ∈ N C = 1, cos b−a b−a es una conjunto ortogonal. Escribir en los casos especiales SC[−π, π], SC[−L, L] y SC[0, L] 6. (Calculando series de Fourier) Encontrar las series de Fourier para las funciones siguientes en SC[−π, π] a) x + sin x b) ex   c)    d) 

1

si x ≤ 0

x2 si x > 0 x

si x ≤ 0

x3 si x > 0

e) sin3 x 7. (La identidad de Parseval) Utilizar la serie de Fourier de f (x) = x en [−π, π] para calcular el valor de

∞ X 1 k2 k=1

8. (Funciones pares e impares) Clasificar las siguientes funciones en par, impar o ninguna de las anteriores A) tan x B) xex

2

296

Apuntes Mat023 (versi´on preliminar actualizada 23-05-2014) x+1 x−1 D) ln |x| C)

E) arcsin x F) f (|x|) definida en [−1, 1] donde f : R → R es una funci´on cualquiera. G) x cos x − cos 2x 9. (Parte par e impar de una serie) En Mat021 se mostr´o que toda funci´on f : [−a, a] → R (donde a ∈ R+ ) se puede descomponer en su parte par mas su parte impar, esto es f (x) = fp (x) + fi (x) donde f (x) + f (−x) 2 f (x) − f (−x) fi (x) = 2

fp (x) =

Suponiendo que no hay problemas de convergencia, encontrar las partes par e impar de



a0 X (ak cos (kx) + bk sin (kx)) + 2 k=1 donde ak , bk son constantes. 10. (Convergencia puntual) Encontrar el desarrollo en serie de Fourier de:   −1 −π < x < 0 f (x) =  1 0 0 Sea, ahora, y ∈ B (x, δ). Luego, ky − xk < δ y adem´as: ky − ak = k(y − x) + (x − a)k ≤ ky − xk + kx − ak < δ + kx − ak = ε − kx − ak + kx − ak = ε As´ı, ky − ak < ε, y por tanto, y ∈ B (a, ε). Esto implica que, B (x, δ) ⊆ B (a, ε). Es decir, que todo punto x ∈ B (a, ε) es interior. Ejemplo 6.3.4. Todo intervalo abierto en R es un conjunto abierto. Ejemplo 6.3.5. Sea:  U = (x, y) ∈ R2 : x > 2 Entonces, U es abierto. Proposici´ on 6.3.1. Los conjuntos abiertos de Rn poseen las siguientes propiedades: 1. ∅ y Rn son abiertos. 2. La uni´on arbitraria de conjuntos abiertos en Rn es un conjunto abierto en Rn . 3. La intersecci´on de un n´ umero finito de conjuntos abiertos en Rn es un conjunto abierto en Rn . Ejemplo 6.3.6. Sea Hn = a − n1 , b +

1 n



⊆ R, para cada n ∈ N. Notamos que cada Hn

es un conjunto abierto en R, pero: H=

∞ \

Hn = [a, b]

n=1

el cual no es abierto.

307

Apuntes Mat023 (versi´on preliminar actualizada 23-05-2014)

Definici´ on 6.3.4. Sean U ⊆ Rn y a ∈ Rn . Diremos que a es un punto de adherencia a U si: ∀ε > 0,

B (a, ε) ∩ U 6= ∅

El conjunto de todos los puntos de adherencia de U se denomina la clausura de U y se anota U . ˚ ⊆ U ⊆ U. Observaci´ on 6.3.2. Para todo U ⊆ Rn , U Ejemplo 6.3.7. La bola cerrada con centro en a y radio ε est´a dada por: B (a, ε) = {x ∈ Rn : kx − ak ≤ ε} Ejemplo 6.3.8. N = Z = ∅ = ∅. Ejemplo 6.3.9. Q = R. Definici´ on 6.3.5. Sea U ⊆ Rn . Diremos que U es un conjunto cerrado si U = U . Ejemplo 6.3.10. Los conjuntos ∅ y Rn son cerrados. Teorema 6.3.2. Sea U ⊆ Rn . Entonces, U es cerrado en Rn , si y solo si, U C es un conjunto abierto. Definici´ on 6.3.6. Sean U ⊆ Rn y a ∈ Rn . Diremos que a es un punto de acumulaci´ on de U si: ∀ε > 0,

(B (a, ε) r {a}) ∩ U 6= ∅

El conjunto de todos los puntos de acumulaci´on de U se denomina el conjunto derivado de U y se anota como U 0 . Ejemplo 6.3.11. Sea U = (0, 1] ∪ {2, 3}. Luego, U 0 = [0, 1]. Ejemplo 6.3.12. Sea X =

1 n

: n ∈ N . Luego, X 0 = {0}. Note que lo anterior quiere

decir que hay infinitos puntos distintos de X suficientemente cercanos a 0. Ejemplo 6.3.13. Sea A =



m 1 , n n



: m, n ∈ N, n 6= 0 ⊆ R2 . Calcule A0 .

Observaci´ on 6.3.3. Si U ⊆ Rn tal que U 0 6= ∅, entonces U es infinito. As´ı, para todo conjunto finito A ⊆ Rn , se tiene que A0 = ∅. 308

Apuntes Mat023 (versi´on preliminar actualizada 23-05-2014)

Teorema 6.3.3. Sea U ⊆ Rn . Entonces, U es cerrado en Rn , si y solo si, U 0 ⊆ U . Definici´ on 6.3.7. Sean U ⊆ Rn y a ∈ Rn . Diremos que a es un punto frontera de U si: ∀ε > 0,

B (a, ε) ∩ U 6= ∅



B (a, ε) ∩ U C 6= ∅

El conjunto de todos los puntos frontera de U se llama la frontera de U y se anota como ∂U . Observaci´ on 6.3.4. Note que, desde la definici´on, se obtiene que ∂U = U ∩ U C . Ejemplo 6.3.14. La esfera con centro en a y radio ε > 0 se obtiene como: S (a, ε) = ∂B (a, ε) = {x ∈ Rn : kx − ak = ε} As´ı, la esfera unitaria S n−1 en Rn se obtiene como: S n−1 = ∂B (0, 1) = {x ∈ Rn : kxk = 1} umero real Definici´ on 6.3.8. Diremos que un conjunto X ⊆ Rn es acotado si existe un n´ M > 0 tal que: kxk ≤ M para todo x ∈ X.  Observaci´ on 6.3.5. Note que la cota anterior es uniforme, es decir: sup kxk : x ∈ X ≤ M. Definici´ on 6.3.9 (Heine-Borel). Un conjunto K ⊆ Rn se dice compacto si es cerrado y acotado. Ejemplo 6.3.15. La bola cerrada B (a, ε) es un conjunto compacto sobre en Rn . En efecto, como B (a, ε) es cerrado, basta verificar que es acotado. Note que para todo x ∈ B (x, ε), se tiene que: kxk ≤ kx − ak + kak < ε + kak

309

Apuntes Mat023 (versi´on preliminar actualizada 23-05-2014)

Ejemplo 6.3.16. Sean L ⊆ Rn y K ⊆ Rm conjuntos compactos, respectivamente. Entonces, L × K es compacto en Rn+m = Rn × Rm Definici´ on 6.3.10. Un conjunto A ⊆ Rn se llama disconexo si existen dos conjuntos abiertos U y V de Rn tales que: 1. U ∩ A 6= ∅ ∧ V ∩ A 6= ∅. 2. (U ∩ A) ∩ (V ∩ A) = ∅. 3. (U ∩ A) ∪ (V ∩ A) = A Por otro lado, un conjunto A ⊆ Rn se dice conexo si no es disconexo. Ejemplo 6.3.17. N y Q son disconexos. Ejemplo 6.3.18. R es conexo. Ejemplo 6.3.19. Todo intervalo real es conexo. Teorema 6.3.4. Sean A ⊆ Rn un conjunto conexo y C ⊆ Rn tales que A ⊆ C ⊆ A, entonces C es conexo. Teorema 6.3.5. Sea {Aλ }λ∈Λ una familia de conjuntos conexos en Rn con al menos un punto en com´ un, entonces: [



λ∈Λ

es conexo.

Ejercicios del cap´ıtulo 1. Hallar un subespacio vectorial W de R3 tal que:     x = 2t + 1 L: y = −2 + t    z = 3t − 1 se escriba como: L = W + {u0 } con u0 ∈ R3 adecuado. 310

Apuntes Mat023 (versi´on preliminar actualizada 23-05-2014)

2. En los siguientes conjuntos, determine cuales son: Abiertos, Cerrados. Determine en cada caso el interior, la clausura y frontera del conjunto.

c)

{(x, y, z) ∈ R3 / z < x2 + y 2 }

1 1 , y = , n, m ∈ N n m 3 d) {(x, y, z) ∈ R / x = 0}

e)

Una recta en R2 o R3

f)

a) {(x, y) ∈ R2 / y = 2x}

b) {(x, y) ∈ R2 / x =

g) {(x, y, z) ∈ R3 / x = y = z = (−1)n , n ∈ N} i)

{x ∈ Rn : kxk ≤ 1}

Un plano en R3

h) {(x, y, z) ∈ R3 / x < y} j)

{x ∈ Rn : kxk = 1}

3. Demuestre rigurosamente que: a) (0, 1) no es un punto interior de A = {(x, y) ∈ R2 / |x| + |y| < 1}. b) (0, 0) es un punto interior de A. c) (1, 0) es un punto de acumulaci´on de A.   1 1 d) , es un punto de frontera de A. 2 2 4. Probar que B (a; r) = {x ∈ Rn : kx − ak < r} es abierto en Rn . umero racional r ∈ [0, 1], probar que 5. Si A es un conjunto cerrado que contiene cada n´ [0, 1] ⊆ A. 6. Encuentre un conjunto en R2 que no sea Abierto ni Cerrado. 7. Sea R = [−1, 2] × [3, 4[ a) ¿Es Abierto?, ¿Es Cerrado? b) Determine R. 8. Si K = [0, 1] × [0, 1] ∪ {(2, 2)} a) ¿Es Abierto? b) ¿Es cerrado? 9. Sea {Ai : i ∈ I} una colecci´on de subconjuntos abiertos de Rn muestre que: 311

Apuntes Mat023 (versi´on preliminar actualizada 23-05-2014)

a) ∪i∈I Ai es un conjunto abierto (la uni´on arbitraria de abiertos es un abierto) b) ∩i∈I 0 Ai es un conjunto abierto, donde I 0 ⊆ I es un conjunto finito (la intersecci´on finita de abiertos es un abierto) c) Muestre que la intersecci´on arbitraria de abiertos no es necesariamente un  abierto, para ello considere en Rn la colecci´on de bolas abiertas B θ; n1 con n ∈ N. 10. Encontrar A◦ , A, A0 y ∂A si  A = (x, y) ∈ R2 : x > 0, y > 0 ∪ {(−1, −1)} 11. Si T : Rn → Rm es una transformaci´on lineal y P ∈ Rn , r > 0 ¿En que consiste el conjunto T (B (P ; r))?¿Es abierto? 12. Muestre que p ∈ A si y solo si existe una sucesi´on de puntos en A, digamos {an }n∈N tal que an → p (esto es l´ım d (an , p) = 0

n→∞

en el sentido de los limites de sucesiones en R) 13. Considere el subconjunto de R   1 1 para alg´ un n, m ∈ N U = x∈R:x= + n m encontrar U ◦ , U , U 0 y ∂U . 14. Muestre que si U y V son abiertos en R entonces U × V es abierto en R2 . 15. Sean U, V subconjuntos de Rn probar: a) U ⊂ V ⇒ U ⊂ V b) U ∪ V = U ∪ V 16. Si U ⊆ Rn muestre que: a) U ◦ y ∂U son disjuntos y U = U ◦ ∪ ∂U 312

Apuntes Mat023 (versi´on preliminar actualizada 23-05-2014)

b) Muestre que la frontera es vac´ıa si y solo si el conjunto es abierto y cerrado. c) Muestre que U es abierto si y solo si ∂U = U − U ◦ 17. Si U es abierto ¿Es verdad que U = U ? 18. Muestre que todo abierto de R2 es una uni´on numerable de cajas abiertas de la forma ]a, b[ × ]c, d[ y tambi´en es una uni´on numerable de bolas abiertas.

313

Cap´ıtulo 7 : Funciones de varias variables

Definiciones b´ asicas Definici´ on 7.1.1. Consideremos una funci´on f : U ⊆ Rn → Rm . Diremos que: 1. f es una funci´on real de varias variables si n ≥ 2 y m = 1. 2. f es una funci´on vectorial de una variable real si n = 1 y m ≥ 2. 3. f es una funci´on vectorial de varias variables si n ≥ 2 y m ≥ 2. En general, las funciones reales de varias variables se denotan con letras min´ usculas, como por ejemplo: f, g, h, etc. y las funciones vectoriales se anotan con letras may´ usculas, tales como: F, G, H, etc. Ejemplo 7.1.1. Son funciones reales de varias variables las siguientes funciones: 1.

a) f : R2 → R,

(x, y) 7→ f (x, y) = x2 + 2y.

b) g : R3 → R,

(x, y, z) 7→ g (x, y, z) = sin (xyz).

c) h : R3 → R,

(x, y, z) 7→ h (x, y, z) = e− 2 (x

1

2 +y 2 +z 2

).

Ejemplo 7.1.2. Un ejemplo importante de funci´on de varias variables corresponde a la k−´esima proyecci´on sobre R de un vector x = (x1 , x2 , . . . , xn ) ∈ Rn . M´as precisamente, se define la k−´ esima proyecci´ on de x ∈ Rn como la funci´on πk : Rn → R definida por: πk (x1 , x2 , . . . , xn ) = πk Esta funci´on, en particular, nos permitir´a representar funciones tales como: 2x2 + y 3 f (x, y, z) = 2 x + z2 + 1 como ´algebra de proyecciones. Es decir, podemos escribir f (x, y, z) en la forma: f (x, y, z) =

2 πx2 (x, y, z) + πy3 (x, y, z) πx2 (x, y, z) + πz2 (x, y, z) + 1

Ejemplo 7.1.3. Son funciones vectoriales las siguientes funciones: 314

Apuntes Mat023 (versi´on preliminar actualizada 23-05-2014)

1. F : R → R2 ,

t 7→ F (t) = (C1 et , C2 e−t ), con C1 , C2 ∈ R.

2. G : R → R3 , t 7→ G (t) = (α cos t, α sin t, βt), con α, β > 0. 3. H : R2 → R3 ,

(x, y) 7→ H (x, y) = (ln (x2 + y 2 + 1) , cos (x + y) , x + 3y).

Ejemplo 7.1.4. Notamos que, si en el ejemplo (3) anterior definimos: f1 (x, y) = ln x2 + y 2 + 1



f2 (x, y) = cos (x + y) f3 (x, y) = x + 3y entonces: H (x, y) = (f1 (x, y) , f2 (x, y) , f3 (x, y)) Ejemplo 7.1.5. Sean fi : Ui ⊆ Rn → R funciones de varias variables con dominio Ui , con i = 1, 2, . . . , m. Suponga que: U=

m \

Ui

i=1 m

n

Entonces, la transformaci´on F : U ⊆ R → R definida por: F (x) = (f1 (x) , f2 (x) , . . . , fm (x)) ,

x = (x1 , x2 , . . . , xn ) ∈ U

es una funci´on vectorial de varias variables. Las funciones reales de varias variables fi se llaman funciones componentes de F . Definici´ on 7.1.2. Sea f : D ⊆ Rn → R, (x1 , x2 , . . . , xn ) → f (x1 , x2 , . . . , xn ). Llamaremos dominio m´ aximo de f al conjunto: dom ( f ) = {x ∈ Rn : f (x) ∈ R} As´ı mismo, el dominio m´ aximo de F : D ⊆ Rn → R, x → F (x) = (f1 (x) , f2 (x) , . . . , fm (x)) es dado por: dom (F) =

m \

dom ( fi )

i=1

Ejemplo 7.1.6. Hallar el dominio m´aximo de: p x2 + y 2 − 1 f (x, y) = ln (|x| − y) 315

Apuntes Mat023 (versi´on preliminar actualizada 23-05-2014)

Soluci´ on. El dominio m´aximo en R2 para que esta expresi´on represente una funci´on a valores reales en tal conjunto es 

(x, y) ∈ R2 : x2 + y 2 − 1 ≥ 0 ∧ |x| − y > 0 ∧ |x| − y 6= 1

Ejemplo 7.1.7. Si  F (x, y, z) =

sin (xyz) , y cos xyz

  1 x

Determine el dominio m´aximo de F . Soluci´ on. El dominio m´aximo corresponde a 

(x, y, z) ∈ R3 : xyz 6= 0

Definici´ on 7.1.3. El recorrido de f : U ⊆ Rn → Rm se define como: rec ( f ) = {y ∈ Rm : ∃x ∈ U, x = f (x)} Observaci´ on 7.1.1. En general, para funciones de varias variables es dif´ıcil calcular el recorrido. Sin embargo, puede ser u ´ til tomar restricciones. Esto es im´agenes directas de conjuntos adecuados. Ejemplo 7.1.8. El recorrido de f : R2 → R, (x, y) → f (x, y) = xy 2 + yx2 es todo R. En efecto, si T =



(x, y) ∈ R2 : x = y



= {(x, x) : x ∈ R} Entonces, f T = x · x2 + x · x2 = 2x3 , se sigue que f (T ) = R.

Gr´ aficos, conjuntos de nivel y trazas Definici´ on 7.2.1. Sea f : U ⊆ Rn → R una funci´on. Llamaremos gr´ afico de f al conjunto definido por:  Graf (f ) = (x, f (x)) ∈ Rn+1 : x ∈ U Observaci´ on 7.2.1. Evidentemente, esta definici´on posee un sentido geom´etrico en el caso de funciones reales de varias variables s´olo cuando n = 1, 2. 316

Apuntes Mat023 (versi´on preliminar actualizada 23-05-2014)

Observaci´ on 7.2.2. Si f : U ⊆ R2 → R es una funci´on real de dos variables reales, diremos que: z = f (x, y) ,

(x, y) ∈ U

es una superficie en R3 . Observaci´ on 7.2.3. Respecto de lo anterior, hay algunas superficies cl´asicas que es conveniente reconocer. Tales superficies son conocidas como superficies cu´adricas. Definici´ on 7.2.2. Una superficie cu´adrica es una ecuaci´on del tipo: P (x, y, z) = 0 donde P (x, y, z) es un polinomio de segundo grado en tres variables. En particular, se consideran la superficies cu´adricas en su forma normal, es decir, considerando las ecuaciones: Ax2 + By 2 + Cz 2 + D = 0 o bien: Ax2 + By 2 + Cz = 0 Observaci´ on 7.2.4. Las superficies cu´adricas m´as importantes son las siguientes: 1. Esfera: x2 + y 2 + z 2 = r 2

317

Apuntes Mat023 (versi´on preliminar actualizada 23-05-2014)

2. Elipsoide: x2 a2

+

y2 b2

+

z2 c2

= 1,

a, b, c 6= 0

x2 a2

+

y2 b2



z2 c2

= 1,

a, b, c 6= 0

3. Hiperboloide de una hoja:

318

Apuntes Mat023 (versi´on preliminar actualizada 23-05-2014)

4. Hiperboloide de dos hojas: 2

− xa2 +

y2 b2



z2 c2

= 1,

a, b, c 6= 0

5. Paraboloide: x2 a2

+

y2 b2

= cz,

319

a, b 6= 0 ∧ c > 0

Apuntes Mat023 (versi´on preliminar actualizada 23-05-2014)

6. Paraboloide hiperb´olico: x2 a2



y2 b2

= cz,

a, b 6= 0 ∧ c > 0

Definici´ on 7.2.3. Sea f : U ⊆ Rn → R y c ∈ R. Llamaremos conjunto de nivel c de f al conjunto definido por: Lc (f ) = {x ∈ U : f (x) = c} ⊆ Rn En particular, si n = 2 diremos que Lc (f ) es la curva de nivel c de f . Si n = 3, diremos que Lc (f ) es la superficie de nivel c de f. Observaci´ on 7.2.5. Note que, simplemente: Lc (f ) = f −1 ({c}) Observaci´ on 7.2.6. De los conjuntos anteriores, nos interesa el aspecto gr´afico para n = 2 y n = 3. La colecci´on de los gr´aficos superpuestos de un n´ umero adecuado de curvas de nivel o superficies de nivel para una funci´on permite esbozar el gr´afico de la funci´on dada. Ejemplo 7.2.1. Las curvas de nivel c de la funci´on f (x, y) = x2 + y 2 son circunferencias √ de centro en (0, 0) y radio c para c > 0. Para c = 0 es solo un punto y para c < 0 es conjunto vacio. 320

Apuntes Mat023 (versi´on preliminar actualizada 23-05-2014)

Ejemplo 7.2.2. Las curvas de nivel c de la funci´on f (x, y) =

p x2 + y 2 son circunferencias

de centro en (0, 0) y radio c para c > 0. Para c = 0 es solo un punto y para c < 0 es conjunto vacio. Observaci´ on 7.2.7. Se puede observar que para los ejemplos anteriores las curvas de nivel son, b´asicamente, las mismas, es decir, c´ırculos conc´entricos desde el origen. En este caso, las curvas de nivel no son suficiente para determinar el gr´afico de las funciones anteriores. Debemos considerar la noci´on de traza. Definici´ on 7.2.4. Llamaremos traza de una superficie S : z = f (x, y) a la intersecci´on de dicha superficie con alguno de los planos coordenados. Denotaremos las trazas de una superficie S mediante los s´ımbolos Ty y Tx , si la intersecci´on se efect´ ua con los planos xz e yz, respectivamente. Ejemplo 7.2.3. Por ejemplo, las trazas de las superficies S1 : z = x2 + y 2 y S2 : z = p x2 + y 2 est´an dadas por: (x, y, z) ∈ Ty ⇐⇒ (x, y, z) ∈ S1 ∩ {(x, y, z) : y = 0} ⇐⇒ z = x2 ,

y=0

para S1 , y para S2 por: (x, y, z) ∈ Ty ⇐⇒ (x, y, z) ∈ S1 ∩ {(x, y, z) : y = 0} ⇐⇒ z = |x| ,

y=0

En particular, son algunas trazas las que nos permiten distinguir los gr´aficos de las superficies S1 y S2 .

Ejercicios del cap´ıtulo 1. Encontrar los dominios de las siguientes funciones: p y − x2 2 a) f : D ⊆ R → R (x, y) → 2 x − y2 b) f : D ⊆ R2 → R

c)

f : D ⊆ R2 → R3

p y − x2 ln (1 − x2 + y 2 ) (x, y) → ln (x2 − y 2 )

(x, y) →

! √ x x x2 p , , x + y x + y 2 |x| − x2 + y 2 321

Apuntes Mat023 (versi´on preliminar actualizada 23-05-2014)

2. Esbozar curvas de nivel y gr´aficas de las siguientes funciones: a)

f : R2 → R (x, y) → x − y + 2

b) f : R2 → R (x, y) → x2 + 9y 2 c)

f : R2 → R (x, y) → m´ax {|x| , |y|}

3. Describir el comportamiento de las curvas de nivel f (x, y) = c para c en R con: a) f (x, y) = x2 + 4y 2 + 5

b)

f (x, y) = 4 − x2 − y 2

2 2 f (x, y) = (x2 + y 2 ) e−(x +y )   4. Considere la funci´on f (x, y) = x2 + y 2 y defina para θ ∈ 0, π2 el conjunto

c)

f (x, y) = |8 − |x2 + y 2 − 4|| + 1

d)

 Sθ = (x, y, z) ∈ R3 : y = (tan θ) x ver la forma de la curva G (f ) ∩ Sθ donde G (f ) corresponde a la gr´afica de la funci´on p 2 2 f . Hacer lo mismo con la funci´on f (x, y) = x2 + y 2 + ex +y . 5. Describir los conjuntos de nivel de las funciones: a)

f : R3 → R (x, y, z) → −x2 − y 2 − z 2

b) f : R3 → R (x, y, z) → 4x2 − y 2 + 9z 2 c)

f : R3 → R (x, y, z) → z − x2 − 4y 2

6. Representar en R3 los conjuntos:   y2 3 2 =1 a) A = (x, y, z) ∈ R : z + 4  b) B = (x, y, z) ∈ R3 : x = y 2  c) B = (x, y, z) ∈ R3 : x2 + y 2 = 1 ∧ x + y + z = 0 7. Utilizando coordenadas polares describir las curvas de nivel de la funci´on  2xy    x2 + y 2 si (x, y) 6= (0, 0)  f (x, y) =     0 si (x, y) = (0, 0)

322

Cap´ıtulo 8 : L´ımites y continuidad

Definiciones Definici´ on 8.1.1. Sean U ⊆ Rn , a ∈ U 0 y f : U → R una funci´on. Diremos que el n´ umero real L es el l´ımite de f (x) cuando x tiende a a si: ∀ ε > 0, ∃ δ > 0, 0 < kx − akRn < δ =⇒ |f (x) − L| < ε

(8.1)

Lo anterior se representa mediante el s´ımbolo: l´ım f (x) = L

x→a

Observaci´ on 8.1.1. En particular, si f : U ⊆ R2 → R y (a, b) ∈ U 0 , la definici´on de l´ımite en (8.1) queda como: q ∀ ε > 0, ∃ δ > 0, 0 < (x − a)2 + (y − b)2 < δ =⇒ |f (x, y) − L| < ε y en s´ımbolos: l´ım

f (x, y) = L

(x,y)→(a,b)

Usualmente, a los l´ımites del tipo anterior, se les conoce como l´ımites dobles. Ejemplo 8.1.1. Demuestre que: l´ım

(3x + 2y) = 2

(x,y)→(2,−1)

Soluci´ on. Por demostrar que, dado cualquier ε > 0, existe δ > 0 tal que si: q 0 < (x − 2)2 + (y + 1)2 < δ implica que: |3x + 2y − 4| < ε En efecto, note que: |3x + 2y − 4| = |3 (x − 2) + 2 (y + 1)| ≤ 3 |x − 2| + 2 |y + 1| < 3δ + 2δ = 5δ q Por tanto, dado ε > 0, existe δ = ε/5 tal que si 0 < (x − 2)2 + (y + 1)2 < δ, entonces |3x + 2y − 4| < ε. Por tanto, se concluye que l´ım(x,y)→(2,−1) (3x + 2y) = 2. 323

Apuntes Mat023 (versi´on preliminar actualizada 23-05-2014)

Ejemplo 8.1.2. Demuestre que: x2 cos (x2 + y 2 ) p =0 (x,y)→(0,0) x2 + y 2 l´ım

Soluci´ on. Note que si (x, y) 6= (0, 0) x2 cos (x2 + y 2 )  |x|2 cos x2 + y 2 − 0 = p p 2 x + y2 x2 + y 2 |x| |x| ≤ p x2 + y 2 p p x2 + y 2 x 2 + y 2 p ≤ x2 + y 2 p = x2 + y 2 = δ q por lo tanto, dado ε > 0 existe δ = ε tal que si 0 < (x − 0)2 + (y − 0)2 < δ entonces x2 cos (x2 + y 2 ) − 0 < ε. p 2 x + y2 Observaci´ on 8.1.2. Es importante destacar que la noci´on de l´ımite es un concepto que puede tratarse de manera m´as general que en el caso de funciones reales de varias variables. En particular, el concepto de l´ımite se puede extender a funciones vectoriales considerando la norma correspondiente al espacio de llegada. M´as precisamente, tenemos: Definici´ on 8.1.2. Sean U ⊆ Rn , a ∈ U 0 y F : U → Rm una funci´on vectorial. Diremos que el vector L ∈ Rm es el l´ımite de F (x) cuando x tiende a a si: ∀ ε > 0, ∃ δ > 0, 0 < kx − akRn < δ =⇒ kF (x) − LkRm < ε Sin embargo, para nuestro prop´ositos de c´alculo, es u ´til el siguiente teorema: Teorema 8.1.1. Sean U ⊆ Rn , x ∈ U 0 y F : U → Rm una funci´on vectorial tal que: F (x) = (f1 (x) , f2 (x) , . . . , fm (x)) Entonces, el vector L = (L1 , L2 , . . . , Lm ) ∈ Rm es el l´ımite de F (x) cuando x tiende a a, si y solo si: l´ım fi (x) = Li

x→a

para todo i = 1, 2, . . . , m. En otras palabras, F (x) tiende al vector L cuando x tiende a a, si y solo si, la convergencia se da en cada coordenada de F (x) a la correspondiente coordenada de L. 324

Apuntes Mat023 (versi´on preliminar actualizada 23-05-2014)

La demostraci´on de este resultado se sigue de la desigualdad |fi (x) − Li | ≤ k(f1 (x) , f2 (x) , . . . , fm (x)) − (L1 , L2 , . . . , Lm )k m √ X m |fi (x) − Li | ≤ i=1

Observaci´ on 8.1.3. En vista del resultado anterior, centraremos nuestra atenci´on en funciones reales de varias variables. Teorema 8.1.2. Sean F : U ⊆ Rn → Rm una funci´on vectorial y a ∈ U 0 . Suponga que: l´ım F (x) = L

x→a



l´ım F (x) = M

x→a

entonces, L = M. Demostraci´on. Sea ε > 0. Por hip´otesis, existen δ1 , δ2 > 0 tales que: 0 < kx − ak < δ1 =⇒ kf (x) − Lk
0, ∃ δ > 0, kx − akRn < δ =⇒ |f (x) − f (a)| < ε

(8.2)

Adem´as, diremos que f es continua en U si f es continua en cada punto de U . Observaci´ on 8.3.1. Note que la definici´on dada anteriormente y por la proposici´on en  (8.2) , dice que f es continua en a si: l´ım f (x) = f (a)

x→a

Definici´ on 8.3.2. Diremos que f : U ⊆ Rn → R es discontinua en a ∈ U ∩ U 0 , si f no es continua en a. Ejemplo 8.3.1. Sea πk : Rn → R la k−´esima proyecci´on de x ∈ Rn . Entonces, πk es continua en Rn . En efecto, para cada a ∈ Rn , tenemos que: |πk (x) − πk (a)| = |xk − ak | ≤ kx − ak bastar´a tomar δ = ε. 332

Apuntes Mat023 (versi´on preliminar actualizada 23-05-2014)

Ejemplo 8.3.2. Demuestre que f : R2 → R definida por:  2   xy , (x, y) 6= (0, 0) x4 + y 2 f (x, y) =   0 , (x, y) = (0, 0) es continua en (0, 0). Soluci´ on. Basta notar que:

xy 2 |x| y 2 x4 + y 2 ≤ y 2 = |x|

As´ı, si (x, y) → (0, 0), entonces |x| → 0. Por el Teorema del Sandwich se concluye que: xy 2 =0 (x,y)→(0,0) x4 + y 2 l´ım

como f (0, 0) = 0, se tiene que f es continua en (0, 0). Ejemplo 8.3.3. Sean U = {(x, y) : |x| < y 2 } ⊆ R2 y f : U → R definida por:  xy  , (x, y) ∈ U 2 x + y2 f (x, y) =  0 , (x, y) ∈ /U ¿Es continua en (0, 0)? Soluci´ on. Note que si (x, y) ∈ U xy |x| |y| |y|3 = ≤ 2 |f (x, y) − 0| = 2 x + y 2 x2 + y 2 x + y2 ≤

|y|3 = |y| |y|2

y si (x, y) 6∈ U entonces |f (x, y) − 0| = 0 ≤ |y| se sigue que, para todo (x, y) ∈ R2 |f (x, y) − 0| ≤ |y| y por el teorema de acotamiento l´ım

f (x, y) = 0

(x,y)→(0,0)

La funci´on es continua en (0, 0). 333

Apuntes Mat023 (versi´on preliminar actualizada 23-05-2014)

Definici´ on 8.3.3. Se dice que una funci´on f : U ⊆ Rn → Rm es Lipschitziana en U si: ∃K > 0, ∀x, y ∈ U,

kf (x) − f (y)k ≤ K kx − yk

La constante K se llama constante de Lipschitz. Toda funci´on lipschitziana es continua en U . En efecto, supongamos que f : U ⊆ Rn → Rm es una funci´on lipschitziana en U y que ε > 0, entonces: kf (x) − f (a)k ≤ K kx − ak < Kδ As´ı, tomando δ =

ε , K

se verifica que f es continua en a. Como a ∈ U es cualquiera, se

obtiene, finalmente, la continuidad de f en U . Ejemplo 8.3.4. Toda transformaci´on lineal T : Rn → Rm es una funci´on lipschitziana.

´ Algebra de funciones continuas

El siguiente teorema facilita la identificaci´on de funciones continuas: ´ Teorema 8.4.1 (Algebra de funciones continuas). Sean f, g : U ⊆ Rn → R funciones continuas en a ∈ U 0 ∩ U y α ∈ R, entonces αf, f + g, f − g y f g son continuas en a. Si g (a) 6= 0, entonces f /g es continua en a. Ejemplo 8.4.1. Son funciones continuas en R3 : 1. f (x, y, z) = 3xy 2 + z 2 2. g (x, y, z) =

sin x+sin y+sin z x2 +y 2 +z 2 +1 2

3. h (x, y, z) = ex sin (y) + z 2 Teorema 8.4.2. Sean f : U ⊆ Rn → Rm y g : V ⊆ Rm → Rp tales que f (U ) ⊆ V . Suponga, adem´as, que f es continua en a ∈ U 0 ∩ U y que g es continua en b = f (a) ∈ V 0 , entonces: g ◦ f : U ⊆ Rn → Rp es continua en a. 334

Apuntes Mat023 (versi´on preliminar actualizada 23-05-2014)

Demostraci´on. Sea ε > 0. Como g es continua en b = f (a), existe η > 0 de modo que si y ∈ V , entonces: ky − bk < η =⇒ kg (y) − g (b)k < ε Ahora bien, para y = f (x), existe δ > 0 tal que: kx − ak < δ =⇒ kf (x) − f (a)k < η As´ı, para x ∈ U tal que: kx − ak < δ =⇒ f (x) ∈ V ∧ kf (x) − f (a)k < η =⇒ kg (f (x)) − g (f (a))k < ε

Ejemplo 8.4.2. Una aplicaci´on inmediata del teorema anterior es el cambio de variables. Ilustramos lo anterior con el siguiente ejemplo: sea f : R2 → R definida por:   sin(x2 +y2 ) , si x2 + y 2 6= 0 x2 +y 2 f (x, y) =  1 , si x2 + y 2 = 0 Estudie la continuidad de f en (0, 0). Soluci´ on. Naturalmente, haciendo u = x2 + y 2 , tenemos que: (x, y) → (0, 0) =⇒ u → 0 As´ı:

sin u sin (x2 + y 2 ) = l´ım =1 2 2 u→0 (x,y)→(0,0) x +y u l´ım

Como f (0, 0) = 1, obtenemos que f es continua en (0, 0). Ejemplo 8.4.3. Usando compuesta de continuas podemos garantizar la continuidad de: p x2 + sin2 y + 1 √ |x|+y 2 +z 2 2. g (x, y, z) = z2 +y2 +x|y| 1. f (x, y) =

335

Apuntes Mat023 (versi´on preliminar actualizada 23-05-2014)

Continuidad de funciones vectoriales Observaci´ on 8.5.1. La continuidad para una funci´on vectorial de varias variables F : U ⊆ Rn → Rm en a ∈ U ∩ U 0 se define por: ∀ε > 0, ∃δ > 0, kx − akRn < δ =⇒ kF (x) − F (a)kRm < ε Observaci´ on 8.5.2. Al igual que en los l´ımites para funciones vectoriales, tenemos que la continuidad de F : U ⊆ Rn → Rm definida por x 7→ F (x) = (f1 (x) , f2 (x) , . . . , fm (x)) en a ∈ U 0 ∩ U se obtiene a trav´es de la continuidad de las funciones componentes en x. M´as precisamente: Teorema 8.5.1. Sean F : U ⊆ Rn → Rm una funci´on vectorial definida por: F (x) = (f1 (x) , f2 (x) , . . . , fm (x)) y a ∈ U ∩ U 0 . Entonces, F es continua en a ∈ U ∩ U 0 , si y solo si, fi es continua en a, para cada i = 1, 2, . . . , m. Ejemplo 8.5.1. La funci´on F : R2 → R3 definida por: F (x, y) = x2 y, sin x, 3x2 cos y



es continua en el plano. Ejemplo 8.5.2. Sea f : R2 → R la funci´on definida por:   √2x−y2 +2 , si x2 + y 2 ≤ 1 1−x2 −y 2 f (x, y) =  0 , si x2 + y 2 > 1 Analizar, completamente, la continuidad de f (x, y). Soluci´ on. Consideremos los siguientes subconjuntos del plano: (x, y) ∈ R2 : x2 + y 2 > 1  B = (x, y) ∈ R2 : x2 + y 2 = 1  C = (x, y) ∈ R2 : x2 + y 2 < 1 A =



Tenemos los siguientes casos:

336

Apuntes Mat023 (versi´on preliminar actualizada 23-05-2014)

1. Si (x, y) ∈ A, entonces f (x, y) = 0. Luego, f es continua en A por ser una funci´on constante en A. 2

2. Si (x, y) ∈ C, entonces f (x, y) = √2x−y 2+2 2 , con 1 − x2 − y 2 > 0. Luego, f es continua 1−x −y

en C por ´algebra de funciones continuas y composici´on de funciones continuas. 3. Si (a, b) ∈ B, entonces, f (x, y) no puede ser continua en (a, b) ∈ B si: 2a − b2 + 2 6= 0 pues, si:  (x, y) → (a, b) =⇒ 1 − x2 − y 2 → 0



2x − y 2 + 2 → k, con k 6= 0

2

con lo cual la expresi´on √2x−y 2+2 2 se indefine. Por tanto, los puntos (a, b) ∈ B tales 1−x −y

que: 2a − b2 + 2 = 0 est´an dados por el sistema:   2a − b2 + 2 = 0  a2 + b2 = 1 Es decir, (a, b) = (−1, 0). Por tanto, investigamos el l´ımite: 2x − y 2 + 2 p (x,y)→(−1,0) 1 − x2 − y 2 l´ım

Note que: 2x − y 2 + 2 1 − x2 − y 2 + x2 + 2x + 1 p p = 1 − x2 − y 2 1 − x2 − y 2 p (x + 1)2 2 2 = 1−x −y + p 1 − x2 − y 2 Por tanto, para que el l´ımite: 2x − y 2 + 2 p (x,y)→(−1,0) 1 − x2 − y 2 l´ım

exista es necesario y suficiente que el l´ımite: (x + 1)2 p (x,y)→(−1,0) 1 − x2 − y 2 l´ım

337

Apuntes Mat023 (versi´on preliminar actualizada 23-05-2014)

exista. Sin embargo, note que: ( l´ım

x→ −1−

(x + 1)2

l´ım p 1 − x2 − y 2

y→0−

)

(x + 1)2 = l´ım − √ =0 x→ −1 1 − x2

y que adem´as, al considerar la trayectoria:   ϕ = (x, y) ∈ C : − x4 + 4x3 + 7x2 + 4x = y 2 vemos que: (x + 1)2 (x + 1)2 p =1 = l´ım √ (x,y)→(−1,0) 1 − x2 − y 2 x→−1 1 − x2 + x4 + 4x3 + 7x2 + 4x l´ım

Por tanto:

(x + 1)2 p (x,y)→(−1,0) 1 − x2 − y 2 l´ım

no existe. En este caso, el l´ımite: 2x − y 2 + 2 p (x,y)→(−1,0) 1 − x2 − y 2 l´ım

no existe. As´ı, f no es continua en (−1, 0). Por lo tanto, f es continua en:  R2 − (x, y) ∈ R2 : x2 + y 2 = 1 . Finalmente enunciamos un teorema que nos habla sobre el comportamiento de las funciones continuas sobre los conjuntos compactos y conexos: Teorema 8.5.2. Sea f : U ⊆ Rn → Rm una funci´on continua en U , entonces: 1. Si A ⊆ U es conexo, entonces f (A) es conexo. 2. Si K ⊆ U es compacto, entonces f (K) es compacto.

Ejercicios del cap´ıtulo 1. Determine el valor de α ∈ R de modo que el l´ımite: √   x − x cos xy (x2 − xy) 1 − α2 l´ım + (x,y)→(0,0) x2 − y 2 y sin2 (3x) exista. 338

Apuntes Mat023 (versi´on preliminar actualizada 23-05-2014)

2. Sea f : R2 − {(0, 0)} → R la funci´on definida por   1 2 x sin x+y f (x, y) = 2 x + |x| + 2 calcular

l´ım

f (x, y)

(x,y)→(0,0)

3. Considere la funci´on definida por

f (x, y) =

 x  si x 6= −y 2    x + y2    

0

si x = −y 2

analizar la continuidad de f en R2 . 4. Sean f y g funciones definidas en R2 − {(0, 0)} por p  sin x2 + y 2 f (x, y) = p x2 + y 2 + x2 + y 2  3  x − y3 g (x, y) = α |x| + |y| que valor debe tener α para que l´ım

{f (x, y) + g (x, y)}

(x,y)→(0,0)

exista. 5. Estudiar la continuidad de la funci´on   x3 + sin (y 2 )   f (x, y) =    2 x − xy + y 3 + cos (y 2 ) 6. Estudiar la continuidad de la funci´on  2 2 x −y    x2 +y2 −1 f (x, y) =    0 339

y≤x y>x

x2 + y 2 6= 1 x2 + y 2 = 1

Apuntes Mat023 (versi´on preliminar actualizada 23-05-2014)

7. Determine si los siguientes l´ımites existen x2 y 2 |sin x sin y| 2) l´ım 1) l´ım 2 (x,y)→(0,0) x2 y 2 + (y − x) (x,y)→(0,0) |xy|

3)

xy sin (y 3 ) (x,y)→(0,0) x4 + y 4

4)

sin (xy 4 ) (x,y)→(0,0) x2 + y 8

5)

p y 3 |x| l´ım (x,y)→(0,0) |x| + y 4

6)

x3 + y 2 (x,y)→(0,0) |x| + |y|

l´ım

l´ım

l´ım

cos

p  |xy| − 1

sin x + sin y (x,y)→(0,0) x+y

8)

9)

(x − 1)4 y 2 l´ım 8 (x,y)→(1,0) (x − 1) + y 4

p x3 y x2 + y 2 10) l´ım (x,y)→(0,0) sin (xy)

11)

x2 y (x,y)→(0,0) x2 + y

12)

2x2 − y 2 (x,y)→(0,0) x2 + 2y 2

13)

x4 + 2x2 y 2 + 2y 4 (x,y)→(0,0) (x2 + y 2 )2

14)

x6 + y 6 (x,y)→(0,0) x4 + y 4

7)

l´ım

l´ım

l´ım

l´ım (x,y)→(1,0)

340

l´ım

l´ım

y

Cap´ıtulo 9 : Diferenciaci´ on en varias variables

En secciones anteriores hemos estudiado algunos m´etodos para dibujar la gr´afica de funciones de varias variables. Utilizando solo esos m´etodos es muy dif´ıcil obtener suficiente informaci´on para captar las caracter´ısticas generales de una funci´on con expresi´on algo complicada. Sabemos del c´alculo de una variable que la derivada nos puede ayudar mucho en esta tarea; por ejemplo, nos permite cuantificar los cambios en la gr´afica, localizar m´aximos, m´ınimos, estudian concavidad, etc. La derivada tambi´en tiene otras muchas aplicaciones como el estudiante habr´a descubierto en c´alculo de una variable. Una funci´on diferenciable de R2 en R deber´ıa ser una funci´on con gr´afica suave, que no tenga dobleces bruscos. Para hacer precisas estas ideas necesitamos una definici´on de lo que entenderemos por diferenciable en el caso de funciones de varias variables. Comenzamos con una noci´on de derivada que descansa en nuestros conocimientos de una variable.

Derivadas parciales Observaci´ on 9.1.1. En lo que sigue denotaremos por ei , con i = 1, 2, . . . , n, a los n vectores de la base can´onica de Rn , m´as precisamente, ei representa el vector de Rn que tiene todas su componentes cero salvo la i-´esima componente la cual es igual a uno. ◦

Definici´ on 9.1.1. Sean f : U ⊆ Rn → R y a ∈ U . Se define la derivada parcial de f en a respecto a la i-´esima variable como el l´ımite: ∂f f (a + tei ) − f (a) (a) = l´ım t→0 ∂xi t f (a1 , a2 , . . . , ai + t, . . . , an ) − f (a1 , a2 , . . . , an ) = l´ım t→0 t si este existe. Observaci´ on 9.1.2. Para f : U ⊆ Rn → R, las derivadas parciales se denotan por: ∂f , ∂xi

fxi , Di f , etc. ◦

Observaci´ on 9.1.3. Si n = 2, f : U ⊆ R2 → R, (x, y) → f (x, y) y a = (a, b) ∈ U ,

341

Apuntes Mat023 (versi´on preliminar actualizada 23-05-2014)

entonces: ∂f f (a + te1 ) − f (a) (a, b) = l´ım t→0 ∂x t f ((a, b) + t (1, 0)) − f (a, b) = l´ım t→0 t f (a + t, b) − f (a, b) = l´ım t→0 t y f (a + te2 ) − f (a) ∂f (a, b) = l´ım t→0 ∂y t f ((a, b) + t (0, 1)) − f (a, b) = l´ım t→0 t f (a, b + t) − f (a, b) = l´ım t→0 t note que corresponden a l´ımites de una variable, a saber, la variable t. Observaci´ on 9.1.4. En particular, si n = 2, 3 y 4, anotamos: y

∂f ∂x4

=

∂f , ∂w

∂f ∂x1

=

∂f , ∂f ∂x ∂x2

=

∂f , ∂f ∂y ∂x3

=

∂f ∂z

seg´ un corresponda respecto al n´ umero de variables de f y el nombre y orden

que a estas asignemos. Ejemplo 9.1.1. Sea f : R2 → R, (x, y) → f (x, y) = x3 y + 2x2 y 3 + 2x, calcular y

∂f ∂y

∂f ∂x

(−1, 1)

(1, 2) si estas existen.

Soluci´ on. Por definici´on ∂f f (−1 + t, 1) − f (−1, 1) (−1, 1) = l´ım t→0 ∂x t   3 (−1 + t) 1 + 2 (−1 + t)2 13 + 2 (−1 + t) − (−1)3 1 + 2 (−1)2 13 + 2 (−1) = l´ım t→0 t 2 t (t − t + 1) = l´ım t→0 t  2 = l´ım t − t + 1 t→0

= 1

342

Apuntes Mat023 (versi´on preliminar actualizada 23-05-2014)

y ∂f f (1, 2 + t) − f (1, 2) (1, 2) = l´ım t→0 ∂y t  13 (2 + t) + 2 (12 ) (2 + t)3 + 2 (1) − (13 2 + 2 (12 ) (23 ) + 2) = l´ım t→0 t t (2t2 + 12t + 25) = l´ım t→0 t  2 = l´ım 2t + 12t + 25 t→0

= 25 note que en estos l´ımites, una de las variables se deja fija y se calcula la derivada usual de una variable respecto a la otra, en otras palabras ∂f f (x + t, y) − f (x, y) (x, y) = l´ım t→0 ∂x t h (x + t) − h (x) = l´ım t→0 t = h0 (x) donde hemos pensado h (x) = f (x, y), al estar y fija esta es solo una funci´on de la variable x. En el ejemplo  ∂f ∂ (x, y) = x3 y + 2x2 y 3 + 2x ∂x ∂x = 3x2 y + 4xy 3 + 2 evaluando en (−1, 1) ∂f (−1, 1) = 3 (−1)2 + 4 (−1) + 2 = 1 ∂x y  ∂f ∂ (x, y) = x3 y + 2x2 y 3 + 2x ∂y ∂y = x3 + 6x2 y 2 luego  ∂f (1, 2) = 13 + 6 22 = 25 ∂y

343

Apuntes Mat023 (versi´on preliminar actualizada 23-05-2014)

Ejemplo 9.1.2. Si f (x, y, z) = 2xyz 2 + sin (xy 2 ) + 2x entonces    ∂f ∂ (x, y, z) = 2xyz 2 + sin xy 2 + 2x = 2yz 2 + y 2 cos xy 2 + 2 ∂x ∂x    ∂f ∂ (x, y, z) = 2xyz 2 + sin xy 2 + 2x = 2xz 2 + 2xy cos xy 2 ∂y ∂y   ∂f ∂ (x, y, z) = 2xyz 2 + sin xy 2 + 2x = 4xyz ∂z ∂z Ejemplo 9.1.3. Verifique que: ∂u ∂u ∂u + + =1 ∂x ∂y ∂z si: u=x+

x−y y−z

Soluci´ on. Derivando  ∂ x+ ∂x  ∂ x+ ∂y  ∂ x+ ∂z

 x−y y−z+1 = y−z y−z  x−y x−z = − y−z (y − z)2  x−y x−y = y−z (y − z)2

se sigue 

y−z+1 y−z



    x−z x−y + − + =1 (y − z)2 (y − z)2

Ejemplo 9.1.4. Verifique que: xzx + yzy = xy + z si: y

z = xy + xe x Soluci´ on. Derivando   y 1 ∂  1  1y y x x x xy + xe = xe − ye + xy ∂x x  y 1 ∂  xy + xe x = x + exy ∂y

344

Apuntes Mat023 (versi´on preliminar actualizada 23-05-2014)

luego xzx + yzy      1 1 1 1 y y xe x − ye x + xy + y x + exy = x x 1

1

1

= xe x y − ye x y + xy + xy + ye x y 1

= xe x y + 2xy y

= xy + xy + xe x = xy + z Ejemplo 9.1.5. Sea f : R2 → R la funci´on definida por:   xy , si (x, y) 6= (0, 0) x2 +y 2 f (x, y) =  0 , si (x, y) = (0, 0) Calcule las derivadas parciales de f en (0, 0). Soluci´ on. Es importante recordar que f no es continua en (0, 0). Sin embargo, la continuidad no afecta la existencia de las derivadas parciales. En efecto: f (t, 0) − f (0, 0) ∂f (0, 0) = l´ım t→0 ∂x t 0−0 = l´ım =0 t→0 t An´alogamente, se tiene que f (0, t) − f (0, 0) ∂f (0, 0) = l´ım t→0 ∂y t 0t 2 2 − 0 = l´ım 0 +t t→0 t = l´ım 0 t→0

= 0 Ejemplo 9.1.6. Sea f : R2 → R la funci´on definida por:   x2 , y ≤ x f (x, y) =  xy , y > x Calcule

∂f ∂y

(1, 1). 345

Apuntes Mat023 (versi´on preliminar actualizada 23-05-2014)

Soluci´ on. Calculamos

∂f ∂y

l´ım+

t→0

(1, 1). Note que:

f (1, 1 + t) − f (1, 1) = t

1 · (1 + t) − 1 t→0 t t = l´ım+ t→0 t = 1 l´ım+

y, an´alogamente, se obtiene que: f (1, 1 + t) − f (1, 1) → 0, si t → 0− t Por tanto,

∂f ∂y

(1, 1), no existe.

en la figura se muestra el dominio y en dos colores distintos la parte y ≤ x e y > x, al aproximarnos al punto (1, 1) por los puntos (1, 1 + t) se obtienen dos valores distintos de l´ımite para t > 0 y t < 0. Observaci´ on 9.1.5. La sola existencia de las derivadas parciales no es un buen concepto de ser derivable en el caso de funciones de varias variables. Por ejemplo, la funci´on     1 si xy = 0 f (x, y) =    0 si xy 6= 0 no es continua en (0, 0) pero las derivadas parciales existen en ese punto, luego la existencia de las derivadas parciales no implica continuidad, si la existencia de las derivadas parciales fuera el concepto de “ser derivable” no podr´ıamos recuperar el teorema de una variable, 346

Apuntes Mat023 (versi´on preliminar actualizada 23-05-2014)

derivable en un punto implica continua en el punto. Veremos que el concepto de ser derivable estudia las variaciones de la funci´on en una forma m´as global y no en una direcci´on espec´ıfica como las derivadas parciales.

Ejercicios de la secci´ on 1. Hallar

∂f ∂x

y

∂f ∂y

para las siguientes funciones

a) f (x, y) = xy 2 b) f (x, y) = ex

3 +y 2

c) f (x, y) = x ln (x2 + y 2 ) 2. Si f (x, y) =

   

x2 y 3 x2 +y 2

si (x, y) 6= (0, 0)

  

0 si (x, y) = (0, 0) n o (x,y) determine el conjunto D = (x, y) ∈ R2 : ∂f∂x existe y estudiar si la funci´on D ⊆ R2 → R ∂f (x, y) (x, y) → G (x, y) = ∂x G

:

es continua en todo D.  2 2 −y 3. Si u (x, y) = arctan x xy hallar ux , uy y verificar que xux + yuy = 0 4. Si u (x, y, z) = x2 y + y 2 z + z 2 x verificar que ∂u ∂u ∂u + + = (x + y + z)2 ∂x ∂y ∂z 5. Determine todas las funciones f : R3 → R, (x, y, z) → f (x, y, z) que cumplen ∂f = ex+y + 3x2 y + 4yz + y 2 + 2 ∂x ∂f = ex+y + 2yz 3 + 2xy + 4xz + x3 ∂y ∂f = 3y 2 z 2 + 4xy ∂z 347

Apuntes Mat023 (versi´on preliminar actualizada 23-05-2014)

6. Si f (x, y) =

 p  |x| + y 2 + |x|   si   x+y     

determine si

∂f ∂x

(0, 0) y

∂f ∂y

x+y >0

x3

si x + y ≤ 0 ∧ x2 + y 2 ≤ 4

y2 + x

si x + y ≤ 0 ∧ x2 + y 2 > 4

(0, 0) existen.

Interpretaci´ on de la derivada parcial En esta secci´on veremos una interpretaci´on de las derivadas parciales y buscaremos la ecuaci´on del plano tangente a la gr´afica de una funci´on de dos variables. Considere una funci´on f : D ⊆ R2 → R, su gr´afica esta contenida en R3 . Suponga que estamos interesados en estudiar el comportamiento de la funci´on en un punto (x0 , y0 ) de su dominio, como no contamos con t´ecnicas de varias variables, intentamos estudiar el comportamiento descendiendo a una variable, para ello, vamos a intersectar la gr´afica con dos planos x = x0 e y = y0 . La intersecci´on de la gr´afica con el plano x = x0 es una curva, la cual puede ser interpretada como el gr´afico de la funci´on de una variable z = f (x0 , y) = g (y)

en el plano x = x0 (note que si x = x0 y z = f (x, y) entonces z = f (x0 , y)). En este plano podemos aplicar las t´ecnicas de una variable para conocer el crecimiento y decrecimiento, concavidad y otros, sin embargo, este an´alisis solo es un comportamiento de la curva, no necesariamente un comportamiento global, por ejemplo un m´aximo sobre esta curva puede no ser un m´aximo de la funci´on en su dominio. La recta tangente a la gr´afica de esta funci´on de una variable es z − g (y0 ) = g 0 (y0 ) (y − y0 ) esto es z − f (x0 , y0 ) =

∂f (x0 , y0 ) (y − y0 ) ∂y

348

Apuntes Mat023 (versi´on preliminar actualizada 23-05-2014)

pues g (y0 + h) − g (y0 ) h→0 h f (x0 , y0 + h) − f (x0 , y0 ) = l´ım h→0 h ∂f (x0 , y0 ) = ∂y

g 0 (y0 ) = l´ım

note que la recta est´a en el plano x = x0 , se sigue que los puntos de ella cumplen x = x0 y = y z = f (x0 , y0 ) +

∂f (x0 , y0 ) (y − y0 ) ∂y

o en ecuaciones param´etricas    x x0     y = 0    z f (x0 , y0 ) − y0 ∂f (x0 , y0 ) ∂y donde y ∈ R, el vector director es en este caso  0   1  ∂f (x0 , y0 ) ∂y





0



  +y  

1

  

∂f ∂y

(x0 , y0 )

   

De manera similar, la intersecci´on de la gr´afica con el plano y = y0 es una curva, la cual puede ser interpretada como el gr´afico de la funci´on de una variable z = f (x, y0 ) = p (x)

sobre el plano y = y0 (note que si y = y0 y z = f (x, y) entonces z = f (x, y0 )). La recta tangente a la gr´afica de esta funci´on de una variable es z − p (x0 ) = p0 (x0 ) (x − x0 ) esto es z − f (x0 , y0 ) =

∂f (x0 , y0 ) (x − x0 ) ∂x 349

Apuntes Mat023 (versi´on preliminar actualizada 23-05-2014)

pues p (x0 + h) − p (x0 ) h→0 h f (x0 + h, y0 ) − f (x0 , y0 ) = l´ım h→0 h ∂f (x0 , y0 ) = ∂x

p0 (x0 ) = l´ım

se sigue que los puntos de esta recta cumplen x = x y = y0 ∂f (x0 , y0 ) (x − x0 ) ∂x

z = f (x0 , y0 ) +

o en ecuaciones param´etricas    0 x     y = y0    f (x0 , y0 ) − x0 ∂f z (x0 , y0 ) ∂x





1



   + x  

0

  

∂f ∂x

(x0 , y0 )

el vector director corresponde a 

1



  

0

  

∂f ∂x

(x0 , y0 )

de estos c´alculos se sigue que en el punto (x0 , y0 , f (x0 , y0 )) de la gr´afica los vectores     1 0       y   0 1     ∂f ∂f (x0 , y0 ) (x0 , y0 ) ∂x ∂y son tangentes, se sigue que el vector    1     × 0    ∂f (x0 , y0 ) ∂x

0





(x0 , y0 ) − ∂f ∂x



  ∂f   =  − (x0 , y0 )    ∂y  (x0 , y0 ) 1 1

∂f ∂y

es perpendicular a la superficie, as´ı   ∂f ∂f − (x0 , y0 ) , − (x0 , y0 ) , 1 · (x − x0 , y − y0 , z − f (x0 , y0 )) = 0 ∂x ∂y 350

Apuntes Mat023 (versi´on preliminar actualizada 23-05-2014)

deber´ıa corresponder a un plano tangente a la superficie, el problema para definir esto directamente como el plano tangente a la gr´afica es el de formalizar en que sentido queremos decir que la funci´on y el plano se parecen cerca del punto, el c´alculo realizado depende solo de la existencia de las derivadas parciales pero por ejemplo en el caso de la funci´on     1 si xy = 0 f (x, y) =

  

0 si xy 6= 0

los c´alculos nos llevar´ıan a que el plano tangente es z = 1 sin embargo los valores que toma la funci´on en todo entorno abierto de (0, 0) no se parecen a uno (tan cerca como queramos existen valores cero). Veamos un ejemplo concreto, consideremos la funci´on f : R2 → R, (x, y) → f (x, y) = (x2 − 2xy) e−2x

2 −y 4 +2y

y estudiemos la funci´on en el punto (x0 , y0 ) = (1, 0). La gr´afica de f

es

351

Apuntes Mat023 (versi´on preliminar actualizada 23-05-2014)

2

si cortamos con el plano y = 0 se obtiene la curva z = f (x, 0) = x2 e−2x

la recta tangente en x = 1 es d  2 −2x2  (x − 1) z − f (1, 0) = xe dx x=1 esto es 2 2 z − e−2 = 2xe−2x − 4x3 e−2x (x − 1) x=1  z − e−2 = 2e−2 − 4e−2 (x − 1) es decir x = x y = 0 z = e−2 − 2e−2 (x − 1)

352

Apuntes Mat023 (versi´on preliminar actualizada 23-05-2014)

de manera similar, si cortamos con el plano x = 1 se obtiene la curva z = f (1, y) = (1 − 2y) e−2−y

4 +2y

la recta tangente en y = 0 es  d  −2−y 4 +2y z − f (1, 0) = (1 − 2y) e (y − 0) dy y=0

esto es z − e−2 = −4ye−y

4 +2y−2

 −2y 3 + y 2 + 1

y y=0

z − e−2 = 0 es decir x = 1 y = y



1

 los directores de estas rectas son  

0

a    

z = e−2  

0



    y  1  se sigue que el normal corresponde    −2 −2e 0      1 0 2e−2      × 1 = 0  0      −2 −2e 0 1 353

Apuntes Mat023 (versi´on preliminar actualizada 23-05-2014)

y as´ı el plano tangente corresponde a   2e−2 , 0, 1 · x − 1, y − 0, z − e−2 = 0 es decir 2e−2 (x − 1) + z − e−2 = 0 o equivalentemente z = e−2 − 2e−2 (x − 1)

en la figura, en blanco est´an las curvas y las rectas tangentes, la intersecci´on de estas rectas y curvas corresponde al punto (1, 0, f (1, 0)) y el plano que contiene a estas rectas tangentes es el plano tangente, como se puede apreciar la propiedad de tangencia es local, no significa que toque a la gr´afica en un u ´nico punto.

Ejercicios de la secci´ on 1. Considere la funci´on f : R2 → R, (x, y) → f (x, y) = ex sin y + e2y cos x. Determine las ecuaciones param´etricas de las rectas tangentes a la gr´afica de f en el punto (0, 0, 1) que son paralelas a los planos x = 0 e y = 0. 354

Apuntes Mat023 (versi´on preliminar actualizada 23-05-2014)

2. Determine una recta normal a la superficie x2 + en el punto



√1 , 1, 1 2 2



y2 + z2 = 1 4

. 2 −y 4 +2y

3. Si f : R2 → R, (x, y) → f (x, y) = (x2 − 2xy) e−2x

estudiar los extremos

(m´aximos y m´ınimos) locales y globales de la funci´on g : R → R, t → g (t) = f (t, 0).

Diferenciabilidad Definici´ on 9.3.1. Sean U ⊆ Rn y f : U → R una funci´on. Diremos que f es diferenciable ◦

en a ∈ U si existe una transformaci´on lineal T : Rn → R tal que: |f (a + h) − f (a) − T (h)| =0 h→0 khk l´ım

Observaci´ on 9.3.1. Si n = 2, por ejemplo, el l´ımite en la definici´on anterior toma la forma: |f (a + h, b + k) − f (a, b) − T (h, k)| √ =0 (h,k)→(0,0) h2 + k 2 Recordemos, adem´as, que si T : Rn → R es una transformaci´on lineal, entonces l´ım

existen constantes A1 , A2 , . . . , An ∈ R (respecto, por ejemplo, de las bases can´onicas, respectivamente) tales que: [T ] =



A1 A2 · · · An



∈ M1×n (R)

El siguiente teorema indica la condici´on necesaria que deben cumplir las constantes A1 , A2 , . . . , An en caso de que una funci´on f sea diferenciable en a: ◦

Teorema 9.3.1. Sean U ⊆ Rn y f : U → R una funci´on diferenciable en a ∈ U , entonces: Ai =

∂f (a) ∂xi

para cada i = 1, 2, . . . , n.

355

Apuntes Mat023 (versi´on preliminar actualizada 23-05-2014)

Demostraci´on. Como f : U → R es diferenciable en a, existe una transformaci´on lineal T : Rn → R tal que:

|f (a + h) − f (a) − T · h| =0 h→0 khk l´ım

Note que haciendo: h = t · ei el l´ımite anterior implica el siguiente l´ımite: |f (a + t · ei ) − f (a) − T (t · ei )| =0 t→0 kt · ei k

l´ım

(9.1)

Por otro lado, suponga que la transformaci´on lineal T : Rn → R tiene representaci´on matricial: [T ]1C =



A1 A2 · · · An



donde C es la base can´onica de Rn y 1 representa la base can´onica de R. As´ı: [T · (t · ei )]1 = [T ]1C [t · ei ]C = t · Ai As´ı: |f (a + t · ei ) − f (a) − t · Ai | |f (a + t · ei ) − f (a) − T (t · ei )| = l´ım t→0 t→0 kt · ei k |t| f (a + t · ei ) − f (a) = l´ım − Ai t→0 t ∂f = (a) − Ai ∂xi

l´ım

y como el valor del l´ımite en (9.1) es 0 (por la diferenciabilidad de f en a) se tiene que: ∂f ∂xi (a) − Ai = 0 Esto es: ∂f (a) = Ai ∂xi para cada i = 1, 2, . . . , n. Observaci´ on 9.3.2. Se sigue entonces que si f es diferenciable en x = a las derivadas parciales deben existir en el punto. 356

Apuntes Mat023 (versi´on preliminar actualizada 23-05-2014)

Podemos entonces reformular la definici´on como: Definici´ on 9.3.2. Sean U ⊆ Rn y f : U → R una funci´on. Diremos que f es diferenciable ◦

en a ∈ U si: 1. Para cada i = 1, 2, . . . , n, existen las derivadas parciales

∂f ∂xi

(a).

2. Asumiendo, h = (h1 , h2 , . . . , hn ) ∈ Rn , se tiene que: Pn ∂f (a) h f (a + h) − f (a) − i=1 ∂x i i l´ım =0 h→0 khk Observaci´ on 9.3.3. Con el cambio de variables h = x − a el l´ımite Pn ∂f (a) h f (a + h) − f (a) − i=1 ∂x i i l´ım =0 h→0 khk se puede escribir en la forma Pn ∂f (a) (x − a ) f (x) − f (a) − i=1 ∂x i i i l´ım =0 x→a kx − ak Observaci´ on 9.3.4. Se debe hacer notar que la suma: n X ∂f (a) · hi ∂xi i=1

se obtiene v´ıa el isomorfismo can´onico que existe entre M1×1 (R) y el espacio vectorial real unidimensional. En efecto sabemos que:  ∂f [T ]1C = ∂x (a) 1

∂f ∂x2

(a) · · ·

∂f ∂xn

(a)



y como h = (h1 , h2 , . . . , hn ), tenemos: [T · h]1 = [T ]1C [h]C

=



∂f ∂x1

(a)

∂f ∂x2

(a) · · ·

∂f ∂xn



h1



   (a)   

h2 .. .

     

hn =

n X ∂f (a) · hi ∂x i i=1

!

357

1×1

Apuntes Mat023 (versi´on preliminar actualizada 23-05-2014)

Ejemplo 9.3.1. Considere la funci´on f : R2 → R definida por:  2 4   xy si (x, y) 6= (0, 0) x2 + y 2   0 si (x, y) = (0, 0) estudiar la diferenciabilidad de f en (0, 0). Soluci´ on. Notemos que ∂f f (h, 0) − f (0, 0) (0, 0) = l´ım h→0 ∂x h h2 04 2 2 − 0 = l´ım h +0 h→0 h = l´ım 0 h→0

= 0 de manera similar ∂f f (0, h) − f (0, 0) (0, 0) = l´ım h→0 ∂y h 02 h4 2 2 − 0 = l´ım 0 +h h→0 h = l´ım 0 h→0

= 0 luego las derivadas parciales existen, notemos que (0, 0) (x − 0) − f (x, y) − f (0, 0) − ∂f ∂x p l´ım (x,y)→(0,0) x2 + y 2 2 4 xy x2 +y2 = l´ım p (x,y)→(0,0) x2 + y 2 x2 y 4 = l´ım (por polares) (x,y)→(0,0) (x2 + y 2 )3/2 = 0

∂f ∂y

(0, 0) (y − 0)

se sigue que f es diferenciable en (0, 0). Ejemplo 9.3.2. Considere la funci´on g : R3 → R definida por:    sin (xyz) , (x, y, z) = 6 (0, 0, 0) x2 + y 2 + |z| g (x, y, z) =   0 , (x, y, z) = (0, 0, 0) 358

Apuntes Mat023 (versi´on preliminar actualizada 23-05-2014)

Demuestre que g es diferenciable en el punto (0, 0, 0). Soluci´ on. Calculamos, primeramente, las derivadas parciales de f en (0, 0, 0). Esto es: ∂f f (t, 0, 0) − f (0, 0, 0) (0, 0, 0) = l´ım t→0 ∂x t 0 = l´ım t→0 t = 0 An´alogamente, se tiene que

l´ım

∂f ∂f (0, 0, 0) = (0, 0, 0) = 0. Luego: ∂y ∂z

f (h, k, l) − f (0, 0, 0) −

∂f ∂x

(h,k,l)→(0,0,0)

= =

(0, 0, 0) · h + (0, 0, 0) · k + √ h2 + k 2 + l2

∂f ∂z

|f (h, k, l)| √ (h,k,l)→(0,0,0) h2 + k 2 + l2 l´ım l´ım

(h,k,l)→(0,0,0) (h2



l´ım (h,k,l)→(0,0,0)



∂f ∂y

|l|

+



k2

|sin (hkl)| √ + |l|) h2 + k 2 + l2

|hkl| + k 2 + l2

h2

|hkl| (h,k,l)→(0,0,0) |l| |k| l´ım

=

l´ım

|h|

(h,k,l)→(0,0,0)

=0 Por tanto, f es diferenciable en (0, 0, 0). Ejemplo 9.3.3. Consideremos f : R2 → R la funci´on definida por:   sin(xy2 ) , (x, y) 6= (0, 0) x2 +|y| f (x, y) =  0 , (x, y) = (0, 0) Verifique que f no es diferenciable en (0, 0). Soluci´ on. Notamos, primeramente que: ∂f f (t, 0) − f (0, 0) (0, 0) = l´ım t→0 ∂x  t  1 0 = l´ım · 2 =0 t→0 t t +0 359

(0, 0, 0) · l

=

Apuntes Mat023 (versi´on preliminar actualizada 23-05-2014)

An´alogamente se obtiene que: ∂f (0, 0) = 0 ∂y Ahora bien, como: (0, 0) · h − f (h, k) − f (0, 0) − ∂f ∂x √ l´ım (h,k)→(0,0) h2 + k 2

∂f ∂y

(0, 0) · k

=

|f (h, k)| √ (h,k)→(0,0) h2 + k 2 l´ım

=

l´ım (h,k)→(0,0)



|sin (hk)| 1 h2 + k 2 h2 + |k|

Ahora bien, note que: 

|sin (hk)| 1 l´ım l´ım √ h→0 k→0 h2 + k 2 h2 + |k|

 = l´ım 0 = 0 h→0

y si consideramos la trayectoria ϕ : h = k, tenemos que: l´ım √

h→0

1 |sin (h2 )| 1 |sin (h2 )| √ = l´ ım h→0 h2 + h2 h2 + |h| 2 h3 + h2 1 |sin (h2 )| 1 = √ l´ım h2 1 + |h| 2 h→0 1 = √ 2

Por tanto, f no es diferenciable en (0, 0). Ejemplo 9.3.4. Toda transformaci´on lineal es diferenciable, la transformaci´on lineal que mas se parece es la misma funci´on. Observaci´ on 9.3.5. Supongamos que f : U ⊆ R2 → R es una funci´on diferenciable en ◦

(a, b) ∈ U . Esto quiere decir que: (a, b) h − f (a + h, b + k) − f (a, b) − ∂f ∂x √ l´ım (h,k)→(0,0) h2 + k 2 Ahora bien, haciendo el cambio de variables:   x=a+h  y =b+k tenemos que: (h, k) → (0, 0) ⇐⇒ (x, y) → (a, b) 360

∂f ∂y

(a, b) k

=0

(9.2)

Apuntes Mat023 (versi´on preliminar actualizada 23-05-2014)

Por tanto, el l´ımite en (9.2) queda: (a, b) · (x − a) − f (x, y) − f (a, b) − ∂f ∂x q l´ım (x,y)→(a,b) (x − a)2 + (y − b)2

∂f ∂y

(a, b) · (y − b)

=0

Intuitivamente, para (x, y) suficientemente cercano a (a, b), tenemos que: ∂f ∂f f (x, y) − f (a, b) − ∂x (a, b) · (x − a) − ∂y (a, b) · (y − b) q '0 2 2 (x − a) + (y − b) As´ı: f (x, y) − f (a, b) −

∂f ∂f (a, b) · (x − a) − (a, b) · (y − b) ' 0 ∂x ∂y

y por tanto: f (x, y) '

∂f ∂f (a, b) · (x − a) + (a, b) · (y − b) + f (a, b) ∂x ∂y

Es decir, para (x, y) suficientemente cercano a (a, b), f (x, y) puede ser aproximada por el plano: z = f (a, b) +

∂f ∂f (a, b) · (x − a) + (a, b) · (y − b) ∂x ∂y

el cual es tangente a la superficie: S : z = f (x, y) en una vecindad de (a, b). As´ı, tenemos la siguiente definici´on: Definici´ on 9.3.3. Sea f : U ⊆ R2 → R una funci´on diferenciable en (a, b). Llamaremos plano tangente a la superficie z = f (x, y) en (a, b) al plano de ecuaci´on: z − f (a, b) = (x − a)

∂f ∂f (a, b) + (y − b) (a, b) ∂x ∂y

Ejemplo 9.3.5. Encontrar la ecuaci´on del plano tangente a la superficie S dada por la gr´afica de la funci´on: z = (x + y)2 − 2x en el punto (1, 1, 0).

361

Apuntes Mat023 (versi´on preliminar actualizada 23-05-2014)

Soluci´ on. Calculamos las derivadas parciales ∂z ∂x

 ∂ (x + y)2 − 2x = 2x + 2y − 2 ∂x

= ⇒

∂z (1, 1) = 2 ∂x y ∂z ∂y

=

 ∂ (x + y)2 − 2x = 2x + 2y ∂y

⇒ ∂z (1, 1) = 4 ∂y el plano tangente es z = 0 + 2 (x − 1) + 4 (y − 1) es decir z = 2x + 4y − 6 Ejemplo 9.3.6. Sean a, b, c n´ umeros reales positivos. Verifique si el cono: x2 y 2 z2 + = a2 b2 c2 y la esfera: 2   b2 b2 + c 2 = 2 b2 + c 2 x +y + z− c c 2

2

son tangentes entre s´ı en los puntos (0, ±b, c). Soluci´ on. Notemos que los puntos entregados pertenecen a la intersecci´on 02 (±b)2 c2 + = a2 b2 c2 y  2  b2 + c 2 b2 0 + (±b) + c − = 2 b2 + c 2 c c 2

2

Notemos que x2 y 2 + 2 a2 b

=

z2 c2

⇒ r z = c 362

x2 y 2 + 2 a2 b

Apuntes Mat023 (versi´on preliminar actualizada 23-05-2014)

(el punto (0, ±b, c) tiene tercera coordenada positiva, por eso consideramos solo la ra´ız positiva). El vector normal al plano tangente es ! ! ! r r ∂ x2 y 2 x2 y 2 ∂ − c c + 2 ,− + 2 ,1 ∂x a2 b ∂y a2 b   −cy −cx   , q , 1 =  q 2 y 2 +b2 x2 2 y 2 +b2 x2 a a a2 b2 a2 b2 a2 b2 (x,y)=(0,±b) (x,y)=(0,±b)  c  = 0, ∓ , 1 b para la superficie  2  b2 + c 2 b2 x +y + z− = 2 b2 + c 2 c c 2

se tiene

2

r 2 2 2 b + c = b (b2 + c2 ) − (x2 + y 2 ) z − c c2

luego b2 + c 2 =± z− c

r

b2 2 (b + c2 ) − (x2 + y 2 ) c2

evaluando en (0, ±b, c) se tiene b2 + c 2 c− c

r = ±

b2 2 (b + c2 ) − b2 c2

⇔ 2

b +c c

c−

2

= ±

b2 c

se sigue que el signo correcto es −. Luego r b2 + c 2 b2 2 z= − (b + c2 ) − (x2 + y 2 ) c c2 as´ı el normal corresponde a (−zx , −zy , 1)|(0,±b) donde zx =

∂ ∂x

= q

2

2

b +c − c x

r

b2 c2

b4 +b2 c2 −c2 x2 −c2 y 2 c2

363

! (b2 + c2 ) − (x2 + y 2 )

Apuntes Mat023 (versi´on preliminar actualizada 23-05-2014)

zy =

∂ ∂y

= q

2

2

b +c − c y

r

b2 c2

! (b2 + c2 ) − (x2 + y 2 )

b4 +b2 c2 −c2 x2 −c2 y 2 c2

evaluando zx = 0 zy = q

±b b4 +b2 c2 −c2 (±b)2 c2

=

±b



b2 c

c b

as´ı (−zx , −zy , 1)|(0,±b)  c  = 0, ∓ , 1 b se sigue que tienen los mismos planos tangentes. ◦

Teorema 9.3.2. Sean U ⊆ Rn y f : U → R una funci´on diferenciable en a ∈ U , entonces f es continua en a. Demostraci´on. Dado ε > 0, existe δ > 0 tal que: n X ∂f f (a + h) − f (a) − (a) · h i ≤ ε khk ∂xi i=1

para todo khk < δ. Entonces, por la desigualdad triangular, tenemos que: n X ∂f |f (a + h) − f (a)| ≤ ε khk + (a) · hi ∂x i i=1 n X ∂f (a) ≤ ε khk + khk ∂xi i=1

n ∂f donde A = m´ax ∂x i

≤ (ε + n A) khk o  (a) : i = 1, 2, . . . , n . Ahora bien, eligiendo δ˜ = m´ın δ,

obtiene que: khk < δ˜ =⇒ |f (a + h) − f (a)| < ε

364

ε ε+n A



se

Apuntes Mat023 (versi´on preliminar actualizada 23-05-2014)

Observaci´ on 9.3.6. Como ya es sabido desde el c´alculo diferencial en una variable, el teorema anterior dice que la continuidad es una condici´on necesaria para la diferenciabilidad, pero no es suficiente. En efecto, considere: Ejemplo 9.3.7. Verifique que f : R2 → R definida por:  3/2   x |y| , si (x, y) 6= (0, 0) x2 + y 2 f (x, y) =   0 , si (x, y) = (0, 0) es continua, pero no diferenciable en (0, 0). Soluci´ on. La continuidad es inmediata. En efecto: |xy| p |x| |y|3/2 = |y| x2 + y 2 x2 + y 2 1 x2 + y 2 p |y| = ≤ 2 x2 + y 2

p |y| 2

donde la u´ltima expresi´on converge a 0, cuando (x, y) → (0, 0). Ahora bien, por el Teorema del Sandwich, se obtiene que: x |y|3/2 l´ım = 0 = f (0, 0) (x,y)→(0,0) x2 + y 2 Por tanto, f es continua en (0, 0). Por otro lado, es f´acil ver que: ∂f ∂f (0, 0) = (0, 0) = 0 ∂x ∂y Luego: (0, 0) h − f (h, k) − f (0, 0) − ∂f ∂x √ l´ım (h,k)→(0,0) h2 + k 2

∂f ∂y

(0, 0) k

=

=

|f (h, k)| √ (h,k)→(0,0) h2 + k 2 l´ım

l´ım (h,k)→(0,0)

|h | |k|3/2 (h2 + k 2 )3/2

Al tomar la recta de aproximaci´on h = k en el u ´ltimo l´ımite se observa que no existe. Por tanto, la funci´on f no es diferenciable en el (0, 0). ´ Teorema 9.3.3 (Algebra de funciones diferenciables). Sean f, g : U ⊆ Rn → R funciones ◦

diferenciables en a ∈ U y α ∈ R una constante, entonces αf, f + g, f − g, f g son diferenciables en a. Si, adem´as, g (a) 6= 0, entonces f /g tambi´en es diferenciable en a. 365

Apuntes Mat023 (versi´on preliminar actualizada 23-05-2014) ◦

Teorema 9.3.4. Sean f : U ⊆ Rn → R y a ∈ U . Si las derivadas parciales

∂f ∂xi

existen en

una bola alrededor de a y son continuas en a entonces f es diferenciable en a. Observaci´ on 9.3.7. La condici´on en el teorema anterior es s´olo suficiente, mas no necesaria como puede verse con el siguiente ejemplo: Ejemplo 9.3.8. Verificar que la funci´on f : R2 → R definida por:  !  1   (x2 + y 2 ) sin p , si (x, y) 6= (0, 0) x2 + y 2 f (x, y) =    0 , si (x, y) = (0, 0) es diferenciable en todo punto de R2 pero las funciones derivadas parciales no son continuas en R2 . Soluci´ on. Del a´lgebra de funciones diferenciables se obtiene que f es diferenciable en todo punto distinto de (0, 0). Notemos que para (x, y) 6= (0, 0) se cumple !  1 ∂ x2 + y 2 sin p ∂x x2 + y 2 ! !    1 1 −1 2 2 2 2 −3/2 = 2x sin p + x + y cos p x +y 2x 2 x2 + y 2 x2 + y 2 ! ! x 1 1 −p = 2x sin p cos p x2 + y 2 x2 + y 2 x2 + y 2 y por simetr´ıa  ∂ x2 + y 2 sin ∂y = 2y sin

1

!

p x2 + y 2 ! 1 y p −p cos x2 + y 2 x2 + y 2

1 p x2 + y 2

en (0, 0) calculamos las derivadas parciales f (h, 0) − f (0, 0) ∂f (0, 0) = l´ım h→0 ∂x h h2 sin

= l´ım

h→0

= l´ım h sin h→0

= 0 366

h 

1 |h|

1 |h|



!

Apuntes Mat023 (versi´on preliminar actualizada 23-05-2014)

y por simetr´ıa

∂f ∂y

(0, 0) = 0. La funci´on derivada parcial esta definida por      1 x 1   2x sin √ 2 2 − √ 2 2 cos √ 2 2 si (x, y) 6= (0, 0)   x +y x +y x +y

∂f (x, y) =  ∂x   

0

si (x, y) = (0, 0)

esta funci´on no es continua en (0, 0), en efecto, considere los puntos   1 (xn , yn ) = ,0 2nπ + π4 entonces si n → +∞, (xn , yn ) → (0, 0) pero 2 ∂f (xn , yn ) = ∂x 2nπ + 2 = 2nπ +

π 4 π 4

sin 1 √ 2

 π π 2nπ + − cos 2nπ + 4 4 1 −√ 2



luego cuando n → +∞ se sigue (xn , yn ) → (0, 0) pero

∂f ∂x

(xn , yn ) → − √12 , el l´ımite no es

cero (usando otra sucesi´on se muestra que en realidad el l´ımite no existe por ejemplo con   1 (f xn , yen ) = 2nπ+ π , 0 da otro l´ımite). Se sigue que f no es de clase C 1 sin embargo 3

|f (x, y)| p (x,y)→(0,0) x2 + y 2 l´ım

  2 (x + y 2 ) sin √ 1 x2 +y 2 p = l´ım (x,y)→(0,0) x2 + y 2 ! p 1 = l´ım x2 + y 2 sin p 2 2 (x,y)→(0,0) x +y = 0

aqu´ı hemos usado el acotamiento p 2 2 0 ≤ x + y sin

! p p ≤ x2 + y 2 2 2 x +y 1

luego f es diferenciable en (0, 0) y en todos los otros puntos por ´algebra de funciones diferenciables pero f no tiene derivadas parciales continuas en el origen..

Ejercicios de la secci´ on 1. Hallar la ecuaci´on del plano tangente a la superficie z = x2 + y 3 en el punto (3, 1, 10). 367

Apuntes Mat023 (versi´on preliminar actualizada 23-05-2014)

2. ¿Por qu´e deben llamarse tangentes en (0, 0) las gr´aficas de f (x, y) = x2 + y 2 y g (x, y) = −x2 − y 2 + xy 3 ? 3. Sea f : R2 → R definida por

f (x, y) =

   

x2 y 4 x2 +y 2

si (x, y) 6= (0, 0)

  

0

si (x, y) = (0, 0)

Probar que f es de clase C 1 (R2 ). 4. Sea p ≥ 1. Estudiar la diferenciabilidad en el origen de la funci´on k·kp

:

x→ kxkp =

Rn → R n X

!1/p p

|xi |

k=1

5. ¿Para que valores de α la funci´on     fα (x, y) =   

x3 +xy 2 |x|α +|y|α

si (x, y) 6= (0, 0)

0

si (x, y) = (0, 0)

es diferenciable en (0, 0)? 6. Considere la superficie definida por     x 3 S = (x, y, z) ∈ R : x sin −z =0 y Si (a, b, c) ∈ S determine el plano tangente a la superficie en ese punto y mostrar que pasa por el origen. 7. Utilizar la aproximaci´on mediante el plano tangente para estimar el valor de: 10

a) (0,99e0,1 ) q b) (3,99)2 + (4,01)2 + (2,01)2 8. Sean α > 1 y f : Rn → R, x →f (x) una funci´on tal que, para todo x, y ∈ Rn |f (x) − f (y)| ≤ kx − ykα Muestre que f es diferenciable.

368

Apuntes Mat023 (versi´on preliminar actualizada 23-05-2014)

Derivadas de orden superior y funciones de clase C n Observaci´ on 9.4.1. Sean U ⊆ Rn un conjunto abierto no vac´ıo y f : U → R una funci´on ∂f ∂xi

tal que las derivadas parciales

(x) existen para todo x ∈ U . Entonces, podemos definir ∂f ∂xi

la funci´on derivada parcial de f como la funci´on

: U ⊆ Rn → R dada por x 7→

∂f ∂xi

(x).

Las derivadas parciales de orden superior son, entonces, derivadas parciales de la funci´on ∂f ∂xi

: U ⊆ Rn → R. M´as precisamente:

Definici´ on 9.4.1. Sean U ⊆ Rn un conjunto abierto no vac´ıo y f : U → Rm una funci´on tal que las derivadas parciales

∂f ∂xi

(x) existen para todo x ∈ U . Se define la derivada parcial

de segundo orden para f como: ∂ ∂ 2f (x) = ∂xj ∂xi ∂xj



∂f ∂xi

 (x)

M´as precisamente: 1 ∂ 2f (x) = l´ım t→0 ∂xj ∂xi t Adem´as, si i 6= j la derivada anotamos

∂2f ∂x2i

∂2f ∂xj ∂xi



 ∂f ∂f (x + tej ) − (x) ∂xi ∂xi

(x) recibe el nombre de derivada parcial mixta. Si i = j,

(x).

La derivada parcial de tercer orden para f se define como:  2  ∂ 3f ∂ ∂ f (x) = (x) ∂xk ∂xj ∂xi ∂xk ∂xj ∂xi considerando

∂2f ∂xj ∂xi

: U ∗ ⊆ Rn → R, en donde U ∗ corresponde al conjunto de puntos de

U para los cuales la segunda derivada parcial est´a definida. En particular, si i = j = k, anotamos entonces:

∂ 3f (x) ∂x3i

Finalmente, las derivadas parciales de orden superior se definen como las derivadas parciales sucesivas de

∂f ∂xi

: U ⊆ Rn → R.

Ejemplo 9.4.1. Hallar las derivadas parciales de segundo orden de la funci´on: f (x, y) = arctan

369

x y

Apuntes Mat023 (versi´on preliminar actualizada 23-05-2014)

Soluci´ on.   ∂f ∂ x y = arctan = 2 ∂x ∂x y x + y2   ∂ x x ∂f = arctan =− 2 ∂y ∂y y x + y2 y las de segundo orden ∂ 2f ∂x2 ∂ 2f ∂y∂x ∂ 2f ∂x∂y ∂ 2f ∂y 2

= = = =

  y −2xy ∂ = 2 2 2 ∂x x + y (x + y 2 )2   x2 − y 2 y ∂ = ∂y x2 + y 2 (x2 + y 2 )2   ∂ x x2 − y 2 − 2 = ∂x x + y2 (x2 + y 2 )2   ∂ x 2xy − 2 = 2 ∂y x +y (x2 + y 2 )2

notar que, en este caso,

∂ 2f ∂ 2f + =0 ∂x2 ∂y 2

y

∂ 2f ∂ 2f = ∂y∂x ∂x∂y

Ejemplo 9.4.2. Hallar

∂ 3z , si: ∂x∂y 2 z = sin xy

Soluci´ on. zy =

∂ (sin xy) = x cos xy ∂y

luego zyy =

∂ (x cos xy) = −x2 sin xy ∂y

y finalmente zyyx =

 ∂ −x2 sin xy = −2x sin xy − x2 y cos xy ∂x

Definici´ on 9.4.2. Sean U ⊆ Rn un conjunto abierto no vac´ıo. Diremos que la funci´on f : U → R es de clase C n sobre U, o bien que f ∈ C n (U ) si todas las derivadas parciales hasta el orden n−´esimo son funciones continuas en U .

370

Apuntes Mat023 (versi´on preliminar actualizada 23-05-2014)

Observaci´ on 9.4.2. A modo de resumen, tenemos que: f ∈ C 1 (U ) =⇒ f es diferenciable en U =⇒ f es continua en U Teorema 9.4.1 (Schwarz). Sean U ⊆ Rn un conjunto abierto no vac´ıo y f ∈ C 2 (U ), entonces, para todo x ∈ U y para todo i 6= j ∂ 2f ∂ 2f (x) = (x) ∂xj ∂xi ∂xi ∂xj Observaci´ on 9.4.3. Del teorema anterior y bajo la hip´otesis de clase C n adecuada se puede establecer igualdad de derivadas parciales mixtas de tercer orden y superior, note por ejemplo que   ∂2 ∂f ∂ 3f = 2 ∂y∂x ∂y∂x ∂x   ∂2 ∂f = ∂x∂y ∂x  2  ∂ f ∂ = ∂x ∂y∂x  2  ∂ ∂ f = ∂x ∂x∂y ∂ 3f = ∂x2 ∂y si f ∈ C 3 . Observaci´ on 9.4.4. La continuidad de las derivadas parciales de segundo orden es fundamental en el teorema anterior. Esto se puede ilustrar considerando el siguiente ejemplo: Ejemplo 9.4.3. Sea f : R2 → R la funci´on definida por:     xy x22 −y22 si (x, y) 6= (0, 0) x +y f (x, y) =  0 si (x, y) = (0, 0) Verifique que:

∂ 2f ∂ 2f (0, 0) 6= (0, 0) ∂y∂x ∂x∂y

Soluci´ on. Note que   2  ∂f ∂ x − y2 (x, y) = xy ∂x ∂x x2 + y 2 y (x4 + 4x2 y 2 − y 4 ) = (x2 + y 2 )2 371

Apuntes Mat023 (versi´on preliminar actualizada 23-05-2014)

para (x, y) 6= (0, 0) y ∂f f (h, 0) − f (0, 0) (0, 0) = l´ım =0 h→0 ∂x h luego   y(x4 +4x2 y2 −y4 ) si (x, y) 6= (0, 0) ∂f (x2 +y 2 )2 (x, y) =  ∂x 0 si (x, y) = (0, 0) se sigue que   1 ∂f ∂f ∂ 2f (0, 0) = l´ım (0, h) − (0, 0) h→0 h ∂y∂x ∂x ∂x   1 h (0 − h4 ) = l´ım h→0 h (02 + h2 )2 = −1 por otro lado   2  ∂f ∂ x − y2 (x, y) = xy ∂y ∂y x2 + y 2 −x (−x4 + 4x2 y 2 + y 4 ) = (x2 + y 2 )2 y ∂f (0, 0) = 0 ∂y se sigue   ∂ 2f 1 ∂f ∂f (0, 0) = l´ım (h, 0) − (0, 0) h→0 h ∂x∂y ∂y ∂y   1 −h (−h4 ) = l´ım h→0 h (h2 + 02 )2 = 1 se sigue

∂2f ∂y∂x

(0, 0) 6=

∂2f ∂x∂y

(0, 0) en este caso la funci´on no es de clase C 2 (R2 ).

Ejemplo 9.4.4. Si u = ln (x2 + y 2 ) verificar que ∂ 2u ∂ 2u + =0 ∂x2 ∂y 2 Soluci´ on. Derivando  ∂u ∂ 2x ln x2 + y 2 = 2 = ∂x ∂x x + y2  ∂u ∂ 2y = ln x2 + y 2 = 2 ∂y ∂y x + y2 372

Apuntes Mat023 (versi´on preliminar actualizada 23-05-2014)

y de segundo orden   ∂ 2u 2x x2 − y 2 ∂ = −2 = ∂x2 ∂x x2 + y 2 (x2 + y 2 )2   ∂ 2u 2y x2 − y 2 ∂ = 2 = ∂y 2 ∂y x2 + y 2 (x2 + y 2 )2 as´ı

∂ 2u ∂ 2u x2 − y 2 x2 − y 2 + 2 =0 + = −2 ∂x2 ∂y 2 (x2 + y 2 )2 (x2 + y 2 )2

Ejercicios de la secci´ on   p −3/2 1. Si u = arctan xy/ 1 + x2 + y 2 probar que uxy = (1 + x2 + y 2 ) y uxxyy =

15xy (1 + x2 + y 2 )7/2

2. Sean a, b, c constantes positivas. Si z = z (x, y) es tal que a2 x2 + b2 y 2 = c2 z 2 entonces muestre que zxx zyy = (zxy )2 3. Si u = ln (x2 + y 2 + z 2 ) probar que xuyz = yuzx = zuxy 4. Si f (x, y) =

xy x+y

demuestre que x2

2 ∂ 2f ∂ 2f 2∂ f + 2xy + y =0 ∂x2 ∂x∂y ∂y 2

Gradiente y matriz jacobiana Definici´ on 9.5.1. Sean U ⊆ Rn y f : U → R una funci´on tal que todas las derivadas parciales

∂f ∂xi



(a), i = 1, 2, . . . , n de f existen en a ∈ U . Se define el gradiente de f en a

como el vector en Rn dado por:  ∇f (a) =

 ∂f ∂f ∂f (a) , (a) , . . . , (a) ∂x1 ∂x2 ∂xn 373

Apuntes Mat023 (versi´on preliminar actualizada 23-05-2014)

Ejemplo 9.5.1. Sea f (x, y, z) = 3x2 + sin y 2 + 2xz 2 . Calcule ∇f (x, y, z). Soluci´ on. En este caso  ∇f (x, y, z) =

∂f (x, y, z) ∂f (x, y, z) ∂f (x, y, z) , , ∂x ∂y ∂z



es decir ∇f (x, y, z) = 2z 2 + 6x, 2y cos y 2 , 4xz



Ejemplo 9.5.2. Considere f (x, y, z) = 3x2 + 2xy − 4z 3 . Sean S la superficie dada por: S : f (x, y, z) = 1 y Π el plano tangente a S en (1, 1, 1). Si u, v ∈ Π linealmente independientes, entonces pruebe que existe λ ∈ R tal que: u × v = λ · ∇f (1, 1, 1) Soluci´ on. Si u, v ∈ Π son linealmente independientes entonces u × v es normal al plano, pero de las secciones anteriores sabemos que el normal a la gr´afica de una funci´on z = h (x, y) tiene la forma k (−hx , −hy , 1) donde k es una constante, la superficie f (x, y, z) = 1 la podemos mirar como la gr´afica de r z=

3

1 3 2 1 x + yx − 4 2 4

se sigue (−zx , −zy , 1) q   1 1 3 3 2 3 1 x + yx − x + 2y 2√ 4 2 4 3 2 = − 2 , 1 2 , −2x 2 3 9x + 6yx − 3 (3x2 + 2yx − 1) 3 evaluando en (1, 1) 

r

3 2

1 2

q 3

3 4

+ 12 −

1 4



+ 43 3 1 1 (−zx , −zy , 1) = − + − , −2 , 1 3 4 2 43+2−1 9+6−3   2 1 = − ,− ,1 3 6 374

Apuntes Mat023 (versi´on preliminar actualizada 23-05-2014)

note que ∇f (x, y, x) = 6x + 2y, 2x, −12z 2



luego ∇f (1, 1, 1) = (6 + 2, 2, −12) = (8, 2, −12) se sigue que existe λ ∈ R tal que   2 1 u × v =k − , − , 1 = λ∇f (1, 1, 1) 3 6 Observaci´ on 9.5.1. El isomorfismo can´onico ϕ : Rn ' M1×n (R) vuelve a jugar un rol importante en relacionar el vector gradiente ∇f (a) y la matriz de la diferencial Df (a). Sabemos que la diferencial de una funci´on f : Rn → R est´a dada por la transformaci´on lineal Df (a) : Rn → R tal que: |f (a + h) − f (a) − Df (a) · h| =0 h→0 khk l´ım

pero ϕ−1 (Df (a)) = ∇f (a), por lo tanto, podemos escribir: |f (a + h) − f (a) − ∇f (a) · h| =0 h→0 khk l´ım

Observaci´ on 9.5.2. Nuestro prop´osito, ahora, es tratar la diferenciabilidad para funciones vectoriales de la forma f : U ⊆ Rn → Rm .. Teniendo en cuenta que la topolog´ıa del espacio euclidiano Rm se obtiene considerando la norma k·kRm , tenemos que: Definici´ on 9.5.2. Sean U ⊆ Rn y f : U → Rm una funci´on vectorial. Diremos que f es ◦

diferenciable en a ∈ U si existe una transformaci´on lineal T : Rn → Rm tal que: kf (a + h) − f (a) − T (h)k =0 h→0 khk l´ım

Definici´ on 9.5.3. Sean U ⊆ Rn y f : U → Rm una funci´on tal que todas las derivadas ∂fi parciales existen en a. Llamaremos matriz jacobiana de f en a a la matriz de derivadas ∂xj

375

Apuntes Mat023 (versi´on preliminar actualizada 23-05-2014)

parciales:  J f (a) =

 ∂fi (a) ∂xj m×n

      =      

∂f1 (a) ∂x1 ∂f2 (a) ∂x1 .. .

 ∂f1 ∂f1 (a) · · · (a)  ∂x2 ∂xn   ∂f2 ∂f2 (a) · · · (a)   ∂x2 ∂xn  ∈ Mm×n (R)  .. .. ..  . . .    ∂fm ∂fm ∂fm (a) (a) · · · (a) ∂x1 ∂x2 ∂xn

Ejemplo 9.5.3. Sea f : R2 → R2 definida por f (x, y) = (2x2 y + y, 2xy − y 2 ). Luego, si anotamos u = 2x2 y + y y v = 2xy − y 2 , entonces  ∂u  ∂x Jf (x, y) =   ∂v ∂x  4xy =  2y

la matriz jacobiana de f est´a dada por:  ∂u ∂y   ∂v  ∂y 2x

2

2x − 2y

 

Teorema 9.5.1. Sean f : U ⊆ Rn → Rm una funci´on definida por f (x) = (f1 (x) , f2 (x) , . . . , fm (x)) ◦

y a ∈ U . f es diferenciable en a, si y solo si, fi : U → R es diferenciable en a, para cada i = 1, 2, . . . , m. Adem´as: 

∇f1 (a)

  ∇f (a) 2  Df (a) =  ..  .  ∇fm (a) ϕ

donde ∇fi (a) ∼



∂fi ∂x1

(a)

∂fi ∂x2 n

(a) · · ·

     ∂fi  (a) = Jf (a) =  ∂xj m×n 

∂fi ∂xn

(a)



, para cada i = 1, 2, . . . , m, mediante el

isomorfismo can´onico ϕ : R → M1×n (R). Ejemplo 9.5.4. ¿Es diferenciable g (x, y, z) = x + sin x cos y 2 , x2 y + 1,



 z 2 + 1 en R3 ?

Soluci´ on. La respuesta es si, pues las funciones componentes son todas diferenciables en R3 . 376

Apuntes Mat023 (versi´on preliminar actualizada 23-05-2014)

Observaci´ on 9.5.3. Por definici´on, tenemos que si f : U ⊆ Rn → Rm es diferenciable en a, entonces existe una transformaci´on lineal T : Rn → Rm tal que: kf (a + h) − f (a) − T (h)k =0 h→0 khk l´ım

Adem´as, note que para cada i = 1, 2, . . . , m, se tiene que: |fi (a + h) − fi (a) − Ti (h)| ≤ kf (a + h) − f (a) − T (h)k para todo h. As´ı, si h = t · ej , tenemos que: fi (a + t · ej ) − fi (a) − Ti (ej ) 0 ≤ t fi (a + t · ej ) − fi (a) − Ti (tej ) = t kf (a + t · ej ) − f (a) − T (tej )k ≤ →0 ktej k si acaso t → 0. Lo anterior implica que: [Ti ]j =

∂fi (a) ∂xj

en donde [Ti ]j representa el j−´esimo coeficiente de la matriz [Ti ], respecto de las bases can´onicas. Todo lo anterior implica que en caso de que f sea diferenciable en a, la matriz jacobiana Jf (a) es la matriz de la diferencial Df (a). Desde ahora en adelante, anotamos: Df (a) = Jf (a)

La regla de la cadena Teorema 9.6.1. Sean f : U ⊆ Rn → Rm , con U abierto y g : V ⊆ Rm → Rp , con V abierto, tales que f (U ) ⊆ V . Suponga que f es diferenciable en a ∈ U y que g es diferenciable en b = f (a) ∈ V , entonces: g ◦ f : U ⊆ Rn → Rp es diferenciable en a. Adem´as: D (g ◦ f ) (a) = Dg (f (a)) Df (a) = Dg (b) Df (a) 377

Apuntes Mat023 (versi´on preliminar actualizada 23-05-2014)

Observaci´ on 9.6.1. Es importante destacar que Dg (b) ∈ Mp×m (R) y que Df (a) ∈ Mm×n (R). Observaci´ on 9.6.2. Supongamos que z = g (f (x)) e y = f (x), con b = f (a), entonces la regla de la cadena en t´erminos de las matrices jacobianas queda en la forma: D (g ◦ f ) (a) = Dg (b) · Df (a)  ∂z ∂z1 1 (b) (b)  ∂y1 ∂y 2   ∂z2 ∂z2   ∂y1 (b) ∂y2 (b) =   .. ..  . .    ∂z ∂zp p (b) (b) ∂y1 ∂y2 =

∂z1 (b) ∂ym ∂z2 ··· (b) ∂ym .. ... . ···

···

! m X ∂yk ∂zi (b) (a) ∂y ∂x k j k=1

∂zp (b) ∂ym

            ·          

∂y1 (a) ∂x1 ∂y2 (a) ∂x1 .. .

 ∂y1 ∂y1 (a) · · · (a)  ∂x2 ∂xn   ∂y2 ∂y2 (a) · · · (a)   ∂x2 ∂x2   .. .. ..  . . .    ∂ym ∂ym ∂ym (a) (a) · · · (a) ∂x1 ∂x2 ∂xn

∈ Mp×n (R) i×j

Por tanto, en notaciones cl´asicas, mirando las componentes del producto, la regla viene dado por las f´ormulas: ∂zi ∂xj

=

m X ∂zi ∂yk ∂yk ∂xj k=1

=

∂zi ∂y1 ∂zi ∂y2 ∂zi ∂ym + + ··· + ∂y1 ∂xj ∂y2 ∂xj ∂ym ∂xj

para i = 1, 2, . . . , p y j = 1, 2, . . . , n. Ejemplo 9.6.1. Sean g : R3 → R y f : R3 → R3 tales que: f (x, y, z) = u (x, y, z) ; v (x, y, z) ; w (x, y, z) Definamos F : R3 → R mediante la ecuaci´on: F (x, y, z) = (g ◦ f ) (x, y, z)

378



Apuntes Mat023 (versi´on preliminar actualizada 23-05-2014)

Por la regla de la cadena: DF (x, y, z) = Dg (u, v, w) · Df (x, y, z)  ∂u     ∂x  ∂v ∂g ∂g ∂g = ·  ∂x ∂u ∂v ∂w   ∂w ∂x

∂u ∂y ∂v ∂y ∂w ∂y

∂u ∂z ∂v ∂z ∂w ∂z

       

As´ı, por ejemplo: ∂F ∂g ∂u ∂g ∂v ∂g ∂w = + + ∂x ∂u ∂x ∂v ∂x ∂w ∂x Ejemplo 9.6.2. Sea g : R3 → R y consideremos el cambio de variables a coordenadas esf´ericas: ϕ (r, θ, φ) :

    x = r cos θ sin φ y = r sin θ sin φ

  

z = r cos φ  Defina z (r, θ, φ) = g x (r, θ, φ) , y (r, θ, φ) , z (r, θ, φ) . Calcule

∂z ∂z , ∂r ∂θ

y

∂z . ∂φ

Soluci´ on. ∂g ∂x ∂g ∂y ∂g ∂z ∂z = + + ∂r ∂x ∂r ∂y ∂r ∂z ∂r ∂g ∂g ∂g = (cos θ sin φ) + (sin θ sin φ) + (cos φ) ∂x ∂y ∂z ∂z ∂g ∂x ∂g ∂y ∂g ∂z = + + ∂θ ∂x ∂θ ∂y ∂θ ∂z ∂θ ∂g ∂g ∂g = (−r sin θ sin φ) + (r cos θ sin φ) + (0) ∂x ∂y ∂z ∂g ∂g = (−r sin θ sin φ) + (r cos θ sin φ) ∂x ∂y ∂z ∂g ∂x ∂g ∂y ∂g ∂z = + + ∂φ ∂x ∂φ ∂y ∂φ ∂z ∂φ ∂g ∂g ∂g = (r cos θ cos φ) + (r sin θ cos φ) + (−r sin φ) ∂x ∂y ∂z

379

Apuntes Mat023 (versi´on preliminar actualizada 23-05-2014)

Ejemplo 9.6.3. Considere: w = x3 yez Suponga que:  2 2    x=s −t y = s2 + t2

  

z =s+t

Pruebe que: ∂w ∂w (1, 0) + (1, 0) = 10e ∂s ∂t Soluci´ on. Usando la regla de la cadena ∂w ∂x ∂w ∂y ∂w ∂y ∂w = + + ∂s ∂x ∂s ∂y ∂s ∂y ∂s ∂w ∂w ∂x ∂w ∂y ∂w ∂y = + + ∂t ∂x ∂t ∂y ∂t ∂y ∂t recordar que las derivadas de w quedan evaluadas en x = s2 − t2 , y = s2 + t2 , z = s + t as´ı  ∂x ∂w ∂w 2 (s, t) = s − t2 , s2 + t2 , s + t (s, t) ∂s ∂x ∂s  ∂y ∂w 2 + (s, t) s − t2 , s2 + t2 , s + t ∂y ∂s  ∂y ∂w 2 + s − t2 , s2 + t2 , s + t (s, t) ∂y ∂s y  ∂x ∂w ∂w 2 (s, t) = s − t2 , s2 + t2 , s + t (s, t) ∂t ∂x ∂t  ∂y ∂w 2 + s − t2 , s2 + t2 , s + t (s, t) ∂y ∂t  ∂y ∂w 2 + s − t2 , s2 + t2 , s + t (s, t) ∂y ∂t evaluando en (1, 0) se obtiene ∂w ∂w ∂x (1, 0) = (1, 1, 1) (1, 0) + ∂s ∂x ∂s ∂w ∂w ∂x (1, 0) = (1, 1, 1) (1, 0) + ∂t ∂x ∂t

∂w ∂y (1, 1, 1) (1, 0) + ∂y ∂s ∂w ∂y (1, 1, 1) (1, 0) + ∂y ∂t

380

∂w ∂z (1, 1, 1) (1, 0) ∂z ∂s ∂w ∂z (1, 1, 1) (1, 0) ∂z ∂t

Apuntes Mat023 (versi´on preliminar actualizada 23-05-2014)

note que ∂w (x, y, z) = ∂x ∂w (x, y, z) = ∂y ∂w (x, y, z) = ∂z

3x2 yez x3 e z





x3 yez



evaluando ∇w (1, 1, 1) = (3e, e, e) por otro lado ∂x ∂x (s, t) = 2s y (s, t) = −2t ∂s ∂t ∂y ∂y (s, t) = 2s y (s, t) = 2t ∂s ∂t ∂z ∂z (s, t) = 1 y (s, t) = 1 ∂s ∂t evaluando ∂x ∂x (1, 0) = 2 y (1, 0) = 0 ∂s ∂t ∂y ∂y (1, 0) = 2 y (1, 0) = 0 ∂s ∂t ∂z ∂z (1, 0) = 1 y (1, 0) = 1 ∂s ∂t se sigue ∂w ∂w ∂x ∂w ∂y ∂w ∂z (1, 0) = (1, 1, 1) (1, 0) + (1, 1, 1) (1, 0) + (1, 1, 1) (1, 0) ∂s ∂x ∂s ∂y ∂s ∂z ∂s = (3e) (2) + (e) (2) + (e) (1) = 9e y ∂w ∂w ∂x ∂w ∂y ∂w ∂z (1, 0) = (1, 1, 1) (1, 0) + (1, 1, 1) (1, 0) + (1, 1, 1) (1, 0) ∂t ∂x ∂t ∂y ∂t ∂z ∂t = (3e) (0) + (e) (0) + (e) (1) = e se sigue ∂w ∂w (1, 0) + (1, 0) = 10e ∂s ∂t 381

Apuntes Mat023 (versi´on preliminar actualizada 23-05-2014)

Ejemplo 9.6.4. Sea w = f (r) una funci´on diferenciable y r =

p

x2 + y 2 + z 2 . Demuestre

que: 

dw dr

2

= k∇f k2

Soluci´ on. La derivada de w respecto a r es dw = f 0 (r) dr luego 2 dw 2 = (f 0 (r)) dr por otro lado, usando la regla de la cadena 

x r y 0 0 = f (r) ry = f (r) r z 0 0 = f (r) rz = f (r) r

wx = f 0 (r) rx = f 0 (r) wy wz se sigue

 x 2  0 y 2  0 z 2 f 0 (r) + f (r) + f (r) r r r  0 2  f (r) = x2 + y 2 + z 2 r 2  0  f (r) r2 = r

k∇f k2 =

2

= (f 0 (r)) como se quer´ıa probar.

Ejemplo 9.6.5. Sean f : [0, +∞) → R una funci´on diferenciable y u : R → R definida por: u = f k(x, y, z)k2 donde:



    x = ρ cos ϕ cos ψ y = ρ cos ϕ sin ψ

   Demuestre que

z = ρ sin ϕ

∂u = 0. ∂ϕ 382

Apuntes Mat023 (versi´on preliminar actualizada 23-05-2014)

Soluci´ on. Usando la regla de la cadena, note que u (ρ, ϕ, ψ) = f (g (h (ρ, ϕ, ψ))) donde p → f (p) (x, y, z) → g (x, y, z) = x2 + y 2 + z 2 (ρ, ϕ, ψ) → h (ρ, ϕ, ψ) = (ρ cos ϕ cos ψ, ρ cos ϕ sin ψ, ρ sin ϕ) se sigue ∂g ∂x ∂g ∂y ∂g ∂z ∂u = f0 + f0 + f0 ∂ϕ ∂x ∂ϕ ∂y ∂ϕ ∂z ∂ϕ  0 2 = f ρ (2ρ cos ϕ cos ψ) (−ρ sin ϕ cos ψ)  +f 0 ρ2 (2ρ cos ϕ sin ψ) (−ρ sin ϕ sin ψ)  +f 0 ρ2 (2ρ sin ϕ) (ρ cos ϕ)   = 2ρf 0 ρ2 − cos ϕ sin ϕ cos2 ψ − cos ϕ sin ϕ sin2 ψ + sin ϕ cos ϕ  = 2ρf 0 ρ2 (0) = 0 otra forma de verlo es u = f k(x, y, z)k2 con



    x = ρ cos ϕ cos ψ y = ρ cos ϕ sin ψ

  

z = ρ sin ϕ

queda u = f ρ2



luego la derivada es 0. Ejemplo 9.6.6. Cambiar las variables x, y, z en la ecuaci´on x

∂u ∂u ∂u +y +z = nu ∂x ∂y ∂z

por ξ, η, ϕ de la forma ξ=

x y ,η= yϕ=z z z

383

Apuntes Mat023 (versi´on preliminar actualizada 23-05-2014)

y probar que la ecuaci´on se transforma en ϕ

∂v = nv ∂ϕ

deducir de esto que u tiene la forma u (x, y, z) = z n F

x y  , z z

para una funci´on F arbitraria. Soluci´ on. Definimos v (ξ (x, y, z) , η (x, y, z) , ϕ (x, y, z)) = u (x, y, z) con el cambio de variable propuesto ∂v ∂ξ ∂v ∂η ∂v ∂ϕ ∂u = + + ∂x ∂ξ ∂x ∂η ∂x ∂ϕ ∂x ∂u ∂v ∂ξ ∂v ∂η ∂v ∂ϕ = + + ∂y ∂ξ ∂y ∂η ∂y ∂ϕ ∂y ∂v ∂ξ ∂v ∂η ∂v ∂ϕ ∂u = + + ∂z ∂ξ ∂z ∂η ∂z ∂ϕ ∂z se sigue   ∂u ∂v 1 ∂v ∂v = (0) + (0) + ∂x ∂ξ z ∂η ∂ϕ   ∂u ∂v ∂v 1 ∂v = (0) + (0) + ∂y ∂ξ ∂η z ∂ϕ ∂u ∂v  x  ∂v  y  ∂v = − 2 + − 2 + (1) ∂z ∂ξ z ∂η z ∂ϕ se sigue que x

∂u ∂u ∂u +y +z = nu ∂x ∂y ∂z

se transforma en ∂v  x  ∂v  y  ∂v  x  ∂v  y  ∂v + + − + − +z = nv ∂ξ z ∂η z ∂ξ z ∂η z ∂ϕ es decir ϕ

∂v = nv ∂ϕ 384

Apuntes Mat023 (versi´on preliminar actualizada 23-05-2014)

esta ecuaci´on es de variables separadas, la resolvemos ln |v| = n ln |ϕ| + C luego v = Kϕn note que la constante es respecto a la variable ϕ, luego puede depender de ξ, η, ponemos v = K (ξ, η) ϕn luego u (x, y, z) = v (ξ (x, y, z) , η (x, y, z) , ϕ (x, y, z)) = K (ξ (x, y, z) , η (x, y, z)) ϕ (x, y, z)n x y  , zn = K z z Observaci´ on 9.6.3. Muchas veces abusando de la notaci´on, en lugar de poner v (ξ (x, y, z) , η (x, y, z) , ϕ (x, y, z)) = u (x, y, z) simplemente se trabaja con u (ξ, η, ϕ) y se escribe ∂u ∂ξ ∂u ∂η ∂u ∂ϕ ∂u = + + ∂x ∂ξ ∂x ∂η ∂x ∂ϕ ∂x Observaci´ on 9.6.4. Otra manera de enfrentar el problema anterior, cuando es posible despejar en forma expl´ıcita es la siguiente, note que ξ=

x y ,η= yϕ=z z z

implica x = ξϕ y = ηϕ z = ϕ luego P (ξ, η, ϕ) = u (x (ξ, η, ϕ) , y (ξ, η, ϕ) , z (ξ, η, ϕ)) 385

Apuntes Mat023 (versi´on preliminar actualizada 23-05-2014)

luego ∂P ∂u ∂x = + ∂ϕ ∂x ∂ϕ ∂u = (ξ) + ∂x

∂u ∂y ∂u ∂z + ∂y ∂ϕ ∂z ∂ϕ ∂u ∂u (η) + ∂y ∂z

luego ϕ

∂u ∂u ∂u ∂P = (ξϕ) + (ηϕ) + ϕ ∂ϕ ∂x ∂y ∂z ∂u ∂u ∂u = x +y +z ∂x ∂y ∂z = nu = nP

de donde P (ξ, η, ϕ) = ϕn K (ξ, η) y as´ı u (x (ξ, η, ϕ) , y (ξ, η, ϕ) , z (ξ, η, ϕ)) = ϕn K (ξ, η) esto es n

u (x, y, z) = z K



x z , y y



Estudiaremos el comportamiento de la regla de la cadena en derivadas de orden superior Ejemplo 9.6.7. Suponiendo z ∈ C 2 (R2 ), reescriba la ecuaci´on diferencial: ∂ 2z ∂2 ∂ 2z +2 −3 2 =0 ∂x2 ∂x∂y ∂y en nuevas variables u y v, definidas por: u = 3x − y y v = x + y. Soluci´ on. Definamos P (u (x, y) , v (x, y)) = z (x, y) entonces zx = Pu ux + Pv vx esto es zx = 3Pu + Pv 386

Apuntes Mat023 (versi´on preliminar actualizada 23-05-2014)

derivando nuevamente respecto a x zxx = 3 (Pu )x + (Pv )x = 3 ((Pu )u ux + (Pu )v vx ) + ((Pv )u ux + (Pv )v vx ) = 3 ((Pu )u 3 + (Pu )v ) + ((Pv )u 3 + (Pv )v ) = 9Puu + 6Puv + Pvv y zxy = (3Pu + Pv )y = 3 (Pu )y + (Pv )y = 3 (Puu uy + Puv vy ) + (Pvu uy + Pvv vy ) = 3 (Puu (−1) + Puv ) + (Pvu (−1) + Pvv ) = −3Puu + 2Puv + Pvv con respecto a la variable y zy = Pu uy + Pv vy = −Pu + Pv derivando nuevamente zyy = − (Puu uy + Puv vy ) + (Pvu uy + Pvv vy ) = − (−Puu + Puv ) + (−Pvu + Pvv ) = Puu − 2Puv + Pvv se sigue que

∂2 ∂ 2z ∂ 2z + 2 − 3 =0 ∂x2 ∂x∂y ∂y 2

queda reescrita como (9Puu + 6Puv + Pvv ) + 2 (−3Puu + 2Puv + Pvv ) − 3 (Puu − 2Puv + Pvv ) = 0 es decir (9 − 6 − 3) Puu + (6 + 4 + 6) Puv + (1 + 2 − 3) Pvv = 0 387

Apuntes Mat023 (versi´on preliminar actualizada 23-05-2014)

as´ı Puv = 0 esta ecuaci´on implica P (u, v) = F (u) + G (v) donde F , G son funciones de clase C 2 de una variable se sigue z (x, y) = P (u (x, y) , v (x, y)) = F (u (x, y)) + G (v (x, y)) = F (3x − y) + G (x + y) como ejercicio, verificar que si z (x, y) = F (3x − y)+G (x + y) entonces

∂2z ∂2 ∂2z +2 ∂x∂y −3 ∂y 2 ∂x2

=

0. Ejemplo 9.6.8. Las ecuaciones u = f (x, y) , x = X (s, t) e y = Y (s, t) definen u como una funci´on de s y t: u = F (s, t). Si las funciones son de clase C 2 pruebe que: 2 2   ∂X ∂Y ∂ 2 f ∂ 2F ∂f ∂ 2 X ∂ 2 f ∂X ∂f ∂ 2 Y ∂ 2 f ∂Y +2 = + 2 + + 2 ∂s2 ∂x ∂s2 ∂x ∂s ∂x ∂s ∂x∂y ∂y ∂s2 ∂y ∂s Soluci´ on. Por la regla de la cadena F s = f x xs + f y y s Fss = (fx )s xs + fx xss + (fy )s ys + fy ys = (fxx xs + fxy ys ) xs + fx xss + (fyx xs + fyy ys ) ys + fy ys = fxx (xs )2 + 2fxy ys xs + fyy (ys )2 + fx xss + fy ys se sigue ∂ 2f ∂ 2F = ∂s2 ∂x2



∂x ∂s

2

∂x ∂y ∂ 2 f ∂ 2f +2 + 2 ∂s ∂s ∂x∂y ∂y



∂y ∂s

2 +

∂f ∂ 2 x ∂f ∂ 2 y + ∂x ∂s2 ∂y ∂s2

Gradiente y planos tangentes Observaci´ on 9.7.1. Sea f : U ⊆ Rn → R una funci´on diferenciable tal que ∇f (a) = 6 0. Consideremos el conjunto de nivel: S : Lc (f ) = {x ∈ U : f (x) = c} 388

Apuntes Mat023 (versi´on preliminar actualizada 23-05-2014)

Suponga que λ : t 7→ λ (t) una curva diferenciable de R → Rn tal que Im(λ) ⊆ S. Note que, en vista de las hip´otesis para la curva λ, tenemos que: f (λ (t)) = c,

∀t

entonces, por la regla de la cadena, tenemos que: n X ∂f ∇f (λ (t)) · λ (t) = (λ (t)) λ0i (t) = 0 ∂x i i=1 0

en particular, si existe t0 ∈ R tal que λ (t0 ) = a, entonces, de la ecuaci´on anterior conclu´ımos que: h∇f (a) , λ0 (t0 )i = 0 como λ es cualquier curva diferenciable tal que se tiene que: ∇f (a) ⊥ S puesto que λ0 es tangente a S. Se tiene entonces: Teorema 9.7.1. Sean U un conjunto abierto no vac´ıo, f : U ⊆ Rn → R una funci´on diferenciable y S : Lc (f ) el conjunto de nivel de c para f . Suponga que que f (a) = c (es decir, a ∈ S) y que ∇f (a) 6= 0 entonces: ∇f (a) ⊥a S donde ⊥a significa que la ortogonalidad de ∇f (a) al conjunto de nivel S : Lc (f ) se da en el punto a. Ejemplo 9.7.1. Hallar un vector normal unitario a la superficie definida por: 2xy 3 z + z ln x + y sin y = 0 en (1, 2π, 0). Soluci´ on. El punto dado pertenece a la superficie 2xy 3 z + z ln x + y sin y = 0 la cual corresponde al conjunto de nivel cero de la funci´on f : D ⊆ R3 → R dada por f (x, y, z) = 2xy 3 z + z ln x + y sin y 389

Apuntes Mat023 (versi´on preliminar actualizada 23-05-2014)

Sabemos que el gradiente es normal a la superficie,   1 3 2 3 z + 2y z, sin y + y cos y + 6xy z, 2xy + ln x ∇f (x, y, z) = x evaluando ∇f (1, 2π, 0) = =

0, 2π, 2 (2π)3  0, 2π, 16π 3



es un vector normal a la superficie en el punto dado. Observaci´ on 9.7.2. Recordemos que si u0 ∈ Rn la ecuaci´on del plano Π que pasa por u0 y que es normal n es dada por: n · (x − u0 ) = 0 Por lo tanto, el plano tangente Π a una superficie S : f (x) = c en a, est´a dado por: Π : ∇f (a) · (x − a) = 0 pues ∇f (a) ⊥a S y a ∈ S Ejemplo 9.7.2. Encontrar el plano tangente a la superficie: x2 + y 2 + z 2 = 1 en (1, 0, 0). Soluci´ on. La superficie es el conjunto de nivel 1 de la funci´on h (x, y, z) = x2 + y 2 + z 2 y el punto (1, 0, 0) pertenece a la superficie, se sigue que el vector normal al plano tangente es dado por ∇h (x, y, z) = (2x, 2y, 2z) as´ı el plano tangente es ∇h (1, 0, 0) · (x − 1, y − 0, z − 0) = 0 es decir (2, 0, 0) · (x − 1, y − 0, z − 0) = 0 luego x=1 es el plano tangente. 390

Apuntes Mat023 (versi´on preliminar actualizada 23-05-2014)

Observaci´ on 9.7.3. En secciones anteriores encontramos la ecuaci´on del plano tangente a la gr´afica de una funci´on diferenciable z = f (x, y) en un punto (x0 , y0 , f (x0 , y0 )) el cual tiene ecuaci´on z = f (x0 , y0 ) +

∂f ∂f (x0 , y0 ) (x − x0 ) + (x0 , y0 ) (y − y0 ) ∂x ∂y

Note que la gr´afica corresponde al conjunto  S = (x, y, z) ∈ R3 : z − f (x, y) = 0 el cual puede ser interpretado como el conjunto de nivel cero de F (x, y, z) = z − f (x, y), luego en el punto (x0 , y0 , f (x0 , y0 )) = (x0 , y0 , z0 ) el plano tangente a S tiene ecuaci´on ∇F (x0 , y0 , z0 ) · (x − x0 , y − y0 , z − z0 ) = 0 pero ∂f ∂F (x0 , y0 , z0 ) = − (x0 , y0 ) ∂x ∂x ∂f ∂F (x0 , y0 , z0 ) = − (x0 , y0 ) ∂y ∂y ∂F (x0 , y0 , z0 ) = 1 ∂z as´ı el plano es 

 ∂f ∂f (x0 , y0 ) , − (x0 , y0 ) , 1 · (x − x0 , y − y0 , z − z0 ) = 0 − ∂x ∂y

es decir ∂f ∂f (x0 , y0 ) (x − x0 ) + (x0 , y0 ) (y − y0 ) ∂x ∂y La propiedad del gradiente entonces nos permite obtener el resultado obtenido por otros z = f (x0 , y0 ) +

m´etodos.

Derivada direccional y direcciones de crecimiento m´ aximo ◦

Definici´ on 9.8.1. Sean a ∈ U , f : U ⊆ Rn → R una funci´on cualquiera y u ∈ Rn un vector unitario en Rn (es decir, un vector tal que kuk = 1). Se define la derivada direccional 391

Apuntes Mat023 (versi´on preliminar actualizada 23-05-2014)

de f en a en la direcci´on de u como el l´ımite: Du (a) =

f (a + tu) − f (a) ∂f (a) = l´ım t→0 ∂u t

si acaso existe. Ejemplo 9.8.1. Calcular la derivada direccional de f : R2 → R, (x, y) → f (x, y) =   1 √1 3 2 3 √ x y + 2x y en el punto a = (1, 2) en la direcci´on u = 2 , 2 Soluci´ on. Por definici´on f (a + tu) − f (a) ∂f (a) = l´ım t→0 ∂u t    − f (1, 2) f (1, 2) + t √12 , √12 = l´ım t→0 t  f 1 + √t2 , 2 + √t2 − f (1, 2) = l´ım t→0 t  3    2  1 + √t2 2 + √t2 + 2 1 + √t2 2+ = l´ım t→0 t 63 √ = 2 (Usando la regla de L’ Hˆopital) 2

√t 2

3

− (2 + 24 )

Observaci´ on 9.8.1. Note que los vectores can´onicos ei , con i = 1, 2, . . . , n, de Rn son vectores unitarios, luego las derivadas parciales son casos particulares de la derivada direccional. M´as precisamente, las derivadas parciales son derivadas direccionales en la direcci´on de los vectores de la base can´onica de Rn . Observaci´ on 9.8.2. Se debe hacer notar, que

∂f ∂u

(a) mide la raz´on de cambio de f en la

direcci´on de la recta vectorial: x = a + tu,

t∈R

as´ı como las derivadas direccionales miden la raz´on de cambio de f es la direcci´on de los ejes coordenados. La interpretaci´on de estas derivadas es la misma. Ejemplo 9.8.2. Sea f : R2 → R la funci´on definida por:  |x| y   p , si x3 − y 2 = 6 0 2 2 x +y f (x, y) =   0 , si x3 − y 2 = 0 392

Apuntes Mat023 (versi´on preliminar actualizada 23-05-2014)

Verifique que f posee derivada direccional en (0, 0) en cualquier direcci´on y que no es diferenciable en dicho punto. Soluci´ on. Sea u = (u, v) un vector unitario ∂f f (tu, tv) − f (0, 0) (0, 0) = l´ım t→0 ∂u t |tu| tv = l´ım q t→0 t (tu)2 + (tv)2 |t| |u| tv √ t→0 t |t| u2 + v 2 = l´ım |u| v

= l´ım t→0

= |u| v luego las derivadas direccionales existen en toda direcci´on. Note que

∂f ∂x

(0, 0) =

∂f ∂y

(0, 0) = 0

para ello tomar las direcciones (u, v) = (1, 0) y (0, 1) respectivamente. Para la diferenciabilidad (0, 0) (x − 0) − f (x, y) − f (0, 0) − ∂f ∂x l´ım (x,y)→(0,0) k(x, y) − (0, 0)k |xy| p x2 + y 2 p = l´ım (x,y)→(0,0) x2 + y 2 |xy| = l´ım 2 (x,y)→(0,0) x + y 2

∂f ∂y

(0, 0) (y − 0)

note que este l´ımite no existe, usando las trayectorias y = x e y = 0 obtenemos l´ımites distintos. La funci´on no es diferenciable. Observaci´ on 9.8.3. Del ejemplo anterior podemos afirmar que la existencia de todas las derivadas direccionales en un punto no implica diferenciabilidad en el punto. ◦

Teorema 9.8.1. Sea f : U ⊆ Rn → R una funci´on diferenciable en a ∈ U . Si u = (u1 , u2 , . . . , un ) es un vector unitario en Rn , entonces: ∂f (a) = h∇f (a) , ui ∂u n X ∂f = (a) ui ∂xi i=1 393

Apuntes Mat023 (versi´on preliminar actualizada 23-05-2014)

Demostraci´on. Definamos g (t) = f (a + tu), esta funci´on nes diferenciable en 0 por se compuesta de diferenciables, adem´as por la regla de la cadena g 0 (t) = ∇f (a + tu) · u evaluando en t = 0 se obtiene g 0 (0) = ∇f (a) · u por otro lado g (t) − g (0) t→0 t−0 f (a + tu) − f (a) = l´ım t→0 t ∂f = (a) ∂u

g 0 (0) = l´ım

as´ı ∂f (a) = ∇f (a) · u ∂u

Ejemplo 9.8.3. Verifique que la derivada direccional de z =

y2 x

en cualquier punto de la

elipse: 2x2 + y 2 = c2 en la direcci´on de la normal a la curva es nula. Soluci´ on. Note que la curva es el conjunto de nivel c2 de la funci´on g (x, y) = 2x2 + y 2 luego en el punto (x0 , y0 ) de la elipse, la direcci´on normal es ∇g (x0 , y0 ) 1 =p (4x0 , 2y0 ) k∇g (x0 , y0 )k 16x20 + 4y02 1 = p 2 (2x0 , y0 ) 4x0 + y02

n =

394

Apuntes Mat023 (versi´on preliminar actualizada 23-05-2014)

luego la derivada direccional es ∂z (x0 , y0 ) ∂n ∇g (x0 , y0 ) = ∇z (x0 , y0 ) · k∇g (x , y0 )k  2 0  1 y0 2y0 = p 2 − , · (2x0 , y0 ) x20 x0 4x0 + y02   2y02 2y02 1 − + = p 2 x0 x0 4x0 + y02 = 0 como se quer´ıa demostrar. Observaci´ on 9.8.4. Suponiendo las condiciones del teorema anterior, utilizando las propiedades euclidianas de Rn , obtenemos: ∂f (a) = h∇f (a) , ui ∂u = k∇f (a)k kuk cos θ = k∇f (a)k cos θ en donde θ = ∠ (∇f (a) ; u) ∈ [0, π]. Note que: − k∇f (a)k ≤

∂f (a) ≤ k∇f (a)k ∂u

As´ı, si en particular ∠ (∇f (a) ; u) = 0 entonces ∂f (a) = k∇f (a)k ∂u Es decir, la m´axima raz´on de cambio de f se obtiene en la direcci´on del gradiente ∇f (a) y de manera similar, si ∠ (∇f (a) ; u) = π entonces ∂f (a) = − k∇f (a)k ∂u m´aximo decrecimiento. Teorema 9.8.2. Sean U ⊆ Rn abierto y f una funci´ on diferenciable en a ∈ U , entonces ∇f (a) la direcci´on de m´aximo crecimiento de f es k∇f . Adem´as, obtenemos que: (a)k   ∂f k∇f (a)k = m´ax (a) : kuk = 1 ∂u

395

Apuntes Mat023 (versi´on preliminar actualizada 23-05-2014)

De manera similar, la menor derivada direccional de f se obtiene en la direcci´on de ∇f (a) − k∇f y (a)k

 − k∇f (a)k = m´ın

 ∂f (a) : kuk = 1 ∂u

Ejercicios del cap´ıtulo 1. Calcule las derivadas parciales de las siguientes funciones:   √ a) f (x, y) = ln sin x+a y b) f (x, y, z) = xy c) f (x, y) =

z

1 y arctan( x )

d ) f (x, y, z) = (x + z)x+y e) f (x, y) = arcsin √

x x2 +y 2

2. Suponga que g : R → R es una funci´on continua. Calcule las derivadas parciales de f si: a) f (x, y) =

R x+y a

Z

g (t) dt

sin(x sin(y+z 2 ))

b) f (x, y, z) =

z g (t) dt xy

3. Calcular las derivadas parciales de las siguientes funciones: a) f (x, y) = 31 x3 − 3x2 y + 3xy 2 + y 3 p b) f (x, y) = ln (x + 2 x2 + 3y 2 ) x2 − 2xy x+y 4 d ) f (x, y) = x − x3 y + x2 y 2 − xy 3 + y 4 x+y e) f (x, y) = x−y f ) f (x, y) = ln(x2 + y 2 ) c) f (x, y) =

g) f (x, y) = xy + y x 396

Apuntes Mat023 (versi´on preliminar actualizada 23-05-2014)

  x h) f (x, y) = x cos y   2 i ) f (x, y) = arctan x+y j ) f (x, y, z) = xy+x − y z−y + z x+y k ) f (x, y, z) = logx (x + y + z) 4. Pruebe que si f (x, y) =

2x entonces fx (3, 1) + fy (3, 1) = 1 xy

5. Pruebe que si f (x, y) = Ax4 + 28x2 y 2 + Cy 4 , entonces x 6. Sea f (x, y) = ex+y . Pruebe que:

∂f ∂f +y = 4f (x, y) ∂x ∂y

∂ m+n f (x, y) = ex+y . m n ∂x ∂y

∂ 2z ∂ 2z = ∂x∂y ∂y∂x

7. Sea z = f (x, y). Verifique que a) z = x2 − 4xy + 3y 2 b) z = ln(x + y) c) z = (x3 + y 3 )10 d ) z = (x + y) sec(xy) e) z = x2 cos(y −2 ) 4 f ) z = arctan(xy) π 8. Verifique que f (x, y) = arctan

y

9. Hallar

si z =

∂ 2f ∂ 2f + =0 ∂x2 ∂y 2

∂ 2z (x, y), ∂x∂y p 2xy + y 2 , siendo 2xy + y 2 > 0. −n2 kt

10. Muestre que u(x, t) = e 11. Sea

x

satisface la ecuaci´on

∂u ∂ 2u sin(nx) satisface la ecuaci´on =k 2 ∂t ∂x   

x2 y si (x, y =6= (0, 0) x4 + y 2 f (x) =   0 si (x, y) = (0, 0) 397

Apuntes Mat023 (versi´on preliminar actualizada 23-05-2014)

Pruebe que existen todas las derivadas direccionales de f en (0,0) pero f no es continua en el origen.  xy 3     x2 + y 2 si (x, y) 6= (0, 0) 12. Sea f (x, y) =     0 si (x, y) = (0, 0) a) ¿f es continua en (0,0)? ∂f ∂f (0, 0), (0, 0)? b) ¿Existen ∂x ∂y

13. Sea f (x, y) =

 4 x (y + 2)2   si (x, y) 6= (0, −2)   x2 + y 2    

si (x, y) = (0, −2)

0

a) Analice la continuidad de f en (0,-2). ∂f (0, −2) b) Calcule, si existe, ∂x 14. Sea

f (x, y) =

    (x2 + (y − 1)2 )k sin       

p

x2 + (y − 1)2

0

f (x, y) =

∂f ∂f (0, 1) y (0, 1) existan. ∂x ∂y

   y x 2 2   x arctan − y arctan si xy 6= 0   x y    

Calcule, si existen,

si (x, y) 6= (0, 1)

si (x, y) = (0, 1)

Determine todos los valores de k ∈ R de modo que 15. Sea

!

1

0

si xy = 0

∂f ∂f (0, 1) y (1, 0) ∂x ∂y

16. Verifique que la funci´on f : R → R2 definida por f (x) = (x2 , x3 ) es diferenciable en R. Calcule Df (x). 398

Apuntes Mat023 (versi´on preliminar actualizada 23-05-2014)

17. Sea f : R3 → R2 la funci´on definida por: f (x, y, z) = xy 2 , yz 2



Demuestre, por definici´on, que f es diferenciable en R3 . Calcule Df (x, y, z) 18. Considere f : Rn → Rm . Suponga que existe M > 0 tal que: kf (x) k < M kxk2 para todo x ∈ Rn . Verifique que f es derivable en 0 y que adem´as Df (0) = 0. 19. Sea f : R2 → R la funci´on f (x, y) = 20. Sea:

  

x2 y 2 , x2 + y 4 f (x, y) =   0 ,

a) Calcular b) Decidir

p |xy|. ¿Es f diferenciable en (0, 0)?

∂f ∂x

∂f ∂x

si (x, y) 6= (0, 0) si (x, y) = (0, 0)

en cada punto donde exista.

es continua en el punto (0, 0) y si acaso f es diferenciable en (0, 0).

21. Demuestre que las siguientes funciones son derivables y calcule su derivada: a) f (x, y) = ey , sin xy, x2 + 2y 3



b) g (x, y, z, w) = xyzw2 , esin w , x2



22. Calcule ∇f (x, y), en t´erminos de derivadas de g (x) y h (x) , para las funciones f (x, y) siguientes: a) f (x, y) = g (x) b) f (x, y) = g (y) c) f (x, y) = g (x + y)2 + g (x)h(y) − 3g (x2 − 3y 2 ) 23. Verifique lo siguiente: ∂z ∂z a) Si z = xy + xey/x , entonces x ∂x + y ∂y = xy + z.

399

Apuntes Mat023 (versi´on preliminar actualizada 23-05-2014)

b) Si u = (x − y) (y − z) (z − x), entonces a) Se supone que

∂z ∂y

=

x . x2 +y 2

b) Se supone que

∂u ∂x

=

x2 +y 2 x

∂u ∂x

+

∂u ∂y

= − ∂u . ∂z

Calcule z = z (x, y). y que u (1, y) = sin y. Calcule u = u (x, y).

∂z 24. Sea z = logy x. Calcule ∂y .   x+y 25. Sea z = f x−y , con f una funci´on clase C 1 (R). Verifique que:

∂z ∂z +y =0 ∂x ∂y   x+y 26. Sean f : R → R y u (x, y) = xy f xy . Hallar una funci´on escalar g (x, y) tal que x

se verifique la relaci´on: x2

∂u ∂u − y2 = g (x, y) u (x, y) ∂x ∂y

27. Calcule: d (g ◦ f ) (t) dt t=2 2 t−2 3 si f (t) = (t, t − 4, e ) y g : R → R es tal que: ∂g (2, 0, 1) = 4 ∂x



∂g (2, 0, 1) = 2 ∂y



∂g (2, 0, 1) = 2 ∂z

28. Calcule D h (x, y, z), si: h (x, y, z) = f (u (x, y, z) , v (x, y) , w (y, z)) 29. Se define

∂2u ∂x∂y

=

∂ ∂x



∂u ∂y



. Considere z = u (x, y) eax+by tal que

∂2u ∂x∂y

= 0. Hallar los

valores de las constantes a, b ∈ R de modo que: ∂ 2z ∂z ∂z − − +z =0 ∂x∂y ∂x ∂y 30. Sea f : Rp → R una funci´on. Se dice que f es homogenea de grado m si, para cada x ∈ Rp y cualquier t ∈ R se tiene que: f (t x) = tm f (x) Demuestre que si f es homogenea de grado m y diferenciable, entonces: hDf (x) , xi = m f (x) 400

Apuntes Mat023 (versi´on preliminar actualizada 23-05-2014)

31. Calcule

∂z , ∂x

si: z = f (u)

con u = xy + xy . 32. Hallar u0 (x), si: u = f (x, y, z) donde y = ϕ (x) y ψ (x, y).   33. Sea f (x, y, z) = x sin xy − z definida sobre:  A = (x, y, z) ∈ R3 : y 6= 0 Considere la superficie:  S = (x, y, z) ∈ R3 : f (x, y, z) = 0 Verifique que el plano tangente a S en cualquier punto de S pasa por el origen. 34. Encontrar la ecuaci´on del plano tangente a la superficie S dada por la gr´afica de la funci´on: a) z = x2 + y 2 en el punto (0, 0, 0). b) z = (x + y)2 − 2x en el punto (1, 1, 0). p c) f (x, y) = x2 + y 2 + (x2 + y 2 ) en (1, 0, 2). p √  d ) f (x, y, z) = x2 + 2xy − y 2 + 1 en el punto 1, 1, −2, 3 . 35. Considere la superficie:  π S : z 2 = −3x2 y + cos 2xy + 4 √  a) ¿Se puede calcular el plano tangente a S en el punto 0, 0, 2 ? Explique.  √  π b) ¿Se puede calcular el plano tangente a S en el punto 1, − 4 , 23π ? Explique. 36. Sea f (x, y) = x2 − xy + 2y 2 . Hallar la derivada direccional de f en el punto (1, 2) y ◦

en direcci´on que forma con el eje x positivo un a´ngulo de 60 . 401

Apuntes Mat023 (versi´on preliminar actualizada 23-05-2014)

37. Sea f (x, y) = x3 − 2x2 y + xy 2 + 1. Hallar la derivada direccional de f en el punto (1, 2) en la direcci´on que va desde este punto al punto (4, 6). 38. Determinar los valores de las constantes a, b, c ∈ R de modo que la derivada direccional de: f (x, y, z) = a xy 2 + b yz + c z 2 x3 en el punto (1, 2, −1) tenga un valor m´aximo de 64 en una direcci´on paralela al eje → − X. 39. Sea f : R2 → R la funci´on definida por:   x2 y , x3 −y 2 f (x, y) =  0 ,

si x3 − y 2 6= 0 si x3 − y 2 = 0

Verifique que f tiene en (0, 0) derivada en cualquier direcci´on, pero que no es diferenciable en dicho punto. 40. Calcule la derivada direccional de la funci´on f en el punto P y en direcci´on del vector v. a) f (x, y) = 2x2 + 3xy + 4y 2 , P (2, 1), v = (1, 1).   x b) f (x, y) = arctan , (−3, 3), v = (3, 4). y c) f (x, y, z) = x2 + 3xy + 4y 2 , (1, 1, 2), v = (2, 3, 0).  z x d ) f (x, y, z) = , P (1, 1, 1), v = (2, 1, −1) y 41. Calcule la derivada de f (x, y) = x4 + x3 y 2 + y en (1, 1) y en la direcci´on de la tangente a la curva y = x4 . 42. Calcule la derivada direccional de f (x, y) = xy + x2 + x4 y, en (1, 1) y en la direcci´on del vector que forma una ´angulo de 60◦ con el eje x. 43. Encuentre la m´axima derivada direccional de f en P . a) f (x, y) = 2x2 + 3xy + 4y 2 , en P (1, 1). b) f (x, y, z) = e−(x+y+z) , en P (5, 2, 3). 402

Apuntes Mat023 (versi´on preliminar actualizada 23-05-2014)

44. Dada la funci´on f (x, y) se sabe que: f 0 (x0 ; v) = 2

si x0 = (1, 2) y v = (2, 2)

f 0 (x0 ; v) = −2 si x0 = (1, 2) y v = (1, −1)

Calcule ∇f (1, 2) y f 0 (x0 ; v) cuando x0 = (1, 2) y v = (4, 6) 45. Suponga que la temperatura en el punto (x, y, z) est´a dada por T (x, y, z) = 3x2 + 2y 2 − 4z. Determine el valor de la raz´on de cambio de T en P (−1, −3, 2) en direcci´on a Q(−4, 1, −2). ¿Cu´al es la taza de m´axima variaci´on?. 46. Hallar los puntos (x, y) y las direcciones para las que la derivada direccional de f (x, y) = 3x2 + y 2 tiene el valor m´aximo si (x, y) est´a en el c´ırculo x2 + y 2 = 1. 47. Calcule la derivada direccional de f (x, y, z) = x2 + y 2 − z 2 en (3, 4, 5) a lo largo de la curva intersecci´on de las superficies 2x2 + 2y 2 − z 2 = 25 y x2 + y 2 = z 2 . 48. Calcular la ecuaci´on del plano tangente a la superficie dada por: a) f (x, y) = 3x2 + 8xy en el punto (x0 , y0 ) = (1, 0). p b) f (x, y) = x2 + y 2 en el punto (x0 , y0 ) = (1, 2) Z x2 +y2 √ e− t dt en el punto (x0 , y0 ) = (1, 1) c) f (x, y) = 0   π 1 1 d ) f (x, y, z) = sen(xyz) − en el punto 3, , 2 6 3 49. Hallar una constante c tal que en todo punto de la intersecci´on de las dos esferas: (x − c)2 + y 2 + z 2 = 3

x2 + (y − 1)2 + z 2 = 1

los planos tangentes correspondientes sean perpendiculares el uno al otro. 50. Demuestre que la superficie x2 − 2yz + y 3 = 4 es ortogonal a cualquiera de las superficies de la familia x2 + 1 = (2 − 4a)y 2 + az 2 en el punto de intersecci´on (1, −1, 2). 51. Determinar en que punto de la superficie z = 3xy − x3 − y 3 el plano tangente es horizontal (paralelo al plano z = 0) 403

Apuntes Mat023 (versi´on preliminar actualizada 23-05-2014)

52. Pruebe que si u = x2 y + y 2 z + z 2 x, entonces

∂u ∂u ∂u + + = (x + y + z)2 ∂x ∂y ∂z

53. Si w = x2 + y 2 + z 2 encuentre el diferencial total de w 54. Sea f (x, y) =

xy x+y

Demuestre que : x2

2 ∂ 2f ∂ 2f 2∂ f + y + 2xy =0 ∂x2 ∂x∂y ∂y 2

55. Probar que el volumen formado por el plano tangente a la superficie xyz = m3 , en cualquier punto, y los planos coordenados es constante. 56. Hallar

si z =

∂ 2z (x, y), ∂x∂y p 2xy + y 2 , siendo 2xy + y 2 > 0.

57. Calcular la ecuaci´on del plano tangente a la superficie dada por

z = 3x2 + 8xy en el punto (x, y) = (1, 0). 58. Calcular las derivadas de segundo orden: a) f (x, y) = x3 + 3x2 y + 6xy 2 − y 3 b) 2x4 − 3x2 y 2 + y 4 = z 59. Sea f (x, y) = ϕ(x2 + y 2 ), considere ϕ diferenciable, pruebe que:

y

∂f ∂f −x =0 ∂x ∂y

2 ∂ 2u 2∂ u 60. Demuestre que la funci´on u(x, t) satisface la ecuaci´on 2 = a para: ∂t ∂x2

a) u(x, t) = ϕ(x − at) + ψ(x + at) donde ϕ y ψ son funciones de clase C 2 . b) u(x, t) = sen(akt) sen(kx) con k ∈ Z.

404

Apuntes Mat023 (versi´on preliminar actualizada 23-05-2014)

61. Se dice que una funci´on es arm´onica si verifica que ∇2 f =

∂ 2f ∂ 2f + =0 ∂x2 ∂y 2

Sea f : R2 → R una funci´on de clase C 2 arm´onica y sean x = eu cos v

y = eu sen v

Consideremos la funci´on g : R2 → R definida como g(u, v) = f (x(u, v) , y(u, v)). a) Pruebe que 

∂g ∂u

2

 +

∂g ∂v

2

= e2u

"

∂f ∂x

2

 +

∂f ∂y

2 #

b) Pruebe que g es arm´onica. Es decir

∇2 g =

∂ 2g ∂ 2g + =0 ∂u2 ∂v 2

62. Si x2 = y 2 + f (x2 + z 2 ), Calcular: xy

∂y ∂y ∂z ∂z + yz − zx ∂x ∂x ∂x ∂x

63. Sea f (x, y) = x2 g(x2 y). Pruebe que x 64. Sea f (x, y) = g(x, y) eax+by con y b para que

∂f ∂f − 2y = 2f (x, y). ∂x ∂y

∂ 2g = 0. Determine los valores de las constantes a ∂x∂y

∂ 2f ∂f ∂f − − +f =0 ∂x∂y ∂x ∂y

65. Sea f : R2 → R la funci´on definida por:   √|x|y , x2 +y 2 f (x, y) =  0 ,

si x3 − y 2 6= 0 si x3 − y 2 = 0

Verifique que f es continua en (0, 0), que posee derivada direccional en (0, 0) en cualquier direcci´on que no es diferenciable en dicho punto. 405

Apuntes Mat023 (versi´on preliminar actualizada 23-05-2014)

66. Sea f : R2 → R la funci´on definida por:   x , y f (x, y) =  0 , a) Calcule

∂f ∂x

(0, 0) y

∂f ∂y

si y 6= 0 si y = 0

(0, 0).

b) Sea u = (α, β) tal que αβ 6= 0. ¿Existe

∂f ∂u

(0, 0)?

67. Una mosca se encuentra volando, circularmente, en alguna cocina. La due˜ na de casa, ante la presencia del insecto, roc´ıa el aire con alg´ un insecticida. La distribuci´on del veneno en el aire se puede modelar mediante la superficie: z = 10 + 6 cos x cos y + 3 cos 2x + 4 cos 3y Ante el peligro, la mosca establece (r´apidamente) un sistema de referencia y determina  que se encuentra en el punto π3 , π3 , 6 . ¿En qu´e direcci´on deber´ıa arrancar la mosca si quiere sobrevivir? a) Encontrar la ecuaci´on del plano tangente a la superficie x = e2y−z en (1, 1, 2) y un vector unitario normal a este plano.  b) Sea f (x, y, z) = z − ex sin y y sea a = ln 3, 3π , −3 . Calcular el valor de c tal 2 que a ∈ S, donde: (x, yz) ∈ S ⇐⇒ f (x, y, z) = c Hallar, adem´as, el plano tangente a esta superficie en a. c) Sea f (x, y) = √ x2 2 . Hallar el plano tangente a la superficie f (x, y) = z en el  x +y 3 punto 3, −4, 5 . 68. Una funci´on u est´a definida por una ecuaci´on de la forma:   x+y u = xy f xy Demostrar que u satisface una ecuaci´on en derivadas parciales de la forma: x2

∂u ∂u − y2 = u G (x, y) ∂x ∂y

Calcule, adem´as, G (x, y). 406

Apuntes Mat023 (versi´on preliminar actualizada 23-05-2014)

69. Sean k una constante positiva y g (x, t) = Z

1 √x . 2 kt g(x,t)

f (x, t) =

Considere: 2

e−u du

0

Demuestre que: k

∂ 2f ∂f = 2 ∂x ∂t

70. Las ecuaciones: ∧

u = f (x, y)

x = X (s, t)



y = Y (s, t)

definen u como una funci´on de s y t, digamos: u = F (s, t). a) Emplee una forma adecuada de la regla de la cadena para expresar las derivadas parciales

∂F ∂s

y

∂F ∂t

en funci´on de

∂f ∂f ∂X ∂X ∂Y , , , , ∂x ∂y ∂s ∂t ∂s

y

∂Y ∂t

.

b) Asumiendo que f ∈ C 2 , demuestre que: ∂ 2F ∂f ∂ 2 X ∂ 2 f = + 2 ∂s2 ∂x ∂s2 ∂x



∂X ∂s

2

∂f ∂ 2 Y ∂ 2f ∂X ∂Y ∂ 2 f + + +2 ∂s ∂s ∂x∂y ∂y ∂s2 ∂y 2



∂Y ∂s

2

71. Resuelva el ejercicio anterior para los casos: a) X (s, t) = s + t e Y (s, t) = st. b) X (s, t) = st e Y (s, t) = st . c) X (s, t) = 21 (s − t) e Y (s, t) = 12 (s + t). 72. La sustituci´on x = es e y = et transforma f (x, y) en g (s, t) siendo g (s, t) = f (es , et ). Se sabe, adem´as, que f satisface la ecuaci´on en derivadas parciales: x2

2 ∂f ∂f ∂ 2f 2∂ f + y + x + y =0 ∂x2 ∂y 2 ∂x ∂t

Demuestre que g satisface otra ecuaci´on de la forma: a

∂ 2g ∂ 2g ∂g ∂g + b + c + d =0 ∂s2 ∂t2 ∂s ∂t

para ciertas constantes a, b, c, d ∈ R. Calcule los valores de dichas constantes.

407

Apuntes Mat023 (versi´on preliminar actualizada 23-05-2014)

73. Escriba la ecuaci´on: x

∂z ∂z +y = x2 y ∂x ∂y

en las variables u y v mediante el cambio de variables: u=x 74. Represente la ecuaci´on:



v=

y x

∂2 ∂ 2z ∂ 2z − 3 + 2 =0 ∂x2 ∂x∂y ∂y 2

en las variables u y v definidas por el cambio de variables: u = 3x − y 75. Escriba la ecuaci´on: y



v =x+y

∂ 2z ∂ 2z ∂ 2z + (x + y) + x =0 ∂x2 ∂x∂y ∂y 2

en t´erminos de u y v las cuales vienen dadas por el cambio de variables: u = x2 − y 2

408



v =y−x

Cap´ıtulo 10 : M´ aximos y m´ınimos

Extremos locales Definici´ on 10.1.1. Sean U ⊆ Rn un conjunto abierto y f : U ⊆ Rn → R una funci´on. Diremos que x0 ∈ U es un punto de: 1. M´ınimo local, si existe un δ > 0 tal que ∀x ∈ B (x0 , δ) ∩ U, f (x0 ) ≤ f (x) 2. M´aximo local, si existe un δ > 0 tal que ∀x ∈ B (x0 , δ) ∩ U, f (x0 ) ≥ f (x) 3. Extremo local si es m´aximo o m´ınimo local.

Cuando trabajamos con funciones diferenciables de una variable los extremos locales ocurren en los puntos en los cuales f 0 (x) = 0, este resultado se extiende a funciones de varias variables. Suponga que x0 = (x01 , x02 , . . . , x0n ) es un punto de m´aximo local de f : U ⊆ Rn → R entonces existe un δ > 0 tal que ∀x ∈ B (x0 , δ) ∩ U, f (x0 ) ≥ f (x)

409

Apuntes Mat023 (versi´on preliminar actualizada 23-05-2014)

es decir, ∀x ∈ U, kx − x0 k < δ ⇒ f (x0 ) ≥ f (x). Definamos la funci´on g

:

I⊆R→R

t → g (t) = f t, x02 , x03 , . . . , x0n



donde I es un intervalo abierto con x01 ∈ I entonces

   

t, x02 , x03 , . . . , x0n − x01 , x02 , . . . , x0n < δ ⇒ g x01 = f (x0 ) ≥ f t, x02 , x03 , . . . , x0n = g (t) pero

 

t, x02 , x03 , . . . , x0n − x01 , x02 , . . . , x0n < δ ⇔ t − x01 < δ luego |t − x01 | < δ ⇒ g (x01 ) ≥ g (t) de esta forma g tiene un m´aximo local en t = x01 se sigue que como g es diferenciable  g 0 x01 = 0 pero g 0 x01



g (x01 + h) − g (x01 ) h→0 h f (x01 + h, x02 , x03 , . . . , x0n ) − f (x01 , x02 , x03 , . . . , x0n ) = l´ım h→0 h f (x0 + he1 ) − f (x0 ) ∂f (x0 ) = l´ım = h→0 h ∂x1 = l´ım

se sigue ∂f (x0 ) =0 ∂x1 el mismo procedimiento se puede realizar para todas las variables. Teorema 10.1.1. Sea f : U ⊆ Rn → R una funci´on diferenciable, donde U es un conjunto abierto. Si x0 ∈ U es un extremo local de f entonces ∇f (x0 ) = 0. Definici´ on 10.1.2. Sea f : U ⊆ Rn → R una funci´on diferenciable, donde U es un conjunto abierto. Llamaremos puntos cr´ıticos de f a todos aquellos puntos x0 ∈ U que cumplen ∇f (x0 ) = 0. Si x0 es un punto cr´ıtico que no es un extremo local entonces x0 se dice punto de silla.

410

Apuntes Mat023 (versi´on preliminar actualizada 23-05-2014)

Ejemplo 10.1.1. Considere la funci´on f : R2 → R, (x, y) → f (x, y) = x2 + y 2 , esta funci´on es diferenciable en todo R2 , sus puntos cr´ıticos son aquellos (x, y) tales que ∇f (x, y) = (0, 0) pero   ∂f ∂f (x, y) , (x, y) = (2x, 2y) = (0, 0) ∂x ∂y es decir, el u ´ nico punto cr´ıtico es (0, 0). Por la forma de la funci´on sabemos que (0, 0) es un punto de m´ınimo ya que f (0, 0) = 0 ≤ x2 + y 2 = f (x, y) m´as a´ un es un m´ınimo global (en ´el, la funci´on asume el menor valor que puede tomar en todo su dominio) Ejemplo 10.1.2. Considere la funci´on f : R2 → R, (x, y) → f (x, y) = −x2 − y 2 , esta funci´on es diferenciable en todo R2 , sus puntos cr´ıticos son aquellos (x, y) tales que ∇f (x, y) = (0, 0) pero   ∂f ∂f (x, y) , (x, y) = (−2x, −2y) = (0, 0) ∂x ∂y es decir, el u ´nico punto cr´ıtico es (0, 0). Por la forma de la funci´on sabemos que (0, 0) es un punto de m´aximo ya que f (0, 0) = 0 ≥ −x2 − y 2 = f (x, y) m´as a´ un es un m´aximo global (en ´el, la funci´on asume el mayor valor que puede tomar en todo su dominio). 411

Apuntes Mat023 (versi´on preliminar actualizada 23-05-2014)

Ejemplo 10.1.3. Considere la funci´on f : R2 → R, (x, y) → f (x, y) = x2 − y 2 , esta funci´on es diferenciable en todo R2 , sus puntos cr´ıticos son aquellos (x, y) tales que ∇f (x, y) = (0, 0) pero 

 ∂f ∂f (x, y) , (x, y) = (2x, −2y) = (0, 0) ∂x ∂y

es decir, el u ´nico punto cr´ıtico es (0, 0). Note que, si x, y 6= 0 entonces −y 2 = f (0, y) < 0 = f (0, 0) < f (x, 0) = x2 de esta forma, arbitrariamente cerca de (0, 0) hay puntos en los cuales la funci´on toma valores mayores y menores. (0, 0) es un punto de silla.

Figura: La silla de montar Claramente, no siempre es posible realizar este an´alisis para ver si los puntos son m´aximos, m´ınimos o puntos de silla, necesitamos a´lg´ un criterio que nos permita distinguir entre estos tipos de puntos. En c´alculo de una variable tenemos el siguiente criterio: Si x0 es un punto cr´ıtico de f , entonces f 00 (x0 ) > 0 implica que x0 es punto de m´ınimo local y si f 00 (x0 ) < 0 de m´aximo local. Si f 00 (x0 ) = 0 no hay informaci´on. Este m´etodo requiere el signo de la segunda derivada veamos como se puede generalizar a funciones de varias variables:

412

Apuntes Mat023 (versi´on preliminar actualizada 23-05-2014)

Proposici´ on 10.1.1. Sea n ∈ N y f : [a, b] → R una funci´on tal que f 0 , f 00 , . . . , f (n) existen en [a, b] adem´as f (n) es continua en [a, b] y diferenciable en ]a, b[ entonces existe c ∈ ]a, b[ tal que f (b) =

n X f (k) (a)

k!

k=0

(b − a)k +

f (n+1) (c) (b − a)n+1 (n + 1)!

Demostraci´on. La demostraci´on se basa en el teorema del valor medio, Para x ∈ [a, b] defina P (x) =

n X f (k) (a)

k!

k=0

(x − a)k

y F : [a, b] → R  x → F (x) = f (x) − P (x) −

f (b) − P (b) (b − a)n+1



(x − a)n+1

la funci´on esta bien definida adem´as F (a) = F (b) = 0 por el teorema de Rolle existe un c1 ∈ ]a, b[ tal que F 0 (c1 ) = 0 pero note que F 0 (a) = 0 entonces existe c2 ∈ ]a, c1 [ tal que F 00 (c2 ) = 0 pero F 00 (a) = 0... este proceso continua hasta la existencia de un c = cn+1 ∈ ]a, cn [ tal que F (n+1) (c) = 0 como P (n+1) (x) ≡ 0 se sigue   f (b) − P (b) (n+1) (n + 1)! f (c) = (b − a)n+1 es decir

f (n+1) (c) (b − a)n+1 + P (b) = f (b) (n + 1)!

Si f : [a, b] → R es una funci´on tal que f 0 , f 00 existen en [a, b] adem´as f 00 es continua en [a, b] y diferenciable en ]a, b[ entonces existe c ∈ ]a, b[ tal que f (b) =

2 X f (k) (a) k=0

k!

(b − a)k + 413

f (3) (c) (b − a)3 (n + 1)!

Apuntes Mat023 (versi´on preliminar actualizada 23-05-2014)

es decir f (b) = f (a) + f 0 (a) (b − a) +

f (3) (c) f 00 (a) (b − a)2 + (b − a)3 2 3!

pongamos x0 y x0 + h en lugar de a, b entonces  f 00 (x0 ) 2 f (3) cxx00 +h 3 h + h f (x0 + h) = f (x0 ) + f (x0 ) h + 2 3! 0

si x0 es un punto cr´ıtico f 0 (x0 ) = 0 y as´ı  f 00 (x0 ) 2 f (3) cxx00 +h 3 f (x0 + h) − f (x0 ) = h + h 2 3!  ! f 00 (x0 ) f (3) cxx00 +h 2 = h + h 2 3! Suponga que f 00 (x0 ) 6= 0, como  f (3) cxx00 +h l´ım h=0 h→0 3! se sigue que existe un δ > 0 tal que 0 < |h| < δ ⇒ f (3) cx0 +h  |f 00 (x )| 0 x0 < 3! 4 luego para 0 < |h| < δ ⇒  f 00 (x0 ) |f 00 (x0 )| f 00 (x0 ) f (3) cxx00 +h f 00 (x0 ) |f 00 (x0 )| − < + h< + 2 4 2 3! 2 4 de donde podemos concluir que para 0 < |h| < δ la cantidad

f 00 (x0 ) 2

+

  x +h f (3) cx00 3!

h tiene el

mismo signo que f 00 (x0 ). As´ı, si f 00 (x0 ) > 0 entonces f (x0 + h) − f (x0 ) = h2

 ! f 00 (x0 ) f (3) cxx00 +h + h 2 3!

> h2 luego, para 0 < |h| < δ se cumple f (x0 + h) > f (x0 ) es decir x0 es un m´ınimo. El mismo argumento entrega que x0 es un m´aximo local cuando f 00 (x0 ) < 0 (notar que el signo de h no influye por estar al cuadrado).

414

Apuntes Mat023 (versi´on preliminar actualizada 23-05-2014)

Teorema 10.1.2 (F´ormula de Taylor de orden 2 en varias variables). Suponga que U ⊆ Rn es un abierto y f : U → R tiene derivadas parciales continuas hasta el tercer orden entonces 1 f (x0 + h) = f (x0 ) + ∇f (x0 ) h + hT Hf (x0 ) h + R2 (h, x0 ) 2 donde R2 (h, x0 ) =0 khk→0 khk2   h1    h   2  h= .   ..    hn l´ım

y 

∂ 2f (x0 ) ∂x21

      ∂ 2f  (x )  ∂x2 ∂x1 0     ∂ 2f Hf (x0 ) =  (x )   ∂x3 ∂x1 0     ..  .      ∂ 2f (x0 ) ∂xn ∂x1

∂ 2f (x0 ) ∂x1 ∂x2

∂ 2f (x0 ) ∂x1 ∂x3

···

∂ 2f (x0 ) ∂x22

∂ 2f (x0 ) ∂x2 ∂x3

···

∂ 2f (x0 ) ∂x3 ∂x2

∂ 2f (x0 ) ∂x23

···

.. .

.. .

..

∂ 2f (x0 ) ∂xn ∂x2

∂ 2f (x0 ) ∂xn ∂x3

···

.

 ∂ 2f (x0 )  ∂x1 ∂xn     2  ∂ f (x0 )   ∂x2 ∂xn     2 ∂ f  (x0 )   ∂x3 ∂xn     ..  .     2  ∂ f (x ) 0 ∂x2n

La demostraci´on es el teorema de Taylor de una variable y la regla de la cadena, en efecto, si ponemos g (t) = f (x0 + ht) entonces g (1) = f (x0 + h) g (0) = f (x0 )

415

Apuntes Mat023 (versi´on preliminar actualizada 23-05-2014)

y por la regla de la cadena g 0 (t) = Df (x0 + ht) D (x0 + ht) es decir g 0 (t) = ∇f (x0 + ht) · h n X ∂f = (x0 + ht) hi ∂xi i=1 as´ı

n X ∂f (x0 ) hi = ∇f (x0 ) · h g (0) = ∂x i i=1 0

adem´as ! n X ∂f (x0 + ht) hi ∂x i i=1   n X d ∂f (x0 + ht) = hi dt ∂xi i=1

d g 00 (t) = dt

usando la regla de la cadena 00

g (t) = = = = de manera similar g 000 (t) =

n X i=1 n X

 hi ∇ hi

i=1 n X n X

 ∂f (x0 + ht) · h ∂xi

n X ∂ 2f (x0 + ht) hj ∂xj ∂xi j=1

!

∂ 2f (x0 + ht) hj hi ∂xj ∂xi

i=1 j=1 n 2 X

∂ f (x0 + ht) hj hi ∂xj ∂xi i,j=1 n X

∂ 2f (x0 + ht) hj hi hk ∂x k ∂xj ∂xi i,j,k=1

se sigue g (1) = g (0) + g 0 (0) 1 +

g 00 (0) 2 g 000 (c) 3 1 + 1 2! 3!

reemplazando f (x0 + h) n n n X  ∂f 1 X ∂ 2f 1 X ∂ 2f = f (x0 ) + (x0 ) hi + (x0 ) hj hi + x0 + hc10 hj hi hk ∂xi 2! i,j=1 ∂xj ∂xi 3! i,j,k=1 ∂xk ∂xj ∂xi i=1 416

Apuntes Mat023 (versi´on preliminar actualizada 23-05-2014)

note que n 1 1 X ∂ 2f (x0 ) hj hi = hT Hf (x0 ) h 2! i,j=1 ∂xj ∂xi 2

donde 

h1



   h=  

h2 .. .

     

hn y 

∂ 2f (x0 ) ∂x21

      ∂ 2f  (x )  ∂x2 ∂x1 0     ∂ 2f Hf (x0 ) =  (x )   ∂x3 ∂x1 0     ..  .      ∂ 2f (x0 ) ∂xn ∂x1

∂ 2f (x0 ) ∂x1 ∂x2

∂ 2f (x0 ) ∂x1 ∂x3

···

∂ 2f (x0 ) ∂x22

∂ 2f (x0 ) ∂x2 ∂x3

···

∂ 2f (x0 ) ∂x3 ∂x2

∂ 2f (x0 ) ∂x23

···

.. .

.. .

..

∂ 2f (x0 ) ∂xn ∂x2

∂ 2f (x0 ) ∂xn ∂x3

···

.

 ∂ 2f (x0 )  ∂x1 ∂xn     2  ∂ f (x0 )   ∂x2 ∂xn     ∂ 2f  (x0 )   ∂x3 ∂xn     ..  .     2  ∂ f (x ) 0 ∂x2n

Observaci´ on 10.1.1. Bajo las hip´otesis de continuidad del teorema se tiene ∂ 2f ∂ 2f = ∂xi ∂xj ∂xj ∂xi por lo tanto la matriz Hf (x0 ) llamada Matriz Hessiana es una matriz sim´etrica. Ahora bien, si x0 es punto cr´ıtico entonces ∇f (x0 ) = 0 se sigue n X ∂f (x0 ) hi = 0 ∂xi i=1

luego la expansi´on en Taylor quedar´a 1 f (x0 + h) = f (x0 ) + hT Hf (x0 ) h + R2 (h, x0 ) 2 y podemos generalizar el criterio de la segunda derivada de funciones reales. 417

Apuntes Mat023 (versi´on preliminar actualizada 23-05-2014)

Definici´ on 10.1.3. Sea A una matriz sim´etrica. Diremos que una forma cuadr´atica CA : Rn → R, x → CA (x) = xT Ax es definida positiva si para todo x ∈ Rn se cumple CA (x) = xT Ax ≥ 0 y CA (x) = 0 solo para x = 0. Diremos que es definida negativa si para todo x ∈ Rn se cumple CA (x) = xT Ax ≤ 0 y CA (x) = 0 solo para x = 0. Recordemos que toda matriz sim´etrica A es diagonalizable ortogonalmente, se sigue que existe una matriz Q invertible QQT = I tal que A = QT DQ donde D = diag (λ1 , λ2 , . . . , λn ) es una matriz diagonal entonces xT Ax = (Qx)T D (Qx) si ponemos 

y1



   y = Qx =   

y2 .. .

     

yn entonces T

x Ax =

n X

λi yi2

i=1 T

la forma x Ax es definida positiva (negativa) si y solo si todos los valores propios de A son positivos (negativos). Proposici´ on 10.1.2. Si la forma xT Ax es definida positiva entonces existe c > 0 tal que xT Ax ≥ c kxk2 En efecto, basta considerar c como el m´ınimo de los valores propios de la matriz A. entonces, siguiendo el c´alculo anterior con xT Ax = (Qx)T D (Qx)   y1    y  2   y = Qx =  .   ..    yn 418

Apuntes Mat023 (versi´on preliminar actualizada 23-05-2014)

se tiene xT Ax =

n X

λi yi2 ≥ c

n X

i=1

yi2 = c kyk2

i=1

pero kyk2 = kQxk2 = (Qx)T (Qx) = xT QT Qx = xT Ix = kxk2 dando el resultado deseado.

Teorema 10.1.3. Sean U ⊆ Rn abierto, f : U ⊆ Rn → R una funci´on de clase C 3 (U ) y x0 es un punto cr´ıtico de f . Si f (h) = 12 hT Hf (x0 ) h es definida positiva entonces x0 es un m´ınimo relativo, si es definida negativa es un m´aximo relativo. Demostraci´on. Supongamos que es definida positiva, por la proposici´on anterior existe un c > 0 tal que 1 T h Hf (x0 ) h ≥ c khk2 2 y de la expansi´on de Taylor tenemos f (x0 + h) = f (x0 ) + ∇f (x0 ) h +

hT Hf (x0 ) h + R2 (h, x0 ) 2

donde R2 (h, x0 ) =0 khk→0 khk2 l´ım

como x0 es punto cr´ıtico ∇f (x0 ) · h = 0 y as´ı hT Hf (x0 ) h f (x0 + h) = f (x0 ) + + R2 (h, x0 ) 2 ≥ f (x0 ) + c khk2 + R2 (h, x0 ) como R2 (h, x0 ) =0 khk→0 khk2 (h,x0 ) c existe un δ > 0 tal que 0 < khk < δ ⇒ R2khk < 2 se sigue para 0 < khk < δ 2 l´ım

 R2 (h, x0 ) f (x0 + h) − f (x0 ) ≥ khk c + khk2  c  c khk2 ≥ khk2 c − = >0 2 2 2

419



Apuntes Mat023 (versi´on preliminar actualizada 23-05-2014)

as´ı 0 < khk < δ implica f (x0 + h) ≥ f (x0 ) luego x0 es un punto de m´ınimo local ( es lo mismo quer decir, si x ∈ B (x0 ; δ) entonces f (x) ≥ f (x0 )). La demostraci´on de M´aximo local es similar. Teorema 10.1.4. Sean U ⊆ Rn abierto, f : U ⊆ Rn → R una funci´on de clase C 3 (U ) y x0 es un punto cr´ıtico de f . Si Hf (x0 ) tiene todos sus valores propios no nulos y existen dos con distinto signo entonces x0 es un punto silla. Existen otros m´etodos para determinar si una matriz es definida positiva o negativa. Sea

       A=     

a11 a12 . . . a1k . . . a1n



 a21 a22 . . . a2k . . . a2n   .. .. . . .. .. ..  . . . .  . .   ak1 ak2 . . . akk . . . akn   .. .. .. ..  . . ... . ... .   an1 an2 . . . ank . . . ann

Denotaremos por Ak a la submatriz     Ak =   

a11 a12 . . . a1k



a21 a22 . . . a2k .. .. . . . . .. . .

     

ak1 ak2 . . . akk de esta forma A1 = a11  A2 =  

a11 a12 a21 a22

 

a11 a12 a13



   A3 =  a a a  21 22 23  a31 a32 a32 .. . An = A 420

Apuntes Mat023 (versi´on preliminar actualizada 23-05-2014)

si los determinantes de todas estas submatrices son positivos la matriz generar´a una forma cuadr´atica definida positiva y el punto cr´ıtico es un m´ınimo. Si los determinantes se alternan en signo comenzando con el determinante de A1 negativo, A2 positivo, etc, entonces la forma cuadr´atica ser´a definida negativa y el punto cr´ıtico es un m´aximo local. Si los determinantes son no nulos y no cumplen con los ordenes de signos anteriores, el punto critico es punto silla. Observaci´ on 10.1.2 (Importante). Los criterios anteriores no entregan informaci´on si existen valores propios nulos o los subdeterminates son cero. Ejemplo 10.1.4. Analizar los extremos locales de la funci´on f : R2 → R, (x, y) → f (x, y) = ln (x2 + y 2 + 1) Soluci´ on. Los extremos locales est´an en los puntos cr´ıticos. Los puntos cr´ıticos de f son aquellos puntos (x, y) tales que ∇f (x, y) = (0, 0) pero 2x ∂f (x, y) = 2 ∂x x + y2 + 1 2y ∂f (x, y) = 2 ∂y x + y2 + 1 se sigue que el u´nico punto cr´ıtico es (x, y) = (0, 0). Veamos si es un m´aximo, m´ınimo local o un punto de silla. Calculemos la Hessiana, para ello necesitamos las derivadas de segundo orden: ∂ 2f 2 (x2 + y 2 + 1) − 2x (2x) 2y 2 − 2x2 + 2 (x, y) = = ∂x2 (x2 + y 2 + 1)2 (x2 + y 2 + 1)2 ∂ 2f −4xy (x, y) = ∂x∂y (x2 + y 2 + 1)2 ∂ 2f 2 (x2 + y 2 + 1) − 2y (2y) 2x2 − 2y 2 + 2 (x, y) = = ∂y 2 (x2 + y 2 + 1)2 (x2 + y 2 + 1)2   2y 2 − 2x2 + 2 −4xy  (x2 + y 2 + 1)2 (x2 + y 2 + 1)2      Hf (x, y) =     2 2  −4xy 2x − 2y + 2  (x2 + y 2 + 1)2 (x2 + y 2 + 1)2 421

Apuntes Mat023 (versi´on preliminar actualizada 23-05-2014)

entonces

 Hf (0, 0) = 

2 0 0 2

 

2 0 aplicando el criterio de los subdeterminantes vemos que |A1 | = 2 > 0 y 0 2 luego el extremo local es un m´ınimo.

=4>0

Ejemplo 10.1.5. Analizar los extremos de f (x, y) = x3 y + xy 5 + xy. Soluci´ on. Buscamos los puntos cr´ıticos ∇f (x, y) = (0, 0) es decir  ∂f (x, y) = 3x2 y + y 5 + y = y 3x2 + y 4 + 1 = 0 ∂x  ∂f (x, y) = x3 + 5xy 4 + x = x x3 + 5y 4 + 1 = 0 ∂y luego el u ´nico punto cr´ıtico es (x, y) = (0, 0). La Hessiana en este caso es   Hf (x, y) =  

3x2 + 5y 4 + 1

6xy 3x2 + 5y 4 + 1

20xy 3

   

luego 

0

1

 Hf (0, 0) =  

   

1

0

entonces no funciona el m´etodo de los subdeterminantes pues el primero da cero. Note que los valores propios de esta matriz son 1 y −1 por lo tanto es un punto de silla. Esto se puede ver de la funci´on misma pues f (x, y) = xy x2 + y 4 + 1



cerca del (0, 0) hay puntos donde la funci´on es negativa y positiva luego es un punto de silla. 422

Apuntes Mat023 (versi´on preliminar actualizada 23-05-2014)

Ejemplo 10.1.6. Clasificar los puntos cr´ıticos de f : R3 → R, (x, y, z) → f (x, y, z) = x2 z + y 2 z + 23 z 3 − 4x − 4y − 10z Soluci´ on. La funci´on esta definida en un abierto y es de clase C ∞ , buscamos los puntos cr´ıticos  ∇f (x, y, z) = 2xz − 4, 2yz − 4, x2 + y 2 + 2z 2 − 10 = (0, 0, 0) esto es 2xz − 4 = 0 2yz − 4 = 0 x2 + y 2 + 2z 2 − 10 = 0 de las dos primeras ecuaciones se sigue que x, y, z 6= 0 y 2 2 x = ,y = z z reemplazado en la tercera 8 + 2z 2 − 10 = 0 z2 as´ı z = ±1, ±2 tenemos 4 puntos cr´ıticos (2, 2, 1) , (−2, −2, −1) , (1, 1, 2) , (−2, −2, −1) los clasificaremos usando el criterio de la hessiana  2z  Hf (x, y, z) =   0 2x

y subdeterminantes  0 2x  2z 2y   2y 4z

entonces ∆1 = 2z ∆2 = (2z)2 ∆3 = −8z x2 + y 2 − 2z 2 423



Apuntes Mat023 (versi´on preliminar actualizada 23-05-2014)

y as´ı Punto/Determinante

∆1

∆2

∆3

Tipo de punto

P1 = (2, 2, 1)

+

+

-

Punto silla

P2 = (−2, −2, −1)

-

+

+

Punto silla

P3 = (1, 1, 2)

+

+

+

Punto de m´ınimo local

P4 = (−2, −2, −1)

-

+

-

Punto de m´aximo local

notemos que 2 f (0, 0, z) = z 3 − 10z 3 luego l´ım f (0, 0, z) = +∞

z→+∞

l´ım f (0, 0, z) = −∞

z→−∞

luego la funci´on no tiene m´aximo ni m´ınimo en todo R3 . Ejemplo 10.1.7. Muestre que

π π π , , 2 2 2



es un punto de m´aximo de la funci´on f (x, y, z) =

sin x + sin y + sin z − sin (x + y + z) Soluci´ on. Notemos que

π π π , , 2 2 2



es un punto cr´ıtico pues

∇f (x, y, z) = ∇ (sin x + sin y + sin z − sin (x + y + z)) = (cos x − cos (x + y + z) , cos y − cos (x + y + z) , cos z − cos (x + y + z)) as´ı π π π  , , ∇f 2 2 2    π π π π π π  = cos − cos π + , cos − cos π + , cos − cos π + 2 2 2 2 2 2 = (0, 0, 0)

424

Apuntes Mat023 (versi´on preliminar actualizada 23-05-2014)

la Hessiana es Hf (x, y, z)  sin (x + y + z) − sin x     =  sin (x + y + z)     sin (x + y + z)

sin (x + y + z)



sin (x + y + z)

sin (x + y + z) − sin y

sin (x + y + z)

sin (x + y + z)

sin (x + y + z) − sin z

        

as´ı π π π  , , Hf 2 2 2   sin π + π2 − sin      π =  sin π +  2     sin π + π2  −2 −1 −1     =  −2 −1  −1    −1 −1 −2

π 2



sin π + sin π +

π 2



π 2



− sin

sin π +

π 2



sin π + π 2



sin π + sin π +

π 2



π 2

π 2







− sin

          π 2

         

los subdeterminantes son ∆1 = −2 ∆2 = 3 ∆3 = −4 luego el punto cr´ıtico es un punto de m´aximo local.

M´ aximos y m´ınimos en compactos y/o con restricciones El siguiente teorema generaliza el teorema de una variable que afirma que toda funci´on continua f : [a, b] → R alcanza su m´aximo y m´ınimo absolutos, es decir, existen x0 , x1 ∈ 425

Apuntes Mat023 (versi´on preliminar actualizada 23-05-2014)

[a, b] tales que f (x0 ) ≤ f (x) ≤ f (x1 ) para todo x ∈ [a, b]. Teorema 10.2.1 (De Weiertrass). Sean K ⊆ Rn un conjunto cerrado y acotado y f : K → R una funci´on continua entonces existen puntos x0 y x1 en K tales que ∀x ∈ K, f (x0 ) ≤ f (x) ≤ f (x1 ) esto es f alcanza su m´aximo y m´ınimos absolutos en K. Para utilizar tal teorema es conveniente la siguiente proposici´on. Proposici´ on 10.2.1. La imagen inversa de un conjunto abierto por una funci´on continua es un conjunto abierto. La imagen inversa de un cerrado por una funci´on continua es un conjunto cerrado. Por ejemplo, si consideramos la funci´on continua f (x, y) = y − x2 entonces f −1 (]0, +∞[) = f −1 ({u ∈ R : u > 0})  = (x, y) ∈ R2 : f (x, y) ∈ ]0, +∞[  = (x, y) ∈ R2 : y − x2 > 0  = (x, y) ∈ R2 : y > x2 es un conjunto abierto. Si consideramos h (x, y) = x2 + y 2 entonces h es continua y h−1 ([1, 2]) =

(x, y) ∈ R2 : h (x, y) ∈ [1, 2]  = (x, y) ∈ R2 : 1 ≤ x2 + y 2 ≤ 2 

es la imagen inversa de un cerrado por una funci´on continua, entonces es un conjunto cerrado.

Ejemplo 10.2.1. Hallar los valores extremos de f (x, y) = x2 + y 2 − x − y + 1 en el disco D = {(x, y) ∈ R2 : x2 + y 2 ≤ 1}

426

Apuntes Mat023 (versi´on preliminar actualizada 23-05-2014)

Soluci´ on. La funci´on f (x, y) = x2 + y 2 − x − y + 1 es continua. Note que el disco lo podemos ver como D = h−1 (]−∞, 1]) donde h (x, y) = x2 + y 2 entonces D es cerrado y esta contenido en la bola B ((0, 0) ; 2) luego es acotado. Por el teorema de los extremos absolutos se tiene que f (x, y) alcanza su m´aximo y m´ınimos absolutos en D, es decir, existen puntos (u0 , v0 ) y (u1 , v1 ) en D tales que ∀ (x, y) ∈ D, f (u0 , v0 ) ≤ f (x, y) ≤ f (u1 , v1 ) Note que tales extremos pueden estar en la bola abierta B = {(x, y) ∈ R2 : x2 + y 2 < 1} o en la circunferencia unitaria  S 1 = (x, y) ∈ R2 : x2 + y 2 = 1 Primero busquemos los puntos cr´ıticos en B.   ∂f ∂f ∇f (x, y) = (x, y) , (x, y) = (0, 0) ∂x ∂x = (2x − 1, 2y − 1) = (0, 0) el u´nico punto cr´ıtico en B es (x, y) =

1 1 , 2 2



. El conjunto S 1 lo podemos parametrizar por

c : [0, 2π] → S 1 tal que t → c (t) = (cos t, sin t), de esta forma la funci´on f en el conjunto S 1 es f (cos t, sin t) = 2 − cos t − sin t para t ∈ [0, 2π] como es una funci´on de una variable buscamos sus valores extremos con el c´alculo diferencial y el teorema de los extremos en R. f 0 = 0 ⇔ sin t − cos t = 0 tiene las soluciones t = cr´ıticos 

1 1 , 2 2

π 4

yt=

π 4

+ π en el intervalo ]0, 2π[ luego se tienen los puntos

     1 1 1 1 , √ , √ , −√ , −√ y (1, 0) = (cos 0, sin 0) = (cos 2π, sin 2π) 2 2 2 2

427

Apuntes Mat023 (versi´on preliminar actualizada 23-05-2014)

evaluando en la funci´on obtenemos   2  2  1 1 1 1 1 1 1 , = + − − +1= f 2 2 2 2 2 2 2   √ 1 1 1 1 1 1 f √ ,√ = + − √ − √ +1=2− 2 2 2 2 2 2 2   √ 1 1 1 1 1 1 = + + √ + √ +1=2+ 2 f −√ , −√ 2 2 2 2 2 2 f (1, 0) = 1 − 1 + 1 = 1    de donde − √12 , − √12 es el punto de m´aximo y 21 , 12 es el punto de m´ınimo en el disco. 2 2 Nota: La funci´on es f (x, y) = x − 12 + y − 12 + 12 el valor m´ınimo se alcanza en   1 1 , y el m´aximo en el disco ser´a en el punto del disco que encuentre m´as lejos de 12 , 21 2 2   este es − √12 , − √12 .

Extremos restringidos Multiplicadores de Lagrange Es com´ un en problemas querer maximizar o minimizar alguna funci´on sujeta a restricciones dadas. Por ejemplo; En econom´ıa suponga que se quiere vender dos tipos de productos I y II. Sean x e y la cantidad de productos I y II respectivamente. Representemos por f (x, y) la ganancia obtenida cuando se vende una cantidad x de I y y de II, nuestra producci´on esta limitada o controlada por nuestro capital, de esta forma tenemos una restricci´on del tipo g (x, y) = c. El problema podr´ıa plantearse como m´ax f (x, y) (x,y)∈R2 ,g(x,y)=c

Supongamos que x0 es un punto extremo del problema Optimizar f (x) x∈Rn ,g(x)=c

donde f : U ⊆ Rn → R y optimizar es m´aximo o m´ınimo. Sea c : R → Rn donde g (c (t)) = c y c (0) = x0 entonces por la regla de la cadena ∇g (c (t)) · c0 (t) = 0 428

Apuntes Mat023 (versi´on preliminar actualizada 23-05-2014)

se sigue que ∇g (x0 ) · c0 (0) = 0 como x0 es extremo de la f se sigue que f (c (t)) tiene un m´aximo o m´ınimo en t = 0 luego ∇f (c (t)) · c0 (t) = D (f ◦ c) (t) ∇f (x0 ) · c0 (0) = 0 de esta forma ∇f (x0 ) y ∇g (x0 ) deben ser paralelos es decir ∃λ ∈ R tal que ∇f (x0 ) = λ∇g (x0 ) el n´ umero λ se llama multiplicador de Lagrange y la funci´on de n + 1 variables L (x, λ) = f (x) − λ (g (x) − c) es llamado Lagrangiano del problema. Podemos entonces formular el siguiente teorema: Teorema 10.3.1. Sean f, g : U ⊆ Rn → R funciones de clase C 1 . Sea x0 ∈ U un punto tal que g (x0 ) = c y ∇g (x0 ) 6= 0. Si S = {x ∈ U : g (x) = c} y f |S tiene un extremo local en x0 entonces existe λ0 ∈ R tal que ∇f (x0 ) = λ0 ∇g (x0 )

Observaci´ on 10.3.1. Es lo mismo que buscar los puntos cr´ıticos del Lagrangiano L (x, λ) = f (x) − λ (g (x) − c)

Ejemplo 10.3.1. Sea S ⊆ R2 la recta que pasa por (−1, 0) inclinada en un a´ngulo de 45◦ y sea f : R2 → R, (x, y) → x2 + y 2 . Hallar los extremos de f |S . Soluci´ on. La pendiente es tan (45) = 1 entonces y−0=x−1 429

Apuntes Mat023 (versi´on preliminar actualizada 23-05-2014)

se sigue que la restricci´on es g (x, y) = y − x + 1 = 0 buscamos los puntos cr´ıticos del Lagrangiano L (x, y, λ) = x2 + y 2 − λ (y − x + 1) as´ı ∂L (x, y, λ) = 2x + λ = 0 ∂x ∂L (x, y, λ) = 2y − λ = 0 ∂y ∂L (x, y, λ) = y − x + 1 = 0 ∂λ Note que de la primera y segunda se obtiene λ = 2y = −2x ⇔ y = −x =

λ 2

reemplazando en la tercera se obtiene 1 1 y=− ⇒x= 2 2 geom´etricamente podemos deducir que este punto es un m´ınimo (x0 , y0 ) =

1 , − 12 2



. Note

que el punto es un extremo de f |S pero no de f . Ejemplo 10.3.2. Maximizar la funci´on f (x, y, z) = x + z sujeta a la restricci´on x2 + y 2 + z 2 = 1. Soluci´ on. El conjunto S = {(x, y, z) ∈ R3 : x2 + y 2 + z 2 = 1} es cerrado y acotado y f es una funci´on continua en todo R2 de donde obtenemos que f |S debe tener un m´aximo y un m´ınimo absolutos. Los buscamos con el m´etodo de Lagrange. Pongamos el Lagrangiano  L (x, y, z, λ) = x + z − λ x2 + y 2 + z 2 − 1 y debemos buscar sus puntos cr´ıticos ∂L (x, y, z, λ) ∂x ∂L (x, y, z, λ) ∂y ∂L (x, y, z, λ) ∂z ∂L (x, y, z, λ) ∂λ

= 1 − 2λx = 0 = −2λy = 0 = 1 − 2λz = 0 = x2 + y 2 + z 2 − 1 = 0 430

Apuntes Mat023 (versi´on preliminar actualizada 23-05-2014)

note que λ = 0 o y = 0. λ = 0 no puede ser (sistema inconsistente) entonces y = 0 se sigue de la primera y tercera ecuaciones 1 2λ 1 z = 2λ

x =

as´ı x = z y reemplazando en la u ´ltima 1 2x2 = 1 ⇒ x = ± √ 2 as´ı obtenemos dos puntos cr´ıticos     1 1 1 1 √ , 0, √ y − √ , 0, − √ 2 2 2 2 reemplazando se tiene que el primero es un m´aximo y el segundo un m´ınimo. Ejemplo 10.3.3. Hallar el mayor volumen que pueda tener una caja rectangular con tapa sujeta a la restricci´on de que el a´rea de la superficie sea 10 m2 . Soluci´ on. Sean x, y, z los lados de la caja entonces el volumen es V (x, y, z) = xyz, vemos que esta funci´on esta restringida a [0, 10] × [0, 10] × [0, 10] luego debe alcanzar el m´aximo y m´ınimo absolutos. El a´rea ser´a 2 (xy + xz + zy) = 10 as´ı el Lagrangiano ser´a L (x, y, z, λ) = xyz − λ (xy + xz + zy − 5) buscamos los puntos cr´ıticos ∂L (x, y, z, λ) ∂x ∂L (x, y, z, λ) ∂y ∂L (x, y, z, λ) ∂z ∂L (x, y, z, λ) ∂λ

= yz − λ (y + z) = 0 = xz − λ (x + z) = 0 = xy − λ (x + y) = 0 = xy + xz + zy − 5 = 0

note que x 6= 0 pues si fuese cero zy = 5 pero λz = 0 se sigue λ = 0 y as´ı de la primera yz = 0 contradicci´on, lo mismo para y, z. 431

Apuntes Mat023 (versi´on preliminar actualizada 23-05-2014)

como son cantidades positivas tambi´en las sumas ser´an distintas de cero xz xy yz = = y+z x+z x+y esto implica x = y = z sustituyendo en la u ´ltima ecuaci´on nos da 3x2 = 5 de donde obtenemos el punto cr´ıtico r

el volumen m´aximo es V =

q 3 5 3

5 , 3

r

5 , 3

r ! 5 3

.

Este resultado tiene una extensi´on a m´as restricciones: Si tenemos las restricciones una restricci´on S dada por los puntos que satisfacen g1 (x1 , x2 , . . . , xn ) = c1 g2 (x1 , x2 , . . . , xn ) = c2 .. . gk (x1 , x2 , . . . , xn ) = ck y f restringida a S tiene un extremo en x0 ∈ S entonces, si ∇g1 (x0 ) , ∇g2 (x0 ) , · · · , ∇gk (x0 ) son l.i. existir´an λ1 , λ2 , . . . , λk ∈ R tales que ∇f (x0 ) = λ1 ∇g1 (x0 ) + λ2 ∇g2 (x0 ) + · · · + λk ∇gk (x0 ) Ejemplo 10.3.4. Determine los extremos absolutos de f (x, y, z) = x + 2y + z sujeto a las restricciones x2 + y 2 = 1 y y + z = 1. Soluci´ on. f (x, y, z) = x + 2y + z

es una funci´on continua. El conjunto definido por las

restricciones (x, y, z) ∈ R3 : x2 + y 2 = 1 ∧ y + z = 1   = (x, y, z) ∈ R3 : x2 + y 2 = 1 ∩ (x, y, z) ∈ R3 : y + z = 1

S =



note que el primer conjunto restringe a x, y al conjunto [−1, 1] y eso restringe a z en el segundo conjunto. Es un conjunto acotado e intersecci´on de dos cerrados. Luego es cerrado y acotado la funci´on alcanza un m´aximo y m´ınimo absolutos en S. 432

Apuntes Mat023 (versi´on preliminar actualizada 23-05-2014)

El Lagrangiano ser´a  L (x, y, z, λ, µ) = (x + 2y + z) − λ x2 + y 2 − 1 − µ (y + z − 1) buscamos los puntos cr´ıticos ∂L ∂x ∂L ∂y ∂L ∂z ∂L ∂λ ∂L ∂µ

= 1 − 2λx = 0 = 2 − 2λy − µ = 0 = 1−µ=0 = x2 + y 2 − 1 = 0 = y+z−1=0

resolvemos µ = 1 reemplazando en la segunda obtenemos 1 = 2λy de donde λ ni y pueden ser cero. De la primera obtenemos x=

1 =y 2λ

de donde obtenemos en la cuarta 1 x = y = ±√ 2 de donde podemos obtener z en la 5. As´ı     1 1 1 1 1 1 √ , √ ,1 − √ y −√ , −√ , 1 + √ 2 2 2 2 2 2 evaluando en la funci´on f (x, y, z) = x + 2y + z se tiene   1 1 1 1 2 1 2 f √ , √ ,1 − √ = √ + √ +1− √ = √ +1 2 2 2 2 2 2 2   1 1 1 1 2 1 2 f −√ , −√ , 1 + √ = −√ − √ + 1 + √ = 1 − √ 2 2 2 2 2 2 2 el primer es m´aximo y el segundo un m´ınimo.

433

Apuntes Mat023 (versi´on preliminar actualizada 23-05-2014)

Criterio de la segunda derivada para extremos condicionados En esta secci´on presentamos un teorema que nos permite clasificar el punto obtenido con los multiplicadores de Lagrange:

Teorema 10.3.2 (Hessiana orlada). Sean f, g : U ⊆ Rn → R funciones de clase C 2 . Sean v0 ∈ U, g (v0 ) = c, ∇g (v0 ) = 6 0 y existe un λ0 ∈ R tal que ∇f (v0 ) = λ0 g (v0 ). Si L (λ, x) = f (x) − λ (g (x) − c) y 0 − ∂g ∂x1 ∂g H = HL (v0 ) = − ∂x . 2 .. − ∂g ∂xn

∂g − ∂x 1

∂g − ∂x 2

···

∂2L ∂x21

∂2L ∂x1 ∂x2

···

∂2L

∂2L

∂x1 ∂x2

∂x22

··· .. .

∂2L ∂x1 ∂xn

∂2L ∂x2 ∂xn

.. .

.. .

···

∂g − ∂x n 2 ∂ L ∂x1 ∂xn ∂2L ∂x2 ∂xn .. . ∂2L 2 ∂xn

entonces: 1. Si 0 ∂g H3 = − ∂x ∂g1 − ∂x2

∂g − ∂x 1

∂g − ∂x 2

∂2L

∂2L

∂x21

∂x1 ∂x2

∂2L ∂x1 ∂x2

∂2L ∂x22

0 ∂g − < 0, H4 = ∂x1 − ∂g ∂x2 ∂g − ∂x3

∂g − ∂x 1

∂g − ∂x 2

∂2L ∂x21

∂2L ∂x1 ∂x2

∂2L ∂x1 ∂x2

∂2L ∂x22

∂2L ∂x1 ∂x3

∂2L ∂x2 ∂x3

∂g − ∂x 3 ∂2L ∂x1 ∂x3 < 0, . . . ∂2L ∂x2 ∂x3 ∂2L 2 ∂x3

entonces v0 es m´ınimo local de f sujeto a g (x) = c. 2. Si los determinantes se alternan en la forma H3 > 0, H4 < 0, H5 > 0 etc, entonces v0 es m´aximo local de f sujeto a g (x) = c 3. Si los determinantes anteriores son distintos de cero pero siguen los patrones anteriores entonces el punto es de silla.

Ejemplo 10.3.5. Estudiar los extremos de la funci´on f (x, y) = cos2 x + cos2 y sujeta a la restricci´on x − y = π4 .

434

Apuntes Mat023 (versi´on preliminar actualizada 23-05-2014)

Soluci´ on. la funci´on de Lagrange es F (λ, x, y) = cos2 x + cos2 y − λ x − y −  ∂F π = − x−y− ∂λ 4 ∂F = −2 cos x sin x − λ ∂x ∂F = −2 cos y sin y + λ ∂y luego sin (2x) = −λ sin (2y) = λ π x−y = 4 se sigue   π  = − sin (2y) sin 2 y + 4 cos 2y = − sin (2y) as´ı cos 2y + sin 2y = 0 ⇔ π sin 2y + = 0 4 ⇒ π k π − con k ∈ Z y = 2 8 

se sigue 

 k π k π π+ , π− con k ∈ Z 2 8 2 8 son los puntos cr´ıticos. Calculamos la Hessiana orlada 2 ∂F ∂F ∂F −1 1 ∂λ2 ∂x∂λ ∂y∂λ 0 2F ∂F ∂ ∂F 0 ∂x∂λ ∂x2 ∂x∂y = −1 −2 cos 2x ∂F 2F ∂F ∂ 1 0 −2 cos 2y ∂y∂λ ∂x∂y ∂y 2 pero 2 cos 2x = 2 cos 2y = 435

√ √

2 (−1)k 2 (−1)k



π 4



se sigue

Apuntes Mat023 (versi´on preliminar actualizada 23-05-2014)

se sigue

∂2F ∂λ2 ∂F ∂x∂λ ∂F ∂y∂λ

∂F ∂x∂λ ∂2F ∂x2 ∂F ∂x∂y

∂F ∂y∂λ ∂F ∂x∂y ∂2F ∂y 2

se sigue que si k es impar

0 −1 1 √ k+1 = −1 2 (−1) 0 √ 1 0 2 (−1)k+1 ∂2F ∂λ2 ∂F ∂x∂λ ∂F ∂y∂λ

∂F ∂x∂λ ∂2F ∂x2 ∂F ∂x∂y

∂F ∂y∂λ ∂F ∂x∂y ∂2F ∂y 2

√ = 2 (−1)k 2

< 0 y el punto es m´ınimo y si k es par es

un m´aximo. Note que f (x, y) = cos2 x + cos2 y ⇒  f

π k π k π+ , π− 2 8 2 8

 = = = = =

   π π k k 2 π+ π− cos + cos 2 8 2 8   π 1 + cos kπ − π4 1 + cos kπ + 4 + 2 2  1  π π  1+ cos kπ + + cos kπ − 2 4 4  1 √ 1+ 2 (−1)k 2√ 2 (−1)k 1+ 2 2



para k impar  f

k π k π π+ , π− 2 8 2 8



π k π k π+ , π− 2 8 2 8





para k par  f

2 2

=1−

√ =1+

2 2

Ejemplo 10.3.6. Hallar las dimensiones del mayor paralelep´ıpedo rectangular de aristas paralelas a los ejes coordenados que puede ser inscrito en el elipsoide de ecuaci´on:  x 2 3

+

 y 2 4

+

436

 z 2 5

=1

Apuntes Mat023 (versi´on preliminar actualizada 23-05-2014)

Soluci´ on. El volumen del paralelep´ıpedo es V (x, y, z) = (2x) (2y) (2z) = 8xyz pero (x, y, z) debe estar sobre el elipsoide  x 2  y 2  z 2 + + =1 3 4 5 y sus coordenadas son positivas o cero. Tenemos que resolver el problema m´ax V (x, y, z) r(x,y,z)=0 x,y,z≥0

donde V (x, y, z) = 8xyz y r (x, y, z) =

 x 2 3

+

 y 2 4

+

 z 2 5

− 1. Este problema tiene

soluci´on pues el elipsoide es un conjunto cerrado y acotado adem´as la funci´on es continua. Determinaremos el valor m´aximo utilizando los multiplicadores de Lagrange ∇V (x, y, z) = λ∇r (x, y, z) r (x, y, z) = 0 esto es  (8yz, 8xz, 8xy) = λ  x 2  y 2  z 2 + + = 1 3 4 5 437

2x 2y 2z , , 32 42 52



Apuntes Mat023 (versi´on preliminar actualizada 23-05-2014)

el sistema a resolver es 2λx 32 2λy 8xz = 42 2λz 8xy = 52  x 2  y 2  z 2 + + = 1 3 4 5 8yz =

multiplicando las primeras tres ecuaciones por x, y, z respectivamente se obtiene λx2 32 λy 2 4xyz = 42 λz 2 4xyz = 52 4xyz =

sumando las tres ecuaciones λx2 λy 2 λz 2 + 2 + 2 2 3 4 5  x 2  y 2  z 2  = λ + + 3 4 5 = λ

12xyz =

as´ı

λx2 λy 2 λz 2 λ = 4xyz = 2 = 2 = 2 3 3 4 5

si λ 6= 0 entonces 32 ⇒x= 3 42 = ⇒y= 3 52 = ⇒z= 3

x2 = y2 z2

3 √ 3 4 √ 3 5 √ 3

si λ = 0 entonces xyz = 0 y yz = 0 xz = 0  x 2 3

+

 y 2 4

xy = 0  z 2 + = 1 5

438

Apuntes Mat023 (versi´on preliminar actualizada 23-05-2014)

esto nos da los puntos (3, 0, 0) , (0, 0, 5) , (0, 4, 0). Estos 3 puntos son puntos de m´ınimo V =0y  V

3 4 5 √ ,√ ,√ 3 3 3



 =8

3 √ 3



4 √ 3



5 √ 3

 =

160 √ 3 3

es el valor m´aximo.

Ejercicios del cap´ıtulo 1. Determine los valores de a, b ∈ R para los cuales la funci´on f (x, y) = ax2 + by 2 tiene un m´aximo, m´ınimo o punto silla en (0, 0). 2. Sea C ∈ R. Muestre que si f : U ⊆ Rn → R tiene un m´aximo (m´ınimo) local en x = x0 entonces la funci´on F : U → R definida por F (x) = f (x) + C tambi´en tiene un m´aximo (m´ınimo) local en x = x0 . 3. Determinar los valores de a, b ∈ R para los cuales la funci´on f : R2 → R definida por f (x, y) = x3 + 3ax + y 3 + by + 1 no tiene extremos locales. 4. Para cada una de las siguientes funciones determinar los puntos cr´ıticos (o puntos estacionarios), clasificarlos en m´aximos, m´ınimos y puntos silla: a) z = x2 + (y − 1)2

i)

z = x2 − xy + y 2 − 2x + y

b) z = x2 − (y − 2)2

j)

c) z = 1 − x2 − y 2

k)

z = sin (x) sin (y) sin (x + y) 2 2 z = (5x + 7y − 25) e−(x +xy+y )

d) z = (x − y − 1)2

l)

z = (x2 + y 2 ) e−(x

e) z = 2x2 − xy − 3y 2 − 3x + 7y

m) z = xy 2 (x + 2y − 1)

)

n)

z = x3 − x2 y − x2 + y 2

g) z = x3 + y 3 − 3 (x + y) + 1

o)

z = (x + y) (x2 + y 2 − 6)

h) z = xy + (x − y)3

p)

z = (x − y)3 + x4 + y 4

f)

z = x (x2 + y 2 − a2 )

2 +y 2

5. Sea f (x, y) = 3x4 − 4x2 y + y 2 . Demostrar que sobre toda recta de la forma y = mx la funci´on tiene un m´ınimo en (0, 0) pero no es m´ınimo en ning´ un entorno bidimensional del origen. (Estudiar los puntos donde f (x, y) > 0 y f (x, y) < 0). 439

Apuntes Mat023 (versi´on preliminar actualizada 23-05-2014)

6. Considere la funci´on z = f (x, y) que en los alrededores del punto (1, 1, 1) esta definida impl´ıcitamente por z 3 + 3x2 y − y 3 z + y 2 − 3x − 1 = 0 obtener la expansi´on de Taylor para z en (1, 1). 7. Sea f : R4 − {(0, 0, 0, 0)} → R dada por f (x, y, z, u) = x +

y z u 1 + + + x y z u

tiene un punto cr´ıtico en (1, 1, 1, 1) y clasificarlo. 8. Sea g : R → R una funci´on diferenciable que no se anula. Analizar los extremos de Z

(x−1)2

f (x, y) =

Z g (t) dt +

0

(y−1)2

g (t) dt 0

si g (0) > 0. 9. Determine los puntos cr´ıticos de la funci´on f (x, y) = 2x4 + y 4 − 4x2 − 2y 2 y clasificarlos (obs.: Son 9 puntos). 11.* Considere la funci´on z = f (x, y) definida impl´ıcitamente por la expresi´on F (x, y, z) = x3 + y 3 + z 3 − 3x − 3y + z + 4 = 0 obtener sus puntos cr´ıticos y clasificarlos. 10. Determine los valores de las constantes a, b ∈ R para los cuales la integral Z 1 (a + bx − f (x))2 dx 0

sea m´ınimo (suponer f es continua en [0, 1]), estudiar los casos f (x) = x2 y f (x) = x3 + x. 11. Sea f (x, y) = Ax2 + 2Bxy + Cy 2 + 2Dx + 2Ey + F donde A > 0 y B 2 < AC. a) Demostrar que f tiene un m´ınimo. 440

Apuntes Mat023 (versi´on preliminar actualizada 23-05-2014)

b) Demostrar que en el punto de m´ınimo (x1 , y1 ) se cumple f (x1 , y1 ) = Dx1 + Ey1 + F c) Demostrar que A B D 1 B C E f (x1 , y1 ) = AC − B 2 D E F



umeros distintos x1 , x2 , . . . , xn y 12. M´ etodo de los m´ınimos cuadrados: Dados n n´ otros n n´ umeros (no necesariamente distintos) es en general imposible encontrar una recta f (x) = ax + b que pase por todos los puntos (xi , yi ) esto es f (xi ) = yi ∀i, no obstante se puede encontrar los valores de a y b que hacen que el error cuadr´atico total E (a, b) =

n X

(f (xi ) − yi )2

i=1

sea m´ınimo. Encontrar tales valores!!! 13. Comprobar que f (x, y, z) = x4 + y 4 + z 4 − 4xyz tiene un punto cr´ıtico en (1, 1, 1) y clasificarlo como m´aximo, m´ınimo o punto silla. Expandir esta funci´on en Taylor entorno al punto (1, 1, 1) para visualizar directamente la forma cuadr´atica asociada a la Hessiana Hf (1, 1, 1). 14. Estudiar los extremos relativos de la funci´on f (x, y, z) = xyz (1 − x − y − z) 15. Sea A ∈ Mn×n (R), B ∈ M1×n (R) y C ∈ R, se define F : Rn → R F (X) = X T AX + BX + C donde X ∈ Mn×1 (R) (el vector se mira como columna). Estudiar las condiciones bajo las cuales esta funci´on tiene m´aximos, m´ınimos o puntos silla. 16. Considere la funci´on f : R3 → R dada por f (x, y, z) = sin x + sin y + sin z −  sin (x + y + z) determine la expansi´on de Taylor de f en π2 , π2 , π2 ´, muestre adem´as que el punto es un m´ınimo.

441

Apuntes Mat023 (versi´on preliminar actualizada 23-05-2014)

17. Considere la funci´on z = f (x, y) que en los alrededores del punto (1, 1, 1) esta definida impl´ıcitamente por z 3 + 3x2 y − y 3 z + y 2 − 3x − 1 = 0 obtener la expansi´on de Taylor para z en (1, 1). 18. La Hessiana de cierta funci´on f : D ⊆ R3 → R  x2  Hf (x, y, z) =   y xz

es y

xz



 y 2 yx   2 yx z

¿En que subconjunto de R3 debe estar el punto critico (x, y, z) para que sea m´aximo, m´ınimo o punto silla? ¿Cuando esta Hessiana no entrega informaci´on?. 19. Determine los extremos de la funci´on f (x, y) = 2x − y con la restricci´on g (x, y) = 3x2 + 2y 2 − 33/2 = 0 20. Determine los extremos de la funci´on f (x, y) = x2 + 8y si (x, y) se encuentra sobre la elipse x2 + 4y 2 = 5. 21. Utilizar el m´etodo de los multiplicadores para determinar los semiejes de la elipse 5x2 − 6xy + 5y 2 − 32 = 0. 22. Usando multiplicadores de Lagrange, determinar la menor distancia entre las rectas x+4 y−4 z+1 = = 2 −1 −2 x+5 y−5 = =z−5−5 4 −3 23. Demostrar que el paralelep´ıpedo de mayor volumen que se puede inscribir en una esfera es un cubo. 24. Hallar los puntos de la curva x2 + 4y 2 − 4 = 0 que se encuentren mas cercanos y mas lejanos a los puntos de la curva x2 + y 2 + 4x + 2y − 20 = 0

442

Apuntes Mat023 (versi´on preliminar actualizada 23-05-2014)

25. Hallar los extremos absolutos de la funci´on f (x, y) =

x3 3

− 32 x2 + 2x + y 2 − 2y + 1 en

la regi´on K = {(x, y) : x ≥ 0, y ≥ 0, x + y ≤ 1}. 26. Hallar los extremos absolutos de la funci´on f (x, y, z) = (x − 1)2 + (y − 1)2 + (z − 1)2 en la regi´on  K = (x, y, z) ∈ R3 : x2 + y 2 + z 2 ≤ 12 28.* Suponga que queremos determinar los extremos de la funci´on f (x, y, z) sujeta a la restricci´on g (x, y, z) = 0 y utilizando el teorema de Lagrange obtenemos el punto cr´ıtico p0 = (x0 , y0 .z0 ), utilizando el teorema de la funci´on impl´ıcita demostrar que el problema se puede reducir a estudiar los extremos de una funci´on de 2 variables (con el mismo punto cr´ıtico), calcular la Hessiana de esta funci´on de 2 variables y expresarla en t´erminos de f y g para obtener un criterio que permita determinar si el punto cr´ıtico entregado por Lagrange es m´aximo, m´ınimo o silla (Esto es llamado criterio de la hessiana reducida). 27. La fabrica de gelatinas BOB esta interesada en minimizar sus costos de producci´on. Los ingredientes utilizados en la elaboraci´on de su gelatina son tres: Grenetina, az´ ucar y frutas. La f´ormula de producci´on dice que el n´ umero de gelatinas que se puede producir es g (x, y, z) = xα y β z γ donde x es la cantidad de grenetina, y es la cantidad de az´ ucar y z es la cantidad de frutas (todas las cantidades en kilogramos) y las constantes α, β, γ son secretas. Suponga que el precio por kilogramo de los ingredientes es: La grenetina $10,000, az´ ucar $250 y fruta $500. Si se quiere producir 10,000 gelatinas, ¿Qu´e combinaci´on de ingredientes minimiza los costos c (x, y, z) = 10,000x + 250y + 500z? (la respuesta puede quedar en t´erminos de α, β, γ).

443

Cap´ıtulo 11 : Funciones impl´ıcitas e inversas

El teorema de la funci´ on impl´ıcita En los cursos anteriores, frecuentemente nos hemos encontrado con relaciones de la forma F (x, y) = 0 donde F : Ω ⊆ R2 → R es una funci´on de clase C 1 (Ω), por ejemplo, una elipse x2 y 2 + 2 = 1 ⇔ F (x, y) = 0 a2 b donde F (x, y) =

x2 y 2 + 2 −1 a2 b

o la ecuaci´on cuadr´atica general Ax2 + Bxy + Cy 2 + Dx + Ey + F = 0 Estas ecuaciones generalmente definen una curva en el plano que localmente (esto es, en intervalos alrededor de un punto dado) puede ser descrita como la gr´afica de una funci´on y = φ (x) o de una funci´on x = ψ (y). Por ejemplo considere la funci´on F : R2 → R, (x, y) → F (x, y) = x2 + y 2 − 1, esta funci´on es clase C ∞ (R2 ) y F (x, y) = 0 ⇔ x2 + y 2 = 1 Todos los puntos (x, y) con y > 0 que satisfacen x2 + y 2 = 1 corresponden a puntos de la gr´afica de la funci´on ψ (x) =



1 − x2 para x ∈ [−1, 1]

444

Apuntes Mat023 (versi´on preliminar actualizada 23-05-2014)

y=

√ 1 − x2

1

0,5

−1

−0,5

0,5

1

−0,5

esto es, si (x, y) ∈ Graf (ψ), entonces F (x, y) = 0. En efecto (x, y) ∈ Graf (ψ) ⇔ (x, y) = (x, ψ (x)) para alg´ un x ∈ [−1, 1]   √ un x ∈ [−1, 1] ⇔ (x, y) = x, 1 − x2 para alg´ luego (x, y) ∈ Graf (ψ) implica  √  2 F (x, y) = F x, 1 − x √ 2 = x2 + 1 − x2 − 1 = 0 lo que demuestra que todos los puntos de la gr´afica de ψ cumplen la relaci´on F (x, y) = 0 (est´an sobre la circunferencia unitaria). De manera similar, los puntos (x, y) con y < 0 que satisfacen F (x, y) = 0 corresponden a puntos de la gr´afica de la funci´on √ y = ϕ (x) = − 1 − x2 para x ∈ [−1, 1] sin embargo, cerca de los puntos (1, 0) y (−1, 0) no podemos interpretar, los puntos de la curva como puntos del gr´afico de una funci´on de y = f (x), en este caso los puntos corresponden a la gr´afica de x = τ (y) =

p

1 − y 2 para y ∈ [−1, 1] ∧ x = η (y) = −

p 1 − y 2 para y ∈ [−1, 1]

respectivamente. En resumen, en cada punto (x0 , y0 ) ∈ R2 que cumple F (x0 , y0 ) = 0 existe una funci´on ψ : ]x0 − ε, x0 + ε[ → R tal que ψ (x0 ) = y0 y que cumple para cada 445

Apuntes Mat023 (versi´on preliminar actualizada 23-05-2014)

x ∈ ]x0 − ε, x0 + ε[, F (x, ψ (x)) = 0 o bien existe una funci´on ϕ : ]y0 − ε, y0 + ε[ → R tal que ϕ (y0 ) = x0 y F (ϕ (y) , y) = 0 para cada y ∈ ]y0 − ε, y0 + ε[.

y=ψ(x) y0

x0 −ε

x0 x0 +ε

Si tenemos la funci´on G : R2 → R, (x, y) → G (x, y) = x7 + y 7 + xy − 1 y la ecuaci´on G (x, y) = 0 x7 + y 7 + xy = 1 el problema no es tan sencillo. ¿Cerca de que puntos (x, y) podemos encontrar una funci´on y = ψ (x) cuya gr´afica coincida con la curva dada? es decir, los puntos del gr´afico de y = ψ (x) est´en sobre la curva x7 + ψ 7 (x) + xψ (x) = 1 446

Apuntes Mat023 (versi´on preliminar actualizada 23-05-2014)

supuesto que hemos resuelto el problema de la existencia de tal funci´on, ¿Qu´e propiedades de G hereda la funci´on y = ψ (x)? ¿es diferenciable? si es as´ı ¿Cuanto vale su derivada?. Supongamos que la funci´on que encontramos y = ψ (x) es derivable, entonces tenemos G (x, ψ (x)) = 0 por la regla de la cadena ∂G ∂ (x) ∂G (x, ψ (x)) + (x, ψ (x)) ψ 0 (x) = 0 ∂x ∂x ∂y de esto se tiene

− ∂G (x, ψ (x)) ψ (x) = ∂G∂x (x, ψ (x)) ∂y 0

siempre que

∂G ∂y

(x, ψ (x)) 6= 0.

Definici´ on 11.1.1. Sea F : U ⊆ Rn × Rm → Rm una funci´on. Diremos que la funci´on ϕ : V ⊆ Rn → Rm , x → ϕ (x) esta definida impl´ıcitamente por la ecuaci´on F (x, y) = 0 si ∀x ∈ V se cumple F (x, ϕ (x)) = 0 Ejemplo 11.1.1. La funci´on ϕ : R− → R definida por sr 1 6 1 3 ϕ (x) = x − 8x3 + x3 + rq 4 2 3 1 4

2x x6 − 8x3 + 12 x3

esta definida impl´ıcitamente por la ecuaci´on x3 + 6xy = y 3 . En la figura se muestra la curva x3 + 6xy = y 3 y en verde la parte de ella descrita por ϕ. En este caso, puede resultar mas simple trabajar directamente con la ecuaci´on que define la funci´on en lugar de trabajar con la expresi´on expl´ıcita de la funci´on.

447

Apuntes Mat023 (versi´on preliminar actualizada 23-05-2014)

El teorema de la funci´on impl´ıcita nos entrega condiciones sobre la funci´on que define la ecuaci´on, para garantizar todas estas propiedades Teorema 11.1.1. Sean U abierto de Rn+1 y F : U → R es una funci´on de clase C p (U ). Denotemos por (x, z) los puntos de Rn+1 donde x ∈ Rn y z ∈ R. Supongamos que (x0 , z0 ) satisface ∂F (x0 , z0 ) 6= 0 ∂z entonces existe una bola abierta U que contiene a x0 ∈ Rn , una vecindad V de z0 ∈ R F (x0 , z0 ) = 0 y

y una u ´nica funci´on z = g (x) definida para x ∈ U y con recorrido en V que satisface z0 = g (x0 ), F (x, g (x)) = 0 para todo x ∈U adem´as z = g (x) es de clase C p (U) y Dg (x) = −

Dx F (x, g (x)) ∂F (x, g (x)) ∂z

donde Dx F es la derivada respeto a x es decir   ∂F ∂F Dx F = ,..., ∂x1 ∂xn 448

Apuntes Mat023 (versi´on preliminar actualizada 23-05-2014)

en t´erminos de componentes ∂F − ∂x (x, g (x)) ∂g (x) = ∂F i ∂xi (x, g (x)) ∂z

para i = 1, 2, . . . , n.

Ejemplo 11.1.2. Sea G : R3 → R una funci´on de clase C 1 (R3 ). Considere la superficie G (x, y, z) = 0 suponga adem´as que (x0 , y0 , z0 ) es un punto sobre la superficie, es decir G (x0 , y0 , z0 ) = 0 si ∂G (x0 , y0 , z0 ) 6= 0 ∂z por el teorema de la funci´on impl´ıcita existe un conjunto abierto U que contiene (x0 , y0 ) y una funci´on z = z (x, y) definida en U tal que z0 = z (x0 , y0 ) adem´as G (x, y, z (x, y)) = 0 para (x, y) ∈ U el plano tangente a la gr´afica de z = z (x, y) en el punto (x0 , y0 , z0 ) es z − z0 =

∂z ∂z (x0 , y0 ) (x − x0 ) + (x0 , y0 ) (y − y0 ) ∂x ∂y

pero note que − ∂G (x0 .y0 , z0 ) ∂z (x0 , y0 ) = ∂x ∂G(x0 ,y0 ,z0 ) ∂x ∂z

y − ∂G (x0 .y0 , z0 ) ∂z ∂y (x0 , y0 ) = ∂G(x0 ,y0 ,z0 ) ∂y ∂z

reemplazando z − z0 =

− ∂G (x0 .y0 , z0 ) ∂x ∂G(x0 ,y0 ,z0 ) ∂z

(x − x0 ) +

− ∂G (x0 .y0 , z0 ) ∂y ∂G(x0 ,y0 ,z0 ) ∂z

(y − y0 )

as´ı ∂G ∂G ∂G (x0 , y0 , z0 ) (x0 .y0 , z0 ) (x − x0 ) + (x0 .y0 , z0 ) (y − y0 ) + (z − z0 ) = 0 ∂x ∂y ∂z 449

Apuntes Mat023 (versi´on preliminar actualizada 23-05-2014)

que es ∇G (x0 , y0 , z0 ) ((x, y, z) − (x0 .y0 , z0 )) = 0 que es la ecuaci´on del plano tangente a una superficie que obtuvimos por otros medios (el gradiente es perpendicular a los conjuntos de nivel) Ejemplo 11.1.3. Suponga que F : R3 → R es una funci´on de clase C 1 (R3 ) que cumple   ∂F  ∂F  ∂F 6= 0 muestre que las funciones z = z (x, y), x = x (y, z) y y = y (x, z) ∂x ∂y ∂z definidas impl´ıcitamente por F (x, y, z) = 0 cumplen ∂z ∂x ∂y = −1 ∂x ∂y ∂z Soluci´ on. Como

∂F ∂x

6= 0 se sigue que la ecuaci´on define a x como funci´on de y y z, es

decir, x = x (y, z) adem´as ∂F

∂x ∂y = − ∂F ∂y ∂x     ∂F ∂F de manera similar (usando ∂F 6= 0) se tiene y = y (x, z) y z = z (x, y) donde ∂x ∂y ∂z ∂F ∂F ∂z ∂y ∂x ∂z = − ∂F y = − ∂F ∂x ∂z ∂z ∂y

entonces ∂z ∂x ∂y = ∂x ∂y ∂z



∂F ∂x ∂F ∂z

! −

∂F ∂y ∂F ∂x

! −

∂F ∂z ∂F ∂y

!

= −1

Ejemplo 11.1.4. ¿Cerca de cu´ales puntos es posible representar la superficie x3 + 3y 2 + 8xz 2 − 3z 3 y = 1 como gr´afica de una funci´on diferenciable z = k (x, y)? Soluci´ on. Son aquellos puntos para los cuales la derivada parcial respecto a la variable z sea no nula, es decir 16xz − 9z 2 y 6= 0 z (16x − 9zy) 6= 0 450

Apuntes Mat023 (versi´on preliminar actualizada 23-05-2014)

Observaci´ on 11.1.1. Tambi´en se puede intentar despejar las otras variables, formular el teorema en tales casos. Observaci´ on 11.1.2. De la ecuaci´on x3 + 3y 2 + 8xz 2 − 3z 3 y = 1 tambi´en se puede obtener una expresi´on para las derivadas parciales (derivando la ecuaci´on respecto a x) 3x2 + 8z 2 + 16xz as´ı

∂z ∂z − 9z 2 y =0 ∂x ∂x

−3x2 − 8z 2 ∂z = ∂x 16xz − 9z 2 y

Ejemplo 11.1.5. Sea f : R2 → R, (u, v) → f (u, v) una funci´on de clase C 1 (R2 ). Supongamos que

∂f ∂v

6= 0 en todo R2 . Muestre que la ecuaci´on  f x2 − y 2 , y 2 − z 2 = 0

define impl´ıcitamente una funci´on z = z (x, y) para z 6= 0 y que la expresi´on E (x, y, z) = yz

∂z ∂z + zx ∂x ∂y

no depende de f . Soluci´ on. Definamos G (x, y, z) = f (x2 − y 2 , y 2 − z 2 ) entonces G es compuesta de funciones C 1 y por tanto es C 1 , adem´as ∂G ∂f ∂u ∂f ∂v = + ∂z ∂u ∂z ∂v ∂z ∂f = (−2z) 6= 0 ∂v por el teorema de la funci´on impl´ıcita existe una u ´nica funci´on z = z (x, y) de clase C 1 tal que − ∂G ∂z fu ux + fv vx = ∂G∂x = − ∂x fv (−2z) ∂z    fu 2x + fv 0 fu x = − = fv (−2z) fv z 451

Apuntes Mat023 (versi´on preliminar actualizada 23-05-2014)

similarmente − ∂G ∂z fu uy + fv vy ∂y = ∂G = − ∂y fv (−2z) ∂z fu (−2y) + fv (2y) −fu y + fv y = − = fv (−2z) fv z se sigue ∂z ∂z + zx = yz yz ∂x ∂y = xy



fu fv

     x −fu y + fv y + zx z fv z

luego yz

∂z ∂z + zx = xy ∂x ∂y

no depende de f . Estudiaremos ahora el caso m´as general en el cual F : RN × Rm → Rm . Supongamos que tenemos un sistema de ecuaciones lineales 2u + 3v + x − y = 0 u − v + 2x + 3y = 0 sabemos que este sistema tiene infinitas soluciones 4 variables y dos ecuaciones (es cuesti´on de rangos), las infinitas soluciones son porque podemos dejar variables en funci´on de otras, notemos lo siguiente 2u + 3v = b1 = y − x u − v = b2 = −2x − 3y para resolver el sistema 2u + 3v = b1 u − v = b2 podemos utilizar Cramer y se obtiene b1 3 y−x b2 −1 −2x − 3y = u = 2 3 2 3 1 −1 1 −1 452

3 −1 8 7 = − x − y 5 5

Apuntes Mat023 (versi´on preliminar actualizada 23-05-2014)

y 2 1 v = 2 1

b1 b2 = 3 −1

2 y−x 1 −2x − 3y 2 3 1 −1



3 7 = x+ y 5 5

luego las ecuaciones 2u + 3v + x − y = 0 u − v + 2x + 3y = 0 permiten despejar a u y v como funciones de x e y, 7 8 u (x, y) = − x − y 5 5 3 7 v (x, y) = x+ y 5 5 Ahora suponga que tenemos ecuaciones no lineales xu + yvu2 = 2 xu3 + y 2 v 4 = 2 en este caso se nos hace mucho m´as dif´ıcil despejar u, v en funci´on de x e y. Estas ecuaciones las podemos reinterpretar como  F (x, y, u, v) = xu + yvu2 − 2, xu3 + y 2 v 4 − 2 = (F1 , F2 ) = (0, 0) note que las ecuaciones se satisfacen en el punto (1, 1, 1, 1). Sabemos que una buena aproximaci´on de F (x, y, u, v) cerca de (1, 1, 1, 1) es dada por F (1, 1, 1, 1) + DF (1, 1, 1, 1) (x − 1, y − 1, u − 1, v − 1)T donde  DF (x, y, u, v) =   = 

∂F1 ∂x ∂F2 ∂x

u

∂F1 ∂y ∂F2 ∂y 2

vu

u3 2yv 4 453

∂F1 ∂u ∂F2 ∂u

∂F1 ∂v ∂F2 ∂v

  2

x + 2yvu

yu

3xu2

4v 3 y 2

 

Apuntes Mat023 (versi´on preliminar actualizada 23-05-2014)

evaluando en el punto  DF (1, 1, 1, 1) = 

1 1 3 1 1 2 3 4

 

luego  F (x, y, u, v) ≈   ≈ 

1 1 3 1 1 2 3 4

  (x − 1, y − 1, u − 1, v − 1)T

3u + v + x + y − 6 3u + 4v + x + 2y − 10

 

luego la ecuaci´on F (x, y, u, v) = (0, 0) debe ser similar a 3u + v + x + y = 6 3u + 4v + x + 2y = 10 en el cual podemos despejar 3 1 3 4

u y v en funci´on de x e y, para poder hacer esto necesitamos = D(u,v) F (1, 1, 1, 1) ∂F1 1 ∂u (1, 1, 1, 1) ∂F (1, 1, 1, 1) ∂v = ∂F2 ∂F2 ∂u (1, 1, 1, 1) ∂v (1, 1, 1, 1) 6= 0

(permite usar Cramer para despejar las variables) Despejando 6−x−y 1 10 − x − 2y 4 14 2 1 u= = − y − x 9 9 3 3 1 3 4 y 3 6−x−y 3 10 − x − 2y v= 3 1 3 4 454



=

4 1 − y 3 3

Apuntes Mat023 (versi´on preliminar actualizada 23-05-2014)

de donde deber´ıa cumplirse 2 1 ux = − , uy = − 3 9 1 vx = 0, vy = − 3 (note que deber´ıamos poner 14 2 1 − y− x 9 9 3 4 1 v (x, y) ≈ − y 3 3

u (x, y) ≈

(es una aproximaci´on de primer orden de estas funciones) cerca del punto (1, 1, 1, 1)). Estas ideas se resumen en el siguiente: Teorema 11.1.2 (De la funci´on impl´ıcita). Sean Ω abierto de RN , Λ abierto de Rm , F : Ω × Λ → Rm una funci´on de clase C p (Ω × Λ). Pongamos (x, y) para los puntos en RN × Rm . Suponga que (x0 , y0 ) es un punto tal que F (x0 , y0 ) = 0 y que Dy F (x0 , y0 ) es invertible, entonces existe un abierto U ⊆ RN , una funci´on y = g (x) de U a Rm de clase C p (U ) tal que F (x, g (x)) = 0 para todo x ∈ U y0 = g (x0 ) adem´as Dg (x) = −Dy (x, y)−1 Dx (x, y) Ejemplo 11.1.6. Mostrar que cerca del punto (x, y, u, v) = (1, 1, 1, 1) podemos resolver xu + yvu2 = 2 xu3 + y 2 v 4 = 2 de manera u ´nica para u y v como funciones de x y y. Calcular

∂u ∂x



(1, 1)

Soluci´ on. Definamos F

:

((x, y) , (u, v)) →

R2 × R2 → R2  xu + yvu2 − 2, xu3 + y 2 v 4 − 2 455

Apuntes Mat023 (versi´on preliminar actualizada 23-05-2014)

entonces F ((1, 1) , (1, 1)) = (0, 0) calculemos D(u,v) F (x, y, u, v) 

∂ (xu+yvu2 −2) ∂v ∂ (xu3 +y 2 v 4 −2) ∂v

∂ (xu+yvu2 −2) ∂u ∂ (xu3 +y 2 v 4 −2) ∂u

D(u,v) F (x, y, u, v) =   = 

 



2

x + 2yuv

yu

3xu2

4y 2 v 3



en (1, 1, 1, 1) es  D(u,v) F (1, 1, 1, 1) = 

3 1 3 4

 

el determinante es 12 − 3 = 9 6= 0 se sigue que es invertible y as´ı por el teorema de la funci´on impl´ıcita podemos despejar de manera u ´nica para u y v como funciones de x y y. Adem´as g (x, y) = (u (x, y) , v (x, y)) tiene por derivada  Dg (x, y) = − 

x + 2yuv 3xu

2

−1

yu2



2 3

4y v

D(x,y) (x, y, u, v)

donde 

∂ (xu+yvu2 −2) ∂x ∂ (xu3 +y 2 v 4 −2) ∂x

D(x,y) (x, y, u, v) =   = 

u u

3

2

vu

2yv

4

Dg (x, y) = − 

3xu2





2

x + 2yuv





entonces 

∂ (xu+yvu2 −2) ∂y ∂ (xu3 +y 2 v 4 −2) ∂y

yu

4y 2 v 3

−1  



u

2

vu



u3 2yv 4



− 31 − 29



− 13



en el punto en cuesti´on  Dg (1, 1) = − 

3 1 3 4

−1  



456

1 1 1 2





=

0

Apuntes Mat023 (versi´on preliminar actualizada 23-05-2014)

de donde obtenemos 

∂u ∂x ∂v ∂x



(1, 1) (1, 1)

∂u ∂y ∂v ∂y

(1, 1) (1, 1)





=

− 13 0

− 29 − 13

 

se sigue ∂u 1 (1, 1) = − ∂x 3 Ejemplo 11.1.7. Discutir sobre la resolubilidad del sistema 3x + 2y + z 2 + u + v 2 = 0 4x + 3y + z + u2 + v + w + 2 = 0 x + z + w + u2 + 2 = 0 para u, v, w en t´erminos de x, y, z en el punto x = y = z = 0, u = v = 0 y w = −2. Si es posible, obtener la ecuaci´on del plano tangente a w (x, y, z) = 0 en (0, 0, 0). Soluci´ on. Definamos F : R6 → R3 dada por F (x, y, z, u, v, w) =

3x + 2y + z 2 + u + v 2 , 4x + 3y + z + u2 + v + w + 2, x + z + w + u2 + 2

el sistema pude verse como F (x, y, z, u, v, w) = (0, 0, 0) notamos que      D(u,v,w) F (0, 0, 0) =  2u 1 1  2u 0 1 (0,0,0)   1 0 0    =  0 1 1   0 0 1 

457

1

2v 0



Apuntes Mat023 (versi´on preliminar actualizada 23-05-2014)

que tiene determinante 1, por el teorema de la funci´on impl´ıcita se sigue que podemos despejar (u, v, w) en t´erminos de (x, y, z) adem´as    ∂u ∂u ∂u 1  ∂x ∂y ∂z    ∂v ∂v ∂v  =   ∂x ∂y ∂z   0 ∂w ∂w ∂w 0 ∂x ∂y ∂z (0,0,0)  3  =   3 1

0 0

−1 

 1 1   0 1  2 0  3 0   0 1

3 2 0



   4 3 1    1 0 1

de esto obtenemos ∇W = (1, 0, 1) y as´ı el plano es (1, 0, 1) (x, y, z) = 0 x+z = 0

Ejercicios de la secci´ on 1. Sea F : R2 → R, (u, v) → F (u, v) una funci´on de clase C ∞ que satisface F (0, 0) = 0 ∂F ∂F (0, 0) + (0,0) 6= 0 ∂u ∂v Muestre que la ecuaci´on  F x3 − yz, y 3 − xz = 0 define una funci´on z = z (x, y) de clase C ∞ en un entorno del punto (x, y) = (1, 1) que satisface z (1, 1) = 1 y la ecuaci´on  ∂z  ∂z xz + 3y 3 + yz + 3x3 = 9x2 y 2 − z 2 ∂x ∂y  2. Probar que cerca del punto (x0 , y0 , z0 , u0 , v0 ) = 1, 1, 0, π2 , 0 se puede resolver el sistema x2 − y cos (uv) + z 2 = 0 x2 + y 2 − sin (uv) + 2z 2 − 2 = 0 xy − sin u cos v + z = 0 458

Apuntes Mat023 (versi´on preliminar actualizada 23-05-2014)

 π ,0 de manera u ´nica para x, y, z como funciones de u y v. Calcular ∂x ∂v 2   z z 3 3. Sea f : R → R definida por f (x, y, z) = g x + y , y + x donde g : R2 → R y ∂f ∂z

(x, y, z) 6= 0

a) Verificar que la ecuaci´on f (x, y, z) = 0 define impl´ıcitamente a z como una funci´on de clase C 1 de las variables x e y. b) Comprobar que x

∂z ∂z +y = z − xy ∂x ∂y

4. Mostrar que el sistema x2 − y 2 + u2 + 2v 2 = 5 x2 + y 2 − u2 − v 2 = −4 define impl´ıcitamente a u = u (x, y), v = v (x, y) con u (0, 1) = 2 y v (0, 1) = −1. Encontrar las diferenciales Du (0,1) y Dv (0, 1) adem´as de la derivada

∂2v ∂x2

(0, 1).

5. Suponga que F, G : R3 → R son dos funciones de clase C 1 (R3 ) y x0 = (x0 , y0 , z0 ) ∈ R3 es un punto en el cual F (x0 ) = 0, G (x0 ) = 0, esto es un punto de la intersecci´on de las superficies S1 : F (x, y, z) = 0 S2 : G (x, y, z) = 0 Determine una condici´on que permita asegurar que es posible despejar x = x (y), z = z (y) como funciones de clase C 1 en un entorno de y0 luego la intersecci´on se puede parametrizar como x = x (y) y = y z = z (y) para y en un entorno de y0 (el problema nos dice cuando podemos parametrizar con un par´ametro la intersecci´on de dos superficies)

459

Apuntes Mat023 (versi´on preliminar actualizada 23-05-2014)

El teorema de la funci´ on inversa Si f : R → R es una funci´on de clase C 1 (R) y x0 es un punto al que f 0 (x0 ) 6= 0 entonces, por la continuidad de la derivada, podemos garantizar la existencia de un intervalo abierto I tal que f 0 (x) para todo x ∈ I tiene el mismo signo de f 0 (x0 ), entonces la funci´on f es estrictamente creciente o estrictamente decreciente en I (dependiendo si f 0 (x0 ) > 0 o f 0 (x0 ) < 0).

f0

f

f 0 (x0 ) x0

En la gr´afica podemos observar en verde la gr´afica de una funci´on y en azul la de su funci´on derivada, en el punto x0 la derivada es no nula (estrictamente positiva), existe un intervalo (marcado en rojo) en el cual la funci´on derivada es positiva, note que en todo ese intervalo la funci´on f es estrictamente creciente. Si restringimos nuestra funci´on f al intervalo I entonces f es inyectiva (toda funci´on estrictamente creciente o estrictamente decreciente es inyectiva pues env´ıa n´ umeros distintos del dominio en im´agenes distintas), si adem´as restringimos el recorrido a f (I) entonces es sobreyectiva y podemos definir una inversa (no de la funci´on completa, se trata de una inversa local. 460

f0

Apuntes Mat023 (versi´on preliminar actualizada 23-05-2014)

f −1 f

note que solo podemos garantizar la existencia, obtener una expresi´on para ella puede ser muy complicado. ¿Como podemos trabajar entonces con tal funci´on?, ¿Cuales propiedades de f conserva su inversa?. El teorema de la funci´on inversa de una variable asegura que la inversa es una funci´on de la misma clase (en t´erminos de derivadas) adem´as  f ◦ f −1 (x) = x entonces d dx por la regla de la cadena

  f ◦ f −1 (x) = 1

 df −1 df −1 (x) = 1 f (x) dx dx

entonces

df −1 1 (x) = 0 −1 dx f (f (x))

si queremos calcular la derivada de segundo orden d 1 d2 f −1 (x) = dx2 dx f 0 (f −1 (x))  −2 00 −1  0 −1 = − f f (x) f f (x) =

−f 00 (f −1 (x)) (f 0 (f −1 (x)))3

461

1 f 0 (f −1 (x))



Apuntes Mat023 (versi´on preliminar actualizada 23-05-2014)

Si T : Rn → Rn es una transformaci´on lineal, sabemos que es invertible si y solo si su matriz asociada (por ejemplo respecto a las bases can´onicas) es invertible, sabemos que la matriz asociada respecto a las bases can´onicas corresponde a la matriz Jacobiana de la T . JT (x0 ) = [T ]CC luego, si det (JT (x0 )) 6= 0 la transformaci´on lineal es invertible (derivada no nula implica invertible), es razonable esperar que si una funci´on se parece a una transformaci´on lineal localmente entonces podamos obtener informaci´on de ella a trav´es de la transformaci´on lineal a la cual se parece, eso es justamente lo que afirma el teorema de la funci´on inversa de varias variables. Observaci´ on 11.2.1. Supongamos que U, V son abiertos y F : U ⊆ Rn → V ⊆ Rm (diferenciable) tiene inversa (diferenciable) entonces n = m. En efecto, si G : V → U es su inversa entonces F ◦G

:

V →V

x → (F ◦ G) (x) = x y G◦F

:

U →U

x → (G ◦ F ) (x) = x luego D (F ◦ G) = Im D (G ◦ F ) = In as´ı DF (G (x)) DG (x) = Im DG (F (v)) DF (v) = In si ponemos x =F (v) entonces G (x) = v entonces DF (G (x)) DG (x) = Im DG (x) DF (G (x)) = In 462

Apuntes Mat023 (versi´on preliminar actualizada 23-05-2014)

pongamos A = DF (G (x)) y B = DG (x) entonces AB = In BA = Im definamos las transformaciones lineales TA

:

Rn → Rm

x → TA (x) = Ax y TB

:

Rm → Rn

x → TB (x) = Bx entonces TA ◦ TB es IRm y TB ◦ TA es IRn de la primera se obtiene que TB es inyectiva TB (x) = TB (y) ⇒ TA (TB (x)) = TA (TB (y)) ⇒ x=y y de la segunda que TB es sobreyectiva (si y ∈ Rn entonces TA (y) = v ∈ Rm entonces TB (v) = TB (TA (y)) = y de donde Im(TB ) = Rn ) se sigue por el teorema de las dimensiones que Dim (Rn ) = Dim (Rm ) as´ı n = m.

Teorema 11.2.1 (De la funci´on inversa). Sean U un abierto de Rn , F : U ⊆ Rn → Rn una funci´ on de clase C p (U ) y x0 ∈ U un punto en el cual det (DF (x0 )) 6= 0 entonces existe un abierto U1 ⊆ U tal que x0 ∈ U1 y F : U1 → V = F (U1 ) es invertible, su inversa F −1 : V → U1 es de clase C p (V ), adem´as DF −1 (F (x)) = (DF (x))−1 para x ∈ U1 en particular, si F (x0 ) = y0 entonces DF −1 (y0 ) = (DF (x0 ))−1 463

Apuntes Mat023 (versi´on preliminar actualizada 23-05-2014)

Observaci´ on 11.2.2. Si x = (x1 , x2 , . . . , xn ) y F : D ⊆ Rn → Rn , x → F (x) = (F1 (x) , F2 (x) , . . . , Fn (x)) entonces det (DF (x0 )) se representa por el s´ımbolo ∂ (F1 , F2 , . . . , Fn ) ∂ (x1 , x2 , . . . , xn ) x=x0 y es llamado el Jacobiano (es el determinate de la matriz Jacobiana).

Ejemplo 11.2.1. Muestre que la funci´on F :

R2



R2

(x, y) → F (x, y) = (u (x, y) , v (x, y)) = (ex cos y, ex sin y) no tiene una inversa global (definida en R2 ), sin embargo, cerca de todo punto es posible definir una inversa local. Calcular DF −1

√ √  2, 2

√ √   2, 2 y obtener una expresi´on para si F ln 2, π4 = DF −1 (u, v) si (u, v) 6= 0. Soluci´ on. Notemos F (0, 2π) = F (0, 0) = (1, 0) luego F no es inyectiva y por tanto no tiene inversa global. Notemos que F ∈ C ∞ (R2 ) adem´as 

∂F1 ∂x

 DF (x, y) =    = 

∂F1 ∂y

∂F2 ∂x

∂F2 ∂y





  =  

∂u ∂x

∂v ∂x

ex cos y −ex sin y



ex sin y



ex cos y

∂u ∂y

∂v ∂y

   

y 

x

x



e cos y −e sin y ∂ (u, v)  = e2x 6= 0 = det  x x ∂ (x, y) e sin y e cos y 464

Apuntes Mat023 (versi´on preliminar actualizada 23-05-2014)

as´ı, por el teorema de la funci´on inversa, F es invertible cerca de cada punto de R2 (inversa local) adem´as DF −1 (F (x, y)) = (DF (x, y))−1  −1 ex cos y −ex sin y  =  x x e sin y e cos y de esto se obtiene     π   DF −1 F ln 2, =  4    =  

π −1 4    π  π ln 2 ln 2 e cos e sin 4 4 √ √  1 1 2 2 4 4    √ √ 1 1 −4 2 2 4 eln 2 cos

esto es  DF −1

√ √   2, 2 =  

1 4

π 4

√ 2

√ − 41 2

−eln 2 sin

√  2    √ 1 2 4 1 4

si F (x, y) = (u, v) entonces F (u, v) = (x, y) donde x = x (u, v) e y = y (u, v) adem´as     as´ı por ejemplo

∂x ∂u

√ √  2, 2

∂x ∂v

∂y ∂u

√ √  2, 2

∂y ∂v

√ √    1√ 2, 2 2   4 =   √ √  √ − 14 2 2, 2

∂x √ √  1 √ 2, 2 = 2 ∂u 4

465

√  2    √ 1 2 4 1 4

Apuntes Mat023 (versi´on preliminar actualizada 23-05-2014)

en general DF −1 (u, v) = DF −1 (F (x, y))  −1 ex cos y −ex sin y  =  ex sin y ex cos y  −1 u −v  =  v u   v u u2 + v 2   u2 + v 2   =     u v − 2 u + v2 u2 + v 2 Observaci´ on 11.2.3. Note que el teorema de la funci´on inversa afirma DF −1 (y0 ) = (DF (x0 ))−1 luego ∂ (u, v) = ∂ (x, y)

1 ∂(x,y) ∂(u,v)

cuidando donde est´an evaluadas las funciones. Ejemplo 11.2.2. Sea f : R2 → R2 la funci´on definida por f (u, v) = u2 + u2 v + 10v, u + v 3



1. Mostrar que tiene inversa cerca del punto (1, 1) Soluci´ on. Notemos que la funci´on es de clase C ∞ (R2 ) y   Df (1, 1) =   = 

∂F1 ∂u ∂F2 ∂u

∂F1 ∂v ∂F2 ∂v

 ∂ (u2 +u2 v+10v ) ∂v ∂ (u+v 3 ) ∂v

∂ (u2 +u2 v+10v ) ∂u ∂ (u+v 3 ) ∂u

 2u (v + 1) u + 10  =  2 1 3v (1,1)   4 11  =  1 3 

2

466

 

Apuntes Mat023 (versi´on preliminar actualizada 23-05-2014)

luego 

4 11

det 

1

3

  = 1 6= 0

y as´ı, por el teorema de la funci´on inversa, la funci´on es localmente invertible, adem´as f (1, 1) = (12, 2) 2. Calcular la derivada de su inversa en el punto (12, 2). Soluci´ on. Como f (1, 1) = (12, 2) se sigue  Df −1 (12, 2) = Df −1 (f (1, 1)) =   = 

3

−11

−1

4

4 11 1

3

−1 

 

Ejemplo 11.2.3. Sea u : R2 → R, (x, y) → u (x, y) una funci´on que satisface 2xy

 ∂u ∂u − x2 − y 2 =0 ∂x ∂y

1. Utilizar el teorema de la funci´on inversa para determinar condiciones sobre la funci´on ζ de modo que F

:

D ⊆ R2 → R2

F (x, y) = (µ (x, y) , ζ (x, y))   x = , ζ (x, y) x2 + y 2 sea invertible. Soluci´ on. Por el teorema de ∂µ ∂µ ∂x ∂y ∂ζ ∂ζ = ∂x ∂y

la funci´on inversa, si x2 −y 2 2xy − (x2 +y2 )2 − (x2 +y2 )2 ∂ζ ∂ζ ∂x ∂y     x2 − y 2 ∂ζ 2xy ∂ζ − + = ∂y ∂x (x2 + y 2 )2 (x2 + y 2 )2 6= 0 467

Apuntes Mat023 (versi´on preliminar actualizada 23-05-2014)

2. Escribir la ecuaci´on 2xy

 ∂u ∂u − x2 − y 2 =0 ∂x ∂y

en las nuevas variables (µ, ζ) mediante regla de la cadena, resolver esa nueva ecuaci´on y concluir que u debe tener la forma  u (x, y) = f

x 2 x + y2



donde f ∈ C 1 (R) es una funci´on arbitraria. Soluci´ on. ∂u ∂µ ∂u ∂ζ ∂u = + ∂x ∂µ ∂x ∂ζ ∂x ∂u ∂µ ∂u ∂ζ ∂u = + ∂y ∂µ ∂y ∂ζ ∂y de donde   ∂u ∂u x2 − y 2 = − + ∂x ∂µ (x2 + y 2 )2   ∂u ∂u 2xy = − + ∂y ∂µ (x2 + y 2 )2

∂u ∂ζ ∂ζ ∂x ∂u ∂ζ ∂ζ ∂y

se sigue  ∂u ∂u − x2 − y 2 ∂x ∂y     2 ∂u x − y2 ∂u ∂ζ = 2xy − + ∂µ ∂ζ ∂x (x2 + y 2 )2      ∂u 2xy ∂u ∂ζ 2 2 − x −y − + ∂µ ∂ζ ∂y (x2 + y 2 )2  ∂u ∂ζ ∂u ∂ζ = 2xy − x2 − y 2 ∂ζ ∂x ∂ζ ∂y    ∂u ∂ζ 2 2 ∂ζ = 2xy − x −y ∂ζ ∂x ∂y

0 = 2xy

pero    ∂ζ 2 2 ∂ζ 2xy − x −y 6= 0 ∂x ∂y entonces ∂u =0 ∂ζ 468

Apuntes Mat023 (versi´on preliminar actualizada 23-05-2014)

se sigue u (µ, ζ) = f (µ) as´ı  u (x, y) = f

x 2 x + y2



donde f es una funci´on arbitraria.

Ejercicios de la secci´ on 1. Sea f : R → R una funci´on de clase C 1 y sea u = f (x) v = −y + xf (x) si f 0 (x0 ) 6= 0 probar que la funci´on T (x, y) = (u, v) es invertible cerca de (x0 , y0 ) y que la inversa tiene la forma x = f −1 (u) y = −v + uf −1 (u) encontrar DT −1 .   2 2. Considere la funci´on F (x, y) = (x − y)2 , xy para y 6= 0. a) Probar que F admite inversa local en una vecindad de (−1, 1) b) Sea F −1 : V ⊆ R2 → R2 , (x, y) = (g (u, v) , h (u, v)) la inversa local de F , calcular la raz´on de cambio de h en (4, 1) en la direcci´on del vector (2, −1). 3. Sea F (x, y) = (f1 (x, y) , f2 (x, y)) = (x cos y, sin (x − y)). Mostrar que F tiene inversa  local en una vecindad del punto π2 , π2 y obtener la Jacobiana de la inversa en (0, 0). 4. Definimos x : R2 → R, (r, θ) → x (r, θ) = r cos θ y y : R2 → R, (r, θ) → y (r, θ) = r sin θ.

469

Apuntes Mat023 (versi´on preliminar actualizada 23-05-2014)

a) Demostrar que ∂ (x, y) = r0 ∂ (r, θ) (r0 ,θ0 ) b) ¿cu´ando se puede formar una funci´on inversa suave de F : R2 → R2 , (r, θ) → (x (r, θ) , y (r, θ))? Comprobarlo directamente y con el teorema de la funci´on inversa. 5. Definamos F : R3 → R3 , (ρ, φ, θ) → F (ρ, φ, θ) = (x (ρ, φ, θ) , y (ρ, φ, θ) , z (ρ, φ, θ)) donde x (ρ, φ, θ) = ρ sin φ cos θ y (ρ, φ, θ) = ρ sin φ sin θ z (ρ, φ, θ) = ρ cos φ a) Muestre que ∂ (x, y, z) = ρ2 sin φ ∂ (ρ, φ, θ) b) ¿Cu´ando se puede despejar (ρ, φ, θ) en t´erminos de (x, y, z)?

Ejercicios del cap´ıtulo 1. Si u1 , u2 son las ra´ıces de la ecuaci´on cuadr´atica en λ x2 + y 2 z2 + =1 λ−2 λ−1 y u3 es arctan (y/x) probar que 4z ∂ (u1 , u2 , u3 ) =− ∂ (x, y, z) u1 − u2 √ √  2. La ecuaci´on x3 + y 3 = 6xy define cerca del punto 2 3 2, 2 3 4 a y como funci´on de x. √ Muestre que x = 2 3 2 es un punto cr´ıtico de tal funci´on y clasificarlo en m´aximo, m´ınimo o punto silla.

470

Apuntes Mat023 (versi´on preliminar actualizada 23-05-2014)

3. La ecuaci´on y z  , =0 x x define a z como funci´on de x, y, z = g (x, y). Muestre que f

x

∂g ∂g +y =g ∂x ∂y

4. Si ϕ, φ : D ⊆ R3 → R son funciones de clase C 1 tales que ∂ (ϕ, φ) 6= 0 ∂ (y, z) entonces las ecuaciones ϕ (x, y, z) = 0, φ (x, y, z) = 0 determinan y, z como funciones de x y dy dx

=

∂(ϕ,φ) ∂(z,x) ∂(ϕ,φ) ∂(y,z)

dz dx

=

∂(ϕ,φ) ∂(x.y) ∂(ϕ,φ) ∂(y,z)

5. Si z1 = er cos θ, z2 = er sin θ y x = r cos θ, y = r sin θ probar que ∂ (z1 , z2 ) e2r = ∂ (x, y) r 6. Suponga que x, y, z, r, θ, φ est´an relacionados por f (x, y, z) = 0 y x = r sin θ cos φ

y = r sin θ sin φ

z = r cos θ

probar que 

 ∂r ∂r sin θ sin θ + r cos θ + sin φ ∂θ ∂φ   ∂r − sin θ cos φ cos θ − r sin θ = 0 ∂θ ∂z ∂x



7. Sea a > 0. Si y 4 + 3a2 y 2 − 5a2 xy + 2a2 x2 = 0, probar que dy 2 dy = 0 o l´ım = x→0 dx x→0 dx 3 l´ım

8. Si u, v son funciones de las variables x, y por medio de las ecuaciones f (x, y, u) = 0 y g (u, v, x) = 0 probar que ∂v fx gu − fu gx = ∂x f u gv ∂v fy gu = ∂y fu gv 471

Apuntes Mat023 (versi´on preliminar actualizada 23-05-2014)

9. La ecuaci´on

∂f ∂z

(x, z) = u define a z como funci´on de x y u. Si ponemos z = g (x, u)

y se define ϕ (x, u) = f (x, g (x, u)) probar que ∂z ∂ϕ = u ∂u ∂u ∂ϕ ∂f ∂z = +u ∂x ∂x ∂x 10. Suponiendo que u, v son funciones de x, y las cuales definen a u, v en t´erminos de x, y y satisfacen las ecuaciones ∂u ∂v ∂u + +u = 0 ∂x ∂y ∂y ∂v ∂v ∂u + +u = 0 ∂x ∂y ∂y Probar que

∂ 2x ∂ 2x ∂x − 2 = 2 ∂u ∂v ∂v

11. Si u1 x1 = u2 x2 = u3 x3 = u4 x4 = (u1 u2 u3 u4 )1/2 entonces ∂ (u1 , u2 , u3 , u4 ) = −1 ∂ (x1 , x2 , x3 , x4 ) 12. Determinar el Jacobiano de la transformaci´on x = a (ρ cos θ sin φ)α y = b (ρ sin θ sin φ)β z = c (ρ cos φ)γ donde a, b, c, α, β.γ son constantes positivas y determinar cuando es posible invertir tal transformaci´on. Ind.: Compuestas de funciones.

472

Parte III Evaluaciones de a˜ nos anteriores

473

Cap´ıtulo 12 : Controles

Control 1 1. Dada la funci´on f : D ⊆ R2 → R definida por f (x, y) =

q

x−y x+y

a) Determine y grafique el dominio de f . b) ¿El dominio de f es un conjunto abierto? c) Determine la frontera del dominio de f y sus puntos de acumulaci´on. d ) Determine las curvas de nivel f (x, y) = c y grafique para c = 0, c = 1, c =

1 2

y

c = 2. e) Analice que sucede cuando c crece indefinidamente. 2. Sea T : R3 [x] → R2 [x] definida por 00

Z

1

T [p (x)] = p (x) +

p (x) dx 0

a) Pruebe que T es una transformaci´on lineal. b) Sean B1 =



1, x − 1, (x − 1)2 , (x − 1)3



B2 = {1, x, x (x − 1)} bases de R3 [x] y R2 [x] respectivamente. Determine [T ]BB21 y use esta matriz para obtener el n´ ucleo de T .

Control 2 1. Sea T : R4 → R3 una transformaci´on lineal definida por T (x, y, z, w) = (x − y + z + w, x + 2z − w, x + y + 3z − 3w) Encuentre una base y la dimensi´on de Ker(T ) e Im(T ). 2. Describa expl´ıcitamente una transformaci´on lineal T : R3 → R2 tal que Im (T ) = h{(1, 0, −1) , (1, 2, 2)}i 474

Apuntes Mat023 (versi´on preliminar actualizada 23-05-2014)

Control 3 1. Considere la funci´on T : R2 [x] → R3 definida por T (p (x)) = (p (0) , p (1) , p (2)). Sean B = {1, x, x2 } y C = {(1, 0, 0) , (0, 1, 0) , (0, 0, 1)} bases de R2 [x] y R3 respectivamente: a) Pruebe que T es una transformaci´on lineal. b) Determine Ker(T ) y [T ]CB . c) ¿T es un isomorfismo?. 2. Considere la funci´on f : D ⊆ R2 → R, definida por f (x, y) =

p

sin (x2 + y 2 ).

a) Encuentre el dominio de f y dibujarlo. b) Determine si el dominio es abierto o cerrado.

Control 4 1. Sea f (x, y) =

    x + y si |x| + |y| ≥ 2   

¿Existe

l´ım

1

si |x| + |y| < 2

f (x, y)? Justificar.

(x,y)→(1,1)

2. Demuestre o d´e un contraejemplo de la propiedad: Para todo A, B ⊆ Rn ∂ (A ∪ B) ⊆ ∂A ∪ ∂B

Control 5 1. Considere la funci´on f : {(x, y) ∈ R2 : x 6= 0} → R, definida por 2

f (x, y) = x + y

2



arctan

y x

¿es posible definir f en (0, 1) de forma que f sea continua en tal punto?

475

Apuntes Mat023 (versi´on preliminar actualizada 23-05-2014)

2. Sea f : R2 → R definida por

f (x, y) =

Hallar

∂f ∂x

(x, y) y

∂f ∂x

   

xy x2 +y

si x2 6= −y

  

0

si x2 = −y

(x, y) en todos los puntos que estas existan y determinar si son

continuas en (0, 0).

Control 6 1. Considere el sistema x = u+v+w y = u2 + v 2 + w 2 z = u3 + v 3 + w 3 Calcule

∂v ∂y

(2, 6, 8) y

∂w ∂x

(2, 6, 8) donde (x, y, z) = (2, 6, 8) es la imagen de (u, v, w) =

(1, 2, −1). 2. Considere el sistema u3 + xv 2 + y = 0 v 3 + yv + u2 = 0 a) Pruebe que es posible despejar x = x (u, v), y = y (u, v) en vecindades de los puntos (x, y) = (1, −1) y (u, v) = (0, 1). b) Hallar

∂u ∂x

(1, −1) y

∂v ∂x

(1, −1).

Control 7 1. Un servicio de entrega de paquetes requiere que las dimensiones de una caja rectangular sean tales que la longitud m´as el doble del ancho m´as el doble de la altura no rebase 108 cms. ¿Cu´al es el volumen de la caja m´as grande que podr´a enviar la compa˜ n´ıa? 476

Apuntes Mat023 (versi´on preliminar actualizada 23-05-2014)

2. Considerar el sistema xy 2 + xzu + yv 2 = 3 u3 yz + 2xv − u2 v 2 = 2 ¿Es posible despejar u = u (x, y, z) y v = v (x, y, z) en vecindades de U de (x, y, z) y V de (u, v) = (1, 1). Calcular 3. Resuelva la ecuaci´on

∂v ∂y

(1, 1, 1).

dy 1 − xy 2 = dx 2x2 y

haciendo la sustituci´on v = y/xn para alg´ un n. 4. Muestre que la ecuaci´on y y dy = + xm y n f dx x x se transforma en una ecuaci´on de variables separables usando el cambio de variables y = vx donde v = v (x). Use lo anterior para resolver la ecuaci´on  y y sec2 x dy = + dx x y2

Control 8 1. Encuentre la soluci´on general de la ecuaci´on d4 y d3 y d2 y dy − + 2 −3 − 6y = 0 4 3 dx dx dx dx 2. Un estanque contiene 50 litros de agua pura. Al estanque entra salmuera, que contiene C gramos de sal por litro a raz´on de 1,5 litros por minuto. La mezcla bien revuelta, sale a raz´on de 1 litro por minuto. Si despu´es de 30 minutos la concentraci´on de sal en el estanque es de 30 gramos por litro. Hallar el valor de C.

477

Apuntes Mat023 (versi´on preliminar actualizada 23-05-2014)

Control 9 1. Hallar

dz dx

si se cumple x3 + y 3 + z 3 = 0 y x2 + y 2 + z 2 = 1

2. La matriz

 A=

1 −1 1

2

 

representa el jacobiano de una funci´on f de clase C 2 de R2 en R2 en el punto (1, 1). Suponga que f (1, 1) = (−1, 0) y que f (x, y) = (u, v). Pruebe que f es localmente invertible en (1, 1) y encuentre

∂y ∂u

(−1, 0).

Control 10 1. Muestre que la ecuaci´on diferencial 2x4 yy 0 + y 4 = 4x6 se reduce a una ecuaci´on homog´enea mediante el cambio de variables y = z n para cierto n ∈ R. Determine el valor de n y resuelva la ecuaci´on. 2. Resuelva la ecuaci´on x3 yy 0 + 2x2 y 2 − 1 = 0 usando el cambio de variables u = x2 y.

Control 11 1. Usando la transformada de Laplace resuelva el problema de valor inicial  ty 00 − ty 0 + y = 2 et − 1 con y (0) = 0, y 0 (0) = −1. 2. Obtenga la transformada de Laplace inversa de s2 − s s3 + s2 + 9s + 9

478

Apuntes Mat023 (versi´on preliminar actualizada 23-05-2014)

Control 12 1. Resuelva el problema de valores iniciales dy − ex−y + ex = 0 dx y (0) = 1 Ayuda. Use el cambio de variables u = ey . 2. Resuelva la ecuaci´on y 2 dx = (x3 − xy) dy usando un factor integrante de la forma xn y m .

Control 13 Sea α ∈ R − Z. 1. Encontrar el desarrollo en serie de Fourier de la funci´on f : ]−π, π[ → R f (x) = cos (αx) 2. Verificar que " # ∞ 1 1 X 2α cot (απ) = − π α n=1 n2 − α2

Control 14 1. La recta normal, en cada punto (x, y) de la curva dada, pasa por el punto (2, 0). Si la curva pasa por (2, 3) encuentre su ecuaci´on. Justificar. 2. Resolver la ecuaci´on (2x + 1)2 y 00 − 2 (2x + 1) y 0 − 12y = 6x

Control 15 1. Obtenga la soluci´on de las ecuaciones a) (6xy 2 − 3x2 ) dx = − (6x2 y + 3y 2 − 7) dy 479

Apuntes Mat023 (versi´on preliminar actualizada 23-05-2014)

b) xy 0 − y =

y ln y−ln x

2. En la schoper´ıa de “Don Ram´on” se vende cerveza artesanal a 4 o C. Dado que se est´a en ´epoca de verano la temperatura interior de la schoper´ıa es de 30 o C. El se˜ nor Ram´on es muy exigente con la calidad de su producto y pide a los meseros no servir Schops si es que ´estos est´an a m´as de 7 o C. Sabiendo que a los 60 segundos la cerveza llega a los 5 o C y que adem´as un mesero demora 2 minutos en transportar la cerveza desde su fuente hasta la mesa ¿podr´an los meseros cumplir con las exigencias de Don Ram´on? (justifique matem´aticamente)

Control 16 1. Calcular 

 sin t L (s) t 2. Resolver ty 00 + 2y 0 + ty = 0 con y (0) = 1 e y (π) = 0. 3. Encontrar el desarrollo en serie de Fourier para   0 −π < x < 0 f (x) =  π−x 0 0, y > 0, z > 0. Use el resultado para probar que para n´ umeros reales positivos a, b, c tenemos 3



abc ≤ 27

a+b+c 5

5

4. Sea U = {(x, y, z) ∈ R3 : xy 6= 0}. Definamos f : U ⊆ R3 → R por   z z f (x, y, z) = g x + , y + y x donde g : R2 → R es una funci´on de clase C 1 . Suponga que para cada (x, y, z) ∈ U ,

∂f (x, y, z) 6= 0 ∂z

a) Considere un punto (x0 , y0 , z0 ) ∈ U tal que f (x0 , y0 , z0 ) = 0. Argumente que f (x, y, z) = 0 define impl´ıcitamente a z como funci´on de las variables x e y, es decir, z = z (x, y) en una vecindad de (x0 , y0 ) tal que z0 = z (x0 , y0 ). b) Demuestre que la funci´on z de la parte anterior satisface la ecuaci´on x

∂z ∂z +y = z − xy ∂x ∂y 490

Apuntes Mat023 (versi´on preliminar actualizada 23-05-2014)

Certamen 3 1. Resuelva la siguiente ecuaci´on y 0 = e2x y 2 − 2y − 9e−2x con y (0) = 4 sabiendo que tiene una soluci´on particular de la forma y (x) = aekx . 2. Un tanque contiene inicialmente 60 gal. de agua pura. Entra al tanque, a una tasa de 2 gal./min., salmuera que contiene 1 lb. de sal por gal´on, y la soluci´on (perfectamente mezclada) sale de ´el a raz´on de 3 gal. por minuto; el tanque se vac´ıa despu´es de una hora exactamente. a) Encuentre la cantidad de sal que hay en el tanque despu´es de t minutos. b) ¿Cu´al es la cantidad m´axima de sal que llega a tener el tanque?. 3. Resuelva la ecuaci´on x 1 − x2

2

y 00 − 1 − x2

2

y 0 + x3 y = 0

usando el cambio de variables t = − 12 ln (1 − x2 ) 4. Encuentre la soluci´on general de la ecuaci´on  2 x2 − 2x y 00 + 2 (1 − x) y 0 + 2y = 6 x2 − 2x sabiendo que y = x − 1 es una soluci´on de la ecuaci´on homog´enea asociada.

491

Apuntes Mat023 (versi´on preliminar actualizada 23-05-2014)

Certamen 4 1. Resuelva las ecuaciones dy + x2 y = 2y −4/3 a) x3 dx

b) (x2 + y 2 + 1) dx − (xy + y) dy = 0 2. Suponga que un individuo infectado se introduce en una poblaci´on de tama˜ no N, todos los individuos de la cual son susceptibles a la enfermedad . Si suponemos que la tasa de infecci´on es proporcional al producto del n´ umero de infectados y el de susceptibles presentes, ¿cu´al ser´a el n´ umero de infecciones en el tiempo t?. 3. Para x > 0, considere la ecuaci´on  2 xy 00 + x2 − 1 y 0 + x3 y = e−x /4 a) Use el cambio de variables t = x2 /2 para encontrar la soluci´on general de la homog´enea. b) Resuelva la no homog´enea usando variaci´on de par´ametros.

492

Apuntes Mat023 (versi´on preliminar actualizada 23-05-2014)

Certamen 5 1.

a) Pruebe que el cambio de variables u = ax + by + c transforma la ecuaci´on diferencial y 0 = f (ax + by + c) en una ecuaci´on diferencial aut´onoma, donde a, b y c son constantes reales. b) Use el resultado anterior para transformar la ecuaci´on dy = (y − 2x) − (y − 2x)2 + 2 dx

(ec-1)

en una ecuaci´on aut´onoma. c) Haga la linea de fase de la ecuaci´on aut´onoma obtenida en el item b). d ) En el plano xy haga un esbozo de las soluciones de la ecuaci´on (ec-1). e) Obtenga la soluci´on general de (ec-1). 2. Determine una soluci´on de la forma y (x) = xn para la ecuaci´on  x2 − 1 y 00 (x) − 2xy 0 (x) + 2y (x) = 0, x > 1 donde n es alg´ un entero positivo. Usando el resultado anterior, hallar la soluci´on del problema de valores iniciales   (x2 − 1) y 00 (x) − 2xy 0 (x) + 2y (x) = (x2 − 1)2 x > 1  y (2) = 2, y 0 (2) = 10 3. Considere el sistema lineal de ecuaciones diferenciales dx = y dt dy = −4x − 2ay dt con a ∈ R: a) Para los diferentes valores del par´ametro real a, haga un retrato de fase del sistema de ecuaciones indicando las direcciones en las cuales las curvas son recorridas. b) Obtenga la soluci´on general del sistema. 493

Apuntes Mat023 (versi´on preliminar actualizada 23-05-2014)

4. Supongamos que la tasa de cambio del precio x de un bien, crece en el tiempo a una raz´on constante c como resultado de la inflaci´on constante, al mismo tiempo cae en forma proporcional a la diferencia entre la oferta y en el tiempo t y alguna oferta de equilibrio y0 , es decir,

dx dt

= c − α (y − y0 ). Tambi´en asumimos que la tasa de cambio

de la oferta es proporcional a la diferencia entre el precio y alg´ un precio de equilibrio x0 , es decir,

dy dt

= β (x − x0 ). (α y β)son constantes de proporcionalidad positivas.

Asumiendo que en t = 0,x = x0 y y = y0 a) Muestre que el precio y la oferta oscilan alrededor de x0 y y0 + c/α respectivamente. b) Si en el tiempo t = T0 el precio es m´aximo ¿en qu´e tiempo la oferta es m´axima?

494

Apuntes Mat023 (versi´on preliminar actualizada 23-05-2014)

Certamen 6 1. Se ha determinado experimentalmente que la variaci´on de peso de un tipo de pez sigue la ley β dp = αe− 3 t p2/3 − βp dt

donde p = p (t) representa el peso, α y β son constantes positivas. Si p (0) = p0 > 0 determine el peso m´aximo del pez. a) Hallar una ecuaci´on diferencial lineal con soluci´on general c1 et cos (2t) + c2 et sin (2t) + t5 b) Considere la ecuaci´on y 00 + q (x) y = 0 donde q es continua en todo R y dos soluciones y1 , y2 las cuales satisfacen y1 (0) = 1, y10 (0) = 0 y y2 (0) = 3, y20 (0) = 1. Si W (x) es el Wronskiano de y1 e y2 en x demuestre que W (x) = 1 para todo x donde las soluciones est´en definidas. 2

d y dy 2. Muestre que ϕ (x) = ex es una soluci´on de la ecuaci´on x dx 2 − (x + 2) dx + 2y = 0 y

determinar la soluci´on general de x

d2 y dy − (x + 2) + 2y = x3 2 dx dx

3. Sea C la base can´onica de R3 . Sean T, L : R3 → R3 transformaciones lineales tales que T (1, 1, 1) = (1, −3, 3), T (1, 1, 0) = (2, −3, 2), T (1, 0, 0) = (−1, −1, 2) y   −2 −3 −2   [L]CC =  1 1   1  4 6 5 a) Determine T expl´ıcitamente. b) Determine [T ◦ L]CC c) ¿Qu´e relaci´on existe entre T y L? d ) Determinar Ker(T ) e Im(L).

495

Apuntes Mat023 (versi´on preliminar actualizada 23-05-2014)

Certamen 7 1.

a) Calcular la serie de Fourier de la funci´on     1 si 0 < x < π f (x) =    −1 si −π < x < 0 1 1 1 π = 1 − + − + ··· 4 3 5 7 b) Usando la serie de Fourier anterior, determine la serie de Fourier de     a si 0 < x < π g (x) =    b si −π < x < 0 y usando la serie muestre que

para a, b ∈ R (No se entregar´an puntos por calcular esta serie con las f´ormulas de los coeficientes de Fourier). 2. Resolver el P.V.I.

  t si t < 1 dy + 9y =  0 si t > 1 dt2 2

con y (0) = y 0 (0) = 0. 3. Resolver el sistema Z 0

t

e Z

t−u

0

Z

t

(t − u) y (u) du = t2 x (u) du + Z 0t t x (u) du + (t − u)2 y (u) du = t3 0

4. Sea α ∈ R− {1, 2}. Considere el sistema de ecuaciones      α−2 x d  x   α   = dt y 1−α α y a) Clasificar la soluci´on de equilibrio (silla, atractor, repulsor, etc.) para los distintos valores de α. b) Para α =

3 2

determine la soluci´on general del sistema y bosquejar el diagrama

de fases. 496

Apuntes Mat023 (versi´on preliminar actualizada 23-05-2014)

Certamen 8 1. Hallar y clasificar los puntos cr´ıticos de f : R2 → R definida por f (x, y) = x3 + y 3 + 9x2 − 3y 2 + 15x − 9y ¿Posee f extremos globales?. 2. Considere la funci´on

f (x, y) =

 x2 y   si (x, y) 6= 0   |x| + y 2    

0

si (x, y) = 0

a) Determine los puntos del dominio de f en los cuales la funci´on es continua. b) ¿Es f diferenciable en (0, 0)? ∂f (1, 1) es la derivada direccional de f en ∂a (1, 1) en la direcci´on a, determine el valor de

c) Si a es un vector unitario de R2 y

∂f (1, 1) kak=1 ∂a

m´ ax 2

a∈R ,

3. Determine las funciones f : R → R de clase C 1 (R) tales que las superficies  S1 = (x, y, z) ∈ R3 : y = f (x)  S2 = (x, y, z) ∈ R3 : z 2 + 2xz + y = 0 son tangentes en los puntos de intersecci´on. 4. Suponga que la funci´on de clase C 2 (R), f : R2 → R, (x, y) → f (x, y) satisface la ecuaci´on en derivadas parciales ∂ 2f ∂ 2f ∂ 2f −2 −3 2 =0 ∂x2 ∂x∂y ∂y y defina g : R2 → R por g (u, v) = f (αu + βv, δu + γv) donde α, β, δ y γ son constantes. Determine valores enteros no nulos de α, β, δ y γ para los cuales g cumple

∂ 2g =0 ∂u∂v 497

Apuntes Mat023 (versi´on preliminar actualizada 23-05-2014)

Certamen 9 1. Resolver la ecuaci´on diferencial t

d2 y dy − 2y = 0 + (1 − 2t) 2 dt dt

con las condiciones iniciales y (0) = 1, y 0 (0) = 2. 2. En el paraboloide 4x2 + y 2 + 4z 2 − 16x − 6y − 8z + 25 = 0 hallar el punto m´as cercano y mas alejado del plano 2x + 2y + z = 0. 3. Muestre que existen funciones u = u (x, y), v = v (x, y) definidas en un abierto U que contiene a (1, 1) tales que u (1, 1) = 0, v (1, 1) = 1, ux3 + xy 2 + u sin u = 1 y v 3 − 2xv + y 3 = 0 adem´as estudiar si la funci´on F : U ⊆ R2 → R2 , (x, y) → F (x, y) = (u (x, y) , v (x, y)) es localmente invertible cerca de (1, 1) y en caso afirmativo calcular DF −1 (0, 1). 4. Sea f : R2 → R, (x, y) → f (x, y) una funci´on de clase C 2 . a) Muestre que si g (r, θ) = f (r cos θ, r sin θ) entonces ∂ 2f ∂ 2f ∂ 2g 1 ∂ 2 g 1 ∂g + = + + ∂x2 ∂y 2 ∂r2 r2 ∂θ2 r ∂r b) Determine una funci´on f : {(x, y) ∈ R2 : 1 ≤ x2 + y 2 ≤ 9} → R tal que ∂ 2f ∂ 2f + = 0 para 1 < x2 + y 2 < 9 ∂x2 ∂y 2 f (x, y) = 1 si x2 + y 2 = 1 f (x, y) = 3 si x2 + y 2 = 9 suponiendo que g (r, θ) = f (r cos θ, r sin θ) no depende del ´angulo, es decir, ∂g ∂θ

= 0.

498

Apuntes Mat023 (versi´on preliminar actualizada 23-05-2014)

Certamen 10 1. Determine todas la funciones de clase C 1 y positivas f : R → R+ , que cumplan f (0) = 1 y tales que en todo intervalo [a, b] el ´area bajo la gr´afica de la funci´on y sobre el eje x sea igual a la longitud de arco de la gr´afica. 2. Determine la soluci´on general de la ecuaci´on d3 y d2 y dy − 6y = et + t − 6 + 11 3 2 dt dt dt 3. Si se sabe que una soluci´on de d2 y dy + f (x) + g (x) y = 0 2 dx dx es ϕ (x) = x y f (x) = − x1 (x + 2), determine g (x) y la soluci´on general de d2 y dy + f (x) + g (x) y = xex 2 dx dx 4. Construir una transformaci´on lineal T : R4 → R4 tal que ker (T ) = {(x, y, z, w) : x + y + z = 0 ∧ x − 2y + w = 0} e Im (T ) = {(x, y, z, w) : x − y − z = 0 ∧ x + 2w + y − z = 0} y determine [T ]BC donde B = {(1, 0, 0, 0) ; (1, 1, 0, 0) ; (1, 1, 1, 0) ; (1, 1, 1, 1)} y C es la base can´onica de R4 .

499

Apuntes Mat023 (versi´on preliminar actualizada 23-05-2014)

Certamen 11 1. Determine La serie de Fourier de ϕ : [−2, 2] → R     0 si x ∈ [−2, 0] x → ϕ (x) =    x si x ∈ [0, 2] ¿a qu´e converge la serie de Fourier de ϕ al evaluarla en x = 6? 2. Resolver el P.V.I.

    0 si t < 4

d2 y −y =  dt2  

t2 si t > 4

donde y (0) = y (0) = 0. 3. Determine las funciones de orden exponencial ϕ, φ : R → R que cumplen ϕ (0) = 0 y Z t ϕ (t − u) φ (u) du = t2 0 Z t et−u ϕ0 (u) du = t 0

a) Analizar para los distintos valores de µ en R la naturaleza de las soluciones de equilibrio del sistema 









d  x   µ 1  x  = dt y y −1 µ y bosquejar en cada caso el diagrama de fases. b) Resolver el P.V.I. 









2 1 x d  x    =  dt y −1 2 y x (0) = y (0) = 1

500

Apuntes Mat023 (versi´on preliminar actualizada 23-05-2014)

Certamen 12 1. Considere el siguiente modelo de din´amica poblacional:  dy y =r 1− y − Ey donde 0 < E < r, k 6= 0 dt k a) Hallar y clasificar las soluciones de equilibrio.  b) Si y (0) = k2 1 − Er bosquejar la soluci´on y determinar l´ımt→+∞ y (t). c) ¿Existen valores de los par´ametros E, k, r tal que dos puntos cr´ıticos consecutivos sean atractores? 2. Resolver el problema de valores iniciales xy 00 (x) + (2x − 1) y 0 (x) − 2y (x) = x2 e−3x , x > 0 donde y (1/2) = y 0 (1/2) = 0, si se sabe que y (x) = eαx es una soluci´on de la homog´enea asociada para un α adecuado. 3. Considere el problema π u (t) + a u (t) = 2 sin ωt + 4 0 u (0) = u (0) = 0 00

2



donde a es una constante conocida. a) Si a 6= ω resolver la ecuaci´on diferencial con las condiciones dadas. b) ¿Para qu´e valores de ω ocurre el fen´omeno de la resonancia? Justifique su respuesta. 4. Considere el sistema de ecuaciones dx = −5x + 4y dt dy = −4x + 5y dt a) Elaborar el retrato de fases del sistema y clasificar los puntos de equilibrio.

501

Apuntes Mat023 (versi´on preliminar actualizada 23-05-2014)

b) Bosquejar en el plano de fase la curva soluci´on con condiciones iniciales x (0) = 2, y (0) = 3 ¿Existe alg´ un instante t en el cual x (t) = y (t)? c) Bosquejar en el plano de fase la curva soluci´on con condiciones iniciales x (0) = 4, y (0) = 2 ¿Existe alg´ un instante t en el cual x (t) = y (t)?

502

Apuntes Mat023 (versi´on preliminar actualizada 23-05-2014)

Certamen 13 1. Sea f : R3 → R diferenciable y tal que ∇f (0, 2, 1) = (1, −1, −2)

y

f (0, 2, 1) = 4

Considerar adem´as la funci´on g(u, v) = f (u − v 2 , 3u − v , 3u2 − 2v) . Encuentre la ecuaci´on del plano tangente a la superficie z = g(u, v) en el punto (1, 1, 4) . 2. La ecuaci´on x2 + y 3 + z 4 + u5 = 2 define a u como funci´on de x , y y z y la ecuaci´on x + y 2 + z 3 = 1 define a z como funci´on de x e y . Entonces, se puede considerar u como funci´on de x e y . Entonces: du − (1 , −1) , donde → v est´a en la direcci´on (1 , 1). Hallar → d− v 3. Sean F : R3 → R3 y G : R2 → R3 dadas por F (x , y , z) = (u , v , w) determinado por el sistema u = x−y v = x2 − y 2 + z w = x−z y G(x , y) = (x − 2y + 1 , 2x − y , x2 − y 2 ) a) Determinar los puntos de R3 en los que F es localmente invertible. dy b) Determinar (0, 1, 0) dw c) Calcular el diferencial de F −1 ◦ G en (1, 1) 4. Considerar la elipse que se obtiene al interceptar el cilindro x2 + y 2 = 1 y el plano x + y + z = 0 . Encontrar la longitud del semieje mayor y del semieje menor.

503

Apuntes Mat023 (versi´on preliminar actualizada 23-05-2014)

Certamen 14 1. Resolver 00

t

Z

0

y − 2y + y − 2

y(u)du = 5 ;

t≥0

0

con y(0) = 0 ;

y 0 (0) = 0 .

2. Resolver el sistema 

x0 (t)





1 2 3

 

x(t)



       y 0 (t)  =  0 2 3  ·  y(t)        0 z (t) 0 0 2 z(t) 3. Considerar la funci´on f (x) = eλx ; −π < x < π

y f (x + 2π) = f (x) . Pruebe o refute que: ∞ X 1 (−1)n λ π = +2 sinh(λπ) λ λ2 + n2 n=1

504

Apuntes Mat023 (versi´on preliminar actualizada 23-05-2014)

Certamen 15 1. Encuentre la soluci´on general de la ecuaci´on de Euler no homog´enea: x2 y 00 + 2xy 0 − 2y = 10 cos (ln x) 2. Una masa que pesa 8 lbs. estira 4 pies un resorte. Al principio esta masa parte del reposo a 2 pies abajo de la posici´on de equilibrio y el movimiento ocurre en un medio que presenta una fuerza de amortiguamiento igual a la mitad de la velocidad instant´anea. Deducir la ecuaci´on del movimiento si se aplica una fuerza externa igual a f (t) = t cos(2t) 3. Para x > 0 y haciendo el cambio x =

1 t

encuentre la soluci´on general de la ecuaci´on   1 4 00 3 0 4x y + 8x y + y = tg 2x

4. Considere la ecuaci´on diferencial  2 y 00 + 4xy + 6 + 4x2 y = x2 e−x . 2

a) Pruebe que si y(x) = u(x)e−x es soluci´on, entonces u satisface una ecuaci´on diferencial lineal no homog´enea con coeficientes constantes. b) Use el resultado de la parte anterior para determinar la soluci´on y(x) con y(0) = 1 , y 0 (0) = 0

505

Apuntes Mat023 (versi´on preliminar actualizada 23-05-2014)

certamen 16 1. La Ley de Malthus supone que la tasa de crecimiento de una poblaci´on p , es directamente proporcional al tama˜ no de la poblaci´on en cada instante. a) Escribir la ecuaci´on diferencial que representa esta relaci´on y verificar que p(t0 ) = p0 , entonces p(t) = peα(t−t0 ) , para alguna constante α > 0 b) Sabiendo que la poblaci´on de la tierra aument´o, en promedio, el 2 % anual desde 1960 (α = 0,02) y que en 1965 se estimaba en 3.340 millones de personas. Calcular mediante este modelo en cuanto tiempo la poblaci´on se duplico (respecto de 1960). c) Este modelo tiene una correcci´on propuesta por Verhulst en 1837, que asume que al crecer mucho la poblaci´on y tener que competir por el alimento y espacio, el crecimiento se ve afectado por la falta de recursos. Este modelo est´a dado por la ecuaci´on log´ıstica p0 = αp − βp2 con α, β constantes reales positivas. Resuelva esta ecuaci´on. d ) Calcular para ambos modelos el l´ımite cuando t tiene a infinito y explicar su resultado. 2. Determine la distancia m´axima y m´ınima del origen (0, 0, 0) a los puntos de la curva definida por la intersecci´on de las superficies: z = x2 + y 2 ;

x + 2y + z = 10

3. Considere la transformaci´on F : R3 → R3 dada por F (x, y, z) = (u, v, w), donde u = xy 2

v = x + 3y

w = z − x.

Se verifica que F (4, 1, 2) = F (1, 2, −1) = (4, 7, −2). a) Pruebe que en torno a los puntos (4, 1, 2) y (1, 2, −1) existen inversas locales (x, y, z) = G(u, v, w) ;

(x, y, z) = H(u, v, w)

que satisfacen G(4, 7, −2) = (4, 1, 2) y H(4, 7, −2) = (1, 2, −1) respectivamente. 506

Apuntes Mat023 (versi´on preliminar actualizada 23-05-2014)

b) Calcular

∂x ´ en el punto (4, 7, −2) para G ´o H (ESCOGER SOLO UNA DE ∂v

ELLAS).

507

Apuntes Mat023 (versi´on preliminar actualizada 23-05-2014)

Certamen 17 1. Resuelva la ecuaci´on diferencial x3 y 000 + 4x2 y 00 + xy 0 − y = x2 ln x para x > 0. 2. Considerar la funci´on

  0 si 0 ≤ t < 1 f (t) =  1 si t≥1

Resuelva el sistema 9x0 − 32y 0 − 32y = f (t) 0

Z

−2x +

t

x (u) du + 8y 0 + 8y = 0

0

con x (0) = 32 e y (0) = 9. 3. Use el cambio y =

z 1+x2

para resolver la ecuaci´on

  1 + x2 y 00 + 4xy 0 + 2 3 + 2x2 y =

508

2 2 + + 2x cos (2x) sin (2x)

Apuntes Mat023 (versi´on preliminar actualizada 23-05-2014)

Certamen 18 1. Sea T : R3 → R2 una transformaci´on lineal. Sean B = {(1, 1, 1) , (1, 1, 0) , (1, 0, 0)} y U = {(1, 1) , (1, −1)} bases de R3 y R2 respectivamente. Considere

 A = [T ]UB = 

1

0 −1

−1 1

2

 

Determine: a) [T (1, 1, 0)]U b) T (3, 2, 1) c) ¿T es inyectiva? a) Si w = f



y−x z−y , yz xy



. Probar que x2

∂w ∂w ∂w + y2 + z2 =0 ∂x ∂y ∂z

b) Sea f : R2 → R definida por   x2 si x2 ≥ y 2 f (x, y) =  y si x2 < y 2 determine el m´aximo dominio de continuidad de f . 2 2 2

2

2

2

2

2. Pruebe que el m´aximo valor de x y z bajo la condici´on x + y + z = R es Deduzca de esto que p 3

x2 y 2 z 2 ≤



R2 3

3

.

x2 + y 2 + z 2 3

3. Un canaleta cuya secci´on transversal tiene forma de trapecio, con a´ngulos en la base iguales, se va a construir doblando bandas iguales a lo largo de ambos lados de una larga pieza de metal, de 12 pulgadas de ancho. Encuentre los a´ngulos de la base y las dimensiones de los lados que producen la m´axima capacidad de acarreo.

509

Apuntes Mat023 (versi´on preliminar actualizada 23-05-2014)

Certamen 19 1. Dada la funci´on definida por f (x, y) =

 

(y−2)2 sin(xy) x2 +y 2 −4y+4

si

(x, y) 6= (0, 2)



0

si

(x, y) = (0, 2)

¿Es f una funci´on continua en R2 ? 2. El volumen de un elipsoide de semiejes a, b, c es

4π abc. 3

Hallar el elipsoide con centro

(0, 0, 0) de volumen m´ınimo que pasa por (2, −3, 5). 3. Considere las ecuaciones uv − 3x + 2y = 0 u4 − v 4 = x2 − y 2 Verifique que ellas definen funciones u = u (x, y), v = v (x, y) en torno al punto (u, v, x, y) = (1, 1, 1, 1), adem´as, determine la ecuaci´on del plano tangente a la superficie u = u (x, y) en (1, 1, 1). 4. Sea f : R2 → R de clase C 2 a) Pruebe que si f es homog´enea de grado p, es decir, f (tx, ty) = tp f (x, y) entonces x2

2 ∂ 2f ∂ 2f 2∂ f + 2xy + y = p (p − 1) f ∂x2 ∂x∂y ∂y 2

Ind.: g (t) = f (tx, ty) derivar dos veces respecto a t. b) Probar que si (x, y) · ∇f (x, y) = pf (x, y) para todo (x, y) entonces p es homog´enea de grado p. Ind.: Defina g (t) = f (tx, ty) − tp f (x, y) y calcule la derivada.

510

Apuntes Mat023 (versi´on preliminar actualizada 23-05-2014)

Certamen 20 1. Sea f (x, y) =

 

xy 3 x2 +y 2

si (x, y) 6= (0, 0)



0

si (x, y) = (0, 0)

a) ¿Es f continua en (0, 0)? b) Hallar fx (x, y) y fy (x, y). 2. (Plano tangente y regla de la cadena) a) Dada la superficie S : x2 + 2y 2 + 3z 2 = 21 hallar la(s) ecuaci´on(es) del (de los) plano(s) tangente(s) a S que es (son) paralelos(s) al plano x + 4y + 6z = 0. b) Si z = f (x, y) es de clase C 1 y x = r cos θ e y = r sin θ, probar que sin θ

∂z cos θ ∂z ∂z + = ∂r r ∂θ ∂y

3. (M´aximos y m´ınimos) a) Encuentre los m´aximos y/o m´ınimos de la funci´on f (x, y, z) = x2 + y 2 + z 2 + xy + yz + xz + x − 2y b) Determine los ´angulos α, β, γ de un tri´angulo de modo que el producto de sus senos sea m´aximo.

511

Apuntes Mat023 (versi´on preliminar actualizada 23-05-2014)

Certamen 21 1. Considere la funci´on f definida como sigue:   2yx3 si (x, y) 6= (0, 0) x2 +y 2 f (x, y) =  0 si (x, y) = (0, 0) a) Determine si la funci´on f (x, y) es continua en todo R2 . b) Determine si la funci´on f (x, y) es diferenciable en todo R2 . c) Determine el valor de fxy (0, 0) y fyx (0, 0). 2. Sean x la cantidad de sillas e y la cantidad de mesas producidas por un fabricante. Si las funciones f (x, y) = 256 − 3x − y y g (x, y) = 222 + x − 5y corresponden a los precios unitarios de venta de los productos respectivamente, hallar las cantidades de sillas y mesas de modo de obtener m´aximas utilidades sabiendo que el costo de producci´on total es C (x, y) = x2 + xy + y 2 . 3. Dada la ecuaci´on sin (yz) + sin (xz) + sin (xy) = 0 a) Encuentre las condiciones para que z este definida impl´ıcitamente como funci´on de las variables x e y cerca de (x, y, z) = (1, 0, π). b) Encontrar el plano tangente a la gr´afica de z = g (x, y) en (x, y, z) = (1, 0, π). 4. Dada la funci´on z = f (2x + 3g (y)) a) Encuentre las condiciones para que la funci´on z = f (2x + 3g (y)) sea dos veces diferenciable en R2 . b) Bajo los supuestos encontrados en la parte anterior, determine el valor de k de modo que  k=

∂z ∂ 2 z ∂z ∂ 2 z − ∂x ∂x∂y ∂y ∂x2

512



Apuntes Mat023 (versi´on preliminar actualizada 23-05-2014)

Certamen 22 1. Sea T : R3 → R3 la transformaci´on lineal definida por T (x, y, z) = (x + z, y + 3z, x + y + αz) con α ∈ R: a) Determine el valor de la constante α para que dim Ker(T ) = 1 y en este caso Calcule Ker (T ). b) Para el valor anterior de α calcule Im (T ). 2. Sean p (x) y q (x) dos funciones continuas. Verificar que la sustituci´on y = ez con z = z (x) transforma la ecuaci´on diferencial y 0 + p (x) y = q (x) y ln y en una ecuaci´on lineal de primer orden. Usando lo anterior, resolver la ecuaci´on xy 0 = 2x2 y + (x + 1) y ln y 3. Hallar la soluci´on general de una ecuaci´on diferencial lineal a coeficientes constantes homog´enea, cuya ecuaci´on caracter´ıstica es: λ5 − 2λ4 + 6λ3 − 9λ2 + 8λ − 4 = 0 sabiendo que y = ex/2 cos

√

3 x 2



es una soluci´on de dicha ecuaci´on.

4. Hallar la soluci´on general de la ecuaci´on diferencial dy d2 y + tan (x) + cos2 (x) y = 0 2 dx dx utilizando para ello el cambio de variables t = sin x.

513

Apuntes Mat023 (versi´on preliminar actualizada 23-05-2014)

Certamen 23 1. (40 pts.) Sea f : R2 → R la funci´on definida   −2y 2 ,     0 , f (x, y) =   2 2  x +y    2 sin (x + y) , x + |y|

por: (x, y) ∈ A (x, y) = (0, 0) (x, y) ∈ AC ∧ (x, y) 6= (0, 0)

en donde: A= a) Calcule



(x, y) ∈ R2 : y > 0 ∧ −x ≤ y ≤ x



∂f ∂f (0, 0) y (0, 0). ∂x ∂y

b) ¿Es diferenciable f en (0, 0)? 2. Sean f : R3 → R una funci´on tal que ∇f (1, 1, −2) = (−1, 1, 0) y h : R2 → R otra funci´on definida por h(s, t) = 2s2 + st:  (a) Encuentre un vector unitario normal a la curva de nivel (s, t) : h(s, t) = 2 en el punto (1, 0). (b) Considere ω : R2 → R definida por ω(s, t) = f 3x + 2xy + z 2 , x + y 2 , x − 2z 2 en donde:

x(s, t) = 2st,

y(s, t) = s2 − 2t y



z(s, t) = s + t. Calcule

∇ω(1, 0). (c) Encuentre la derivada direccional de ω en el punto (1, 0) en la direcci´on dada por el vector calculado en la parte (a). 3. Sean f : R2 → R una funci´on definida por: f (x, y) = x − x2 − y 2 y el conjunto U = {(x, y) : x2 + y 2 ≤ 1}: ˚ = {(x, y) : x2 + y 2 < 1}, a) Mediante el criterio del hessiano, determine los extremos de f en U en caso de existencia. 514

Apuntes Mat023 (versi´on preliminar actualizada 23-05-2014)

b) Mediante el m´etodo de los multiplicadores de Lagrange, calcule los extremos de f en ∂U = {(x, y) : x2 + y 2 = 1}, si acaso existen.

515

Apuntes Mat023 (versi´on preliminar actualizada 23-05-2014)

Certamen 24 1. Sea x = x (t) una funci´on que satisface el sistema de ecuaciones x00 = y − z y 00 = x0 + z 0 z 00 = − (1 + x + y) donde x (0) = x0 (0) = 0, y (0) = −1, y 0 (0) = 1 y z (0) = 0, z 0 (0) = 1. Calcule, de ser posible, el valor de x (π). 2. Sea f : R → R la funci´on peri´odica definida por f (x) = e−x para − π < x < π y f (x + 2π) = f (x). Calcule la serie de Fourier de f . 3. Sea f : R2 → R una funci´on de clase C 2 (R2 ). Considere el cambio de variables x = u+v y = uv 2 y la funci´on g : R2 → R definida por g (u, v) = f (x (u, v) , y (u, v)). Calcule el valor de

sabiendo que

∂2f ∂x2

∂ 2g ∂ 2g (1, 1) + (1, 1) ∂u2 ∂v 2 2 ∂2f (2, 1) = ∂y∂x (2, 1) = ∂∂yf2 (2, 1) = 1 y

∂f ∂y

(2, 1) = 2.

4. Se desea construir un tanque que consistir a de un cilindro circular recto de altura h y radio r, una tapa superior semiesf´erica de radio r y finalmente una tapa inferior plana del mismo radio. Suponga que los costos de construcci´on de la tapa semiesf´erica son de $20 por [m2 ]; que los costos de construcci´on de la cara lateral cil´ındrica son de $8 por [m2 ] y de $5 por [m2 ] para la base circular plana. a) Hallar el valor de h y r de modo que el costo de construcci´on sea m´ınimo asumiendo que el volumen debe ser de 200π [m3 ]. b) ¿A cuanto es igual la relaci´on h : r?.

516

Apuntes Mat023 (versi´on preliminar actualizada 23-05-2014)

Certamen 25 1. Resuelva usando la transformada de Laplace el siguiente problema de valores iniciales ty 00 − 2y 0 + ty = 0 con y (0) = 1, y 0 (0) = 0 2. Sea f : [0, π[ → R la funci´on definida por:   2t si 0 ≤ t < π2 π f (t) =  2(π−t) si π ≤ t < π π 2 Desarrollar f (t) en una serie de Fourier en t´erminos del seno. 3. Considere la ecuaci´on diferencial de primer orden √ dx = (x − 1) (x − a) x x2 + x + 1 dt en donde a es un par´ametro real. Determine condiciones sobre a de modo que la soluci´on de equilibrio x = 0 sea un punto atractor.

517

Apuntes Mat023 (versi´on preliminar actualizada 23-05-2014)

Certamen 26 1. Sea T : R2 [x] → R2 la funci´on definida por  Z 0 T (p (x)) = p (1) , 6

1

 p (x) dx

0

a) Pruebe que T es una transformaci´on lineal. b) Hallar el n´ ucleo de T y una base para la imagen de T . c) Sean B = {1, 1 + x, x + x2 } y D = {(1, −1) , (0, 1)} bases ordenadas de R2 [x] y R2 respectivamente. Calcular [T ]D B. 2. Hallar la soluci´on general de la ecuaci´on x2 (1 − x) y 00 + 2x (2 − x) y 0 + 2 (1 + x) = x2 sabiendo que la homog´enea asociada tiene una soluci´on de la forma y = x−2 . 3. La ecuaci´on homog´enea y (4) − 4y 000 + 11y 00 + 8y 0 − 26y = 0 tiene una soluci´on de la forma y = e2x cos 3x. Resolver la ecuaci´on y (4) − 4y 000 + 11y 00 + 8y 0 − 26y = x 4. Se suministra bacterias como alimento a una poblaci´on de protozoos a una tasa  gr  constante de 1 min . Se ha observado que las bacterias son consumidas a una tasa de cuatro veces el cuadrado de su cantidad c (t). Hallar c (t) en funci´on de c (0) = c0 .

518

Apuntes Mat023 (versi´on preliminar actualizada 23-05-2014)

Certamen 27 1. Sea x > 1. Resuelva la ecuaci´on diferencial: (1 − x) y 00 + xy 0 − y = (1 − x)2 cosh x sabiendo que una soluci´on de la ecuaci´on homog´enea asociada es y = ex . 2. Laplace:  a) Calcule L t sen t .  b) Si L f (t) = X (s), calcule f (t), sabiendo para ello que X (s) satisface la ecuaci´on:

 2s (1 − e−2πs ) s2 + 1 X (s) = s2 + 1

3. Considere el siguiente sistema de ecuaciones:  Z t   x0 + 2x + 6 y (u) du = −2 0

 

x0 + y 0 + y =

0

en donde x (0) = −5 e y (0) = 6. Calcule el valor de x (ln 2). 4. Sea f : R2 → R la funci´on definida por:  xy 2   , si y > x2   2 + y2  x   x2 y f (x, y) = , si y ≤ x2 ∧ (x, y) 6= (0, 0)  2 + y2  x      0 , si (x, y) = (0, 0) Determine todos los puntos de R2 para los cuales la funci´on f es continua.

519

Apuntes Mat023 (versi´on preliminar actualizada 23-05-2014)

Certamen 28 1. Considere la funci´on f : R2 → R definida por:    xy sin x , (x, y) 6= (0, 0) x2 + y 2 f (x, y) =   0 , (x, y) = (0, 0) a) Determine si f es continua en (0, 0). b) Determine todas las derivadas direccionales de f en (0, 0). c) Determine si f es diferenciable en (0, 0). 2. Sea z = z (x, y) una funci´on de clase C 2 . Escriba la ecuaci´on: ∂ 2z ∂ 2z ∂ 2z + 2 − 3 =0 ∂x2 ∂x∂y ∂y 2 en las variables u y v definidas por las ecuaciones: u = 3x − y



v =x+y

3. Determine los valores de las constantes a, b y c en R de modo que la derivada direccional de: f (x, y, z) = acy 2 + byz + cz 2 x3 en el punto (1, 2, −1) tenga un valor m´aximo de 64 en la direcci´on del eje X positivo. 4. Hallar las dimensiones del paralelep´ıpedo rectangular de mayor volumen con aristas paralelas a los ejes coordenados que puede ser inscrito en el elipsoide de ecuaci´on:  x 2 3

+

 y 2 4

520

+

 z 2 5

=1

Apuntes Mat023 (versi´on preliminar actualizada 23-05-2014)

Certamen 29 1. Resuelva la ecuaci´on diferencial: (1 + x)2 y 00 − 3 (1 + x) y 0 + 4y = (1 + x)3 utilizando para ello el cambio de variables et = 1 + x. 2. Resuelva la ecuaci´on diferencial: x00 + tx0 − x = 0,

x (0) = 0,

x0 (0) = 1

3. Sea z ∈ C 2 . Simplificar al m´aximo la ecuaci´on: y

∂ 2z ∂ 2z ∂ 2z + (x + y) + x =0 ∂x2 ∂x∂y ∂y 2

considerando para ello el cambio de variables dado por: u = y 2 − x2



v =y−x

4. Sean f : R3 → R una funci´on diferenciable tal que: ∇f (0, 2, 1) = (1, −1, −2)



f (0, 2, 1) = 4

y g la funci´on definida por g (u, v) = f (u − v 2 , 3u − v, 3u2 − 2v). Hallar la ecuaci´on del plano tangente a la superficie z = g (u, v) en el punto (1, 1, 4) . 5. Determine el m´aximo y el m´ınimo absolutos de la funci´on: z = sin x + sin y + sin (x + y) en la regi´on: 0≤x
0, bajo el supuesto que la ecuaci´on homog´enea tiene una soluci´on de la forma y = emx 2. Resuelva la ecuaci´on diferencial: 2 (1 + x)2 y 00 − 6 (1 + x) y 0 + 8y = (x + 1)3 3. Considere la funci´on f : R2 → R definida por:   x|y|3/2 , (x, y) 6= (0, 0) x2 +y 2 f (x, y) =  0 , (x, y) = (0, 0) a) Determine si f es continua en (x, y) = (0, 0) b) Determine si f es diferenciable en (x, y) = (0, 0) 4. Sean f, g : R → R funciones de clase C 2 . Considere z : U ⊆ R2 → R definida por: z (x, y) = x f Calcule el valor de: x2

y x

+g

y x

2 ∂ 2z ∂ 2z 2∂ z + 2xy + y ∂x2 ∂x∂y ∂y 2

5. Determine el m´aximo y el m´ınimo absoluto de la funci´on: z = x3 + y 3 − 3xy en la regi´on: 0≤x≤2



523

−1 ≤ y ≤ 2

Cap´ıtulo : Bibliograf´ıa

[1] Kreyszig, E. Advanced Engineering Mathematics, 9th Ed., John Wiley & Sons Inc., Singapore, 2006. [2] Piskunov, N. C´ alculo Diferencial e Integral, Editorial Limusa S.A. de C.V., M´exico, 2007. [3] Osses, A. Ecuaciones Diferenciales Ordinarias, CMM, Departamento de Ingenier´ıa Matem´atica, U. de Chile, Santiago, 2010. [4] Derrick, W & Grossman, S Ecuaciones diferenciales con aplicaciones, Fondo Educativo Interamericano, M´exico, 1984. [5] Apostol, T. Calculus: volumen 1, Editorial Revert´e, Barcelona, 1967. [6] Fern´andez, C. & Rebolledo, R. Ecuaciones diferenciales ordinarias, Ediciones Universidad Cat´olica de Chile, Santiago, 1995. [7] Hsu, H. An´alisis de Fourier, Addison-Wesley Iberoamericana, EE.UU., 1987. [8] Rocha, J.M. & Villa, G. C´alculo infinitesimal de varias variables: vol. 1, I.P.N, M´exico, 2003. [9] Taylor,H & Wade, T. C´alculo diferencial e integral, Limusa–Wiley, M´exico, 1972. [10] Gavil´an, E. Dossier de problemas resueltos, Departamento de Matem´aticas, U. de Concepci´on, Concepci´on. [11] Mart´ınez, C. C´alculo real y vectorial en varias variables, Instituto de Matem´aticas, P. U. Cat´olica de Valpara´ıso, Valpara´ıso, 2000. [12] Jerrold E. Marsden & Anthony J. Tromba. C´alculo vectorial, 5th Ed. Pearson. Estados Unidos 2010.

524

Anexo 1 : Modificaciones del apunte

En el cap´ıtulo de transformada de Laplace se modific´o la respuesta de t ∗ sin t (en esta versi´on p. 250) En el cap´ıtulo de transformada de Laplace, se repar´o error en el ejemplo de la transformada inversa (en esta versi´on p. 251) Nuevo ejemplo de convoluci´on p. 251.

3